Spine - Spinal Cord Injuries Flashcards

1
Q

CASE 1: Terminology of Spinal Cord Injuries HISTORY: A 19-year-old male noted neck pain and immediate weakness in all limbs after making a tackle while playing football. He was taken to the local emergency room. EXAMINATION: On examination there was no evidence of trauma except tenderness of the mid-cervical region. He was profoundiy weak (i.e. unable to overcome gravity) in his lower extremities but he was able to move his toes, ankles, knees And hips bilaterally. The patient had no movements of the upper extremities below the biceps muscle levels (C6) bilaterally and had absent deep tendon reflexes in all extremities. The patient had a sensory level at C6 bilaterally which was complete below this level in the arms and upper torso with partial sparing of pinprick and joint position sensation in the lower torso, legs and sacral region. The patient’s anal tone was reduced. X-RAY: Cervical spine x-rays showed a fracture-dislocation at C5-5. HOSPITAL COURSE: In the emergency room, the patient’s family inquired about the nature of the injury and the emergency room physician told them the patient was “paralyzed”.

  1. Terms which might correctly be used to describe the motor aspect of this patient’s spinal injury are (Select one or more)

A. Paralysis

B. Paresis

C. Quadriplegia

D. Quadraparesis

E. Central cord syndrome

A

(A,B,D,E) Impairment of strength results in weakness, or paresis; and loss of strength, in paralysis. Plegia is used synonymously with paralysis. This patient is paralyzed in his upper extremities and paretic in his lower extremities. Loss of strength in all four limbs is termed quadraplegia, whereas impaired strength involving four limbs is termed quadraparesis. Since the patient has sparing of some motor function in his lower extremities, he is quadraparetic. Central cord syndrome is characterized by more motor impairment of the upper than the lower extremities, bladder dysfunction and varying decrees of sensory loss below the level of the lesion.

How well did you know this?
1
Not at all
2
3
4
5
Perfectly
2
Q

CASE 1: Terminology of Spinal Cord Injuries HISTORY: A 19-year-old male noted neck pain and immediate weakness in all limbs after making a tackle while playing football. He was taken to the local emergency room. EXAMINATION: On examination there was no evidence of trauma except tenderness of the mid-cervical region. He was profoundiy weak ( i . e . unable to overcome gravity) in his lower extremities but he was able to move his toes, ankles, knees And hips bilaterally. The patient had no movements of the upper extremities below the biceps muscle levels (C6) bilaterally and had absent deep tendon reflexes in all extremities. The patient had a sensory level at C6 bilaterally which was complete below this level in the arms and upper torso with partial sparing of pinprick and joint position sensation in the lower torso, legs and sacral region. The patient’s anal tone was reduced. X-RAY: Cervical spine x-rays showed a fracture-dislocation at C5-5. HOSPITAL COURSE: In the emergency room, the patient’s family inquired about the nature of the injury and the emergency room physician told them the patient was “paralyzed”.

  1. The patient has a “complete” spinal injury. (True or False)
A

FALSE

In cases of spinal cord trauma any preservation of volitional motor, sensory or autonomic function below the level of the lesion indicates an “incomplete” injury. Reflex activities alone are preserved below the level of the lesion in “complete” spinal injuries. This patient had sparing of sensation below the level of the lesion and therefore, had an “incomplete” spinal injury.

How well did you know this?
1
Not at all
2
3
4
5
Perfectly
3
Q
  1. The following terms are often used to describe other.spinal injuries. Indicate whether each of the definitions is either true or false.

A. Hemiparesis is defined as weakness involving upper and lower limbs on one side of the body

B. Hemiplegia is defined as paralysis involving upper and lower limbs on one side of the body

C. Hemiplegia alternans is defined as fluctuating weakness of upper and lower limbs on one side of the body

D. Monoparesis is defined as weakness of only one limb

E. Monoplegia is defined as paralysis of only one limb

F. Triplegia is defined as paralysis of three limbs

G. Tetraplegia is defined as paralysis of four limbs

H. Diplegia is defined as paralysis of an upper and lower extremity

I. Brown-Sequard syndrome involves unilateral spinal injury with ipsilateral pinprick and temperature loss and loss of contralateral motor and joint position sense

J. Anterior spinal artery syndrome is defined as paralysis with hypesthesia and hypalgesia below the level of the lesion combined with preservation of joint position, vibratory and touch sensation

A

A. TRUE

B. TRUE

C. TRUE

D. TRUE

E. TRUE

F. TRUE

G. TRUE

H. FALSE

I. FALSE

J. TRUE

Hemiplegia alternans affects one limb on one side of the body and another limb on.the opposite side (i.e. the right upper and left lower extremities). Diplegia isparalysisof like parts on the two sides of the body (i.e. both upper extremities). The Brown-Sequard syndrome, due to hemisection of the spinal cord, is characterized by ipsiiateral paralysis, ipsilateral joint position loss and contralateral pain and temperature loss below the level of the lesion

How well did you know this?
1
Not at all
2
3
4
5
Perfectly
4
Q

CASE 2: Spinal Cord Injury in a 22-Year-Old Man After a HISTORY: A 22-year-old male was seen in the emergency room one hour after a motor vehicle accident. He complained of severe neck pain, numbness of his arms and legs and inability to move his legs. EXAMINATION: Physical examination revealed a complete flaccid paralysis below the C5 level and anesthesia below C5. Paralysis of rectal and bladder sphincters was also present. A clinical diagnosis of cervical spinal cord injury was made. X-RAY: A lateral cervical spine x-ray showed a fracture-dislocation at the C5-6 level.

  1. Males make up a significantly greater proportion of the population who sustain cervical spinal cord injuries than females. (True or False)
A

TRUE

A retrospective study of 356 patients with major cervical spinal cord injuries which occurred between 1963 and 1972, was carried out in southern California. In this study, males were injured five times more frequently than women.

How well did you know this?
1
Not at all
2
3
4
5
Perfectly
5
Q

CASE 2: Spinal Cord Injury in a 22-Year-Old Man After a HISTORY: A 22-year-old male was seen in the emergency room one hour after a motor vehicle accident. He complained of severe neck pain, numbness of his arms and legs and inability to move his legs. EXAMINATION: Physical examination revealed a complete flaccid paralysis below the C5 level and anesthesia below C5. Paralysis of rectal and bladder sphincters was also present. A clinical diagnosis of cervical spinal cord injury was made. X-RAY: A lateral cervical spine x-ray showed a fracture-dislocation at the C5-6 level.

  1. The incidence of significant spinal cord injuries per million population per year is about (Select only one)
    A. less than 10

B. 10-20

C. 30-40

D. more than 50

A

(B)

Statistics on the incidence of spinal cord injuries are rare. A few estimates from Switzerland, Australia and the United States are available. Based on European statistics, a figure of 10-20 new spinal cord injuries per million population per year is reasonable. Kraus estimated a much higher figure of 53. 4 per million based on statistics from northern California. The methods used in the studies vary and probably account for the differences in results. The European figures are based mainly on admission to specialized spinal cord injury centers in relatively small countries with well organized national health care systems. The United States figures included all types of injury, including deaths on hospital arrival and minor injuries without significant sequelae. The U.S. incidence indeed may be higher than European countries or it may represent broader definition and more careful search for such injuries. Only further experience will define the true incidence of spinal cord injury. Kraus estimated that, based on his figures, 7.000 new spinal cord injuries occur per year in surviving patients in the United States.

How well did you know this?
1
Not at all
2
3
4
5
Perfectly
6
Q

CASE 2: Spinal Cord Injury in a 22-Year-Old Man After a HISTORY: A 22-year-old male was seen in the emergency room one hour after a motor vehicle accident. He complained of severe neck pain, numbness of his arms and legs and inability to move his legs. EXAMINATION: Physical examination revealed a complete flaccid paralysis below the C5 level and anesthesia below C5. Paralysis of rectal and bladder sphincters was also present. A clinical diagnosis of cervical spinal cord injury was made. X-RAY: A lateral cervical spine x-ray showed a fracture-dislocation at the C5-6 level.

  1. The most common location of spinal cord injury is (Select only one)

A. cervical

B. thoracic

C. lumbar

D. sacral

A

(A)

Gjone estimated that 53% of spinal cord injuries occur in the cervical region. About 25% occur in the thoracic area, about 12% in the lumbar area and about 9% involve the sacrum and cauda equina region.

How well did you know this?
1
Not at all
2
3
4
5
Perfectly
7
Q

CASE 2: Spinal Cord Injury in a 22-Year-Old Man After a HISTORY: A 22-year-old male was seen in the emergency room one hour after a motor vehicle accident. He complained of severe neck pain, numbness of his arms and legs and inability to move his legs. EXAMINATION: Physical examination revealed a complete flaccid paralysis below the C5 level and anesthesia below C5. Paralysis of rectal and bladder sphincters was also present. A clinical diagnosis of cervical spinal cord injury was made. X-RAY: A lateral cervical spine x-ray showed a fracture-dislocation at the C5-6 level.

  1. The most common age range for occurrence of spinal cord injury is (Select only one)

A. below 20

B. 20-30 years

C. 30-40 years

D. 40-50 years

E. over 50 years

A

(B)

Unquestionably, spinal cord injury is a disease primarily of young adults. The peak incidence in males 20-30 years is twice that for males 40-50 years old. 10 Spinal cord injury is rare in children under 10. Similar age differences are noted in females and males although the female ratio is about hail that for males in all ages.

How well did you know this?
1
Not at all
2
3
4
5
Perfectly
8
Q

CASE 2: Spinal Cord Injury in a 22-Year-Old Man After a HISTORY: A 22-year-old male was seen in the emergency room one hour after a motor vehicle accident. He complained of severe neck pain, numbness of his arms and legs and inability to move his legs. EXAMINATION: Physical examination revealed a complete flaccid paralysis below the C5 level and anesthesia below C5. Paralysis of rectal and bladder sphincters was also present. A clinical diagnosis of cervical spinal cord injury was made. X-RAY: A lateral cervical spine x-ray showed a fracture-dislocation at the C5-6 level.

  1. Which of the following is true concerning cervical spinal cord injuries? (Select only one)

A. Fifty percent occur in patients ages 16-25 years

B. Automobile accidents account for 501% of cases

C. The most common site of fracture-dislocation is at C5-3

D. All of the above

A

(D)

All three are true. Heiden found that one half of 356 patients with cervical spinal cord injury were between the ages of 16 and 25. The same authors noted that 50% of cervical spinal cord injuries resulted from automobile accidents. A significant number of injuries were related to athletic events and of these 88% were due to water sports such as diving, surfing and water skiing.

How well did you know this?
1
Not at all
2
3
4
5
Perfectly
9
Q

CASE 2: Spinal Cord Injury in a 22-Year-Old Man After a HISTORY: A 22-year-old male was seen in the emergency room one hour after a motor vehicle accident. He complained of severe neck pain, numbness of his arms and legs and inability to move his legs. EXAMINATION: Physical examination revealed a complete flaccid paralysis below the C5 level and anesthesia below C5. Paralysis of rectal and bladder sphincters was also present. A clinical diagnosis of cervical spinal cord injury was made. X-RAY: A lateral cervical spine x-ray showed a fracture-dislocation at the C5-6 level.

  1. Which of the following is true concerning the immediate treatment of complete transverse injury to the cervical spinal cord? (Select only one)

A. Immediate decompressive laminectomy is indicated

B. Immediate anterior cervical exploration and fusion is indicated

C. The incidence of recovery following complete transverse injury to the cervical spinal cord is very low and is not favorably affected by immediate operative therapy

D. None of the above

A

(C)

Tidak ada pembahasan

How well did you know this?
1
Not at all
2
3
4
5
Perfectly
10
Q

CASE 3: Mortality, Survival and Spinal Cord Injury HISTORY: A patient (X) years old suffered a (complete/incomplete) spinal cord injury at (Y) level. The patient was successfully resuscitated from all associated injuries and had no fatal complications for the first month following injury.

  1. As a member of a large unselected group of patients with spinal injuries, what is the likelihood that the patient will survive for 10 years? (Select only one)

A. Less than 25%

B. Less than 50%

C. 50%

D. Greater than 50%

E. Greater than 75%

A

(D)

On the average, more than 50% of a random group of patients with traumatic spinal cord lesions will be alive 10 years after the injury. Fifty-two percent of quadriplegics and 70. 3% of paraplegics will survive at least this length of time.

How well did you know this?
1
Not at all
2
3
4
5
Perfectly
11
Q

CASE 3: Mortality, Survival and Spinal Cord Injury HISTORY: A patient (X) years old suffered a (complete/incomplete) spinal cord injury at (Y) level. The patient was successfully resuscitated from all associated injuries and had no fatal complications for the first month following injury.

  1. if the patient survives longer than three months, his overall chance of surviving 10 years (Select only one)

A. does not change significantly from question I

B. changes to 50% or less

C. changes to 50%

D. changes to 50% or more

E. changes to 75% or more

A

(E)
There is an early high mortality rate regardless of the patient’s age and the extent and level of the spinal lesion at the time of injury. However, for those patients that survive the first three months after injury, overall 10 year survival is 80% or greater. 4 In fact, for those who survive beyond the first three months, long term survival approximates closely the expected survival of the general population.

How well did you know this?
1
Not at all
2
3
4
5
Perfectly
12
Q

CASE 3: Mortality, Survival and Spinal Cord Injury HISTORY: A patient (X) years old suffered a (complete/incomplete) spinal cord injury at (Y) level. The patient was successfully resuscitated from all associated injuries and had no fatal complications for the first month following injury.

  1. The patient’s age (X) is 20 years at the time of injury. His likelihood of 10 years, survival is approximately twice that of a patient who at the time of injury (Select only one)

A. is 25-35 years old

B. is 35-45 years old

C. is 45 years old or older

D. age is a variable that does not significantly influence long term survival

A

(C)

Age is an important determining variable influencing survival for spinal cord injury. 1, 5 i n a group of paraplegics, 10 year survival for 15-24 year old was 86% whereas survival decreased to 41% for 45-59 year old.

How well did you know this?
1
Not at all
2
3
4
5
Perfectly
13
Q

CASE 3: Mortality, Survival and Spinal Cord Injury HISTORY: A patient (X) years old suffered a (complete/incomplete) spinal cord injury at (Y) level. The patient was successfully resuscitated from all associated injuries and had no fatal complications for the first month following injury.

  1. The patients neural lesion is complete as opposed to incomplete. This is a detrimental factor with regard to his likelihood of long term survival. (True or False)
A

TRUE

For patients with lesions at the same level, those with incomplete lesions fare significantly better than those with complete lesions. For instance, in the cervical region overall, 10 year survival is 69% and 40% for incomplete and complete lesions, respectively.

How well did you know this?
1
Not at all
2
3
4
5
Perfectly
14
Q

CASE 3: Mortality, Survival and Spinal Cord Injury HISTORY: A patient (X) years old suffered a (complete/incomplete) spinal cord injury at (Y) level. The patient was successfully resuscitated from all associated injuries and had no fatal complications for the first month following injury.

  1. The level of the patient’s lesion (Y) is C4. The patient’s chance of 3 month survival is approximately half that of a patient with a lesion at (Select only one)

A. C6

B. T6

C. T12

D. L2 or lower

E. survival in spinally injured patients is not significantly affected by the level of the lesion

A

(C)

Forty-three percent of patients with C1 - 5 lesions survived the first three months after injury, whereas approximately 85% of patients with T2-11 lesions survived at least this length of time. Approximately 70-75% of patients with intermediate level lesions survived three months or more and patients with lesions below thoracic levels have even better survival rates. The level of a spinal lesion is an important variable determining survival following spinal injury.

How well did you know this?
1
Not at all
2
3
4
5
Perfectly
15
Q

CASE 3: Mortality, Survival and Spinal Cord Injury HISTORY: A patient (X) years old suffered a (complete/incomplete) spinal cord injury at (Y) level. The patient was successfully resuscitated from all associated injuries and had no fatal complications for the first month following injury.

  1. Having survived for the first month., the patient is most likely to die of what cause? (Select only one)

A. Renal failure

B. Cardiovascular disease

C. Pneumonia

D. Decubitus ulcers

E. Suicide

A

(A)

Although all the listed causes of death except B, cardiovascular, are seen with a higher incidence in the spinally injured group than in the general population, renal failure remains the most common fatal complication of spinally injured patients.

How well did you know this?
1
Not at all
2
3
4
5
Perfectly
16
Q

CASE 4: Neck Pain in a 37-Year-Old Man Following Cerebral Concussion HISTORY: A 37-year-old man was a passenger in an automobile involved in an accident. He was thrown clear of the automobile and rendered unconscious. A paramedic at the accident scene requests your advice, via two-way radio, concerning transportation of the patient. The paramedic reports that the patient was unconscious about 10 minutes but is now awake and complains of pain in the neck, left forearm and right leg. EXAMINATION: Neurological examination by the paramedic is normal. QUESTIONS

  1. Concerning transportation to the hospital, you would advise (Select only one)

A. asking the patient to walk to the ambulance impossible

B. log rolling the patient onto a broad piece of plywood or firmly supported stretcher and sandbagging the head and neck

C. lifting the patient under the knees and armpits and transferring him to a stretcher

D. advising the patient to travel by private automobile to the hospital to avoid the expense of ambulance fees

A

(B)

All patients who complain of spine pain following trauma should be considered as having significant injury to the spinal column until proven otherwise. As such, transportation should be carried out with extreme caution. Under no circumstances should a patient who complains of spine pain following trauma be allowed to move about and ambulate prior to thorough examination. Log rolling of the patient onto a firm surface such as a broad piece of plywood, a wooden door or an appropriately supported spinal stretcher should be utilized for transportation of such patients. During transportation, the head and neck should be sandbagged or otherwise appropriately prevented from moving. Longitudinal traction applied by hand to the patient’s head may be of assistance during patient movement, but solidly securing the head and trunk to a firm surface is recommended. Under no circumstances should a patient with potential cervical spine injury be moved by lifting the patient under the knees and armpits. Such movements result in marked flexion and stress upon the entire spine and the possibility exists of converting a simple fracture or minor fracture-dislocation into a major dislocation with spinal cord injury. Patients with suspected spinal cord trauma should travel by stretcher with supervision by appropriately trained emergency personnel. Problems with hypotension, vomiting and respiratory difficulty may ensue in such patients and means for handling such emergencies should be available. If vomiting should occur during transportation, prevention of aspiration and respiratory distress takes precedence over maintaining body positioning. In such instances, it may be necessary to log roll a patient onto his side with proper vertebral support in order to drain or suction vomitus from the oral pharynx and mouth.

How well did you know this?
1
Not at all
2
3
4
5
Perfectly
17
Q

CASE 4: Neck Pain in a 37-Year-Old Man Following Cerebral Concussion HISTORY: A 37-year-old man was a passenger in an automobile involved in an accident. He was thrown clear of the automobile and rendered unconscious. A paramedic at the accident scene requests your advice, via two-way radio, concerning transportation of the patient. The paramedic reports that the patient was unconscious about 10 minutes but is now awake and complains of pain in the neck, left forearm and right leg. EXAMINATION: Neurological examination by the paramedic is normal. QUESTIONS

  1. Examination of the patient in the emergency room reveals restricted movements of the head and neck in all directions. Neurological examination is normal. Portable AP and lateral x-rays of the cervical spine are normal. Concerning portable x-rays for evaluation of cervical spine trauma, which of the following are true? (Select one or more)

A. Are reliable in excluding cervical spine injury if negative

B. Are useful in screening acutely injured patients for major injuries

C. Must extend from C1 to at least C7

D. Should be followed by a full series of cervical spine x-rays as soon as the patient’s condition permits

A

(B,C,D)

Portable cervical spine x-rays taken in the evaluation of potential spinal trauma should be considered as screening procedure. Reliance on such portable screening x-rays to evaluate cervical spine trauma will result in a significant number of missed spinal fractures. It is important that screening x-rays of the cervical spine extend from the atlanto-occipital junction to at least CV 3,7 AP and lateral x-rays are important. It is essential to follow screening cervical spine portable x-rays with a complete set of cervical spine x-rays including an anterior-posterior, lateral, open mouth and oblique views from the right and left sides. If such an examination fails to reveal pathology but the patient continues to complain of symptoms or if bothersome signs continue to be present, a variety of additional techniques may be required to adequately evaluate the cervical spine. These might include special views, particularly in the cervical dorsal junction region, traction on the arms to depress the shoulders, swimmer news or tomography. With proper supervision, flexion or extension views or cinefluoroscopy may be of help in identifying dislocations which are primarily due to ligamentous trauma an therefore not revealed on static film.

How well did you know this?
1
Not at all
2
3
4
5
Perfectly
18
Q

CASE 4: Neck Pain in a 37-Year-Old Man Following Cerebral Concussion HISTORY: A 37-year-old man was a passenger in an automobile involved in an accident. He was thrown clear of the automobile and rendered unconscious. A paramedic at the accident scene requests your advice, via two-way radio, concerning transportation of the patient. The paramedic reports that the patient was unconscious about 10 minutes but is now awake and complains of pain in the neck, left forearm and right leg. EXAMINATION: Neurological examination by the paramedic is normal. QUESTIONS

  1. Which of the following are true concerning the evaluation of potential cervical spine trauma in patients with head injury (Select only one)

A. All patients with significant head trauma should be evaluated radiologic ally for possible associated cervical spine trauma

B. Clinical examination is sufficient to exclude significant cervical spine trauma

C. In the absence of complaints, evaluation for cervical spine trauma is unrewarding

D. Significant cervical spine trauma is associated only with major head injuries with prolonged unconsciousness

A

(A)

The coexistence of significant head trauma with injuries of the cervical spine is well documented in the literatur. Shrago reported that 53% of a series of 50 patient with injuries of the upper cervical spine had evidence of concurrent head trauma. In patients with concomitant head injury and cervical spine injury, the incidence of injury is greatest in the upper cervical spine, particularly in the atlanto-occipital and C1 regions. In Shrago’s series, 56% had injuries involving the upper cervical spine and 34% had injuries at C5 and below. Only 10% had injuries involving the midcervical spine (C3, 4). The wide range of motion at the atlanto-axial and atlanto-occipital joints predisposes to certain traumatic forces. While failure to identify cervical spine trauma in association with head trauma may not always result in immediate spinal cord injury, the potential for delayed injury exists. The occurrence of delayed myelopathy following non-union of unsuspected atlanto-axial dislocations was described by Bachs, et al. Although clinical examination has been correctly stressed in the evaluation of cervical spine trauma, there is no substitute for an adequate radiological examination of the cervical spine to exclude significant spinal trauma. Complaints referrable to the cervical spine may be minimal, or patients may be unconscious or confused following head injury and unable to describe symptoms referrable to the cervical spine. It is essential that in any patient unconscious from head trauma, adequate x-rays of the cervical spine be obtained before manipulations of the head and neck take place. Although the index of suspicion may be high for cervical spine fractures in association with major head injuries, significant cervical spine trauma may also occur with-minor head injuries. Shrago described patients having only minor lacerations and contusions of the scalp or brief periods of unconsciousness with significant associated cervical spine trauma including fracture-dislocation. Thus, a high index of suspicion should be maintained in any patient with head trauma, even though minor, for the possibility of cervical spine injury.

How well did you know this?
1
Not at all
2
3
4
5
Perfectly
19
Q

CASE 4: Neck Pain in a 37-Year-Old Man Following Cerebral Concussion HISTORY: A 37-year-old man was a passenger in an automobile involved in an accident. He was thrown clear of the automobile and rendered unconscious. A paramedic at the accident scene requests your advice, via two-way radio, concerning transportation of the patient. The paramedic reports that thepatient was unconscious about 10 minutes but is now awake and complains of pain in the neck, left forearm and right leg. EXAMINATION: Neurological examination by the paramedic is normal. QUESTIONS

The patient in question was admitted to the hospital for observation. On the day following admission while sitting up, he complained of progressive numbness and weakness of the iower extremities which progressed to paraplegia over an hour. Examination revealed flaccid paraplegia except for slight toe movement and loss of all sensation below T3. Sphincter paralysis was also noted.

  1. Which of the following would be suspected as a potential cause of deterioration? (Select only one)

A. Spinal epidural hematoma

B. Spinal subdural hematoma

C. Acute disc herniation at C7-T1

D. Missed fracture-dislocation C7 –T1

E. All the above

A

(E)

All of the lesions described could be responsible for delayed progressive myelopathy following spine trauma. Rinaldi, et al described a patient with apparently normal cervical spine films who progressed to paraplegia 24 hours following injury. Repeat x-rays indicated that the patient had a fracture-dislocation at the C7-T1 interspace with locked facets. This fracture was not revealed on the original spine x-rays since they extended as low as C7 but did not include the C7-T1 junction. This case illustrates the need for careful radiological evaluation in patients with potential cervical spine trauma. Intraspinal hematomas, both epidural and subdural, may also be responsible for delayed spinal cord injury following spinal trauma and such hematomas are not necessarily associated with bony injury. Thus, radiologic examination of the spine may be truly normal and subsequent deterioration may occur through the formation of an intraspinal hematoma. Such situations are rare but should be kept in mind when delayed deterioration occurs. Acute traumatic ruptured discs with spinal cord compression also may occur following spinal trauma. Such intervertebral disc herniations may be responsible for spinal cord injury and neurological deterioration either in the immediate post injury period or on a delayed basis, weeks, months or even years following the injury. Patients who deteriorate following an initial injury require immediate vigorous radiologic evaluation including plain x-rays, at times tomography and very frequently myelography, for thorough evaluation of the underlying pathology. Recovery of function may be possible if neurologic deterioration is recognized and treated promptly. In the case illustrated by Rinaldi, et al. nearly complete neurological recovery ensued, although the patient was almost completely paraplegic with total sensory loss and sphincter paralysis prior to operation.

How well did you know this?
1
Not at all
2
3
4
5
Perfectly
20
Q

CASE 5: Spinal Shock Following Cervical Spinal Cord Injury HISTORY: A previously healthy 34-year-old woman was admitted to the emergency room following a motorcycle accident. She was alert and complained of neck pain. EXAMINATION: General physical examination was normal. Neurological examination revealed anesthesia, flaccid quadripiegia, areflexia and flaccid sphincter paralysis below C5. Blood pressure was 80/50, pulse 60, T 90.6 ° F . EKG showed sinus bradycardia. QUESTIONS

  1. The hypotension seen in this patient is most likely due to which one of the following? (Select only one)

A. Hypovolemic shock

B. Cardiogenic shock

C. Sympathectomy effect

D. Gram-negative septicemia

A

(C)

Although hypovolemic shock from associated injuries is possible, it is unlikely with a bradycardia of 60. Associated injuries should be sought bat volume replacement is unnecessary in the absence of evidence of hemorrhage. Cardiogenic shock is unlikely with a normal EKG. The possibility of cardiac contusion should be considered but is unlikely with the normal EKG. Transverse spinal cord lesions above C8, T1 produce a complete sympathectomy. This results in moderate hypotension and bradycardia. In the absence of associated injuries, this most likely accounts for hypotension and bradycardia noted in this patient. Although treatment is usually unnecessary, if desired the blood pressure may be elevated with vasopressors. Gram-negative sepsis may produce hypotension in patients with chronic spinal cord injury. This usually occurs secondary to urinary tract infection and secondary gram-negative septicemia. It is unlikely with acute spinal cord injury in a previously healthy individual.

How well did you know this?
1
Not at all
2
3
4
5
Perfectly
21
Q

CASE 5: Spinal Shock Following Cervical Spinal Cord Injury HISTORY: A previously healthy 34-year-old woman was admitted to the emergency room following a motorcycle accident. She was alert and complained of neck pain. EXAMINATION: General physical examination was normal. Neurological examination revealed anesthesia, flaccid quadripiegia, areflexia and flaccid sphincter paralysis below C5. Blood pressure was 80/50, pulse 60, T 90.6 ° F . EKG showed sinus bradycardia. QUESTIONS

  1. The hypothermia seen in this patient is most likely due to which one of the following? (Select only one)

A. Secondary to hypotension

B. Sympathectomy effect

C. Prolonged exposure to a low environmental temperature

D. Hypothalamic injury

A

(B)

Sympathectomy produced by cervical spinai cord injury leads to dilatation of skin blood vessels and anhydrosis. As a result, the normal mechanisms utilized to regulate body temperature are impaired. The patients show poikilothermia or a tendency for body temperature to seek the environmental temperature. Thus, under most ambient outdoor conditions, hypothermia occurs, as in this patient. Prolonged exposure to very low environmental temperatures may lead to hypothermia in normal individuals. There is no evidence that this patient was exposed to such conditions. Mild hypothermia may be associated with hypovolemic shock and marked hypotension. The hypotension seen in this patient is mild and is due to sympathectomy. Erratic fluctuations in body temperature may be seen with hypothalamic injury. This is usuaiiy seen with severe intracranial pathology such as head injury with coma, brain tumor, or intracranial hemorrhage. This patient was mentally alert with no evidence of intracranial pathology.

How well did you know this?
1
Not at all
2
3
4
5
Perfectly
22
Q

CASE 5: Spinal Shock Following Cervical Spinal Cord Injury HISTORY: A previously healthy 34-year-old woman was admitted to the emergency room following a motorcycle accident. She was alert and complained of neck pain. EXAMINATION: General physical examination was normal. Neurological examination revealed anesthesia, flaccid quadripiegia, areflexia and flaccid sphincter paralysis below C5. Blood pressure was 80/50, pulse 60, T 90.6 ° F . EKG showed sinus bradycardia. QUESTIONS

  1. The flaccid quadriplegia seen in this patient is due to which of the following? (Select only one)

A. Spinal shock

B. An upper motor neuron injury

C. A lower motor neuron injury

D. A combination of upper and lower motor neuron injury

E. None of the above

A

(A)

Spinal shock is defined as a total loss of spinal cord function below the level of an acute spinal cord lesion. It is seen not only with spinal cord injury but also with other acute spinal cord lesions such as infarction, hemorrhage, and infection. It accounts for the quadriplegia in this patient. Although upper motor neuron injury is present in this patient, it does not account for flaccid paralysis. Upper motor neuron lesions are associated with spastic paralysis and hyperreflexia. These clinical signs are masked by spinal shock. Lower motor neuron injury produces flaccid paralysis. In this patient, some of the lower motor neurons to the arms may have been injured. No injury has occurred to the lower motor neurons to the legs. The flaccid paralysis is due to spinal shock.

How well did you know this?
1
Not at all
2
3
4
5
Perfectly
23
Q

CASE 5: Spinal Shock Following Cervical Spinal Cord Injury HISTORY: A previously healthy 34-year-old woman was admitted to the emergency room following a motorcycle accident. She was alert and complained of neck pain. EXAMINATION: General physical examination was normal. Neurological examination revealed anesthesia, flaccid quadripiegia, areflexia and flaccid sphincter paralysis below C5. Blood pressure was 80/50, pulse 60, T 90.6 ° F . EKG showed sinus bradycardia. QUESTIONS

  1. Which of the following are true concerning spinal shock? (Select one or more)

A. Lasts for 4-6 weeks in humans

B. Refers to complete lack of spinal cord function below the level of an acute injury

C. Exact physiological mechanism unknown

D. None of the above

A

(A,B,C)

Spinal shock in humans usually lasts 4-6 weeks and subsides gradually. As it subsides the expected results of the upper motor neuron lesion in-the cervical spinal cord, namely spastic paralysis and hyperreflexia in the legs, gradually become apparent. The persistence or early return of very caudal reflexes such as the bulbocavernosus should not be taken as evidence that spinal shock has subsided. Increasing muscle tone and return of reflexes herald the end of the period of spinal shock.

How well did you know this?
1
Not at all
2
3
4
5
Perfectly
24
Q

CASE 5: Spinal Shock Following Cervical Spinal Cord Injury HISTORY: A previously healthy 34-year-old woman was admitted to the emergency room following a motorcycle accident. She was alert and complained of neck pain. EXAMINATION: General physical examination was normal. Neurological examination revealed anesthesia, flaccid quadripiegia, areflexia and flaccid sphincter paralysis below C5. Blood pressure was 80/50, pulse 60, T 90.6 ° F . EKG showed sinus bradycardia. QUESTIONS

  1. Which of the following have been proposed as mechanisms for the occurrence of spinal shock? (Select one or more)

A. Interruption of descending facilitation to spinal reflexes

B. Persistent inhibition below the level of injury

C. Axonal degeneration of inter neurons

D. Hyperpolarization of motor neurons

E. None of the above

A

(A,B,C,D)

McCough suggested three mechanisms involved in the production of spinal shock. The first is loss of reticulospinal and vestibulospinal. Fulton also implicated interruption of descending corticospinal pathways as a mechanism in spinal shock. Interruption of descending facilitatory influences probably represents the commonest theory for the mechanism of spinal shock. As a result of spinal transection there is reduced activity of gamma motor neurons, and increase in presynaptic inhibition locally. Thus, the alpha motor neurons are inhibited, the muscle is flaccid and the afferent input from stretching the muscle tendon is unable to excite the alpha motor neurons, with resulting areflexia. McCough also suggested that persisting inhibitory influences from below the level of injury could also affect spinal reflexes. Inhibitory influences in the lumbar spinal cord have been shown to depress reflexes in the thoracic and cervical regions in decerebrate rigidity and this effect can be abolished by retransection of the spinal cord in the thoracic region. McCough also suggested that degeneration of interneuronal axons might play a role in spinal shock. Barnes found hyperpolarization of the motor neuronal membrane resting potential in spinal shock and felt this was the major physiological derangement responsible for areflexia and. flaccidity.

How well did you know this?
1
Not at all
2
3
4
5
Perfectly
25
Q

CASE 5: Spinal Shock Following Cervical Spinal Cord Injury HISTORY: A previously healthy 34-year-old woman was admitted to the emergency room following a motorcycle accident. She was alert and complained of neck pain. EXAMINATION: General physical examination was normal. Neurological examination revealed anesthesia, flaccid quadripiegia, areflexia and flaccid sphincter paralysis below C5. Blood pressure was 80/50, pulse 60, T 90.6 ° F . EKG showed sinus bradycardia. QUESTIONS

  1. Which of the following clinical phenomena may also be seen with spina! shock due to cervical spinal cord injury? (Select one or more)

A. Nasal obstruction

B. Intestinal paralysis

C. Flaccid urinary bladder

D. None of the above

A

(A,B,C)

Loss of sympathetic control due to cervical spinal cord injury leads to loss of vasoconstriction of vessels in the head and face. Vasodilatation in the supine and particularly in the prone position leads to nasal mucosal congestion and swelling causing nasal obstruction with difficulty breathing and swallowing. Paralysis of intestinal peristalsis ileus occurs with spinal shock. This may be associated with intestinal and gastric distention. Bowel sounds are hypoactive or absent. Abdominal distention may add to respiratory insufficiency and lead to death in patients with compromised respiratory function following cervical spinal cord injury, Thus, nasogastric drainage and intravenous fluids with no oral intake may be required for several days until peristalsis resumes. Since the intestinal muscle has intrinsic contractile ability, recovery of peristalsis occurs much more quickly than the reflexes related to skeletal muscle. Flaccid paralysis of bladder function also occurs acutely. As spinal shock subsides, bladder muscle tone increases and a spastic neurogenic bladder develops.

How well did you know this?
1
Not at all
2
3
4
5
Perfectly
26
Q

CASE 6: Jefferson Fracture HISTORY: A 62-year-old male was involved in an automobile accident as a passenger. He was momentarily unconscious and was amnesic for the event. When questioned in the emergency room he complained of neck pain and numbness and weakness of all limbs. EXAMINATION: The patient was bright and alert but amnesic for the events of the accident, there was a bruise on his forehead, but otherwise, examination of his cranium and cranial nerves was normal. His neck was tender to palpation at the base of the skull. While there was sensory and motor deficit in all limbs, it was considerably and symmetrically worse in his upper extremities. Deep tendon reflexes were diminished throughout. X-RAY: Cervical spine x-rays showed a Jefferson fracture with out evidence of bony abnormality of other cervical elements (Figure 6. 1). QUESTIONS

  1. Lateral x-ray views of the cervical spine may show which of the following changes with a Jefferson fracture? (Select one or more)

A. A fracture of the posterior arch of the atlas

B. Displaced fragments of the anterior arch of the atlas

C. Soft tissue swelling in the prevertebral space

D. An increase in the distance between the anterior aspect of the dens and the posterior aspect of the anterior arch of the atlas

E. None of the above

A

(A,B,C,D)

Although all of the lateral cervical x-ray findings mentioned may be seen with Jefferson fracture, routine lateral views do not always reveal the full extent of injuries of the atlas. if the posterior arch is fractured on one side only, a slightly oblique lateral projection may show the fracture to better advantage.

How well did you know this?
1
Not at all
2
3
4
5
Perfectly
27
Q

CASE 6: Jefferson Fracture HISTORY: A 62-year-old male was involved in an automobile accident as a passenger. He was momentarily unconscious and was amnesic for the event. When questioned in the emergency room he complained of neck pain and numbness and weakness of all limbs. EXAMINATION: The patient was bright and alert but amnesic for the events of the accident, there was a bruise on his forehead, but otherwise, examination of his cranium and cranial nerves was normal. His neck was tender to palpation at the base of the skull. While there was sensory and motor deficit in all limbs, it was considerably and symmetrically worse in his upper extremities. Deep tendon reflexes were diminished throughout. X-RAY: Cervical spine x-rays showed a Jefferson fracture with out evidence of bony abnormality of other cervical elements (Figure 6. 1). QUESTIONS

  1. Open mouth anterior-posterior x-ray views of the cervical spine are likely to show which of the following changes associated with a Jefferson fracture? (Select one or more)

A. A fracture of the anterior arch of the atlas

B. A fracture of the posterior arch of the atlas

C. Outward displacement of the lateral masses of the atlas

D. A fracture of the lateral mass or transverse process of the atlas

E. None of the above

A

(A,C)

A lateral displacement of both lateral masses of the atlas with respect to the lateral borders of the body of the axis is highly suggestive of Jefferson fracture. A fracture through the anterior arch of the atlas can occasionally be seen, but fractures of the posterior arch are visually located just behind the lateral masses and are hidden by the lateral masses on AP views. Fractures of the lateral masses or transverse process of the atlas are extremely uncommon (see discussion for question 3) but do occasionally occur. As with the lateral view, the open mouth AP x-ray may not reveal injuries of the atlas.

How well did you know this?
1
Not at all
2
3
4
5
Perfectly
28
Q

CASE 6: Jefferson Fracture HISTORY: A 62-year-old male was involved in an automobile accident as a passenger. He was momentarily unconscious and was amnesic for the event. When questioned in the emergency room he complained of neck pain and numbness and weakness of all limbs. EXAMINATION: The patient was bright and alert but amnesic for the events of the accident, there was a bruise on his forehead, but otherwise, examination of his cranium and cranial nerves was normal. His neck was tender to palpation at the base of the skull. While there was sensory and motor deficit in all limbs, it was considerably and symmetrically worse in his upper extremities. Deep tendon reflexes were diminished throughout. X-RAY: Cervical spine x-rays showed a Jefferson fracture with out evidence of bony abnormality of other cervical elements (Figure 6. 1). QUESTIONS

  1. The likely mechanism of bony injury in a Jefferson fracture may include which of the following? (Select one or more)

A. Direct force applied to the atlas

B. Force applied to the skull and transmitted through the occipital condyles to the lateral masses of the atlas

C. Force applied to the lower spine and transmitted through the spinal axis to the lateral masses of the atlas

D. Hyperextension of the skull exerting a levering action on the posterior arch of the atlas

E. None of the above

A

(B,C,D)

Jefferson’s original account of the mechanisms of atlas fractures is still considered accurate. Direct trauma to the atlas is extremely rare, because of the well protected position of the atlas beneath the overhanging output and thick muscular barrier. Indirect forces are usually responsible for atlas fractures, the most common being the mechanism of transmitted forced either from above via the skull, often a blow to the vertex, or from below via the spine, often a fall on the buttocks. The atlas is subjected to forces from above and below which are divergent (Figure 6.2) and the net consequence is a lateral spread of the atlas and one or more fractures at weak points of the atlas ring (Figures 6.3 and 6.4). Fracture of the atlas can also occur indirectly by hyperextension of the head, crushing or cracking the posterior arch between the occiput and the axis. Fractures through the posterior arch commonly occur at the weakest point, namely the anteriorly placed grooves for the vertebral artery and suboccipital nerve. The lateral masses and transverse processes are not usually subject to these indirect forces, although occasionally if a patient’s head is tilted or turned at the time of injury forces may be more directly applied to these structures causing a crush injury of the lateral mass. *pict* *pict* *pict*

How well did you know this?
1
Not at all
2
3
4
5
Perfectly
29
Q

CASE 6: Jefferson Fracture HISTORY: A 62-year-old male was involved in an automobile accident as a passenger. He was momentarily unconscious and was amnesic for the event. When questioned in the emergency room he complained of neck pain and numbness and weakness of all limbs. EXAMINATION: The patient was bright and alert but amnesic for the events of the accident, there was a bruise on his forehead, but otherwise, examination of his cranium and cranial nerves was normal. His neck was tender to palpation at the base of the skull. While there was sensory and motor deficit in all limbs, it was considerably and symmetrically worse in his upper extremities. Deep tendon reflexes were diminished throughout. X-RAY: Cervical spine x-rays showed a Jefferson fracture with out evidence of bony abnormality of other cervical elements (Figure 6. 1). QUESTIONS

  1. Jefferson fractures (Select one or more)

A. represent a small percentage of the total fractures of the cervical spine

B. more than half the time are associated with other fractures of the upper cervical spine

C. more than half the time are associated with spinal cord injury

D. are associated with approximately 10% mortality

E. none of the above

A

(A,D)

Atlas fractures comprise only 2 to 3 o of fractures of the cervical spine. Fractures at lower cervical levels are much more common. Approximately 30% of fractures of the atlas are associated with fractures at other cervical levels, most notably at C2. The space within the ring of the atlas is approximately 1/3 odontoid, 1/3 spinal cord and 1/3 CSF and soft tissues and, therefore, spinal injury is thought to be uncommon, with isolated Jefferson fractures. In Jefferson’s original reports 19 of 46 patients with fractures of the atlas had spinal injury, but in most of these there was bony injury at other cervical levels. Hinchey and Bickel reviewed 112 fractures of the atlas with a mortality of 10%. However, in this report, as in Jefferson’s, more than half of the patients manifested cervical fractures at other levels. Fractures at multiple levels increase the hazard of morbidity and mortality. In addition, some patients with Jefferson fractures probably sustain immediately fatal injuries and are not included in statistics.

How well did you know this?
1
Not at all
2
3
4
5
Perfectly
30
Q

CASE 6: Jefferson Fracture HISTORY: A 62-year-old male was involved in an automobile accident as a passenger. He was momentarily unconscious and was amnesic for the event. When questioned in the emergency room he complained of neck pain and numbness and weakness of all limbs. EXAMINATION: The patient was bright and alert but amnesic for the events of the accident, there was a bruise on his forehead, but otherwise, examination of his cranium and cranial nerves was normal. His neck was tender to palpation at the base of the skull. While there was sensory and motor deficit in all limbs, it was considerably and symmetrically worse in his upper extremities. Deep tendon reflexes were diminished throughout. X-RAY: Cervical spine x-rays showed a Jefferson fracture with out evidence of bony abnormality of other cervical elements (Figure 6. 1). QUESTIONS

  1. The mechanism of neural injury in this patient might be explained on which of the following bases? (Select one or more)

A. “Cruciate paralysis’

B. Hemiplegia cruciata (crossed hemiplegia)

C. Central cord syndrome

D. Anterior spinal artery syndrome

E. None of the above

A

(A,C)

“Cruciate paralysis” was described by Bell and ascribed weakness of the arms greater than the legs to acute lower medullary compression. In this region the pyramidal decussation of motor fibers to the arms are more cephalad and more superficial than fibers to the legs and, therefore, in some cases, more vulnerable to upper cervical fractures. “Hemiplegia cruciata” also represents a lesion in the pyramidal decussation but results in ipsilateral paresis of the arm and contralateral paresis of the leg. Disproportionately more motor impairment in the upper than the lower extremities is the hallmark of the central spinal cord syndromic. The anterior spinal artery syndrome is characterized by complete paralysis below the level of the lesion with sparing of dorsal column sensation.

How well did you know this?
1
Not at all
2
3
4
5
Perfectly
31
Q

CASE 6: Jefferson Fracture HISTORY: A 62-year-old male was involved in an automobile accident as a passenger. He was momentarily unconscious and was amnesic for the event. When questioned in the emergency room he complained of neck pain and numbness and weakness of all limbs. EXAMINATION: The patient was bright and alert but amnesic for the events of the accident, there was a bruise on his forehead, but otherwise, examination of his cranium and cranial nerves was normal. His neck was tender to palpation at the base of the skull. While there was sensory and motor deficit in all limbs, it was considerably and symmetrically worse in his upper extremities. Deep tendon reflexes were diminished throughout. X-RAY: Cervical spine x-rays showed a Jefferson fracture with out evidence of bony abnormality of other cervical elements (Figure 6. 1). QUESTIONS

  1. Further details of the patient’s bony injury were seen on tomograms. There were fractures through the anterior and posterior arch of the atlas. There was wide lateral offset of both lateral processes of the atlas. There was no fracture of the odontoid and the relationship between the odontoid and the anterior arch of the atlas was normal (i.e. less than 2. 5 mm). An acceptable plan of therapeutic management for this patient’s Jefferson fracture might include (Select one or more)

A. this is a stable fracture and no treatment is indicated

B. this is a potentially unstable fracture and the patient should be treated with a soft collar

C . this is a potentially unstable fracture and the patient should be treated with skull traction or a halo frame

D. this is an unstable fracture and early operative fusion is indicated

E. none of the above

A

(C)

While there is no absolute uniformity of opinion regarding management for patients with Jefferson fractures, most authors agree that nondisplaced fractures can be treated with simple neck support (i.e. cervical collar). Atlantoaxial instability is probable following Jefferson fractures if the transverse ligament is ruptured. If lateral displacement of the lateral masses is greater than 6.9 mm or if the distance from the back of the anterior arch of the atlas to the front of the odontoid is greater than 2. 5 mm, the transverse ligament is probably torn and instability is likely. Because of the hazard of further neural injury in potentially unstable Jefferson fractures, cervical traction utilizing either skull calipers or the halo brace, is recommended. If instability remains after a suitable period of immobilization, operative fusion is recommended.

How well did you know this?
1
Not at all
2
3
4
5
Perfectly
32
Q

CASE 7: Odontoid Fracture HISTORY: A 62-year-old male fell down a flight of stairs. He was a known alcoholic and was apparently intoxicated at the time of his fall. He struck his head during the fall and was noted by an observer to be unconscious immediately. On arriving in the emergency a few moments later, he was awake but somewhat confused and disoriented. Within twenty minutes, he was oriented but amnesic for the events of the fall. He complained of neck pain. EXAMINATION: The only visible evidence of injury was a bruise on the left parietal eminence of his skull. Vital signs were stable and general physical examination was normal. Aside from his mental status, his neurological examination was normal. His neck was splinted with sandbags and he was sent for x-rays. X-RAY: Skull - normal. Cervical spine - the Lateral view showed a fracture of the odontoid process at its base with a 10 mm posterior subluxation of the dens on the body of C2 (Figure 7. 1). More detailed AP views and tomograms did not significantly add to the plain lateral x-ray findings. QUESTIONS

  1. Unlike this patient, most patients with odontoid fractures die suddenly as a direct result of the cervical spinal cord injury. (True or False)
A

FALSE

Accurate mortality figures are not known because a number of patients with odontoid fracture presumably expire acutely and are not included in statistics. A high cervical spinal cord lesion due to odontoid fracture would result in respiratory arrest with rapid demise. However, reported mortality figures for odontoid fractures have ranged from 50% to less than 10%

How well did you know this?
1
Not at all
2
3
4
5
Perfectly
33
Q

CASE 7: Odontoid Fracture HISTORY: A 62-year-old male fell down a flight of stairs. He was a known alcoholic and was apparently intoxicated at the time of his fall. He struck his head during the fall and was noted by an observer to be unconscious immediately. On arriving in the emergency a few moments later, he was awake but somewhat confused and disoriented. Within twenty minutes, he was oriented but amnesic for the events of the fall. He complained of neck pain. EXAMINATION: The only visible evidence of injury was a bruise on the left parietal eminence of his skull. Vital signs were stable and general physical examination was normal. Aside from his mental status, his neurological examination was normal. His neck was splinted with sandbags and he was sent for x-rays. X-RAY: Skull - normal. Cervical spine - the Lateral view showed a fracture of the odontoid process at its base with a 10 mm posterior subluxation of the dens on the body of C2 (Figure 7. 1). More detailed AP views and tomograms did not significantly add to the plain lateral x-ray findings. QUESTIONS

  1. This patient demonstrates the difficulty in diagnosing odontoid fractures by history and physical examination alone. (True or False)
A

TRUE

Neck pain is the most common symptom associated with odontoid fracture. Paravertebral tenderness, muscle spasm and limitation of neck movement are commonly associated signs if pain is present. However, these findings are not pathognomonic of odontoid fracture. Almost invariably the diagnosis of fractured odontoid is made radiographically.

How well did you know this?
1
Not at all
2
3
4
5
Perfectly
34
Q

CASE 7: Odontoid Fracture HISTORY: A 62-year-old male fell down a flight of stairs. He was a known alcoholic and was apparently intoxicated at the time of his fall. He struck his head during the fall and was noted by an observer to be unconscious immediately. On arriving in the emergency a few moments later, he was awake but somewhat confused and disoriented. Within twenty minutes, he was oriented but amnesic for the events of the fall. He complained of neck pain. EXAMINATION: The only visible evidence of injury was a bruise on the left parietal eminence of his skull. Vital signs were stable and general physical examination was normal. Aside from his mental status, his neurological examination was normal. His neck was splinted with sandbags and he was sent for x-rays. X-RAY: Skull - normal. Cervical spine - the Lateral view showed a fracture of the odontoid process at its base with a 10 mm posterior subluxation of the dens on the body of C2 (Figure 7. 1). More detailed AP views and tomograms did not significantly add to the plain lateral x-ray findings. QUESTIONS

  1. As with this patient, fracture-dislocations of the odontoid in survivors are usually not associated with neurological signs of spinal cord injury. (True or False)
A

TRUE

In Schatzker’s, et al. review only 9% of patients showed evidence of spinal cord injury. Apuzzo, et al. indicated a slightly higher percentage: 18% of their patients had evidence of myelopathy. The sagittal diameter of the normal adult spinal canal ranges from 16 to 33 mm at the odontoid level. Spinal compression is not usually clinically detectable until this measurement is radiographically less than 14 mm. There is, therefore, for most patients with odontoid fracture, ample room in the spinal canal to accept encroachment associated with fracture dislocations without producing clinical evidence of spinal cord dysfunction.

How well did you know this?
1
Not at all
2
3
4
5
Perfectly
35
Q

CASE 7: Odontoid Fracture HISTORY: A 62-year-old male fell down a flight of stairs. He was a known alcoholic and was apparently intoxicated at the time of his fall. He struck his head during the fall and was noted by an observer to be unconscious immediately. On arriving in the emergency a few moments later, he was awake but somewhat confused and disoriented. Within twenty minutes, he was oriented but amnesic for the events of the fall. He complained of neck pain. EXAMINATION: The only visible evidence of injury was a bruise on the left parietal eminence of his skull. Vital signs were stable and general physical examination was normal. Aside from his mental status, his neurological examination was normal. His neck was splinted with sandbags and he was sent for x-rays. X-RAY: Skull - normal. Cervical spine - the Lateral view showed a fracture of the odontoid process at its base with a 10 mm posterior subluxation of the dens on the body of C2 (Figure 7. 1). More detailed AP views and tomograms did not significantly add to the plain lateral x-ray findings. QUESTIONS

  1. Posterior dislocation of the fractured odontoid, as with this patient, is more common than anterior dislocation. (True or False)
A

FALSE

Anterior odontoid dislocations are more common than posterior dislocations by a factor of 2:1 to 10:1.

How well did you know this?
1
Not at all
2
3
4
5
Perfectly
36
Q

CASE 7: Odontoid Fracture HISTORY: A 62-year-old male fell down a flight of stairs. He was a known alcoholic and was apparently intoxicated at the time of his fall. He struck his head during the fall and was noted by an observer to be unconscious immediately. On arriving in the emergency a few moments later, he was awake but somewhat confused and disoriented. Within twenty minutes, he was oriented but amnesic for the events of the fall. He complained of neck pain. EXAMINATION: The only visible evidence of injury was a bruise on the left parietal eminence of his skull. Vital signs were stable and general physical examination was normal. Aside from his mental status, his neurological examination was normal. His neck was splinted with sandbags and he was sent for x-rays. X-RAY: Skull - normal. Cervical spine - the Lateral view showed a fracture of the odontoid process at its base with a 10 mm posterior subluxation of the dens on the body of C2 (Figure 7. 1). More detailed AP views and tomograms did not significantly add to the plain lateral x-ray findings. QUESTIONS

  1. A fracture at the base of the odontoid, a? seen with this patient, is an unusual type of axis fracture. (True or False)
A

FALSE

Anderson and D’Alonzo described three types of axis fracture. The most common (over 50%) is a fracture at the junction of the odontoid process with the body of the atlas. Less common are fractures through the body of the atlas. Least common are fractures, usually oblique, through the rostral portion of the odontoid process. Each type of fracture may be displaced or nondisplaced.

How well did you know this?
1
Not at all
2
3
4
5
Perfectly
37
Q

CASE 7: Odontoid Fracture HISTORY: A 62-year-old male fell down a flight of stairs. He was a known alcoholic and was apparently intoxicated at the time of his fall. He struck his head during the fall and was noted by an observer to be unconscious immediately. On arriving in the emergency a few moments later, he was awake but somewhat confused and disoriented. Within twenty minutes, he was oriented but amnesic for the events of the fall. He complained of neck pain. EXAMINATION: The only visible evidence of injury was a bruise on the left parietal eminence of his skull. Vital signs were stable and general physical examination was normal. Aside from his mental status, his neurological examination was normal. His neck was splinted with sandbags and he was sent for x-rays. X-RAY: Skull - normal. Cervical spine - the Lateral view showed a fracture of the odontoid process at its base with a 10 mm posterior subluxation of the dens on the body of C2 (Figure 7. 1). More detailed AP views and tomograms did not significantly add to the plain lateral x-ray findings. QUESTIONS

  1. Younger patients with nondisplaced odontoid fractures have a favorable outlook for stable union (healing) with conservative (nonoperative) management. (True or False)
A

TRUE

Apuzzo, et al. assessed the outcome of conservative management and found nonunion in only 16% of all patients with nondisplaced odontoid fractures, whereas nonunion occurred in 60% of all patients with displaced odontoid fractures. Age also seems to effect the probability for stable fusion of an odontoid fracture with conservative measurement. Seventeen percent versus 53% failed to fuse for patients under 40 years of age versus patients over 40 years of age, respectively. Compromise of blood supply to the fractured and displaced odontoid process may adversciv affect the healing process. The atherosclerotic process may play an analogous role in older patients

How well did you know this?
1
Not at all
2
3
4
5
Perfectly
38
Q

CASE 7: Odontoid Fracture HISTORY: A 62-year-old male fell down a flight of stairs. He was a known alcoholic and was apparently intoxicated at the time of his fall. He struck his head during the fall and was noted by an observer to be unconscious immediately. On arriving in the emergency a few moments later, he was awake but somewhat confused and disoriented. Within twenty minutes, he was oriented but amnesic for the events of the fall. He complained of neck pain. EXAMINATION: The only visible evidence of injury was a bruise on the left parietal eminence of his skull. Vital signs were stable and general physical examination was normal. Aside from his mental status, his neurological examination was normal. His neck was splinted with sandbags and he was sent for x-rays. X-RAY: Skull - normal. Cervical spine - the Lateral view showed a fracture of the odontoid process at its base with a 10 mm posterior subluxation of the dens on the body of C2 (Figure 7. 1). More detailed AP views and tomograms did not significantly add to the plain lateral x-ray findings. QUESTIONS

  1. Fusion (union) with conservative (nonoperative) measures would be expected to take about eight months in the patient described (True or False)
A

FALSE

Union, if it occurs, usually does so by the tenth week.

How well did you know this?
1
Not at all
2
3
4
5
Perfectly
39
Q

CASE 7: Odontoid Fracture HISTORY: A 62-year-old male fell down a flight of stairs. He was a known alcoholic and was apparently intoxicated at the time of his fall. He struck his head during the fall and was noted by an observer to be unconscious immediately. On arriving in the emergency a few moments later, he was awake but somewhat confused and disoriented. Within twenty minutes, he was oriented but amnesic for the events of the fall. He complained of neck pain. EXAMINATION: The only visible evidence of injury was a bruise on the left parietal eminence of his skull. Vital signs were stable and general physical examination was normal. Aside from his mental status, his neurological examination was normal. His neck was splinted with sandbags and he was sent for x-rays. X-RAY: Skull - normal. Cervical spine - the Lateral view showed a fracture of the odontoid process at its base with a 10 mm posterior subluxation of the dens on the body of C2 (Figure 7. 1). More detailed AP views and tomograms did not significantly add to the plain lateral x-ray findings. QUESTIONS

  1. After three months of conservative (nonoperative) managementnon union of the odontoid might be demonstrated radiographically by the following criteria? (Select one or more)

A. A defect in the odontoid with contiguous sclerosis of both fragments

B. A defect in the odontoid with contiguous resorption of both fragments

C. A defect in the odontoid with loss of cortical contiguity

D. Demonstrable movement of the dens fragments on flexion-extension x-rays

E. None of the above

A

(A,B,C,D)

These are the four criteria used to radiographically diagnose nonunion of an odentoid fracture

How well did you know this?
1
Not at all
2
3
4
5
Perfectly
40
Q

CASE 7: Odontoid Fracture HISTORY: A 62-year-old male fell down a flight of stairs. He was a known alcoholic and was apparently intoxicated at the time of his fall. He struck his head during the fall and was noted by an observer to be unconscious immediately. On arriving in the emergency a few moments later, he was awake but somewhat confused and disoriented. Within twenty minutes, he was oriented but amnesic for the events of the fall. He complained of neck pain. EXAMINATION: The only visible evidence of injury was a bruise on the left parietal eminence of his skull. Vital signs were stable and general physical examination was normal. Aside from his mental status, his neurological examination was normal. His neck was splinted with sandbags and he was sent for x-rays. X-RAY: Skull - normal. Cervical spine - the Lateral view showed a fracture of the odontoid process at its base with a 10 mm posterior subluxation of the dens on the body of C2 (Figure 7. 1). More detailed AP views and tomograms did not significantly add to the plain lateral x-ray findings. QUESTIONS

  1. As a plan of management for this patient one might consider (Select one or more)

A. conservative nonoperative) management using a soft cervical collar.

B. initial stabilization and reduction with skull traction followed by external bracing of the neck. Radiological assessment for fusion at three months

C. initial stabilization and reduction with skull traction followed in several days by operative fusion

D. immediate operative reduction and fusion

E. none of the above

A

(B,C)

Stable union in nondisplaced fractures can be expected in greater than 30% of all patients utilizing conservative therapy. However, a cervical splinting device (e.g. braces, Minerva plaster jackets or Halo apparatus) rather than a soft cervical collar is recommended to insure cervical stability. Displaced fractures require initial reduction in all cases. This is usually effected by skull tong traction rather than surgically. There is disagreement as to the most suitable subsequent measure to achieve stable fusion of the reduced displaced fracture. Nonfusion occurs in 30-60% of conservatively treated members of this group. Conservative management, using any one of the splinting devices, followed by radiological assessment, or early operative fusion are both acceptable therapeutic alternatives, for patients with reduced odontoid fractures.

How well did you know this?
1
Not at all
2
3
4
5
Perfectly
41
Q

CASE 8: Os Odontoideum HISTORY: A 17-year-old high school football player was brought to the emergency room complaining of weakness and numbness of the arms and legs. He apparently sustained a flexion injury of his neck 45’ minutes earlier while being tackled but this appeared to be of relatively minor forces; He reported immediate complete paralysis below the neck but significant return of movement subsequently. He did not complain of neck pain. EXAMINATION: All vital signs were within normal limits. Motor examination revealed minimal weakness of the arms and legs. Muscle tone was slightly increased but no pathological reflexes were present. Pinprick sensation was normal but there was slight decrease in joint position sensation in the hands and feet. Sphincter function was normal. There was no tenderness to palpation of the cervical spine. QUESTIONS

  1. Which of the following are part of an appropriate plan of action for this patient? (Select one or more)

A. Passively assess flexion and extension of the cervical spine

B. Immobilize the head and neck

C. Obtain cervical spine x-rays

D. Hospitalize the patient for observation

E. Schedule emergency myelogram

A

(B,C,D)

Immediate immobilization is mandatory in any case of suspected spine or spinal cord injury. 30 The history of injury while playing football is sufficient to indicate a need for immobilization of the head and neck prior to accurate diagnosis. Passive or active head and neck movements must be absolutely avoided until a radiological diagnosis is made. Plain cervical spine x-rays are the cornerstone of radiological diagnosis. Appropriate action may be dictated by findings on these x-rays alone. If plain x-rays are normal or fail to define pathology accurately, polytomography is necessary. The history of temporary paralysis indicates significant spinal cord malfunction. The rapid and marked improvement indicates that the malfunction may be a functional disturbance of spinal cord conduction rather than anatomical disruption of spinal pathways. In spite of rapid recovery, the persistence of increased muscle tone and mild weakness are evidence of possible persistent anatomical spinal cord injury. As such, hospitalization is indicated to observe for further neurological changes or complications of spinal injury such as spinal hematoma. Because of rapid improvement in neurological function, myelography is not indicated on an acute basis.

How well did you know this?
1
Not at all
2
3
4
5
Perfectly
42
Q

CASE 8: Os Odontoideum HISTORY: A 17-year-old high school football player was brought to the emergency room complaining of weakness and numbness of the arms and legs. He apparently sustained a flexion injury of his neck 45’ minutes earlier while being tackled but this appeared to be of relatively minor forces; He reported immediate complete paralysis below the neck but significant return of movement subsequently. He did not complain of neck pain. EXAMINATION: All vital signs were within normal limits. Motor examination revealed minimal weakness of the arms and legs. Muscle tone was slightly increased but no pathological reflexes were present. Pinprick sensation was normal but there was slight decrease in joint position sensation in the hands and feet. Sphincter function was normal. There was no tenderness to palpation of the cervical spine. QUESTIONS

  1. Anteroposterior and lateral polytomographic x-rays of the cervical spine are illustrated in Figures 8.1 and 8.2. They show (Select one or more)

A. normal anatomy

B. fracture of the odontoid process

C. an old unhealed odontoid fracture

D. a congenital nonunion of the odontoid process

E. atlanto-axial dislocation

A

(D, E)

Congenital nonunion of the odontoid process or so called “os odontoideum” represents a failure of the odontoid process, embryologically the body of the C1 vertebrae, to unite with the axis (C2). Wollin described criteria for identification of the os odontoideum and its differentiation from odontoid fracture. The radiological appearance in the patient presented is typical of the os odontoideum (Figures 8.1 and 8. 2). There is a wide separation of the “odontoid” from the main mass of C2, the fragment has a smooth rounded surface and is circular in shape. In odontoid fracture (see Case 7), an irregular surface usually marks the site of fracture which most commonly occurs at the odontoid base resulting in a relatively rectangular rather than rounded fragment. In addition to the presence of the os odontoideum, significant atlanto-axial dislocation is present (Figure 8. 1). The atlas and attached cranium are displaced posteriorly in regard to their usual relationship to the axis and the remainder of the spine. The anterior atlanto-axial and occipito-axiai ligaments attach to the odontoid process. When the odontoid is firmly attached to the main mass of the axis, dislocation is prevented. When the odontoid is separated from the axis either congenitally or secondary to fracture, the ligaments are ineffective in preventing atlanto-axial dislocation. Early reports indicated that up to 23% of individuals ages 30-50 had incomplete union of the odontoid process to C2. Yashon, however, feels that the true incidence, based on modern polytomographic studies, is much lower. The pathological, clinical and radiological features of congenital failure of fusion of the odontoid have been reviewed in a number of reports.

How well did you know this?
1
Not at all
2
3
4
5
Perfectly
43
Q

CASE 8: Os Odontoideum HISTORY: A 17-year-old high school football player was brought to the emergency room complaining of weakness and numbness of the arms and legs. He apparently sustained a flexion injury of his neck 45’ minutes earlier while being tackled but this appeared to be of relatively minor forces; He reported immediate complete paralysis below the neck but significant return of movement subsequently. He did not complain of neck pain. EXAMINATION: All vital signs were within normal limits. Motor examination revealed minimal weakness of the arms and legs. Muscle tone was slightly increased but no pathological reflexes were present. Pinprick sensation was normal but there was slight decrease in joint position sensation in the hands and feet. Sphincter function was normal. There was no tenderness to palpation of the cervical spine. QUESTIONS

  1. The transient quadriplegia and subsequent mild sensory and motor abnormalities may be due to (Selectone or more)

A. spinal shock

B. the central spinal cord syndrome

C. temporary spinal cord compression

D. the anterior spinal cord syndrome

E. vascular insufficiency

A

(C,E)

With posterior atlanto-axial dislocation, the anterior arch of the atlas and odontoid fragment move backward and may compress the spinal cord anteriorly. The spinal cord is thus pinched between the posterior arch of the axis (C2) and the odontoid fragment. Anterior atlanto-axial dislocation may injuries of this type may be fatal. Spinal cord injury at this level may result in complete respiratory paralysis since descending input to both phrenic and intercostal respiratory motor neurons is interrupted. Because of the relatively vide dimensions of the spinal canal at this point, atlanto-axial dislocation may occur with little or no neurological deficit. Persistent facet dislocation is not as common in nonfatal upper cervical spine injuries and, thus, rapid dislocation and reduction may occur with only transient spinal cord compression. In lower cervical spine fractures, facet locking may prevents grossly dislocated spine from spontaneously reducing, thus allowing persistent spinal cord compression. This difference in facet dislocations is likely accounted for by the difference in orientation and overlap of facets in the upper, compared to the lower cervical spine. In the upper cervical spine, especially at the C1 - 2 level, the facets are horizontal with only a slight depression in the C2 superior facet to accommodate the C1 inferior facet. Thus, facet dislocation may occur and reduce spontaneously. In the lower cervical spine, marked facet angulation and overlap occur. Thus, when dislocation occurs, persistent locking of adjacent facets occurs with little chance of spontaneous reduction and significant difficulty in obtaining reduction with traction. Schneider and others emphasize the importance of vascular compression in fracture dislocations in the upper cervical region. These occur most commonly with extension injuries. The vertebral arteries emerge from the foramina transversaria of C2 and turn laterally and superiorly, pass through the foramina of the atlas and run in a groove along the superior aspect of the atlas before penetrating the posterior atiantooccipital membrane and entering the posterior cranial fossa. The arteries are fixed at the point of entry into the posterior fossa and may be stretched, compressed or even ruptured by acute dislocations at the atlanto-axial or atlanto-occipitai junctions. This may result in a few minutes of transient neurological deficit, delayed neurological deficit or even death. Spinal shock implies complete loss of spinal cord function below a particular spinal level and is not present in this patient. The central spinal cord syndrome (see Case 10) is characterized by upper extremity weakness disproportionately severe in relation to lower extremity weakness. This patient presents with a mild diffuse weakness of all extremities. The anterior spinal cord syndrome (see Case 11) is characterized by loss of motor function and preservation of dorsal column function. The mild loss of position sense and vibratory perception in this patient are evidence of dorsal column injury and exclude the anterior spinal cord syndrome.

How well did you know this?
1
Not at all
2
3
4
5
Perfectly
44
Q

CASE 8: Os Odontoideum HISTORY: A 17-year-old high school football player was brought to the emergency room complaining of weakness and numbness of the arms and legs. He apparently sustained a flexion injury of his neck 45’ minutes earlier while being tackled but this appeared to be of relatively minor forces; He reported immediate complete paralysis below the neck but significant return of movement subsequently. He did not complain of neck pain. EXAMINATION: All vital signs were within normal limits. Motor examination revealed minimal weakness of the arms and legs. Muscle tone was slightly increased but no pathological reflexes were present. Pinprick sensation was normal but there was slight decrease in joint position sensation in the hands and feet. Sphincter function was normal. There was no tenderness to palpation of the cervical spine. QUESTIONS

  1. Appropriate immediate treatment for this patient might include (Select one or more)

A. corticosteroids systemically

B. skeletal traction

C. decompressive laminectomy

D. anterior resection of the odontoid process

E. placement of a cervical collar and discharge

A

(A,B)

The role of corticosteroids in the treatment of spinal cord injury is unproven. Nevertheless, there is sufficient suggestion from laboratory research studies that strong recommendations for its clinical use have been made. The patient presented has persistent evidence of mild spinal cord injury as judged by quadriparesis and dorsal column sensory changes and the utilization of all available forms of therapy seem justified in an attempt to treat this problem. Skeletal traction produces excellent reduction in atlanto-axial dislocation due to os odontoideum. Since impacted fracture fragments are not present and since locked facet dislocations at the atlanto-axial region are rare, there is little mechanical resistance to postural reduction by skeletal traction. This situation, is unstable, however, and reduction cannot be maintained with a cervical collar alone. Repeat mild trauma could result in recurrent dislocation with potential neurological catastrophe including quadriplegia or even death due to respiratory arrest. Decompressive laminectomy plays no significant role in the treatment of atlanto-axial dislocation due to os odontoideum. Any residual compression of the spinal cord by the posterior arch of C1 can be relieved most promptly by skeletal traction and reduction of dislocation. Removal of the odontoid anteriorly is unnecessary. This small fragment plays little if any direct role in spinal cord injury if correct alignment is maintained.

How well did you know this?
1
Not at all
2
3
4
5
Perfectly
45
Q

CASE 8: Os Odontoideum HISTORY: A 17-year-old high school football player was brought to the emergency room complaining of weakness and numbness of the arms and legs. He apparently sustained a flexion injury of his neck 45’ minutes earlier while being tackled but this appeared to be of relatively minor forces; He reported immediate complete paralysis below the neck but significant return of movement subsequently. He did not complain of neck pain. EXAMINATION: All vital signs were within normal limits. Motor examination revealed minimal weakness of the arms and legs. Muscle tone was slightly increased but no pathological reflexes were present. Pinprick sensation was normal but there was slight decrease in joint position sensation in the hands and feet. Sphincter function was normal. There was no tenderness to palpation of the cervical spine. QUESTIONS

  1. Appropriate long term treatment for this patient is (Select only one)

A. prolonged immobilization in a halo brace

B. prolonged immobilization with skeletal traction on a spinal frame

C. no long term treatment is required

D. cervical fusion of C1, C2 and C3

E. cervical fusion of C1, C2, C3 and the occiput

A

(D)

It is highly unlikely that healing of such, a congenital defect in fusion will occur with prolonged immobilization alone. Such is not the case with odontoid fractures where the fresh fracture and otherwise normal anatomy lend themselves to healing with immobilization. Thus, the distinction between a congenital and a traumatic lesion is important in regard to treatment. Funk has pointed cut that the initial presentation of congenital instability of the spine may be falsely attributed to an athletic accident and misdiagnosed and mistreated. The situation may be somewhat different in infants and young children where prolonged immobilization may be effective in promoting odontoid fusion when final ossification has not occurred. Failure to institute treatment of any kind would subject the patient to persistent risk of spinal cord injury or even death in the event of minor trauma or even with excessive nontraumatic neck flexion. The most appropriate form of long term treatment is fusion of C1, C2 and C3 after reduction has been achieved by skeletal traction. Originally fusion to the occiput was also recommended for greater stability. This is unnecessary in os odontoideum since excellent stability is obtained by fusion of C1, C2 and C3 alone. In addition, fusion to the occiput imposes a severe disability on the patient, namely total restriction of lateral rotation and flexion and extension at the atlanto-occipital and atlanto-axial joints. These joints provide the major contributions to head rotation and contribute significantly also to flexion and extension of the head on the spine. Fusion of C1 to C2 alone omitting C3 may be effective but fusion to C3 assures that a fully stable vertebral segment unaffected in any way by the pathological process is included in the fusion to assure excellent and prolonged stabilization. In addition, the inclusion of C3 in the fusion has little if any additional effect in reducing overall neck mobility,

How well did you know this?
1
Not at all
2
3
4
5
Perfectly
46
Q

CASE 8: Os Odontoideum HISTORY: A 17-year-old high school football player was brought to the emergency room complaining of weakness and numbness of the arms and legs. He apparently sustained a flexion injury of his neck 45’ minutes earlier while being tackled but this appeared to be of relatively minor forces; He reported immediate complete paralysis below the neck but significant return of movement subsequently. He did not complain of neck pain. EXAMINATION: All vital signs were within normal limits. Motor examination revealed minimal weakness of the arms and legs. Muscle tone was slightly increased but no pathological reflexes were present. Pinprick sensation was normal but there was slight decrease in joint position sensation in the hands and feet. Sphincter function was normal. There was no tenderness to palpation of the cervical spine. QUESTIONS

  1. Which of the following pathological processes have been associated with spontaneous atlanto-axial dislocation? (Select one or more)

A. Nasopharyngeal infection

B. Rheumatoid arthritis

C. Spinal tumor

D. Ankylosing spondylitis

E. None of the above

A

(A,B,C,D)

Bell in 1830 first reported spontaneous atlanto-axial dislocation in a patient with a pharyngeal ulcer. Since then, numerous reports of atlanto-axial dislocation have been presented, mainly in relation to inflammatory conditions either in soft tissues surrounding the upper cervical spine or in relation to inflammatory processes or tumors affecting ligaments, joints and bones primarily. Nasopharyngeal infections, particularly in children, may be associated with neck pain and torticollis. Cervical spine x-rays should be carried out in such instances as atlanto-axial dislocation may be identified. Since excessive mobility is present in the upper cervical spines of children, care must be exercised in interpretation of cervical spine x-rays in children. Flexion and extension views with careful supervision may be helpful in ascertaining whether or not true dislocation is present. The mechanism of atlanto-axial dislocation in such inflammatory conditions is undefined. Ligamentous laxity, joint effusions, hyperemic decalcification with loosening of ligaments and irritative contracture of cervical muscles causing secondary dislocation have all been suggested as mechanisms. In rheumatoid arthritis and ankylosing spondylitis the inflammatory process affects the ligaments and joints directly resulting in laxity and dislocation. Local tumors such as eosinophilic granuloma may produce local bone and ligament destruction and resultant dislocation. Tuberculosis, syphilis, poliomyelitis and Down’s Syndrome have all been reported as etiological factors in spontaneous atlanto-axial dislocation. Fortunately spinal cord injury is uncommon in these cases. Reduction is usually effected easily either by halter traction or skeletal traction. Prolonged immobilization and treatment of the underlying pathological process usually lead to permanent stabilization. In some instances, however, surgical fusion is required.

How well did you know this?
1
Not at all
2
3
4
5
Perfectly
47
Q

CASE 9: Halo Immooilization for Odontoid Fracture HISTORY: A 33-year-old male was a passenger in an automobile accident and struck his forehead with momentary loss of consciousness. When he regained consciousness he complained of severe pain in the upper cervical region. He had no loss of sensation nor loss of strength in the limbs. His neck was immobilized with sandbags and he was taken directly to the emergency room. EXAMINATION: The patient was awake and complained of neck pain. There was generalized tenderness in the midline upper cervical region. The examination of his limb sensation, strength and reflexes was normal. X-RAY: Cervical spine radiographs showed a nondisplaced fracture of the odontoid process. HOSPITAL COURSE: The patient was placed in skeletal tong traction utilizing five pounds of weight. Serial x-rays showed no evidence of odontoid displacement. Tomograms confirmed the presence of the fracture. The patient’s neurological condition remained stable. On the fourth day after admissionthe patient was fitted for a halo external fixation apparatus and, after x-rays showed good alignment, ambulation was begun. QUESTIONS

  1. A rigid cervicothoracic brace immobilizes the cervical spine as effectively as tne haloapparatus. (True or False)
A

FALSE

Johnson et al. in their study of cervical orthoses demonstrated considerably more mobility with a cervicothoracic brace than with the halo apparatus. The halo apparatus appears superior, in their study, to all other forms of orthoses for external immobilization of the neck.

How well did you know this?
1
Not at all
2
3
4
5
Perfectly
48
Q

CASE 9: Halo Immooilization for Odontoid Fracture HISTORY: A 33-year-old male was a passenger in an automobile accident and struck his forehead with momentary loss of consciousness. When he regained consciousness he complained of severe pain in the upper cervical region. He had no loss of sensation nor loss of strength in the limbs. His neck was immobilized with sandbags and he was taken directly to the emergency room. EXAMINATION: The patient was awake and complained of neck pain. There was generalized tenderness in the midline upper cervical region. The examination of his limb sensation, strength and reflexes was normal. X-RAY: Cervical spine radiographs showed a nondisplaced fracture of the odontoid process. HOSPITAL COURSE: The patient was placed in skeletal tong traction utilizing five pounds of weight. Serial x-rays showed no evidence of odontoid displacement. Tomograms confirmed the presence of the fracture. The patient’s neurological condition remained stable. On the fourth day after admissionthe patient was fitted for a halo external fixation apparatus and, after x-rays showed good alignment, ambulation was begun. QUESTIONS

  1. Some cervical mobility is to be expected with the halo apparatus. (True or False)
A

TRUE

Johnson, et al. demonstrated 1-4% of normal expected cervical mobility in all directions of movements with the halo apparatus. Koch and Nickel, showed that cervical motion with alterations of posture (i,e. change from lying, to sitting) of patients in the halo apparatus is as great as 42% of expected unrestrained cervical movement.

How well did you know this?
1
Not at all
2
3
4
5
Perfectly
49
Q

CASE 9: Halo Immobilization for Odontoid Fracture HISTORY: A 33-year-old male was a passenger in an automobile accident and struck his forehead with momentary loss of consciousness. When he regained consciousness he complained of severe pain in the upper cervical region. He had no loss of sensation nor loss of strength in the limbs. His neck was immobilized with sandbags and he was taken directly to the emergency room. EXAMINATION: The patient was awake and complained of neck pain. There was generalized tenderness in the midline upper cervical region. The examination of his limb sensation, strength and reflexes was normal. X-RAY: Cervical spine radiographs showed a nondisplaced fracture of the odontoid process. HOSPITAL COURSE: The patient was placed in skeletal tong traction utilizing five pounds of weight. Serial x-rays showed no evidence of odontoid displacement. Tomograms confirmed the presence of the fracture. The patient’s neurological condition remained stable. On the fourth day after admissionthe patient was fitted for a halo external fixation apparatus and, after x-rays showed good alignment, ambulation was begun. QUESTIONS

  1. In applying the halo ring to the patient’s head

A. The ring should be approximately 1/2 inch greater than the circumference of the patient’s head. (True or False)

B. the skull puis should be tightened to a torque of five and one-half pounds. (True or False)

A

(A) TRUE. See Nickel, et al.

(B) TRUE. See Nickel, et al.

How well did you know this?
1
Not at all
2
3
4
5
Perfectly
50
Q

CASE 9: Halo Immobilization for Odontoid Fracture HISTORY: A 33-year-old male was a passenger in an automobile accident and struck his forehead with momentary loss of consciousness. When he regained consciousness he complained of severe pain in the upper cervical region. He had no loss of sensation nor loss of strength in the limbs. His neck was immobilized with sandbags and he was taken directly to the emergency room. EXAMINATION: The patient was awake and complained of neck pain. There was generalized tenderness in the midline upper cervical region. The examination of his limb sensation, strength and reflexes was normal. X-RAY: Cervical spine radiographs showed a nondisplaced fracture of the odontoid process. HOSPITAL COURSE: The patient was placed in skeletal tong traction utilizing five pounds of weight. Serial x-rays showed no evidence of odontoid displacement. Tomograms confirmed the presence of the fracture. The patient’s neurological condition remained stable. On the fourth day after admissionthe patient was fitted for a halo external fixation apparatus and, after x-rays showed good alignment, ambulation was begun. QUESTIONS

  1. In addition to nondisplaced fractures of the odontoid process, other cervical injuries appropriately treated with the halo apparatus might include (Select one or more)

A. Hangman’s fracture

B. Jefferson fracture

C. complex fracture (fracture and/or subluxation at more than one location)

D. subluxation without bone injurv

E. cervical fractures associated with quadraplegia

A

(A,B,C,D)

In Cooper’s, et al. report, 85% of all cervical ligamentous and bone injuries healed successfully with immobilization in the halo apparatus. They reported good healing in all 17 patients with Hangman’s fractures, in 2 of 3 patients with odontoid fractures, and in 5 of 11 patients with complex fractures. Satisfactory treatment of Jefferson’s fracture with a halo apparatus was reported by Zimmerman, et. al. Although ligamentous injuries have generally been considered an indication for surgical fusion, Norton and Cooper, et al. reported two patients with purely ligamentous injuries and subluxation who achieved stability following treatment in the halo apparatus. Although there is a report of quadriplegic patients treated with the halo apparatus (Zwerling and Riggins), some authors limit the use of the halo apparatus to neurologically intact patients. Cooper, et al. caution against the hazard of cutaneous ulceration with the halo apparatus in patients with anesthetic skin.

How well did you know this?
1
Not at all
2
3
4
5
Perfectly
51
Q

CASE 9: Halo Immooilization for Odontoid Fracture HISTORY: A 33-year-old male was a passenger in an automobile accident and struck his forehead with momentary loss of consciousness. When he regained consciousness he complained of severe pain in the upper cervical region. He had no loss of sensation nor loss of strength in the limbs. His neck was immobilized with sandbags and he was taken directly to the emergency room. EXAMINATION: The patient was awake and complained of neck pain. There was generalized tenderness in the midline upper cervical region. The examination of his limb sensation, strength and reflexes was normal. X-RAY: Cervical spine radiographs showed a nondisplaced fracture of the odontoid process. HOSPITAL COURSE: The patient was placed in skeletal tong traction utilizing five pounds of weight. Serial x-rays showed no evidence of odontoid displacement. Tomograms confirmed the presence of the fracture. The patient’s neurological condition remained stable. On the fourth day after admissionthe patient was fitted for a halo external fixation apparatus and, after x-rays showed good alignment, ambulation was begun. QUESTIONS

  1. Relative contraindications to the management of cervical fractures using the halo apparatus might include (Select one or more)

A. spinal fracture and dislocation which cannot be reduced with skeletal traction

B. an uncooperative patient

C. a patient with chronic pulmonary disease

D. a patient with thoracic gibbus

E. a patient over the age of 40 years with a displaced odontoid fracture

A

(A,C,D,E)

Cooper, et al. discusses many of the contraindications to the use of the halo apparatus and concluded that surgery is the indicated form of treatment for patients with irreducible fracture dislocations, that increased respiratory compromise precludes the use of the halo apparatus in patients with chronic pulmonary disease and that there is a serious hazard of skin breakdown overlying a thoracic globus from pressure exerted by the jacket of the hale apparatus. Apuzzo, et al. pointed out the poor likelihood of healing of displaced fractures in patients over the age of 40 and recommended surgical fusion for this group. Finally, Cooper, et al. suggested that because the halo is extremely difficult to remove and manipulate, it offers an advantage for patients who might not be relied upon to cooperate with other forms of conservative management of cervical fractures.

How well did you know this?
1
Not at all
2
3
4
5
Perfectly
52
Q

CASE 10: Weakness and Loss of Sensation in the Arms of a 64-Year-Old Man HISTORY: A 64-year-old man was a passenger in a motor vehicle which struck a utility pole. The patient was not wearing a seat belt and his head struck the windshield. On admission to the emergency room he complained of neck pain and weakness in his hands. EXAMINATION: He was awake and alert and exhibited contusions and abrasions of the forehead and nose. Motor examination revealed moderate weakness (graded 4/5} bilaterally of shoulder abduction, elbow flexion and extension, and marked weakness of wrist flexion and extension. Grip and intrinsic muscle function were graded 2/5 in the hands. Sensory examination revealed hypalgesia over the C5-T1 dermatomes bilaterally but light touch sensation, vibratory perception and joint position sense was preserved. Reflexes were barelv obtainable in the arms. Strength, sensation and reflexes were all normal below T1. Sphincter function was preserved. X-RAY: A lateral cervical spine x-ray is shown in Figure 10.1. QUESTIONS

  1. The cervical spine x-ray reveals which of the following? (Select one or more)

A. A fracture

B. A dislocation

C. Normal cervical spine

D. Cervical spondylosis

E. None of the above

A

(D)

There is no evidence of fracture or dislocation. Intervertebral disc space narrowing and spur formation particularly evident at C4-5 and C5-6 are compatible with the diagnosis of cervical spondylosis or cervical degenerative arthritis. Spurs resulting from degenerative joint disease narrow the saggital diameter of the spinal canal reducing the space available to accommodate the cervical spinal cord. Such patients are particularly susceptible to spinal cord trauma during hyperextension injuries to the cervical spine. Hyperextension has been shown to result in further narrowing of the spinal canal by posterior impingement of the ligamentum flavum. The contusions and abrasions on the forehead and nose of this patient indicate force applied such as to hyperextend the cervical spine.

How well did you know this?
1
Not at all
2
3
4
5
Perfectly
53
Q

CASE 10: Weakness and Loss of Sensation in the Arms of a 64-Year-Old Man HISTORY: A 64-year-old man was a passenger in a motor vehicle which struck a utility pole. The patient was not wearing a seat belt and his head struck the windshield. On admission to the emergency room he complained of neck pain and weakness in his hands. EXAMINATION: He was awake and alert and exhibited contusions and abrasions of the forehead and nose. Motor examination revealed moderate weakness (graded 4/5} bilaterally of shoulder abduction, elbow flexion and extension, and marked weakness of wrist flexion and extension. Grip and intrinsic muscle function were graded 2/5 in the hands. Sensory examination revealed hypalgesia over the C5-T1 dermatomes bilaterally but light touch sensation, vibratory perception and joint position sense was preserved. Reflexes were barelv obtainable in the arms. Strength, sensation and reflexes were all normal below T1. Sphincter function was preserved. X-RAY: A lateral cervical spine x-ray is shown in Figure 10.1. QUESTIONS

  1. The clinical picture exhibited by this patient is due to (Select only one)

A. central cervical spinal cord injury

B. anterior cervical spinal cord injury

C. multiple cervical nerve root injuries

D. none of the above

A

(A)

The central cervical spinal cord syndrome was described by Schneider. It is characterized clinically by weakness which is most severe in (or confined to) the upper extremities as compared to the lower extremities. Further, the weakness tends to be most severe distally in the small hand muscles and least severe In the shoulder girdle. Sensory abnormalities are minimal but if present tend to be most severe in the upper extremities and may be of the “dissociated type”, i.e. a disproportionately greater loss of pain and temperature sensation as compared to touch. Sphincter function may or may not be affected. Hyperextensior. injuries are usually considered the main cause of the central spinal cord syndrome; however, this is not always the case. Up to 50% of cases may result from flexion injuries. Schneider originally proposed central hematomyelia secondary to mechanical spinal cord compression from hyperextension as the mechanism for the central spinal cord syndrome. This was felt to be aggravated by premorbid narrowing of the spinal canal by cervical spondylosis. Vascular lesions, however, may produce identical lesions. Current research also suggests that a process of spreading spinal cord necrosis may follow initial spinal cord injury and be responsible for the final lesion. Figure 10. 2 shows the anatomical explanation for the central spinal cord syndrome. The diagram is at the lower cervical (C5,C6,C7) area at which point the spinal cord has its largest diameter, related to motor supply to the upper extremities. Central cord injury (shaded area) results in damage to anterior horn cells (motor neurons) which supply muscles in the arms. Since the anterior roots take a slightly caudally directed course before exiting from the spinal cord, injury to motor neurons at the C5, C6, C7 transverse spinal cord level tends to be manifest mainly in the small hand muscles supplied primarily by the C8 and T1 nerve roots. If the lesion is large enough, injury to the ventrally located reticulospinal tract may result in sphincter dysfunction. Variations in lesion size and exact location may alter the details of clinical presentation in an individual patient. Nevertheless, the general features of the central cervical spinal cord syndrome, i.e. motor weakness and dissociated sensory loss mainly or exclusively in the upper extremities, are reliable. The recognition of this clinical syndrome is important since it relates directly to the question of treatment.

How well did you know this?
1
Not at all
2
3
4
5
Perfectly
54
Q

CASE 10: Weakness and Loss of Sensation in the Arms of a 64-Year-Old Man HISTORY: A 64-year-old man was a passenger in a motor vehicle which struck a utility pole. The patient was not wearing a seat belt and his head struck the windshield. On admission to the emergency room he complained of neck pain and weakness in his hands. EXAMINATION: He was awake and alert and exhibited contusions and abrasions of the forehead and nose. Motor examination revealed moderate weakness (graded 4/5} bilaterally of shoulder abduction, elbow flexion and extension, and marked weakness of wrist flexion and extension. Grip and intrinsic muscle function were graded 2/5 in the hands. Sensory examination revealed hypalgesia over the C5-T1 dermatomes bilaterally but light touch sensation, vibratory perception and joint position sense was preserved. Reflexes were barelv obtainable in the arms. Strength, sensation and reflexes were all normal below T1. Sphincter function was preserved. X-RAY: A lateral cervical spine x-ray is shown in Figure 10.1. QUESTIONS

  1. Which of the following may explain the loss of pinprick sensation but preservation of touch, vibration and position sense in the arms? (Select only one)

A. Dorsal column injury

B. Anterior white commissure injury

C. Spinothalamic tract injury

D. B and C

E. None of the above

A

(D)

The anterior white commissure crosses from the dorsal region and passes just ventral to the gray matter of the spinal cord to form the contralateral spinothalamic tract (Figure 10.2). As the size of the spinothalamic tract increases from sacral, to lumbar, to thoracic, to cervical regions, new fiber are added to the medial edge of the tract and the previously present fibers are pushed more laterally. Therefore, the spinothalamic tract in the cervical region is somatotopically organized with the sacral area most lateral and the cervical region most medial (see Figure 24.1). Centrally located cervical spinal cord lesions thus tend to produce pain and temperature loss in the upper extremities and sparing the legs. The preservation of touch, vibration and position sense implies that the dorsal columns are functioning fairly normally. *pict*

How well did you know this?
1
Not at all
2
3
4
5
Perfectly
55
Q

CASE 10: Weakness and Loss of Sensation in the Arms of a 64-Year-Old Man HISTORY: A 64-year-old man was a passenger in a motor vehicle which struck a utility pole. The patient was not wearing a seat belt and his head struck the windshield. On admission to the emergency room he complained of neck pain and weakness in his hands. EXAMINATION: He was awake and alert and exhibited contusions and abrasions of the forehead and nose. Motor examination revealed moderate weakness (graded 4/5} bilaterally of shoulder abduction, elbow flexion and extension, and marked weakness of wrist flexion and extension. Grip and intrinsic muscle function were graded 2/5 in the hands. Sensory examination revealed hypalgesia over the C5-T1 dermatomes bilaterally but light touch sensation, vibratory perception and joint position sense was preserved. Reflexes were barelv obtainable in the arms. Strength, sensation and reflexes were all normal below T1. Sphincter function was preserved. X-RAY: A lateral cervical spine x-ray is shown in Figure 10.1. QUESTIONS

  1. The statement in the physical examination that grip and intrinsic muscle function are graded 2/5 in the hands implies that the grip and intrinsic muscles (Select only one)

A. are normal in strength

B. will function against resistance

C. will function with gravity eliminated

D. show no muscle contraction

E. show muscle contraction but ho finger movement

A

(C)

A fairly standardized grading system has been used to assess muscle function. It is reproduced in slightly modified form here: 0 = no contraction 1 = muscle contraction without joint movement 2 = movement with gravity eliminated 3 = movement against gravity 4 = movement against gravity and resistance 5 = normal strength This method is strongly recommended for assessment and recording of muscle function to allow both quantitation and standardization for comparison. The system lacks some flexibility where most clinical weakness occurs, namely in grade 4, and modifications of 4-, 4 and 4+ have been suggested to indicate movement against slight, moderate and strong resistance respectively.

How well did you know this?
1
Not at all
2
3
4
5
Perfectly
56
Q

CASE 10: Weakness and Loss of Sensation in the Arms of a 64-Year-Old Man HISTORY: A 64-year-old man was a passenger in a motor vehicle which struck a utility pole. The patient was not wearing a seat belt and his head struck the windshield. On admission to the emergency room he complained of neck pain and weakness in his hands. EXAMINATION: He was awake and alert and exhibited contusions and abrasions of the forehead and nose. Motor examination revealed moderate weakness (graded 4/5} bilaterally of shoulder abduction, elbow flexion and extension, and marked weakness of wrist flexion and extension. Grip and intrinsic muscle function were graded 2/5 in the hands. Sensory examination revealed hypalgesia over the C5-T1 dermatomes bilaterally but light touch sensation, vibratory perception and joint position sense was preserved. Reflexes were barelv obtainable in the arms. Strength, sensation and reflexes were all normal below T1. Sphincter function was preserved. X-RAY: A lateral cervical spine x-ray is shown in Figure 10.1. QUESTIONS

  1. Appropriate early treatment for this patient would involve which of the following? (Select only one)

A. Decompressive laminectomy

B. Placement of skull tongs

C. Placement of a halo apparatus

D Anterior cervical fusion

E. None of the above

A

(E)

The central cervical spinal cord syndrome is best managed conservatively. Skull tongs and the halo apparatus are useful in patients requiring reduction and stabilization of fracture-dislocations. These are not present in the patient under discussion and are not present in most patients with the central cord syndrome, Decompressive laminectomy does not benefit such patients as an early measure. Minimal immobilization perhaps with a cervical collar or brace may help reduce neck pain. Steady progressive improvement in neurological function usually occurs in a predictable fashion. The legs tend to recover motor power first, followed by improved bladder function, improved proximal upper extremity strength follows and finally improvement in fine hand and finger movements may occur. Residual disability due to persistent weakness of intrinsic hand muscles is not uncommon.

How well did you know this?
1
Not at all
2
3
4
5
Perfectly
57
Q

CASE 11: Sensory Preservation after Spinal Cord Injury in a 29-Year-Old Man HISTORY: A 29-year-old man was brought to the emergency room after an automobile accident. He had gone off the road on a rainy night while driving his auto mobile and had struck a tree. He was awake and alert and complained of neck pain. EXAMINATION: Examination showed complete voluntary motor paralysis below C6. There was total loss of pain and temperature sensation to the same level. Urinary and rectal sphincter paralysis were present. There was preservation of touch sensation, proprioception and vibration sense throughout the body. QUESTIONS

  1. This patient represents the clinical syndrome of (Select only one)

A. the Brown-Sequard syndrome

B. cervical spinal cord transection

C multiple cervical nerve root injury

D. anterior cervical spinal cord injury

E. none of the above

A

(D)

The anterior cervical spinal cord syndrome was first described by Schneider. The syndrome includes subtotal loss of spinal cord function with preservation of so-called “dorsal column function’’, i.e. touch, proprioception and vibration. There is complete loss of motor and sphincter function and loss of pain and temperature sensation below a given level.

How well did you know this?
1
Not at all
2
3
4
5
Perfectly
58
Q

CASE 11: Sensory Preservation after Spinal Cord Injury in a 29-Year-Old Man HISTORY: A 29-year-old man was brought to the emergency room after an automobile accident. He had gone off the road on a rainy night while driving his auto mobile and had struck a tree. He was awake and alert and complained of neck pain. EXAMINATION: Examination showed complete voluntary motor paralysis below C6. There was total loss of pain and temperature sensation to the same level. Urinary and rectal sphincter paralysis were present. There was preservation of touch sensation, proprioception and vibration sense throughout the body. QUESTIONS

  1. The most common mechanism of cervical spine injury producing this clinical syndrome is (Select only one)

A. hyperflexion

B. hyperextension

C. rotation

D. none of the above

A

(A)

The anterior spinal cord syndrome was felt to be caused primarily by hyperflexion injuries to the cervical spine, with resultant rupture of the posterior longitudinal ligament, tearing of the annulus fibrosis and posterior herniation of the nucleus pulposus of the intervertebral disc. This mechanism leads to compression of the anterior aspect to the cervical spinal cord with sparing of the posterior columns. Subsequently, posterior displacement of fractured bone fragments and incomplete spinal cord necrosis have also been recognized as causes of the anterior cervical spinal cord syndrome. Vertebral and anterior spinal artery injuries have also been suggested as causally related to the anterior spinal cord syndrome.

How well did you know this?
1
Not at all
2
3
4
5
Perfectly
59
Q

CASE 11: Sensory Preservation after Spinal Cord Injury in a 29-Year-Old Man HISTORY: A 29-year-old man was brought to the emergency room after an automobile accident. He had gone off the road on a rainy night while driving his auto mobile and had struck a tree. He was awake and alert and complained of neck pain. EXAMINATION: Examination showed complete voluntary motor paralysis below C6. There was total loss of pain and temperature sensation to the same level. Urinary and rectal sphincter paralysis were present. There was preservation of touch sensation, proprioception and vibration sense throughout the body. QUESTIONS

  1. Appropriate diagnostic studies in this patient might include. (Select one or more)

A. cervical spine x-rays

B. cervical myelogram

C. cerebral computerized tomographic (CT) scan

D. spinal angiogram

E. none of the above

A

(A,B)

Plain cervical roentgenograms often reveal a “tear-drop” fracture of the involved vertebrae with widening of the disc space posteriorly and narrowing anteriorly. The posterior widening suggests rupture of the posterior longitudinal ligament and underlying annulus fibrosis of the intervertebral disc. Plain x-rays, however, may be unremarkable in the anterior spinal cord syndrome. Cervical myelogram is an important diagnostic study in the anterior spinal cord syndrome since it may reveal a herniated cervical intervertebral disc with anterior compression of the spinal cord as described by Schneider. Most, if not all, patients with the anterior cervical spinal cord syndrome should undergo myelography shortly after injury, when their general condition is stabilized. Cerebral CT scan is not indicated unless there is evidence of significant head injury which was not present in the patient illustrated. Spinal angiography is usually reserved for the diagnosis of spinal cord tumor or vascular lesion, neither of which is suggested in this case. *pict*

How well did you know this?
1
Not at all
2
3
4
5
Perfectly
60
Q

CASE 11: Sensory Preservation after Spinal Cord Injury in a 29-Year-Old Man HISTORY: A 29-year-old man was brought to the emergency room after an automobile accident. He had gone off the road on a rainy night while driving his auto mobile and had struck a tree. He was awake and alert and complained of neck pain. EXAMINATION: Examination showed complete voluntary motor paralysis below C6. There was total loss of pain and temperature sensation to the same level. Urinary and rectal sphincter paralysis were present. There was preservation of touch sensation, proprioception and vibration sense throughout the body. QUESTIONS

  1. Anteroposterior and lateral x-rays of the cervical spine are illustrated in Figures 11. 1 and 11.2. They show (Select one or more)

A. normal findings

B. fracture of body of C6

C. fracture of transverse processes of C6 and C7

D. dislocation

E. none of the above

A

(B,C)

The lateral view (Figure 11.2) shows reduced height of the C6 vertebral body and a small fracture of the anteroinferior corner of the body of C6 as well. This is the socalled ‘‘tear-drop’’ fracture. Fractures of the transverse processes on the right side are seen in the anteroposterior view (Figure 11.1). No significant dislocation is seen as judged by alignment of the posterior surfaces of the bodies of the cervical vertebrae. The malalignment of the anterior surfaces of the body of C6 in relation to C5 and C7 is due to compression and fracture of the CC body. Facet joints are normally aligned without evidence of dislocation.

How well did you know this?
1
Not at all
2
3
4
5
Perfectly
61
Q

CASE 11: Sensory Preservation after Spinal Cord Injury in a 29-Year-Old Man HISTORY: A 29-year-old man was brought to the emergency room after an automobile accident. He had gone off the road on a rainy night while driving his auto mobile and had struck a tree. He was awake and alert and complained of neck pain. EXAMINATION: Examination showed complete voluntary motor paralysis below C6. There was total loss of pain and temperature sensation to the same level. Urinary and rectal sphincter paralysis were present. There was preservation of touch sensation, proprioception and vibration sense throughout the body. QUESTIONS

  1. Figures 11.3 and 11.4 illustrate lateral views from a myelogram performed using air (11.3) and pantopaque (11. 4). They show (Select only one)

A. normal findings

B. a herniated intervertebral disc

C. an extradural defect at C5-6

D. an extradural hematoma

E. none of the above

A

(C)

The pantopaque myelogram (Figure 11.4) shows a smooth posterior displacement of the dye column primarily opposite the C5-6 intervertebral disc space. The air myelogram (Figure 11.3) shows obliteration of the column of air ventral to the spinal cord in the region of the C5-6 interspace. The “teardrop” fracture of the antero-inferior corner of the body of C6 is also well seen on the myelographic films. It is impossible to ascertain the pathological process responsible for the deficit front the myelographic findings alone. A herniated intervertebral disc or posteriorly displaced bone fragments are likely etiological agents. Spinal hematoma localized to such a small area is uncommon (see Cases 27 and 28).

How well did you know this?
1
Not at all
2
3
4
5
Perfectly
62
Q

CASE 11: Sensory Preservation after Spinal Cord Injury in a 29-Year-Old Man HISTORY: A 29-year-old man was brought to the emergency room after an automobile accident. He had gone off the road on a rainy night while driving his auto mobile and had struck a tree. He was awake and alert and complained of neck pain. EXAMINATION: Examination showed complete voluntary motor paralysis below C6. There was total loss of pain and temperature sensation to the same level. Urinary and rectal sphincter paralysis were present. There was preservation of touch sensation, proprioception and vibration sense throughout the body. QUESTIONS

  1. Appropriate treatment of this patient might include (Select one or more)

A. initial treatment with skeletal traction tongs

B. decompressive laminectomy

C. anterior cervical disc excision at C5-6

D. anterior cervical fusion

E. none of the above

A

(A,C,D)

Plain roentgenograms of the spine demonstrate a fracture of C6. Although significant dislocation is not identified, skeletal traction with skull tongs is recommended for maintenance of normal alignment. The likely mechanism of injury in this patient is hyperflexion. In such injuries, the interspinous ligaments and facet joint ligaments maybe stretched or torn. The anterior elements (vertebral body and disc space) are also damaged by the compressive effect of flexion. Thus, the injury may be unstable even though gross dislocation is not apparent on x-ray. The myelogram demonstrated an extradural defect opposite the C5-6 interspace. Schneider recommended operative treatment of such lesions particularly in association with the anterior spinal cord syndrome which the patient described exhibited. This is most easily and safely carried out by an anterior approach. Direct access to the injured disc is provided without disturbance of the injured spinal cord. Disc removal and spinal cord decompression may be carried out safely by this approach. Additionally, a bone graft may be inserted into the intervertebral space following disc excision to promote fusion. Hyperflexion injuries of significant severity often produce disruption of the capsular ligaments of the posterior facet joints of the cervical vertebrae. This injury, in addition to disruption of the disc space and longitudinal ligaments, may result in spinal in stability. Thus, the anterior approach allows both removal of the offending pathology compressing the spinal cord as well as fusion if instability is present. Although it was initially recommended for treatment of such lesions, laminectomy in this situation does not permit direct access to the pathological process which is anteriorly located. Manipulation of the already injured spinal cord to obtain anterior access may further injure the cord. Further, the dissection of cervical musculature and removal of laminae may aggravate pre-existing instability and require extensive posterior fusion or a secondary operation for anterior fusion. Thus, laminectomy is not indicated in this situation.

How well did you know this?
1
Not at all
2
3
4
5
Perfectly
63
Q

CASE 12: 43-Year-Old Woman With Neck Injury Following Automobile Accident HISTORY: A 43-year-old woman was admitted to the emergency room complaining of severe neck pain following an automobile accident. The patient did not recall the accident nor the ambulance trip to the hospital. EXAMINATION: Examination showed contusions of the neck anteriorly beneath the mandible. Neurological examination was normal except for amnesia. QUESTIONS

  1. Figure 12.1 is a lateral cervical spine x-ray taken in the emergency room. It shows (Select only one)

A. normal cervical spine

B. fracture of the pedicles of C2

C. fracture of the odontoid process

D. fracture of the arch of C1

E. none of the above

A

(B)

Bilateral fractures of the pedicles of C2 are shown. The fracture is more easily appreciated on a coned view of C2 as shown in Figure 12.2. In another patient (Figure 12.3), the fracture is obvious. This lesion has been termed ‘‘Hangman’s fracture” because of its similarity to the lesion which occurs after judicial hanging. Although methods of judicial hanging have varied over the years, most hangings in relatively recent times have employed a submental knot. A knot in such a location combined with a sudden drop of the victim’s body results in symmetrical fractures across the pedicles or lateral masses of the axis and may extend across the posterior part of the body as well. Fracture of the odontoid process does not occur in this pathological process. *pict* *pict*

How well did you know this?
1
Not at all
2
3
4
5
Perfectly
64
Q

CASE 12: 43-Year-Old Woman With Neck Injury Following Automobile Accident HISTORY: A 43-year-old woman was admitted to the emergency room complaining of severe neck pain following an automobile accident. The patient did not recall the accident nor the ambulance trip to the hospital. EXAMINATION: Examination showed contusions of the neck anteriorly beneath the mandible. Neurological examination was normal except for amnesia. QUESTIONS

  1. The mechanism of the spinal injury illustrated is (Select only one)

A. hyperflexion

B. rotation

C. hyperextension

D. none of the above

A

(C)

A combination of hyperextension and compression leads to pressure on the neural arch of the axis. This leads to bilateral fracture of the pedicles. If a distracting force is present, as in judicial hanging, marked dislocation occurs. In vehicular accidents, the most common mechanism of such injuries, strong distracting forces are absent and minimal or no dislocation occurs. Thus, such lesions may be overlooked on screening radiographs. Figure 12,4 illustrated significant dislocation at C2-3 in a patient with “Hangman’s fracture” following vehicular accident. *pict*

How well did you know this?
1
Not at all
2
3
4
5
Perfectly
65
Q

CASE 12: 43-Year-Old Woman With Neck Injury Following Automobile Accident HISTORY: A 43-year-old woman was admitted to the emergency room complaining of severe neck pain following an automobile accident. The patient did not recall the accident nor the ambulance trip to the hospital. EXAMINATION: Examination showed contusions of the neck anteriorly beneath the mandible. Neurological examination was normal except for amnesia. QUESTIONS

  1. Death in judicial hanging is usually due to (Select only one)

A. brainstem injury

B. strangulation

C. decapitation

D. spinal cord transection

E. none of the above

A

(D)

In judicial hanging, submental knot placement leads to bilateral fracture of the pedicles of the axis. The weight of the falling body results in distraction of the fracture and transection of the spinal cord at C2-3. Respiratory arrest and subsequently cardiac arrest and death ensue. If an insufficient length of drop is utilized in attempted hanging, cervical fracture-dislocation does not occur but death may ensue on a delayed basis, due to strangulation. This frequently occurs with attempted suicides. If an excessive length drop is utilized, decapitation may result. If a subaural instead of a submental knot is utilized, basal skull fracture with brainstem injury may result. Death in such instances is not assured and the subaural knot is not recommended in judicial hanging.

How well did you know this?
1
Not at all
2
3
4
5
Perfectly
66
Q

CASE 12: 43-Year-Old Woman With Neck Injury Following Automobile Accident HISTORY: A 43-year-old woman was admitted to the emergency room complaining of severe neck pain following an automobile accident. The patient did not recall the accident nor the ambulance trip to the hospital. EXAMINATION: Examination showed contusions of the neck anteriorly beneath the mandible. Neurological examination was normal except for amnesia. QUESTIONS

  1. The normal neurological examination in this patient is (Select only one)

A. to be expected

B. unusual

C. likely to deteriorate acutely

D. none of the above

A

(A)

In patients who sustain a “Hangman’s fracture” accidentally and in whom survival occurs, significant neurological deficit may occur but is uncommon. In those patients in whom hyperexrension is severe, death often occurs before the institution of resuscitative measures because of acute upper cervical spinal cord injury with respiratory paralysis. Occasional survivors with quadriplegia have been reported but are rare. If the degree of hyperextension is less severe, and particularly if unaccompanied by a distractive force, bilateral fracture of the pedicles of C2 occurs without spinal cord injury. Williams proposed that the mechanisms of judicial versus accidentally induced “Hangman’s fracture” are different. He felt the former always associated with distraction, was fatal, whereas the latter, often unaccompanied by distraction., only rarely resulted in neurological injury. Hyperextension-distraction injuries have been reporter in vehicle accidents in relation to diagonal seat belts applied too loosely. In such cases, the results are similar to judicial hanging, namely producing a fatal injury. Even in those accidental cases of neurological injury associated with “Hangman’s fracture”, the presence of severe associated injuries (e.g. head injury) make correlation of the spinal injury and neurological deficit difficult. The occasional neurological injuries seen with “Hangman’s fracture” tend to recover with time. Progressive deficit is rare but may occur with missed “Hangman’s fracture”.

How well did you know this?
1
Not at all
2
3
4
5
Perfectly
67
Q

CASE 12: 43-Year-Old Woman With Neck Injury Following Automobile Accident HISTORY: A 43-year-old woman was admitted to the emergency room complaining of severe neck pain following an automobile accident. The patient did not recall the accident nor the ambulance trip to the hospital. EXAMINATION: Examination showed contusions of the neck anteriorly beneath the mandible. Neurological examination was normal except for amnesia. QUESTIONS

  1. Appropriate treatment for this patient is (Select one ormore)

A. decompressive laminectomy

B. anterior fusion

C. halo brace

D. high dose corticosteroids

E. none of the above

A

(C)

“Hangman’s fracture” is a relatively stable injury. Originally, the condition was treated by skeletal traction via skull tongs. Excessive distraction can be produced by this means, however, and judicious addition to the amount of weigh utilized to produce traction and monitoring with lateral cervical spine radiographs is necessarv. More recently, management with the halo apparatus has been recommended. Anterior cervical fusion was occasionally used but this seems to be rarely necessary. Decompressive laminectomy plays no role in the treatment of “Hangman’s fracture” since no lesion is present which could be decompressed. Since most patients are neurologically normal, pharmacological agents such as corticosteroids are also not indicated.

How well did you know this?
1
Not at all
2
3
4
5
Perfectly
68
Q

CASE 13: Burning Hands in a 21-Year-Old Football Flayer HISTORY: A 21-year-old college football player was injured while tackling an opponent. He was brought to the emergency room complaining of severe burning pain and numbness in his hands. He denied neck pain. EXAMINATION: There was no external evidence of trauma nor pain on palpation of the cervical spine. Motor examination disclosed mild diffuse weakness of the upper extremities bilaterally.Lower extremity strength was normal. Sensory examination revealed decreased touch and pinprick perception in the hands and fingers bilaterally. Reflexes were absent in the upper extremities but present in the lower extremities. No pathologic reflexes were noted and sphincter function was normal. QUESTIONS

  1. The “burning hands” described by this patient most likely are (Select only one)

A. doe to abrasions from contact with artificial turf

B. due to bilateral carpal tunnel syndrome from acute wrist extension

C. symptoms of central cervical spinal cord injury

D. of no clinical significance

E. none of the above

A

(C)

Burning dysesthesias are commonly seen after nerve root injuries which most often are unilateral. Maroon, however, recently described “burning hands” in two football players following cervical spine injury. He postulated that a lesion of the spinothalamic tract, concerned with pain and temperature perception, was responsible for the symptom. The spinothalamic tract, like the pyramidal motor tract, is somatotopically organized so that the fibers to the upper extremities, especially the fingers, are located most medially in the cervical segments of the spinal cord. Significant motor loss was noted in the upper extremities with normal motor function in the legs in the patient described in this case and in Maroon’s patients. This is consistent with central cervical spinal cord injury (see Case 10). Maroon demonstrated, utilizing myelography, widening of the cervical spinal cord from C5-C7 consistent with spinal cord contusion and swelling. The presence of sensory changes, arm weakness and loss of reflexes in the arms makes it unlikely that burning pain in the hands is due to local injury such as skin abrasions or carpal tunnel syndrome, although either of those lesions could produce isolated burning pain in the hands. The presence of neurological deficit also confirms the seriousness of the potential problem and suggests the clinical significance of the complaint of burning pain in the hands. Maroon pointed out the absence of neck pain in his patients, a symptom thought to be present almost universally with spine injury. The presence of burning pain should alert the physician to the possibility of spinal injury even in the absence of neck pain. Being so alerted, appropriate steps for careful diagnostic study and treatment may be carried out.

How well did you know this?
1
Not at all
2
3
4
5
Perfectly
69
Q

CASE 13: Burning Hands in a 21-Year-Old Football Flayer HISTORY: A 21-year-old college football player was injured while tackling an opponent. He was brought to the emergency room complaining of severe burning pain and numbness in his hands. He denied neck pain. EXAMINATION: There was no external evidence of trauma nor pain on palpation of the cervical spine. Motor examination disclosed mild diffuse weakness of the upper extremities bilaterally.Lower extremity strength was normal. Sensory examination revealed decreased touch and pinprick perception in the hands and fingers bilaterally. Reflexes were absent in the upper extremities but present in the lower extremities. No pathologic reflexes were noted and sphincter function was normal. QUESTIONS

  1. Appropriate treatment of this patient might include (Select one or more)

A. cervical spine x-rays

B. immobilization of the neck

C. myelography

D. none of the above

A

(A,B,C)

The clinical history and examination suggest cervicai spine injury. Plain x-rays of the spine are essential in such a circumstance. One of Maroon’s two patients with burning hand pain demonstrated a fracture-dislocation at C5-6 and the other had normal x-rays. The importance of adequate immobilization of the head and neck in such circumstances cannot be overemphasized. The possibility of converting a mild spinal cord injury into a catastrophe is present unless the importance of immobilization before and during x-ray studies is appreciated. Myelography may be of value dependent on the patient’s clinical condition. Usually patients with central cervical spinal cord injury recover without surgery. In one of Maroon’s, two cases, a myelogram done because of persistent radiculopathy showed a ruptured disc at the C5-6 interspace and surgical treatment was accomplished. In the other, myelography demonstrated only widening of the cervical spinal cord and the patient recovered with nonsurgical treatment.

How well did you know this?
1
Not at all
2
3
4
5
Perfectly
70
Q

CASE 13: Burning Hands in a 21-Year-Old Football Flayer HISTORY: A 21-year-old college football player was injured while tackling an opponent. He was brought to the emergency room complaining of severe burning pain and numbness in his hands. He denied neck pain. EXAMINATION: There was no external evidence of trauma nor pain on palpation of the cervical spine. Motor examination disclosed mild diffuse weakness of the upper extremities bilaterally.Lower extremity strength was normal. Sensory examination revealed decreased touch and pinprick perception in the hands and fingers bilaterally. Reflexes were absent in the upper extremities but present in the lower extremities. No pathologic reflexes were noted and sphincter function was normal. QUESTIONS

  1. Which of the following are true concerning cervical spine injuries in football players? (Select only one)

A. Occur rarely

B. Are usually fatal

C. Are frequently undetected

D. Occur primarily in offensive and defensive linemen

E. None of the above

A

(C)

Albright examined 179 high school players and college freshman football candidates. He found that 32% of the college players and 8% of the high school players had significant x-ray evidence of cervical spine trauma. This x-ray evidence included compression fractures, abnormal motion, narrowed disc spaces and neural arch fractures. Nearly two-thirds of the college players with ah normal x-rays neither complained of neck pain nor gave a history of neck injury. Only 4 of 42 players with abnormal x-rays had significant findings on physical examination. Thus, the incidence of cervical spine injuries in football players, even of relatively young age, is grossly underestimated. This opinion is strengthened by a survey of coaches conducted by Albright. Of 20.189 players on the responding ‘ teams, coaches reported only 1-2% as having been injured. Albright estimated that only about one of 64 actually injured players in an eight team conference had a reported injury. Thus, the incidence of nonfatal cervical spine injuries is probably much underrated. The risk of death due to cervical spine injury is probably overestimated since the number of nonfatal injuries has been previously unappreciated. The actual incidence of serious cervical spine and spinal cord injuries related to athletic injuries is unknown. Leader attributed 4-5% of all spinal cord injuries resulting in permanent paralysis to athletic injuries. Torg documented one serious spinal cord injury per 44.000 participants in football during 1975. Clark surveyed high school and college football teams during 1973-1975 and identified one permanent spinal cord injury per 28,000 participants. Between 1931 and 1975, 819 deaths were attributed to trauma as a result of playing football, two per 100.000 participants, and 79% of the deaths were due to brain and spinal cord injuries. Torg recently suggested that the incidence of cervical spine and spinal cord injury is increasing and questioned the possible emergence of a significant national health problem. Several studies have indicated that the incidence of injuries is related directly to length of participation and position played. Most injuries occur to the individuals involved in making open field tackles. Thus, defensive backs and linebackers are most often injured. Linemen, involved mainly in blocking and tackling at lower speeds in a crowded area, rarely receive significant cervical spine or spinal cord injuries.

How well did you know this?
1
Not at all
2
3
4
5
Perfectly
71
Q

CASE 13: Burning Hands in a 21-Year-Old Football Flayer HISTORY: A 21-year-old college football player was injured while tackling an opponent. He was brought to the emergency room complaining of severe burning pain and numbness in his hands. He denied neck pain. EXAMINATION: There was no external evidence of trauma nor pain on palpation of the cervical spine. Motor examination disclosed mild diffuse weakness of the upper extremities bilaterally.Lower extremity strength was normal. Sensory examination revealed decreased touch and pinprick perception in the hands and fingers bilaterally. Reflexes were absent in the upper extremities but present in the lower extremities. No pathologic reflexes were noted and sphincter function was normal. QUESTIONS

  1. Cervical spine injuries in football players frequently result in quadriplegia. (True or False)
A

TRUE

Schneider estimated that over 50% of cervical fracture-dislocations incurred in football players between 1959-1963 resulted in quadriplegia of a complete and permanent nature. Of 12 severe neck injuries in football players studied by Torg in 1975, seven resulted in quadriplegia. Thus, the incidence of quadriplegia in cervical spinal cord injuries is very high.

How well did you know this?
1
Not at all
2
3
4
5
Perfectly
72
Q

CASE 13: Burning Hands in a 21-Year-Old Football Flayer HISTORY: A 21-year-old college football player was injured while tackling an opponent. He was brought to the emergency room complaining of severe burning pain and numbness in his hands. He denied neck pain. EXAMINATION: There was no external evidence of trauma nor pain on palpation of the cervical spine. Motor examination disclosed mild diffuse weakness of the upper extremities bilaterally.Lower extremity strength was normal. Sensory examination revealed decreased touch and pinprick perception in the hands and fingers bilaterally. Reflexes were absent in the upper extremities but present in the lower extremities. No pathologic reflexes were noted and sphincter function was normal. QUESTIONS

  1. Cervical spine injuries in football players usually result from tackling using the crown of the helmet as a battering ram. (True or False)
A

TRUE

Although a variety of mechanisms have been identified as the cause of cervical spine injuries in football players, the use of the crown of the helmet as a battering ram is most often the etiological mechanism. Torg indicated that in six of eight players with spinal cord injury and quadriplegia the helmet had been used by the injured player as a battering ram while making a tackle. The mechanism of spinal cord injury in such cases is felt to be axial loading of the spine, the force vector being directed along the axial alignment of the cervical spine. This results in disruption of the intervertebral discs and posterior ligamentous structures with dislocation and acute spinal cord trauma.

How well did you know this?
1
Not at all
2
3
4
5
Perfectly
73
Q

CASE 13: Burning Hands in a 21-Year-Old Football Flayer HISTORY: A 21-year-old college football player was injured while tackling an opponent. He was brought to the emergency room complaining of severe burning pain and numbness in his hands. He denied neck pain. EXAMINATION: There was no external evidence of trauma nor pain on palpation of the cervical spine. Motor examination disclosed mild diffuse weakness of the upper extremities bilaterally.Lower extremity strength was normal. Sensory examination revealed decreased touch and pinprick perception in the hands and fingers bilaterally. Reflexes were absent in the upper extremities but present in the lower extremities. No pathologic reflexes were noted and sphincter function was normal. QUESTIONS

  1. Which of the following are true concerning the use of neck manipulation in the treatment of persistent neck pain in football players? (Select one or more)

A. Effective means of pain relief

B Safe if cervical spine x-rays are normal

C. May result in spinal injury

D. May result in vascular occlusion

E. May result in brainstem infarction

A

(C,D,E)

Manipulation of the neck is commonly used in the treatment of persistent neck pain in athletes as well as the general population. It is generally assumed to be a safe and effective means of obtaining pain relief. Significant risk is present, however, as judged by reports of spinal injury, vascular injury and brainstem infarction. Schneider reviewed the risks of neck manipulation in football players and cautioned against its use. Cervical spine dislocation, occlusion of the vertebrobasilar and anterior spinal arteries, brainstem infarction, and even death may result from cervical spine manipulation. This form of treatment is not recommended for persistent neck pain in either the athlete or nonathlete.

How well did you know this?
1
Not at all
2
3
4
5
Perfectly
74
Q

CASE 14: Coma in a 17-Year-Old Who Was Struck by an Automobile HISTORY: A 17-year-old female was brought to the emergency room after having been struck by an automobile. The patient was a pedestrian. When first seen by paramedics at the accident scene, the patient was unconscious without spontaneous respirations and pupils were noted to be fixed and dilated; peripheral pulses were absent. An endotracheal tube was inserted and cardiopulmonary resuscitation was begun. EXAMINATION: Neurological status was unchanged on arrival in the emergencv room, Resuscitative attempts were continued X-RAYS: A lateral cervical spine x-ray is illustrated in Figure 14. 1. QUESTIONS

1 . The cervical spine x-ray shows (Select one or more)

A. a fracture-dislocation

B. soft tissue swelling anterior to the cervical spine

C. probably a fatal injury

D. fracture of the odontoid process

A

(A,B,C)

The x-ray shows a fracture-dislocation at C3-4 with massive dislocation and displacement. The odontoid process of C2 is intact. Actually, the fracture per se appears small, i.e. a fracture of the tip of the spinous process of C4. The primary injury is a dissolution of ligamentous structures between C3 and C4. There is complete rupture through the intervertebral disc space, complete rupture of the capsular ligaments of the facet joints and rupture of the nuchal ligaments between the spinous processes. There is marked increase in distance between the endotracheal tube and the anterior aspect of the bodies of the cervical vertebrae throughout the cervical area due most likely to a combination of hematoma and soft tissue edema. A transverse injury of the spinal cord with complete transection would almost certainly result from such an injury. In addition, significant injury or even transection of the vertebral arteries may occur from such a lesion with resultant acute massive brainstem infarction. Fracture-dislocations with such massive displacement, located at C4 and above are usually fatal due to respiratory arrest.

How well did you know this?
1
Not at all
2
3
4
5
Perfectly
75
Q

CASE 14: Coma in a 17-Year-Old Who Was Struck by an Automobile HISTORY: A 17-year-old female was brought to the emergency room after having been struck by an automobile. The patient was a pedestrian. When first seen by paramedics at the accident scene, the patient was unconscious without spontaneous respirations and pupils were noted to be fixed and dilated; peripheral pulses were absent. An endotracheal tube was inserted and cardiopulmonary resuscitation was begun. EXAMINATION: Neurological status was unchanged on arrival in the emergencv room, Resuscitative attempts were continued X-RAYS: A lateral cervical spine x-ray is illustrated in Figure 14. 1. QUESTIONS

  1. In fatal cervical spinal cord injuries, the cause of death is most often (Select only one)

A. hypovolemic shock

B. spinal shock

C. primary cardiac arrest

D. respiratory arrest

E. none of the above

A

(D)

Fatal cervical spinal cord injuries result most often in acute primary respiratory arrest followed by secondary cardiac arrest. Thus, most fatal lesions are located at C4 or above. Even complete lesions below C4 are usually not fatal since diaphragmatic respiration often provides sufficient respiratory exchange to sustain life. Fatality maybe avoided in some cases of acute spinal transection above C4 by prompt institution of artificial respiration. Spinal shock results in only mild hypotension due to sympathectomy. Hypovolemic shock may occur due to thoracic or abdominal injuries and may indeed be fatal; however, it does not relate directly to spinal cord injury. Respiratory arrest results from interruption of connections between the respiratory centers of the brainstem and the phrenic (C3-4) and intercostal (Tl-2) respiratory neurons.

How well did you know this?
1
Not at all
2
3
4
5
Perfectly
76
Q

CASE 14: Coma in a 17-Year-Old Who Was Struck by an Automobile HISTORY: A 17-year-old female was brought to the emergency room after having been struck by an automobile. The patient was a pedestrian. When first seen by paramedics at the accident scene, the patient was unconscious without spontaneous respirations and pupils were noted to be fixed and dilated; peripheral pulses were absent. An endotracheal tube was inserted and cardiopulmonary resuscitation was begun. EXAMINATION: Neurological status was unchanged on arrival in the emergencv room, Resuscitative attempts were continued X-RAYS: A lateral cervical spine x-ray is illustrated in Figure 14. 1. QUESTIONS

  1. How frequently are cervical spinal injuries identified in fatal traffic accidents? (Select only one)

A. 20%

B. 40%

C. 60%

D. 80%

A

(A)

Alker, et al. studied 146 fatalities of traffic accidents. Both radiological and autopsy studies were employed. Twenty-one percent of their patients demonstrated significant cervical spinal injuries. The incidence of cervical spine trauma in other series of fatal accidents is lower, ranging from 3 to 15%. The later figures are based exclusively on autopsy studies. Since detection of fractures and dislocations of the cervical spine is difficult at autopsy, the figures in the later two studies are probably artificially low. The utilization of x-ray evaluation by Alker, et al. yielded a much higher figure and probably more closely approximates the true incidence.

How well did you know this?
1
Not at all
2
3
4
5
Perfectly
77
Q

CASE 14: Coma in a 17-Year-Old Who Was Struck by an Automobile HISTORY: A 17-year-old female was brought to the emergency room after having been struck by an automobile. The patient was a pedestrian. When first seen by paramedics at the accident scene, the patient was unconscious without spontaneous respirations and pupils were noted to be fixed and dilated; peripheral pulses were absent. An endotracheal tube was inserted and cardiopulmonary resuscitation was begun. EXAMINATION: Neurological status was unchanged on arrival in the emergencv room, Resuscitative attempts were continued X-RAYS: A lateral cervical spine x-ray is illustrated in Figure 14. 1. QUESTIONS

  1. Which of the following are true concerning fatal cervical spine injuries? (Select one or more)

A. Predominantly occur in males

B. Rarely occur in conjunction with serious intracranial injury

C. Predominantly occur in upper cervical region

D. None of the above

A

(A,C)

The preponderance of male traffic accident victims is astonishing. Ninety percent of fatalities among vehicle drivers were male and 49% of pedestrian fatalities were male. In the series of Alker, et al. 25 of 31 fatal cervical spine injuries occured at C3 or above. These included eight with atlantooccipital injuries, seven with injuries to C1 and ten with injuries to C2. Atlanto-occipital injuries occurred almost exclusively in pedestrians struck anteriorly by motor vehicles with sudden posterior acceleration of the body in relation to the head and resultant acute hyperflexion of the head on the neck. One third of patients in Alker’s series with cervical spine trauma also exhibited serious intracranial injury. In fact, of the total of 146 patients studied by Alker, et al., 42 had demonstrable head injuries ranging from relatively simple linear skull fractures to massive skull damage. Death in the latter resulted from intracranial hematoma, brainstem injury or from massive air embolus. Sixty percent of cases with head injury demonstrated intracranial or intravascular air. Massive air embolization of the right atrium was often associated with intravascular air. Air embolization in these cases was due to injury to dural venous sinuses.

How well did you know this?
1
Not at all
2
3
4
5
Perfectly
78
Q

CASE 15: Cervical Trauma in a Rheumatoid Arthritic HISTORY: A 59-year-old female was seen in the emergency room following an automobile accident, complaining of neck pain. She had a 32-year history of rheumatoid arthritis. She denied motor or sensory symptoms and had been able to urinate since the accident. She described mild intermittent upper cervical pain over several months prior to the accident. EXAMINATION: She had many of the stigmata of rheumatoid arthritis, including rheumatoid nodules and ulnar deviation of the fingers. The range of motion of her neck was limited in all directions but there was no tenderness. Her gait was limited by hip involvement by arthritic disease but was not spastic. On neurological examination, her motor and sensory functions were normal. Deep tendon reflexes in all extremities were slightly brisk, but she had no pathological reflexes. X-RAY: The lateral cervical spine in the emergency room suggested forward displacement of C1 on C2. AP and odontoid views showed no evidence of fracture. Lateral flexion and extension views were obtained and showed mobility of C I on C2 in anterior flexion indicating atlanto-axial subluxation (Figure 15. 1) QUESTIONS

  1. Atlanto-axial subluxation can be quantified in adults by which of the following lateral cervical spine measurements? (Select one or more)

A. Chamberlain’s line

B. The distance between the posterior edge of the odontoid process and the posterior arch of the atlas

C. The distance between the posterior edge of the odontoid process and the edge of the posterior lip of the foramen magnum

D. The distance between the anterior edge of the odontoid process and the anterior arch of the atlas E. None of the above

A

(B,D)

Chamberlain’s line extends from the hard palate to the posterior lip of the foramen magnum0 and is utilized primarily in the diagnosis of basilar impression. Basilar impression is occasionally seen in association with rheumatoid arthritis. B. In adults, the distance between the posterior edge of the odontoid process and the posterior arch of the atlas is normally no less than 17 mm in males and 15.8 mm in females and this distance is reduced in atlanto-axial subluxation. C. The base of the skull moves with the atlas and, therefore, this distance is also compromised in atlanto-axial subluxation. However, use of this measurement has not been standardized in the radiological literature. D. This distance is normally no greater than 2.5 mm and increases if the atlas slides forward with respect to the dens. If this distance is greater than 2.5 mm, it indicates atlantoaxial subluxation and radiographically is termed “increased atlanto-axial distance”.

How well did you know this?
1
Not at all
2
3
4
5
Perfectly
79
Q

CASE 15: Cervical Trauma in a Rheumatoid Arthritic HISTORY: A 59-year-old female was seen in the emergency room following an automobile accident, complaining of neck pain. She had a 32-year history of rheumatoid arthritis. She denied motor or sensory symptoms and had been able to urinate since the accident. She described mild intermittent upper cervical pain over several months prior to the accident. EXAMINATION: She had many of the stigmata of rheumatoid arthritis, including rheumatoid nodules and ulnar deviation of the fingers. The range of motion of her neck was limited in all directions but there was no tenderness. Her gait was limited by hip involvement by arthritic disease but was not spastic. On neurological examination, her motor and sensory functions were normal. Deep tendon reflexes in all extremities were slightly brisk, but she had no pathological reflexes. X-RAY: The lateral cervical spine in the emergency room suggested forward displacement of C1 on C2. AP and odontoid views showed no evidence of fracture. Lateral flexion and extension views were obtained and showed mobility of C I on C2 in anterior flexion indicating atlanto-axial subluxation (Figure 15. 1) QUESTIONS

Examination of the patient’s x-rays demonstrated a distance of 5 mm between the front of the dens and the anterior arch of the atlas and this distance increased by 2 mm with anterior flexion. 2. The etiology of this patient’s atlanto-axial subluxation is most likely (Select one or more)

A. related to trauma

B. related to rheumatoid arthritis and is caused primarily by laxity of the transverse odontoid ligament

C. related to rheumatoid arthritis and is caused primarily by erosion of the odontoid process

D. related to rheumatoid arthritis and is caused primarily by inflammation of the synovial joint between the odon toid and the anterior arch of the atlas

E. none of the above

A

(B)

Instability at the C1-C2 region associated with trauma is almost always due to fracture of the dens and would not be associated with an increased atlanto-axial distance. Laxity of the transverse ligament (Figure 15.2) initiates atlantoaxial dislocation although other factors contribute to instability at this level. Erosion of the odontoid process is commonly inferred if the atlanto-occipital separation exceeds 8 mm. Arthritic inflammatory changes are seen in the synovial joints of the cervicai spine and may be in the form of erosion or ankylosis. *pict*

How well did you know this?
1
Not at all
2
3
4
5
Perfectly
80
Q

CASE 15: Cervical Trauma in a Rheumatoid Arthritic HISTORY: A 59-year-old female was seen in the emergency room following an automobile accident, complaining of neck pain. She had a 32-year history of rheumatoid arthritis. She denied motor or sensory symptoms and had been able to urinate since the accident. She described mild intermittent upper cervical pain over several months prior to the accident. EXAMINATION: She had many of the stigmata of rheumatoid arthritis, including rheumatoid nodules and ulnar deviation of the fingers. The range of motion of her neck was limited in all directions but there was no tenderness. Her gait was limited by hip involvement by arthritic disease but was not spastic. On neurological examination, her motor and sensory functions were normal. Deep tendon reflexes in all extremities were slightly brisk, but she had no pathological reflexes. X-RAY: The lateral cervical spine in the emergency room suggested forward displacement of C1 on C2. AP and odontoid views showed no evidence of fracture. Lateral flexion and extension views were obtained and showed mobility of C I on C2 in anterior flexion indicating atlanto-axial subluxation (Figure 15. 1) QUESTIONS

  1. Atlanto-occipitai subluxation is also seen with rheumatoid arthritis. (True or False)
A

FALSE

Atlanto-occipital dislocation is rare and is almost always traumatic in origin.

How well did you know this?
1
Not at all
2
3
4
5
Perfectly
81
Q

CASE 15: Cervical Trauma in a Rheumatoid Arthritic HISTORY: A 59-year-old female was seen in the emergency room following an automobile accident, complaining of neck pain. She had a 32-year history of rheumatoid arthritis. She denied motor or sensory symptoms and had been able to urinate since the accident. She described mild intermittent upper cervical pain over several months prior to the accident. EXAMINATION: She had many of the stigmata of rheumatoid arthritis, including rheumatoid nodules and ulnar deviation of the fingers. The range of motion of her neck was limited in all directions but there was no tenderness. Her gait was limited by hip involvement by arthritic disease but was not spastic. On neurological examination, her motor and sensory functions were normal. Deep tendon reflexes in all extremities were slightly brisk, but she had no pathological reflexes. X-RAY: The lateral cervical spine in the emergency room suggested forward displacement of C1 on C2. AP and odontoid views showed no evidence of fracture. Lateral flexion and extension views were obtained and showed mobility of C I on C2 in anterior flexion indicating atlanto-axial subluxation (Figure 15. 1) QUESTIONS

  1. Subluxations at cervical levels below the C1-C2 level are also seen with rheumatoid arthritis. (True or False)
A

TRUE

In a series of 130 rheumatoid patients with cervical subluxation, 48% of the subluxations occurred at levels other than the C1-C2 level. Fifty-five percent of patients with subluxation associated with rheumatoid arthritis show multiple level involvement.

How well did you know this?
1
Not at all
2
3
4
5
Perfectly
82
Q

CASE 15: Cervical Trauma in a Rheumatoid Arthritic HISTORY: A 59-year-old female was seen in the emergency room following an automobile accident, complaining of neck pain. She had a 32-year history of rheumatoid arthritis. She denied motor or sensory symptoms and had been able to urinate since the accident. She described mild intermittent upper cervical pain over several months prior to the accident. EXAMINATION: She had many of the stigmata of rheumatoid arthritis, including rheumatoid nodules and ulnar deviation of the fingers. The range of motion of her neck was limited in all directions but there was no tenderness. Her gait was limited by hip involvement by arthritic disease but was not spastic. On neurological examination, her motor and sensory functions were normal. Deep tendon reflexes in all extremities were slightly brisk, but she had no pathological reflexes. X-RAY: The lateral cervical spine in the emergency room suggested forward displacement of C1 on C2. AP and odontoid views showed no evidence of fracture. Lateral flexion and extension views were obtained and showed mobility of C I on C2 in anterior flexion indicating atlanto-axial subluxation (Figure 15. 1) QUESTIONS

  1. The statement which most appropriately describes the consensus of opinion regarding treatment of this patient (Select only one)

A. the patient should be admitted to the hospital and undergo cervical fusion

B. the patient should be given a cervical collar and instructed to wear it the rest of her life

C. no treatment is needed for this patient

D. there is no general consensus of opinion concerning treatment for the patient

E. none of the above

A

(D)

While most authors do not mention asymptomatic atlanto-axial dislocation in association with rheumatoid arthritis as an indication for surgery, there are advocates of aggressive surgical management. Because of the potential for spinal cord compression if atlantoaxial subluxation progresses, some authors recommend treathing even symptomatic atlanto-axial subluxation with a cervical collar. One hundred thirty patients with cervical subluxations without evidence of spinal cord compression were followed for an average of 7,8 years without surgical treatment and only four patients developed myelopathic signs. Comparing this group of 150 patients to life tables of rheumatoid arthritis patients without cervical subluxation showed no evidence that life was shortened in this group. Finally, comparison of patients from this group who wore collars regularly with those who did not wear cervical collars regularly showed no effect on the progression of the severity of subluxation. Based on the above review, there does not appear to be a consensus among authors as to the appropriate therapeutic management of patients with asymptomatic atlanto-axial subluxation.

How well did you know this?
1
Not at all
2
3
4
5
Perfectly
83
Q

CASE 16: Persistent Spinal X-Ray Abnormality Associated with Spinal Cord Injury HISTORY: A 39-year-old man was seen in the emergency room complaining of neck pain following a fall down a flight of fifteen steps. He also complained of tingling paresthesias in the legs and diffuse weakness which were present immediately after the injury. EXAMINATION: .Vital Signs were within normal limits. Respirations were diaphragmatic. Shoulder flexion, extension, abduction and adduction were all performed normally. Elbow flexion was also normal. There was weakness graded 4/5 of wrist extension, elbow extension, grip and intrinsic musculature of both arms and diffuse weakness also graded 4 5 of trunk and lower extremity musculature. Muscle tone was increased in the legs and bilateral extensor plantar signs were present. There was hypalgesia to pinprick extending caudally from the index finger bilaterally. Sensation was preserved is the thumbs and radial aspect of both forearms and ab^ve. There was marked diminution of vibratory and proprioceptive sensation in the same distribution. X-RAYS: A lateral x-ray of the cervical spine illustrated in Figure 15. 1. QUESTIONS

  1. The clinical picture is that of (Select only one)

A. spinal cord transection at C6-7

B. partial spinal cord injury at C6-7

C. spinal cord transection at C5-6

D. partial spinal cord injury at C5-6

E. partial spinal cord injury at C4-5

A

(B)

The clinical picture is that of partial spinal cord injury at the C6-7 level. Structures innervated by motor and sensory portions of the C6 spinal segment and above remain functional. Thus, elbow flexion (biceps muscle) and sensory function of the radial aspect of the forearm and hand (C6 dermatome) are preserved. These functions are lost in C5-6 spinal cord injury. Structures innervated by the C7 spinal segment and below exhibit a diffuse, severe but incomplete sensorimotor deficit. Thus, elbow and wrist extension (triceps, extensor digitorum and extensor carpi muscles) are grossly impaired, as is sensation extending from, the index finger (C7 dermatome) distally. In spinal cord injury at the C4-5 level, weakness of shoulder abduction and sensory loss overlying the deltoid muscle (C5 dermatome) would be expected. The incomplete nature of the neurological loss indicates that neither anatomical nor physiological spinal cord transection has occurred. In addition, spinal shock is not present (see Case 5). The increased muscle tone, bilateral Babinski’s sign and hyperactive reflexes, including the triceps and lower extremity tendon reflexes with normal biceps reflex, also are indicative of incomplete spinal cord injury affecting the corticospinal pathways from the C7 segment caudally.

How well did you know this?
1
Not at all
2
3
4
5
Perfectly
84
Q

CASE 16: Persistent Spinal X-Ray Abnormality Associated with Spinal Cord Injury HISTORY: A 39-year-old man was seen in the emergency room complaining of neck pain following a fall down a flight of fifteen steps. He also complained of tingling paresthesias in the legs and diffuse weakness which were present immediately after the injury. EXAMINATION: .Vital Signs were within normal limits. Respirations were diaphragmatic. Shoulder flexion, extension, abduction and adduction were all performed normally. Elbow flexion was also normal. There was weakness graded 4/5 of wrist extension, elbow extension, grip and intrinsic musculature of both arms and diffuse weakness also graded 4 5 of trunk and lower extremity musculature. Muscle tone was increased in the legs and bilateral extensor plantar signs were present. There was hypalgesia to pinprick extending caudally from the index finger bilaterally. Sensation was preserved is the thumbs and radial aspect of both forearms and above. There was marked diminution of vibratory and proprioceptive sensation in the same distribution. X-RAYS: A lateral x-ray of the cervical spine illustrated in Figure 15.1. QUESTIONS

  1. The cervical spine x-ray shows (Select only one)

A. unilateral facet dislocation

B. fracture dislocation

C. bilateral facet dislocation

D. bilateral facet fracture

E. none of the above

A

(C)

There is marked persistent dislocation at the C6-7 level due to bilateral locked facets (Figure 16.1). No definite fracture is identified. This lesion is due to an acute flexion injury with complete disruption of the capsular ligaments of the facet joints bilaterally. The ligamenta flava, interspinous ligaments, paraspinal muscles and ligamentum nuchae are most likely also disrupted due to excessive stretching. The locked position of the facets prevents spontaneous reduction of the dislocation. Observe the normal relationship of facet joints at other levels, for instance C4-5 and C5-6. In the case of unilateral locked facets, the degree of dislocation seen is usually minimal and identification of the dislocation is difficult on the lateral x-ray due to the overlying shadow of the normal opposite facet. In addition, in the case illustrated the facets are clearly seen and no fracture of the facets is identified.

How well did you know this?
1
Not at all
2
3
4
5
Perfectly
85
Q

CASE 16: Persistent Spinal X-Ray Abnormality Associated with Spinal Cord Injury HISTORY: A 39-year-old man was seen in the emergency room complaining of neck pain following a fall down a flight of fifteen steps. He also complained of tingling paresthesias in the legs and diffuse weakness which were present immediately after the injury. EXAMINATION: .Vital Signs were within normal limits. Respirations were diaphragmatic. Shoulder flexion, extension, abduction and adduction were all performed normally. Elbow flexion was also normal. There was weakness graded 4/5 of wrist extension, elbow extension, grip and intrinsic musculature of both arms and diffuse weakness also graded 4 5 of trunk and lower extremity musculature. Muscle tone was increased in the legs and bilateral extensor plantar signs were present. There was hypalgesia to pinprick extending caudally from the index finger bilaterally. Sensation was preserved is the thumbs and radial aspect of both forearms and above. There was marked diminution of vibratory and proprioceptive sensation in the same distribution. X-RAYS: A lateral x-ray of the cervical spine illustrated in Figure 15.1. QUESTIONS

  1. The most important single mode of immediate treatment for this patient is (Select only one)

A. systemic corticosteroids

B. spinal cord hypothermia

C. decompressive laminectomy

D. surgical reduction

E. eduction and immobilization by skeletal traction

A

(E)

The value of prompt reduction of cervical spine fractures and dislocations by skeletal fraction cannot be overemphasized. This form of therapy offers the best method of internally decompressing the injured spinal cord and maintaining such decompression once appropriate positioning is achieved. Crutchfield in 1933 originally described tongs applied to the skull to provide effective cervical traction. Since then many other types of tongs, including modification of the original Crutchfield tongs, have been utilized. The authors prefer the Gardner-Wells tongs because they can be inserted very rapidly, require no incision or drilling, maintain constant pressure at the tong points due to spring loading, rarely pull out and withstand the very heavy weights necessary to reduce some difficult dislocations. Traction weights of 25-35 pounds or less will reduce most dislocations. With locked facets, however, much greater weight (up to 50-80 pounds) may be required. We have usually applied steadily increasing weight beginning at 20-30 pounds and increasing in increments of 5-10 pounds every 30-60 minutes to obtain reduction in 4-8 hours. Many others favor this approach and some even suggest slower reduction. Yashon has argued in favor of much more rapid reduction with rapid progression from initial weights of 20-30 pounds up to 70-80 pounds over 1-2 hours. Although everyone favors reduction as early as possible, we have found it difficult from a practical standpoint to achieve this, mainly since repeated cervical spine x-rays arc necessary after each weight addition to evaluate the progress of attempted reduction and to avoid excessive weight, which by distracting the injured spine excessively may result in aggravation of neurological deficit. If the method described above is unsuccessful, open operative reduction may be necessary. In our experience the need for open reduction is rare, particularly if one adopts a patient attitude and allows traction sufficient time to be effective. Although theoretically desirable, rapid closed reduction has not been clearly shown to be superior to slower reduction. The value of decompressive laminectomy in the treatment of acute cervical spinal cord injury is unknown. Many physicians with extensive experience, over many years, in the treatment of spinal cord injury, feel that laminectomy is rarely, if ever, indicated in such lesions. No controlled clinical study of the value of laminectomy has been carried out. Some clinical reports suggest that laminectomy may not only be lacking in efficacy in reducing neurological deficit after spinal cord injury but may actually reduce chances for recovery. Several reviews of this complex subject have been carried out. A number of laboratory studies have examined the value of various forms of surgical treatment of acute spinal cord injury. In most of these, laminectomy alone provides little benefit compared to untreated controls in terms of reduction of neurological deficit. The value of opening the dura in addition to the laminectomy has also been discussed clinically and examined in the laboratory setting. No definite conclusion has been reached but, overall, aural opening appears to add little to laminectomy alone in the treatment of spinal cord injury. Systemic corticosteroids and hypothermia represent recent additions to the armamentarium in the treatment of spinal cord injury. Their value at present is unproven (see complete discussion Case 32).

How well did you know this?
1
Not at all
2
3
4
5
Perfectly
86
Q

CASE 16: Persistent Spinal X-Ray Abnormality Associated with Spinal Cord Injury HISTORY: A 39-year-old man was seen in the emergency room complaining of neck pain following a fall down a flight of fifteen steps. He also complained of tingling paresthesias in the legs and diffuse weakness which were present immediately after the injury. EXAMINATION: .Vital Signs were within normal limits. Respirations were diaphragmatic. Shoulder flexion, extension, abduction and adduction were all performed normally. Elbow flexion was also normal. There was weakness graded 4/5 of wrist extension, elbow extension, grip and intrinsic musculature of both arms and diffuse weakness also graded 4 5 of trunk and lower extremity musculature. Muscle tone was increased in the legs and bilateral extensor plantar signs were present. There was hypalgesia to pinprick extending caudally from the index finger bilaterally. Sensation was preserved is the thumbs and radial aspect of both forearms and above. There was marked diminution of vibratory and proprioceptive sensation in the same distribution. X-RAYS: A lateral x-ray of the cervical spine illustrated in Figure 15.1. QUESTIONS

  1. Which of the following are strong indications for decompressive laminectomy in the treatment of spinal cord injury? (Select one or more)

A. An incomplete spinal cord lesion

B. Progressive neurological deficit

C. Spinal block on Queckenstedt test

D. Spinal block on myelogram

E. Central spinal cord syndrome

A

(B)

The indications for decompressive laminectomy in the treatment of spinal cord injury have been the subject of heated controversy. Most authors agree that progressive neurological deficit is an indication for surgical decompression. In addition, persistent cerebrospinal fluid leakage after penetrating injury is another indication which is reasonably agreed upon. Beyond this, however, there is a wide spectrum of approach to the utilization of decompressive laminectomy in the treatment of spinal cord injury. Young compared recovery after cervical spine injury in a group of patients treated at the Southwest Regional System for Treatment of Spinal Cord Injury in Phoenix, Arizona to a similar group cared for at the Spinal Cord Unit in Stoke-Mandeville, England. In the Phoenix series, 39% had undergone surgical procedures, whereas in the English series no patients were treated surgically. The operated patients in the Phoenix series showed less average neurological recovery (Ps feeling that immediate immobilization, adequate safe radiological examination and postural reduction of fracture-dislocation formed the cornerstone of the treatment of spinal cord injury. Some reports also describe long term instability following extensive cervical laminectomy. The authors agree basically with this later opinion, favoring laminectomy only in rarely seen, specific types of neurological problems limited primarily to progressive neurological loss after spinal cord injury and persistent cerebrospinal fluid leakage. The value of the demonstration of spinal block by either the Queckenstedt test or myelography is open to considerable controversy as well. In the Queckenstedt test, compression of the jugular veins is carried out in the neck after a lumbar puncture has been performed. The test is usually performed at the bedside with manual neck compression and casual observation of changes in spinal fluid pressure. As such, the results are unreliable. More standardized methods using neck compression by sphygmomanometer and careful recording of pressure changes make results more reliable. Nevertheless, in the presence of a spinal block in the cervical area, the test may be misleading because distended epidural veins beneath the lesion may still produce a normal response. In addition, the indications for the Queckenstedt test and myelography are nearly identical but the latter provides considerably more information. Both air and positive contrast agents have been used for myelography in the study of spinal cord injury. Demonstration of a block to passage of contrast by myelography has been taken by some to be an indication for laminectomy but the wisdom of this reasoning is very doubtful. Most serious spinal cord injuries with hemorrhage, edema and necrosis result in marked spinal cord enlargement often witn myelographic block. There is no evidence that decompressive laminectomy favorably affects the neurological recovery of such patients especially in regard to the results which may be achieved with nonoperative treatment.

How well did you know this?
1
Not at all
2
3
4
5
Perfectly
87
Q

CASE 16: Persistent Spinal X-Ray Abnormality Associated with Spinal Cord Injury HISTORY: A 39-year-old man was seen in the emergency room complaining of neck pain following a fall down a flight of fifteen steps. He also complained of tingling paresthesias in the legs and diffuse weakness which were present immediately after the injury. EXAMINATION: .Vital Signs were within normal limits. Respirations were diaphragmatic. Shoulder flexion, extension, abduction and adduction were all performed normally. Elbow flexion was also normal. There was weakness graded 4/5 of wrist extension, elbow extension, grip and intrinsic musculature of both arms and diffuse weakness also graded 4 5 of trunk and lower extremity musculature. Muscle tone was increased in the legs and bilateral extensor plantar signs were present. There was hypalgesia to pinprick extending caudally from the index finger bilaterally. Sensation was preserved is the thumbs and radial aspect of both forearms and above. There was marked diminution of vibratory and proprioceptive sensation in the same distribution. X-RAYS: A lateral x-ray of the cervical spine illustrated in Figure 15.1. QUESTIONS

  1. Which of the following might reasonably be considered for long terra treatment of this patient? (Select one or more)

A. Halo cast immobilization

B. Anterior cervical fusion

C. Posterior cervical fusion

D. None of the above

A

(A,B,C)

All three of these options should be considered. Halo cast immobilization provides excellent stabilization of fracture-dislocations once reduction has been achieved. It also allows relatively early mobilization of the patient, an important consideration in avoiding complications of prolonged immobility such as pneumonia, venous thrombosis, pulmonary embolism, etc. In utilizing prolonged immobilization with the halo brace or other methods, the normal healing processes must occur and provide natural stability to the spine. This tends to occur most readily when significant bony fracture occurs. Healing of fractured bone provides solid callus and then solid bone to insure stability in most cases. In the lesion illustrated in this case, however, the primary injury is disruption of the capsular ligaments of the facet joints. Healing of such ligamentous injuries is slow and unreliable probably due to the nature of the tissue involved and its relatively poor vascular supply. Thus, poor healing with early or late instability is more likely with primarily ligamentous as compared to primarily bony injury. Nevertheless, a trial of prolonged immobilization is not unreasonable. Stabilization via surgical fusion utilizing either an anterior or posterior approach might be considered on either an early or late basis in the patient present. The proponents of early fusion feel that such procedures provide a high likelihood of solid reliable stabilization of the spine and the possibility of early mobilization. The proponents of late fusion suggest that many fracture-dislocations will heal without surgical fusion. In fact, Burke found only a 4.2% incidence of failure to achieve stability in 175 patients with spinal injury. In their view, nearly all patients should have a period of 8-12 weeks of immobilization and only those who fail to obtain solid stable healing should be considered for surgical fusion. Our position represents a moderate approach between these extremes. If the character of the injury suggests that stable healing is likely, then immobilization in a halo brace seems appropriate. Such injuries have major elements of direct bony fracture and maintenance of the ligamentous structure of the disc space, the capsular ligaments of the facets or the posterior supporting ligaments and muscle. Injuries which are unlikely to heal with stability should be considered for early fusion. Such injuries represent primarily ligamentous injury and injuries to a combination of the major structures contributing to stability, i.e. intervertebral disc, facet joints and posterior elements. The injury in the patient presented would fall into the latter category since disruption of the disc space and posterior longitudinal ligaments, the capsular ligaments of the facets and the posterior supporting ligaments and muscles have all occurred.

How well did you know this?
1
Not at all
2
3
4
5
Perfectly
88
Q

CASE 16: Persistent Spinal X-Ray Abnormality Associated with Spinal Cord Injury HISTORY: A 39-year-old man was seen in the emergency room complaining of neck pain following a fall down a flight of fifteen steps. He also complained of tingling paresthesias in the legs and diffuse weakness which were present immediately after the injury. EXAMINATION: .Vital Signs were within normal limits. Respirations were diaphragmatic. Shoulder flexion, extension, abduction and adduction were all performed normally. Elbow flexion was also normal. There was weakness graded 4/5 of wrist extension, elbow extension, grip and intrinsic musculature of both arms and diffuse weakness also graded 4 5 of trunk and lower extremity musculature. Muscle tone was increased in the legs and bilateral extensor plantar signs were present. There was hypalgesia to pinprick extending caudally from the index finger bilaterally. Sensation was preserved is the thumbs and radial aspect of both forearms and above. There was marked diminution of vibratory and proprioceptive sensation in the same distribution. X-RAYS: A lateral x-ray of the cervical spine illustrated in Figure 15.1. QUESTIONS

  1. The pathophysiology of this patient’s spinal cord injury is most likely (Select one or more)

A. immediate spinal cord contusion at the time of injury

B. immediate contusion followed by central spinal cord necrosis

C. immediate contusion followed by peripheral spinal cord neurosis

D. steadily progressive spinal cord necrosis without initial contusion

E. anatomical complete transverse section of the spinal cord

A

(B)

The clinical picture of spinal cord injury suggests immediate direct spinal cord contusion. The deficit in the case illustrated and in most cases, occurs immediately at the time of trauma and progresses little if at all. For many years, it was felt that direct disruption of axons occurred in the spinal cord at the time of trauma and, therefore, attempts to treat the spinal cord injury itself were futile. Some authors currently espouse this theory. Studies of experimental spinal cord injury, however, clearly indicate two patterns of injury. First, a direct effect on spinal conduction with minimal irreversible tissue destruction. This is followed by progressive spinal cord necrosis which begins centrally and spreads toward the periphery of the injury and also spreads rostraliy and caudally within the spinal cord. The direct initial spinal cord concussion and resultant spinal cord malfunction likely mask the progressive spinal cord necrosis clinically. The necrosis is usually progressive ever a few hours post injury. Osterholm first postulated that catecholamines were released at the site of injury and led to vasoconstriction, ischemia and necrosis of spinal cord tissue. Osterholm demonstrated elevated levels of norepinephrine in experimentally injured spinal cord. Osterholm injected norepinephrine into the spinal cord of animals and found changes resembling those seen with experimental spinal cord injury. These changes were prevented by previous treatment with anticatecholamine drugs. Although some confirmation of Osterholm’s results were obtained many subsequent reports failed to observe the elevation in catecholamine levels which Osterholm described. Vise suggested that if catecholamine levels were elevated in injured spinal cord it likely was due to systemic release rather than local release. Whatever the responsible mechanism, it appears that ischemia occurs at the site of spinal cord injury. This idea was challenged by Kobrine who found hyperemia in lateral spinal cord white matter after spinal cord injury. In spite of the fact that ischemia is regularly demonstrated, the relationship between ischemia and spinal cord dysfunction in spinal cord injury is questionable. Thus, ischemic may be demonstrated when spinal cord function as measured by evoked potentials is intact. Further, vasodilators such as papaverine and sodium nitroprusside are ineffective in the treatment of experimental spinal cord injury. In addition to catecholamines, other toxic substances such as dopamine, free radicals, proteolytic enzymes released from lysosomes, potassium, 5-hydroxytrptamine, and histamine have all been suggested as responsible for progressive central spinal cord necrosis. Recent experiments, however, suggest that toxic substances, if released at the site of injury, may not be responsible for progressive spinal cord necrosis. Some have suggested that blood leaking from traumatized vessels and necrotic tissue may exert pressure on surrounding uninjured axons with recondary compression, further injury and perpetuation of central spinal cord necrosis and cavitation. The idea that myelotomy, that is, incision into the spinal cord, to release necrotic debris will reverse or prevent neurological injury after spinal cord injury is an old one. Some more recent experiments have suggested that myelotomy may be effective in reducing neurological deficit in animals after experimental spinal cord injury. Tator suggested that, although dorsal myelotomy appeared ineffective, complete midline myelotomy reduced neurological deficit. Little controlled clinical application of myelotomy has been carried out. A recent case report indicated myelotomy was ineffective in a single patient.

How well did you know this?
1
Not at all
2
3
4
5
Perfectly
89
Q

CASE 17: Difficulty Walking One Month After Neck Injury in a 47-Year-Old Policeman HISTORY: A 47-year-old policeman complained of numbness in his arms and legs and difficulty walking. Eight weeks previously he had injured his neck in a fail in which sudden neck flexion had occurred. Cervical spine x-rays at that time showed bilateral facet dislocation at C5-6. No neurological deficit was present at the time of injury. A posterior cervical fusion was carried out because the injury was felt to be unstable. The patient was discharged from the hospital four weeks following injury. At discharge his neurological examination was normal. He complained of mild but persistent neck pain. Four weeks after discharge he complained of more severe persistant aching pain at the base of his neck without radiation. He had noted mild tingling of both arms and legs which had increased progressively in severity for about one week. Also for a week he had noted difficulty in ambulation due to stiffness of his legs. EXAMINATION: General physical examination and vital signs were normal. The posterior cervical wound was well healed. His gait was marked by short steps and difficulty with relaxation of muscles in his legs. There was weakness of the biceps muscles bilaterally. Muscle strength was otherwise normal in all extremities. There was a marked increase in muscle tone in the legs. less so in the arms. Tendon reflexes were diffusely hyper active in both upper and lower extremities except for the biceps tendon reflexes which were absent bilaterally. Bilateral Babinski’s signs were present. Sensory examination revealed scattered hypalgesia to pinprick below C5 with normal vibratory and position sense throughout. X-RAYS: Cervical spine x-rays showed excellent alignment of the cervical spine. QUESTIONS

  1. The motor examination suggests (Select only one)

A. upper motor neuron lesion

B. lower motor neuron lesion

C. both of the above

D. neither of the above

A

(C)

Bilateral biceps weakness and absent biceps reflexes are indicative of a lower motor neuron lesion. The increased tone in the lower extremities, hyperactive reflexes and bilateral Babinski’s signs are indicative of an upper motor neuron lesion. The lower motor neuron includes the motor neuron located in the anterior horn of the spinal cord and its axonal extension into the peripheral nerve as far as the terminal endings at the myoneural junction. The upper motor neuron refers to nerve cells of the corticospinal tract. Cell bodies located in the posterior frontal cortex send their axons through the central cerebral white matter, the internal capsule, cerebral peduncles and pyramids to form the corticospinal tract of the spinal cord. These upper motor neurons end with synapses on the lower motor neuron in the ventral gray matter of the spinal cord.

How well did you know this?
1
Not at all
2
3
4
5
Perfectly
90
Q

CASE 17: Difficulty Walking One Month After Neck Injury in a 47-Year-Old Policeman HISTORY: A 47-year-old policeman complained of numbness in his arms and legs and difficulty walking. Eight weeks previously he had injured his neck in a fail in which sudden neck flexion had occurred. Cervical spine x-rays at that time showed bilateral facet dislocation at C5-6. No neurological deficit was present at the time of injury. A posterior cervical fusion was carried out because the injury was felt to be unstable. The patient was discharged from the hospital four weeks following injury. At discharge his neurological examination was normal. He complained of mild but persistent neck pain. Four weeks after discharge he complained of more severe persistant aching pain at the base of his neck without radiation. He had noted mild tingling of both arms and legs which had increased progressively in severity for about one week. Also for a week he had noted difficulty in ambulation due to stiffness of his legs. EXAMINATION: General physical examination and vital signs were normal. The posterior cervical wound was well healed. His gait was marked by short steps and difficulty with relaxation of muscles in his legs. There was weakness of the biceps muscles bilaterally. Muscle strength was otherwise normal in all extremities. There was a marked increase in muscle tone in the legs. less so in the arms. Tendon reflexes were diffusely hyper active in both upper and lower extremities except for the biceps tendon reflexes which were absent bilaterally. Bilateral Babinski’s signs were present. Sensory examination revealed scattered hypalgesia to pinprick below C5 with normal vibratory and position sense throughout. X-RAYS: Cervical spine x-rays showed excellent alignment of the cervical spine. QUESTIONS

  1. The patient’s gait disturbance is due to (Select only one)

A. spasticity

B. loss of position sense

C. weakness

D. lower motor neuron lesion

E. none of the above

A

(A)

The stiff gait and inability to relax suggest spasticity. The presence of increased muscle tone, hyperactive reflexes and bilateral Babinski’s signs confirm that spasticity is the basis of the patient’s gait disturbance. No specific muscle weakness is present in the legs. This is a frequent finding in gait spasticity, where minimal or no weakness is detectable on specific muscle testing of spastic limbs. Gait disturbances maybe due to loss of position sense but the presence of normal vibratory and position sense on examination exclude this possibility.

How well did you know this?
1
Not at all
2
3
4
5
Perfectly
91
Q

CASE 17: Difficulty Walking One Month After Neck Injury in a 47-Year-Old Policeman HISTORY: A 47-year-old policeman complained of numbness in his arms and legs and difficulty walking. Eight weeks previously he had injured his neck in a fail in which sudden neck flexion had occurred. Cervical spine x-rays at that time showed bilateral facet dislocation at C5-6. No neurological deficit was present at the time of injury. A posterior cervical fusion was carried out because the injury was felt to be unstable. The patient was discharged from the hospital four weeks following injury. At discharge his neurological examination was normal. He complained of mild but persistent neck pain. Four weeks after discharge he complained of more severe persistant aching pain at the base of his neck without radiation. He had noted mild tingling of both arms and legs which had increased progressively in severity for about one week. Also for a week he had noted difficulty in ambulation due to stiffness of his legs. EXAMINATION: General physical examination and vital signs were normal. The posterior cervical wound was well healed. His gait was marked by short steps and difficulty with relaxation of muscles in his legs. There was weakness of the biceps muscles bilaterally. Muscle strength was otherwise normal in all extremities. There was a marked increase in muscle tone in the legs. less so in the arms. Tendon reflexes were diffusely hyper active in both upper and lower extremities except for the biceps tendon reflexes which were absent bilaterally. Bilateral Babinski’s signs were present. Sensory examination revealed scattered hypalgesia to pinprick below C5 with normal vibratory and position sense throughout. X-RAYS: Cervical spine x-rays showed excellent alignment of the cervical spine. QUESTIONS

  1. Which of the following would you suggest for this patient at this time? (Select one or more)

A. A cervical collar

B. Cervical traction at home

C. Physical therapy

D. All of the above

E. None of the above

A

(E)

The history and physical examination suggest progressive spinal cord dysfunction. The measures suggested would be ineffective and could lead to a dangerous delay in pursuing the etiology of the spinal cord pathology.

How well did you know this?
1
Not at all
2
3
4
5
Perfectly
92
Q

CASE 17: Difficulty Walking One Month After Neck Injury in a 47-Year-Old Policeman HISTORY: A 47-year-old policeman complained of numbness in his arms and legs and difficulty walking. Eight weeks previously he had injured his neck in a fail in which sudden neck flexion had occurred. Cervical spine x-rays at that time showed bilateral facet dislocation at C5-6. No neurological deficit was present at the time of injury. A posterior cervical fusion was carried out because the injury was felt to be unstable. The patient was discharged from the hospital four weeks following injury. At discharge his neurological examination was normal. He complained of mild but persistent neck pain. Four weeks after discharge he complained of more severe persistant aching pain at the base of his neck without radiation. He had noted mild tingling of both arms and legs which had increased progressively in severity for about one week. Also for a week he had noted difficulty in ambulation due to stiffness of his legs. EXAMINATION: General physical examination and vital signs were normal. The posterior cervical wound was well healed. His gait was marked by short steps and difficulty with relaxation of muscles in his legs. There was weakness of the biceps muscles bilaterally. Muscle strength was otherwise normal in all extremities. There was a marked increase in muscle tone in the legs. less so in the arms. Tendon reflexes were diffusely hyper active in both upper and lower extremities except for the biceps tendon reflexes which were absent bilaterally. Bilateral Babinski’s signs were present. Sensory examination revealed scattered hypalgesia to pinprick below C5 with normal vibratory and position sense throughout. X-RAYS: Cervical spine x-rays showed excellent alignment of the cervical spine. QUESTIONS

  1. What is the most likely diagnosis? (Select only one)

A. Chronic cervical sprain

B. Herniated cervical disc

C. Spinal hematoma

D. Spinal core tumor

E. None of the above

A

(B)

The presence of conclusive signs of cervical cord dysfunction rule out a diagnosis of chronic cervical sprain. In cervical sprain, signs and symptoms are limited to neck pain and muscle spasm and are characterized by normal neurological examination. The findings could be due to spinal hematoma but the long delay since the time of injury make it unlikely (see Cases 27 and 28). Spinal cord tumor is also unlikely, although not impossible, based on the history and neurological examination. In spinal cord tumor, the history is usually slow and steadily progressive. Nevertheless, on occasion, insidious symptoms are not apparent to the patient until an episode of trauma focuses attention on neurological disability. The most likely diagnosis is that of herniated cervical disc. The patient has experienced neck pain without radiation since the time of the injury. This suggests a central disc herniation since radiating pain into the extremities which is characteristic of lateral disc herniation is absent from the patient’s history. The clinical picture suggests that the spinal cord dysfunction begins at the C5-6 level based on the bilateral weakness of the biceps muscles and absence of biceps reflexes. Consequently, a herniated cervical disc at the C5-6 interspace is likely.

How well did you know this?
1
Not at all
2
3
4
5
Perfectly
93
Q

CASE 17: Difficulty Walking One Month After Neck Injury in a 47-Year-Old Policeman HISTORY: A 47-year-old policeman complained of numbness in his arms and legs and difficulty walking. Eight weeks previously he had injured his neck in a fail in which sudden neck flexion had occurred. Cervical spine x-rays at that time showed bilateral facet dislocation at C5-6. No neurological deficit was present at the time of injury. A posterior cervical fusion was carried out because the injury was felt to be unstable. The patient was discharged from the hospital four weeks following injury. At discharge his neurological examination was normal. He complained of mild but persistent neck pain. Four weeks after discharge he complained of more severe persistant aching pain at the base of his neck without radiation. He had noted mild tingling of both arms and legs which had increased progressively in severity for about one week. Also for a week he had noted difficulty in ambulation due to stiffness of his legs. EXAMINATION: General physical examination and vital signs were normal. The posterior cervical wound was well healed. His gait was marked by short steps and difficulty with relaxation of muscles in his legs. There was weakness of the biceps muscles bilaterally. Muscle strength was otherwise normal in all extremities. There was a marked increase in muscle tone in the legs. less so in the arms. Tendon reflexes were diffusely hyper active in both upper and lower extremities except for the biceps tendon reflexes which were absent bilaterally. Bilateral Babinski’s signs were present. Sensory examination revealed scattered hypalgesia to pinprick below C5 with normal vibratory and position sense throughout. X-RAYS: Cervical spine x-rays showed excellent alignment of the cervical spine. QUESTIONS

  1. Which of the following would be the most definitive laboratory study in this patient? (Select only one)

A. Electromyogram

B. Myelogram

C. Flexion-extension x-rays of the cervicai spine

D. Diagnostic nerve block

E. None of toe above

A

(B)

A cervical myelogram is indicated. This could be carried out by instillation of radio-opaque contrast material into the lumbar subarachnoid space or by instillation of air into the cisterna magna. Flexion-extension views of the cervical spine are probably contraindicated at this point. The patient has had a recent fracture-dislocation and posterior cervical fusion. Sufficient time has not passed to expect solid bony union and flexion-extension movements could result in further spinal cord injury. Electromyography is unlikely to add significant information in this patient. The level of spinal cord dysfunction is apparent from the clinical examination and the history and examination do not suggest either peripheral neuropathy or muscle disease. The electromyogram is particularly useful in the diagnosis of diseases of peripheral nerve, the myoneural junction and muscle. Diagnostic nerve block would be of no value in understanding this patient’s problem.

How well did you know this?
1
Not at all
2
3
4
5
Perfectly
94
Q

CASE 17: Difficulty Walking One Month After Neck Injury in a 47-Year-Old Policeman HISTORY: A 47-year-old policeman complained of numbness in his arms and legs and difficulty walking. Eight weeks previously he had injured his neck in a fail in which sudden neck flexion had occurred. Cervical spine x-rays at that time showed bilateral facet dislocation at C5-6. No neurological deficit was present at the time of injury. A posterior cervical fusion was carried out because the injury was felt to be unstable. The patient was discharged from the hospital four weeks following injury. At discharge his neurological examination was normal. He complained of mild but persistent neck pain. Four weeks after discharge he complained of more severe persistant aching pain at the base of his neck without radiation. He had noted mild tingling of both arms and legs which had increased progressively in severity for about one week. Also for a week he had noted difficulty in ambulation due to stiffness of his legs. EXAMINATION: General physical examination and vital signs were normal. The posterior cervical wound was well healed. His gait was marked by short steps and difficulty with relaxation of muscles in his legs. There was weakness of the biceps muscles bilaterally. Muscle strength was otherwise normal in all extremities. There was a marked increase in muscle tone in the legs. less so in the arms. Tendon reflexes were diffusely hyper active in both upper and lower extremities except for the biceps tendon reflexes which were absent bilaterally. Bilateral Babinski’s signs were present. Sensory examination revealed scattered hypalgesia to pinprick below C5 with normal vibratory and position sense throughout. X-RAYS: Cervical spine x-rays showed excellent alignment of the cervical spine. QUESTIONS

  1. An air myelogram was later performed in this patient (Figure 17. 1). The study shows (Select only one)

A. a diffuse extradural defect

B. an intramedullary defect

C. an intradural extramedullary defect

D. a localized extradural defect

E. none of the above

A

(D)

The myelogram shows a localized extradural defect opposite the C5-6 interspace.

How well did you know this?
1
Not at all
2
3
4
5
Perfectly
95
Q

CASE 17: Difficulty Walking One Month After Neck Injury in a 47-Year-Old Policeman HISTORY: A 47-year-old policeman complained of numbness in his arms and legs and difficulty walking. Eight weeks previously he had injured his neck in a fail in which sudden neck flexion had occurred. Cervical spine x-rays at that time showed bilateral facet dislocation at C5-6. No neurological deficit was present at the time of injury. A posterior cervical fusion was carried out because the injury was felt to be unstable. The patient was discharged from the hospital four weeks following injury. At discharge his neurological examination was normal. He complained of mild but persistent neck pain. Four weeks after discharge he complained of more severe persistant aching pain at the base of his neck without radiation. He had noted mild tingling of both arms and legs which had increased progressively in severity for about one week. Also for a week he had noted difficulty in ambulation due to stiffness of his legs. EXAMINATION: General physical examination and vital signs were normal. The posterior cervical wound was well healed. His gait was marked by short steps and difficulty with relaxation of muscles in his legs. There was weakness of the biceps muscles bilaterally. Muscle strength was otherwise normal in all extremities. There was a marked increase in muscle tone in the legs. less so in the arms. Tendon reflexes were diffusely hyper active in both upper and lower extremities except for the biceps tendon reflexes which were absent bilaterally. Bilateral Babinski’s signs were present. Sensory examination revealed scattered hypalgesia to pinprick below C5 with normal vibratory and position sense throughout. X-RAYS: Cervical spine x-rays showed excellent alignment of the cervical spine. QUESTIONS

  1. The myelogram is most compatible with (Select only one)

A. metastatic spinal tumor

B. spinal meningioma

C. central cervical disc herniation

D. spinal neurofibroma

E. none of the above

A

(C)

A localized extradural defect lying directly opposite an intervertebral disc space is most compatible with a cervical disc herniation. Spinal meningioma and spinal neurofibroma are intradural, extramedullary tumors which would not produce the type of myelographic defect seen in this patient. Metastatic spinal tumors produce extradural defects but these are most commonly more extensive and are rarely located opposite the disc space. Spinal metastatic tumors most often begin by a metastatic deposit in bone with secondary extension into the extradural space. Thus, the defect in the case of spinal metastatic tumors most often overlies the vertebral body. In addition, pathologic changes frequently occur In bone in association with metastatic spinal tumors.

How well did you know this?
1
Not at all
2
3
4
5
Perfectly
96
Q

CASE 17: Difficulty Walking One Month After Neck Injury in a 47-Year-Old Policeman HISTORY: A 47-year-old policeman complained of numbness in his arms and legs and difficulty walking. Eight weeks previously he had injured his neck in a fail in which sudden neck flexion had occurred. Cervical spine x-rays at that time showed bilateral facet dislocation at C5-6. No neurological deficit was present at the time of injury. A posterior cervical fusion was carried out because the injury was felt to be unstable. The patient was discharged from the hospital four weeks following injury. At discharge his neurological examination was normal. He complained of mild but persistent neck pain. Four weeks after discharge he complained of more severe persistant aching pain at the base of his neck without radiation. He had noted mild tingling of both arms and legs which had increased progressively in severity for about one week. Also for a week he had noted difficulty in ambulation due to stiffness of his legs. EXAMINATION: General physical examination and vital signs were normal. The posterior cervical wound was well healed. His gait was marked by short steps and difficulty with relaxation of muscles in his legs. There was weakness of the biceps muscles bilaterally. Muscle strength was otherwise normal in all extremities. There was a marked increase in muscle tone in the legs. less so in the arms. Tendon reflexes were diffusely hyper active in both upper and lower extremities except for the biceps tendon reflexes which were absent bilaterally. Bilateral Babinski’s signs were present. Sensory examination revealed scattered hypalgesia to pinprick below C5 with normal vibratory and position sense throughout. X-RAYS: Cervical spine x-rays showed excellent alignment of the cervical spine. QUESTIONS

  1. Which of the following treatments would you suggest (Select only one)

A. Cervical traction

B. Decompressive cervical laminectomy

C. Anterior cervical disc excision

D. Radiation therapy

E. Hone of the above

A

(C)

Anterior cervical disc excision is most appropriate. This procedure allows removal of the offending herniated nuclear disc fragment without a disturbance of spinal cord position. Opening the posterior longitudinal ligament allows thorough inspection of the extradural space for the presence of extradural herniated disc fragments. Decompressive cervical laminectomy would allow some backward movement of the spinal cord and perhaps reduce the spinal cord compression somewhat. It does not, however, treat the underlying pathology direcuy. Removal of a centrally herniated cervical disc via a decompressive laminectomy is difficult in the cervical regionbecause the presence of the spinal cord makes anterior exploration dangerous. Attempts to explore the anterior extradural space in the cervical region to remove a central herniated disc increase the likelihood of irreversible spinal cord injury. Below the level of L1, an anterior central herniated intervertebral disc may be removed transdurally because the fibers of the cauda equina may be separated allowing anterior exposure safely. Radiation therapy is contraindicated in this situation since the most likely diagnosis is cervical disc herniation which is not treatable by radiotherapy. Even in patients in whom a diagnosis of spinal metastatic tumor is entertained, based on the history and myelographic findings, radiation therapy is not recommended as a definitive treatment unless the patient has a previous tissue diagnosis of malignancy. Cervical traction would not be beneficial in this patient since the myelogram makes it obvious that actual herniation of disc material has occurred into the extradural space. Utilization of cervical traction to provide symptomatic treatment prior to a surgical procedure would be acceptable but is unlikely to provide more than minimal symptomatic relief of neck discomfort.

How well did you know this?
1
Not at all
2
3
4
5
Perfectly
97
Q

CASE 18: Neck and Arm Pain and Arm Weakness After Motor Vehicle Accident. HISTORY: A 27-year-old man was seen in the emergency room 45 minutes following an automobile accident. The patient was a passenger in the front seat of a vehicle which struck another vehicle at about 35 miles per hour. The patient was thrown forward and struck his occipital region on the dashboard. His neck was acutely flexed forward and to the right. He was not unconscious but had the immediate onset of pain in the neck and right shoulder. Subsequently, he noted that pain extended into the right hand. He also complained of weakness of the right arm. EXAMINATION: He was awake and alert. There was a 2 cm laceration in the right posterior-parietal region of the scalp. His neck was tender to palpation particularly over the spinous processes of C6 and C7. Motor examination revealed marked weakness of extension of the right elbow and weakness of extension of the. right wrist and fingers. Otherwise, motor function was normal. Tendon reflexes were all 2+ except for the right triceps reflex which was absent. There was a deficit in pinprick sensation along the ulnar aspect of the right forearm including the right middle and ring fingers. Minimal contact with the skin of this region elicited a painful, burning sensation. QUESTIONS

  1. The anatomical localization of the neurological deficit is most likely the (Select only one)

A. spinal cord at C6-7

B. C6 nerve root

C. C7 nerve root

D. C8 nerve root

E. brachial plexus

A

Tidak ada jawaban dan pembahasan

How well did you know this?
1
Not at all
2
3
4
5
Perfectly
98
Q

CASE 18: Neck and Arm Pain and Arm Weakness After Motor Vehicle Accident. HISTORY: A 27-year-old man was seen in the emergency room 45 minutes following an automobile accident. The patient was a passenger in the front seat of a vehicle which struck another vehicle at about 35 miles per hour. The patient was thrown forward and struck his occipital region on the dashboard. His neck was acutely flexed forward and to the right. He was not unconscious but had the immediate onset of pain in the neck and right shoulder. Subsequently, he noted that pain extended into the right hand. He also complained of weakness of the right arm. EXAMINATION: He was awake and alert. There was a 2 cm laceration in the right posterior-parietal region of the scalp. His neck was tender to palpation particularly over the spinous processes of C6 and C7. Motor examination revealed marked weakness of extension of the right elbow and weakness of extension of the. right wrist and fingers. Otherwise, motor function was normal. Tendon reflexes were all 2+ except for the right triceps reflex which was absent. There was a deficit in pinprick sensation along the ulnar aspect of the right forearm including the right middle and ring fingers. Minimal contact with the skin of this region elicited a painful, burning sensation. QUESTIONS

  1. Which of the following are likely pathological mechanisms to explain the neurological deficit? (Select one or more)

A. Spinal cord contusion

B. Acute disc herniation

C. Direct nerve root injury

D. Epidural hematoma

E. Spinal cord infarction

A

Tidak ada jawaban dan pembahasan

How well did you know this?
1
Not at all
2
3
4
5
Perfectly
99
Q

CASE 18: Neck and Arm Pain and Arm Weakness After Motor Vehicle Accident. HISTORY: A 27-year-old man was seen in the emergency room 45 minutes following an automobile accident. The patient was a passenger in the front seat of a vehicle which struck another vehicle at about 35 miles per hour. The patient was thrown forward and struck his occipital region on the dashboard. His neck was acutely flexed forward and to the right. He was not unconscious but had the immediate onset of pain in the neck and right shoulder. Subsequently, he noted that pain extended into the right hand. He also complained of weakness of the right arm. EXAMINATION: He was awake and alert. There was a 2 cm laceration in the right posterior-parietal region of the scalp. His neck was tender to palpation particularly over the spinous processes of C6 and C7. Motor examination revealed marked weakness of extension of the right elbow and weakness of extension of the. right wrist and fingers. Otherwise, motor function was normal. Tendon reflexes were all 2+ except for the right triceps reflex which was absent. There was a deficit in pinprick sensation along the ulnar aspect of the right forearm including the right middle and ring fingers. Minimal contact with the skin of this region elicited a painful, burning sensation. QUESTIONS

  1. Cervical spine x-rays from this patient are shown in Figures 18.1 and 18.2. The findings are best described as (Select only one)

A. normal

B. bilateral facet dislocation and locking

C. fracture-dislocation

D. unilateral facet dislocation and locking

E. none of the above

A

Tidak ada jawaban dan pembahasan

How well did you know this?
1
Not at all
2
3
4
5
Perfectly
100
Q

CASE 18: Neck and Arm Pain and Arm Weakness After Motor Vehicle Accident. HISTORY: A 27-year-old man was seen in the emergency room 45 minutes following an automobile accident. The patient was a passenger in the front seat of a vehicle which struck another vehicle at about 35 miles per hour. The patient was thrown forward and struck his occipital region on the dashboard. His neck was acutely flexed forward and to the right. He was not unconscious but had the immediate onset of pain in the neck and right shoulder. Subsequently, he noted that pain extended into the right hand. He also complained of weakness of the right arm. EXAMINATION: He was awake and alert. There was a 2 cm laceration in the right posterior-parietal region of the scalp. His neck was tender to palpation particularly over the spinous processes of C6 and C7. Motor examination revealed marked weakness of extension of the right elbow and weakness of extension of the. right wrist and fingers. Otherwise, motor function was normal. Tendon reflexes were all 2+ except for the right triceps reflex which was absent. There was a deficit in pinprick sensation along the ulnar aspect of the right forearm including the right middle and ring fingers. Minimal contact with the skin of this region elicited a painful, burning sensation. QUESTIONS

  1. Which of the following are true concerning unilateral facet dislocation and locking? (Select one or more)

A. Associated only with nerve root injury

B. May be associated with incomplete spinal cord injury

C. May be associated with complete spinal cord injury

D. May occur without neurological deficit

E. None of the above

A

(B,C,D)

Although nerve roots are the commonest neurological structure injured in unilateral facet dislocations, other structures are also frequently damaged. Braakman and Penning indicated that 10% of their patients with unilateral facet dislocations had neurological evidence of complete spinal cord injury 24% had various forms of incomplete spinal cord lesions and another 24% had no neurological signs. The remaining 42% manifested evidence of one or more nerve root lesions ipsilateral to the side of facet dislocation.

How well did you know this?
1
Not at all
2
3
4
5
Perfectly
101
Q

CASE 18: Neck and Arm Pain and Arm Weakness After Motor Vehicle Accident. HISTORY: A 27-year-old man was seen in the emergency room 45 minutes following an automobile accident. The patient was a passenger in the front seat of a vehicle which struck another vehicle at about 35 miles per hour. The patient was thrown forward and struck his occipital region on the dashboard. His neck was acutely flexed forward and to the right. He was not unconscious but had the immediate onset of pain in the neck and right shoulder. Subsequently, he noted that pain extended into the right hand. He also complained of weakness of the right arm. EXAMINATION: He was awake and alert. There was a 2 cm laceration in the right posterior-parietal region of the scalp. His neck was tender to palpation particularly over the spinous processes of C6 and C7. Motor examination revealed marked weakness of extension of the right elbow and weakness of extension of the. right wrist and fingers. Otherwise, motor function was normal. Tendon reflexes were all 2+ except for the right triceps reflex which was absent. There was a deficit in pinprick sensation along the ulnar aspect of the right forearm including the right middle and ring fingers. Minimal contact with the skin of this region elicited a painful, burning sensation. QUESTIONS

  1. Which of the following might be useful in the further evaluation of this patient? (Select one or more)

A. Spinal puncture

B. Queckenstedt test

C. Polytomography

D. Electromyography

E. Flexion and extension views

A

(C)

Polytomography may be very useful in accurate diagnosis of unilateral facet dislocations. A polytomographic view from the patient described is illustrated in Figure 18.4. The abnormal position of the C7 superior facet (single crossed arrow) placed posterior to the inferior facet of C6 (plain arrow) is clearly demonstrated. The marked compromise of the C6-7 intervertebral foramen is also evident (double crossed arrow) and results from persistent rotation of C7 in relation to C6. Such foraminal compromise results in nerve root injury. The normal relationship of articular facets and the configuration of the intervertebral foramina are well seen at the C5-6 level. In general, polytomography should be carried out to carefully evaluate spinal fractures, particularly in cases where bony injury does not seem sufficient to explain symptomatology. In cases of unilateral facet dislocation, poly tomography often shows associated facet or transverse process fractures. Flexion and extension views in general are hazardous. Only if bony injury or dislocation have been excluded by appropriate plain x-rays and polytomography should flexion and extension views be carried out. These must be supervised by a qualified physician and should not be left to the radiological technician to carry out since risk of spinal cord injury is present with neck movement. Spinal puncture and the Queckenstedt test are of no use in the evaluation of the patient presented. The character of the spinal fluid and the presence or absence of subarachnoid block on the Queckenstedt maneuver are useless in determining the appropriate course of action in spinal injury. Not all authors, however, agree with this view. Electromyography (EMG) is of little value on an acute basis for the diagnosis of neuromuscular disease. Evidence of denervation takes several weeks to develop and at that time the EMG may be useful in the differential diagnosis of spinal injuries. The EMG is most helpful in providing a prognosis in root injuries but may be of some value in decisions on delayed surgical approach to brachial plexus lesions associated with spinal trauma.

How well did you know this?
1
Not at all
2
3
4
5
Perfectly
102
Q

CASE 19: Motor and Sensory Loss in One Arm after Injurv in a 37-Year-Old Man HISTORY: A 37-year-old man. was seen in the emergency room complaining of monoplegia of the left arm. One hour prior to admission, he was a spectator at an automobile race in which a race car lost one of its tires. The tire rolled along the race track, jumped a guard rail and protective fence and struck the patient in the left side of the neck, across the left clavicle and left side of the chest. He was knocked unconscious for 3-4 minutes. On awakening he complained of severe left shoulder pain and paralysis and numbness of the entire left arm. EXAMINATION: Vital signs were stable and the patient was conversant. The tire tread design was tatooed on the skin of the anterior aspect of the left shoulder. The gross appearance of the left arm was normal. There was marked swelling about the left clavicle. Breath sounds were decreased in the left lung field. A left Horner’s syndrome was noted. Neurological examination disclosed flaccid paralysis of the entire left arm including the deltoid muscle. There was total anesthesia of the arm beginning at the skin over the deltoid muscle and sparing only a tiny area along the medial aspect of the upper arm near the axilla. Tendon reflexes were absent in the left arm. The remainder of the neurological examination was normal. Strength reflexes and sensation were normal in the other extremities. Sphincter function was normal. No pathological reflexes were noted. QUESTIONS

  1. The most likely pathological process to explain the neurological findings is (Select only one)

A. central spinal cord syndrome

B hemisection of the spinal cord

C. avulsion of the roots of the left brachial plexus

D. spinal cord contusion

E. subclavian artery occlusion

A

(C)

Neurological findings are confined to the left arm and include motor and sensory loss in regions innervated by spinal segments C5-T1. These are the segments which compose the brachial plexus. Flaccid motor paralysis and sensory loss confined to one arm are rare with cervical spinal cord injury alone. Combined injuries to the spinal cord and brachial plexus may present problems in clinical diagnosis due to the coexistence of neurological deficits related to the two lesions. In these circumstances, the brachial plexus injury may be overlooked for some time. In spinal cord injury, such as the central spinal cord syndrome, hemisection of the spinal cord or diffuse spinal cord contusion, neurological findings are usually present in the lower extremities and trunk and may include both upper extremities. Sphincter dysfunction is also common. The presence of swelling about the shoulder, tatooing of the skin about the clavicle and the history of the mechanism of injury all suggest brachial plexus rather than spinal cord injury. Although subclavian artery injury commonly occurs with brachial plexus injury, it does not usually produce complete motor and sensory loss. In addition, absence of pulses in the extremity and pallor, neither of which were present in this patient, suggest subclavian or axillary artery injury. Traumatic aneurysms occurring after arterial injury may cause delayed brachial plexus neuropathy due to direct compression of the plexus.

How well did you know this?
1
Not at all
2
3
4
5
Perfectly
103
Q

CASE 19: Motor and Sensory Loss in One Arm after Injurv in a 37-Year-Old Man HISTORY: A 37-year-old man. was seen in the emergency room complaining of monoplegia of the left arm. One hour prior to admission, he was a spectator at an automobile race in which a race car lost one of its tires. The tire rolled along the race track, jumped a guard rail and protective fence and struck the patient in the left side of the neck, across the left clavicle and left side of the chest. He was knocked unconscious for 3-4 minutes. On awakening he complained of severe left shoulder pain and paralysis and numbness of the entire left arm. EXAMINATION: Vital signs were stable and the patient was conversant. The tire tread design was tatooed on the skin of the anterior aspect of the left shoulder. The gross appearance of the left arm was normal. There was marked swelling about the left clavicle. Breath sounds were decreased in the left lung field. A left Horner’s syndrome was noted. Neurological examination disclosed flaccid paralysis of the entire left arm including the deltoid muscle. There was total anesthesia of the arm beginning at the skin over the deltoid muscle and sparing only a tiny area along the medial aspect of the upper arm near the axilla. Tendon reflexes were absent in the left arm. The remainder of the neurological examination was normal. Strength reflexes and sensation were normal in the other extremities. Sphincter function was normal. No pathological reflexes were noted. QUESTIONS

  1. Horner’s syndrome in this patient is due to injury to (Select only one)

A. spinal cord

B. cervical sympathetic trunk

C. C8 and T1 nerve roots

D. parasympathetic pathways

E. none of the above

A

(C)

Horner’s syndrome consists of ptosis, meiosis, and anhydrosis of the face and is due to dysfunction of the sympathetic pathways. The sympathetic pathways descend through the cervical spinal cord and synapse with cells in the intermediolateral cell column of the spinal cord at the first thoracic segment. Preganglionic fibers exit from the spinal canal with the C8 and T1 nerve roots and ascend to make synaptic connections in the cervical sympathetic ganglia. Postganglionic fibers then pass to the pupillodilator muscle, the tarsal muscle of the upper eyelid and sweat glands in the face and scalp. The clinical picture in this patient is most compatible with avulsion of the nerve roots of the brachial plexus and thus avulsion of the C8 and T1 nerve roots most likely explains the Horner’s syndrome in this patient. Injuries to the cervical sympathetic trunk may also cause Horner’s syndrome but are frequently related to penetrating neck wounds. Horner’s syndrome may also be due to spinal cord injury above the T1 level and in such cases is most often bilateral. However, in hemispinal cord injuries, the Horner’s syndrome may be unilateral.

How well did you know this?
1
Not at all
2
3
4
5
Perfectly
104
Q

CASE 19: Motor and Sensory Loss in One Arm after Injurv in a 37-Year-Old Man HISTORY: A 37-year-old man. was seen in the emergency room complaining of monoplegia of the left arm. One hour prior to admission, he was a spectator at an automobile race in which a race car lost one of its tires. The tire rolled along the race track, jumped a guard rail and protective fence and struck the patient in the left side of the neck, across the left clavicle and left side of the chest. He was knocked unconscious for 3-4 minutes. On awakening he complained of severe left shoulder pain and paralysis and numbness of the entire left arm. EXAMINATION: Vital signs were stable and the patient was conversant. The tire tread design was tatooed on the skin of the anterior aspect of the left shoulder. The gross appearance of the left arm was normal. There was marked swelling about the left clavicle. Breath sounds were decreased in the left lung field. A left Horner’s syndrome was noted. Neurological examination disclosed flaccid paralysis of the entire left arm including the deltoid muscle. There was total anesthesia of the arm beginning at the skin over the deltoid muscle and sparing only a tiny area along the medial aspect of the upper arm near the axilla. Tendon reflexes were absent in the left arm. The remainder of the neurological examination was normal. Strength reflexes and sensation were normal in the other extremities. Sphincter function was normal. No pathological reflexes were noted. QUESTIONS

  1. Appropriate diagnostic studies should include (Select one or more)

A. chest x-ray

B. x-rays of the left shoulder

C. electromyogram

D. cervical spine x-rays

E. none of the above

A

(A,B,C,D)

Chest x-ray may reveal several associated and even life threatening injuries such as fracture of the clavicle or upper ribs, pneumothorax, hemothorax or pulmonary injury. Chest x-ray in this patient revealed an elevated left hemidiaphragm suggesting unilateral phrenic nerve paralysis. X-rays of the shoulder provide analysis of associated bony injury to the clavicle, ribs, scapula and humerus. Cervical spine x-rays are also indicated to evaluate possible associated cervicle spine trauma. Avulsion of cervical transverse processes, if identified on cervical spine x-rays, may be of prognostic value since they are nearly pathognomonic of avulsion of the roots of the brachial plexus. The electromvogram (EMG) is most useful several weeks after injury when signs of denervation appear in affected muscles. These include excessive insertionai activity and fibrillation potentials. Sensory evoked potential studies may also be helpful in deciding whether the brachial plexus injury is proximal or distal to the dorsal root ganglia. The latter contain the cell bodies of the sensory neurons, the proximal processes of which cuter the spinal cord and the distal processes of which travel to the extremity. Such a distinction between so-called postganglionic and preganglionic lesions is of great importance. In preganglionic injuries, i.e. those affecting the dorsal roots between the spinal cord and the sensory ganglion cells, recovery is impossible. Postganglionic injuries, on the other hand, may show recovery. EMG examination of the paraspinal muscles may also be useful in distinguishing preganglionic lesions from postganglionic. If denervation of paraspiual muscles is demonstrated, the lesion is preganglionic The posterior primary divisions of the ventral roots which innervate the paraspinal muscles leave the nerves just proximal to the location of the dorsal root ganglia and are only affected by preganglionic lesions. Occasionally the EMG may be useful a few days after injury in distinguishing functional disruption of neural conduction, so called neurapraxia, from an anatomic disruption of the nerve fibers or nerve, called axonotmesis and neurotmesis respectively. In neurapraxia, the evoked muscle response to electrical stimulation of the plexus proximal to the point of injury may be preserved in spite of absent or poor volitional activity. The evoked muscle response is diminished or absent in neurotmesis or axonotmesis.

How well did you know this?
1
Not at all
2
3
4
5
Perfectly
105
Q

CASE 19: Motor and Sensory Loss in One Arm after Injurv in a 37-Year-Old Man HISTORY: A 37-year-old man. was seen in the emergency room complaining of monoplegia of the left arm. One hour prior to admission, he was a spectator at an automobile race in which a race car lost one of its tires. The tire rolled along the race track, jumped a guard rail and protective fence and struck the patient in the left side of the neck, across the left clavicle and left side of the chest. He was knocked unconscious for 3-4 minutes. On awakening he complained of severe left shoulder pain and paralysis and numbness of the entire left arm. EXAMINATION: Vital signs were stable and the patient was conversant. The tire tread design was tatooed on the skin of the anterior aspect of the left shoulder. The gross appearance of the left arm was normal. There was marked swelling about the left clavicle. Breath sounds were decreased in the left lung field. A left Horner’s syndrome was noted. Neurological examination disclosed flaccid paralysis of the entire left arm including the deltoid muscle. There was total anesthesia of the arm beginning at the skin over the deltoid muscle and sparing only a tiny area along the medial aspect of the upper arm near the axilla. Tendon reflexes were absent in the left arm. The remainder of the neurological examination was normal. Strength reflexes and sensation were normal in the other extremities. Sphincter function was normal. No pathological reflexes were noted. QUESTIONS

  1. A myelogram done in this patient is shown in It shows (Select only one)

A. a herniated intervertebral disc

B. benign nerve root cysts

C. is normal

D. an extradural hematoma

E. multiple nerve root avulsions

A

(E)

Extradural pockets of contrast material are demonstrated on the myelogram at C6-7. C7-T1 and T1-2 (Figure 19.1). These are consistent with avulsion of the C7, C8 and T1 nerve roots. The subarachnoid space usually terminates at the point where dorsal and ventral roots join just proximal to the dorsal root ganglia. With root avulsion the subarachnoid space is breached and contrast material leaks into small pseudomeningoceles. In Figure 19.2, contrast material is retained in the pseudomeningoceles when the patient is placed in the upright position. Such radiological findings correlate with a preganglionic location of the lesion and suggest a hopeless prognosis. The results of myelography must be interpreted with caution, however, in establishing a prognosis in brachial plexus injuries, since false-negative and false-positive results have been reported. A herniated intervertebral disc or extradural hematoma would produce appropriate defects in the contrast column or obstruction to flow of contrast material. In addition, while small benign nerve root cysts may occur normally, primarily in the lumbar region, the size and location of the pseudomeningoceles demonstrated, coupled with the clinical picture, are diagnostic of nerve root avulsions. *pict*

How well did you know this?
1
Not at all
2
3
4
5
Perfectly
106
Q

CASE 19: Motor and Sensory Loss in One Arm after Injurv in a 37-Year-Old Man HISTORY: A 37-year-old man. was seen in the emergency room complaining of monoplegia of the left arm. One hour prior to admission, he was a spectator at an automobile race in which a race car lost one of its tires. The tire rolled along the race track, jumped a guard rail and protective fence and struck the patient in the left side of the neck, across the left clavicle and left side of the chest. He was knocked unconscious for 3-4 minutes. On awakening he complained of severe left shoulder pain and paralysis and numbness of the entire left arm. EXAMINATION: Vital signs were stable and the patient was conversant. The tire tread design was tatooed on the skin of the anterior aspect of the left shoulder. The gross appearance of the left arm was normal. There was marked swelling about the left clavicle. Breath sounds were decreased in the left lung field. A left Horner’s syndrome was noted. Neurological examination disclosed flaccid paralysis of the entire left arm including the deltoid muscle. There was total anesthesia of the arm beginning at the skin over the deltoid muscle and sparing only a tiny area along the medial aspect of the upper arm near the axilla. Tendon reflexes were absent in the left arm. The remainder of the neurological examination was normal. Strength reflexes and sensation were normal in the other extremities. Sphincter function was normal. No pathological reflexes were noted. QUESTIONS

  1. Which of the following are true concerning prognosis in this patient? (Select one or more)

A. Recovery of motor function is likely

B. Recovery of motor function is unlikely

C. Recovery of sensory function is likely

D. Recovery of sensory function is unlikely

E. Intractable pain is likely

A

(B,D,E)

The patient described has clinical and myelographic evidence of a preganglionic injury of a traction type. The clinical evidence is the presence of Horner’s syndrome and physical examination and x-ray evidence of a paralyzed hemidiaphragrn. The myelographic evidence of a preganglionic lesion is the presence of multiple pseudomeningoceles at the cites of nerve root avulsions. In addition, the patient has a closed brachial plexus injury. Such lesions result from stretching or concussive blows to the plexus and produce widespread destruction along the length of nerves and nerve roots. Even when such closed injuries occur in a postganglionic location the prognosis for recovery is poor. Intractable pain is an unfortunate and frequent accompaniment of brachial plexus injury. Treatment of such pain is difficult and often unsuccessful and frequently results in depression and narcotic addiction. Amputation of the useless extremity in an attempt to relieve such pain is also usually unsuccessful. Surgical treatment may be useful in assisting recovery if upper brachial plexus injuries result from penetrating trauma such as knife or gunshot wounds. In such cases, nerve anastomosis or cable grafting of missing segments of nerve may allow it; innervation of proximal shoulder and upper arm muscles. Even in cases of penetrating wounds, with a more localized type of injury, the prognosis for recovery of function for muscles in the forearm and hand is poor. This is due to the excessively long distance the regenerating nerve must travel from the point of injury to the distally located muscle. Such nerve re-growth occurs at a rate of about 1 mm per day. Thus, even if nerve regrowth occurs, irreversible muscle atrophy and fibrosis occurs before reinnervation is accomplished. It is for this reason that many surgeons are pessimistic about improvements in function due to brachial plexus injury with surgical treatment. Microsurgical suture techniques may improve the ability or regenerating nerves to transverse zones of injury but cannot overcome the problem of distal muscle atrophy and fibrosis.

How well did you know this?
1
Not at all
2
3
4
5
Perfectly
107
Q

CASE 20: Persistent Neck Pain and Muscle Spasm after Ice Hockey Injury HISTORY: A 19-year-old male was seen 48 hours following injury in an ice hockey game. He was apparently knocked to the ice by an opponent who fell on top of him. The side of his head struck the ice and his neck was forced acutely into right lateral flexion. Ke noted immediate pain in the right shoulder and right side of the neck. He was seen because of persistent pain in the neck, particularly the right side. The shoulder pain had subsided. EXAMINATION: There was mild muscle spasm on the right side of the neck at rest. Neck motion, however, was full with some aggravation of pain on right lateral neck flexion or rotation. The neurological examination was entirely normal. X-RAYS: An anteroposterior (AP) cervical spine x-ray is illustrated in Figure 20.1. A lateral view appeared normal. QUESTIONS

  1. The AP lateral cervical spine x-ray shows (Select only one)

A. normal findings

B. fracture-dislocation

C. nondisplaced odontoid fracture

D. asymmetry of the atlanto-axial relationship

E. Jefferson’s fracture

A

(D)

There is asymmetry in the atlanto-axial relationship which is only appreciated on the anteroposterior view (Figure 20.1). The distance between the lateral margin of odontoid process and the medial edge of the articular mass of the atlas is asymmetric. The term “rotational subluxation^’ or “rotaryfixation” has been applied to this condition in the past with the view that it represents a pathological condition due to trauma. Although there is not currently full agreement, most writers, including the present authors, feel that the condition rarely represents true pathology. Braakman has discussed this subject fully with excellent illustrations. Recently computerized tomographic scanning has been effectively utilized to evaluate such asymmetry. No fractures or significant dislocations are shown on the x-rays illustrated.

How well did you know this?
1
Not at all
2
3
4
5
Perfectly
108
Q

CASE 20: Persistent Neck Pain and Muscle Spasm after Ice Hockey Injury HISTORY: A 19-year-old male was seen 48 hours following injury in an ice hockey game. He was apparently knocked to the ice by an opponent who fell on top of him. The side of his head struck the ice and his neck was forced acutely into right lateral flexion. Ke noted immediate pain in the right shoulder and right side of the neck. He was seen because of persistent pain in the neck, particularly the right side. The shoulder pain had subsided. EXAMINATION: There was mild muscle spasm on the right side of the neck at rest. Neck motion, however, was full with some aggravation of pain on right lateral neck flexion or rotation. The neurological examination was entirely normal. X-RAYS: An anteroposterior (AP) cervical spine x-ray is illustrated in Figure 20.1. A lateral view appeared normal. QUESTIONS

  1. The most likely cause of these x-ray findings is (Select only one)

A. fracture of the atlas

B. fracture of the axis

C. injury to facet joint ligaments

D. muscle spasm

A

(D) The most frequent cause of unilateral asymmetry of the atlanto-axial relationship on x-ray is rotation of the neck due to head tilt and muscle spasm. Hohl and Baker indicated that lateral atlanto-axial movement was possible only when the atlanto-axial joints are in a position of rotation. Braakman showed that on lateral head flexion the atlas moves laterally while, at the same time, atlanto-axial rotation occurs. Therefore, lateral displacement of the atlas on the axis should not automatically be interpreted as a subluxation. Tomography, when abduction and rotation are present at the atlanto-axial joint, may also be misinterpreted. Particularly with anteroposterior tomograms, both lateral joints are not seen in the same plane. Thus, one side may be sharp and the other poorly defined, leading to diagnosis of pathological conditions such as arthritis. In Jefferson’s fracture of the atlas, there is a bursting separation of the atlas ring (see Case 6). Therefore, the anteroposterior view usually shows bilateral increase in distance between the odontoid and lateral mass of the atlas. In addition, there is lateral displacement of the lateral edge of the articular mass of the atlas on the axis. This is not seen with so-called “rotational subluxation”. Bilateral atlanto-axial separation may also occur in certain congenital circumstances as described by Budin. In fractures of the odontoid, displacement is usually in the anteroposterior plane and best appreciated on the lateral view. In addition, the fracture itself is usually visible on the anteroposterior view and no such lesion is seen in the x-ray illustrated. Considerable emphasis has been placed on ligamentous injury as a cause of “rotational subluxation”. Anatomical studies, however, indicate that asymmetry of the atlanto-axial relationship may occur with simple abduction and rotation of the upper spine in the presence of normal ligaments. It seems clear that most cases of asymmetry of the atlanto-axial relationship are due to positioning of the spine at the time of radiological studies. Computerized tomographic scanning, however, confirms that in some cases true “rotational fixation” may be present.

How well did you know this?
1
Not at all
2
3
4
5
Perfectly
109
Q

CASE 20: Persistent Neck Pain and Muscle Spasm after Ice Hockey Injury HISTORY: A 19-year-old male was seen 48 hours following injury in an ice hockey game. He was apparently knocked to the ice by an opponent who fell on top of him. The side of his head struck the ice and his neck was forced acutely into right lateral flexion. Ke noted immediate pain in the right shoulder and right side of the neck. He was seen because of persistent pain in the neck, particularly the right side. The shoulder pain had subsided. EXAMINATION: There was mild muscle spasm on the right side of the neck at rest. Neck motion, however, was full with some aggravation of pain on right lateral neck flexion or rotation. The neurological examination was entirely normal. X-RAYS: An anteroposterior (AP) cervical spine x-ray is illustrated in Figure 20.1. A lateral view appeared normal. QUESTIONS

  1. Which of the following are true concerning atlanto-axial asymmetry identified radiologically ? (Select one or more)

A. Conclusive proof of cervical trauma

B. Occurs only with ligamentous rupture

C. May be due to congenital odontoid asymmetry

D. May be associated with congenital facial asymmetry

A

(C,D) Brocher described asymmetrical implantation of the odontoid process into the body of the axis which simulated lateral displacement of the atlas. Facial asymmetry with asymmetrical relationships in the occipitocervical region were also reported by Wackenheim. Ligamentous rupture is not necessary in. order to allow lateral shift of the atlas on the axis. This abduction movement can occur with rotation in the presence of normal ligaments. Brav also reported a patient who could voluntarily cause lateral atlanto-axial displacement. Several authors have indicated that an asymmetrical atlantoaxial relationship need not be taken as proof of significant spinal trauma.

How well did you know this?
1
Not at all
2
3
4
5
Perfectly
110
Q

CASE 20: Persistent Neck Pain and Muscle Spasm after Ice Hockey Injury HISTORY: A 19-year-old male was seen 48 hours following injury in an ice hockey game. He was apparently knocked to the ice by an opponent who fell on top of him. The side of his head struck the ice and his neck was forced acutely into right lateral flexion. Ke noted immediate pain in the right shoulder and right side of the neck. He was seen because of persistent pain in the neck, particularly the right side. The shoulder pain had subsided. EXAMINATION: There was mild muscle spasm on the right side of the neck at rest. Neck motion, however, was full with some aggravation of pain on right lateral neck flexion or rotation. The neurological examination was entirely normal. X-RAYS: An anteroposterior (AP) cervical spine x-ray is illustrated in Figure 20.1. A lateral view appeared normal. QUESTIONS

  1. Which of the following treatments would you recommend for the patient illustrated? (Select only one)

A. Skeletal traction

B. Open reduction

C. Halo brace

D. Cervical collar

E. Spinal fusion

A

(D) Specific treatment of atlanto-axial asymmetry is not required. Attempts at reduction either manually, by halter or skeletal traction are unnecessary. General treatment of cervical muscle spasm and torticollis by rest or cervical collar, mild physical therapy, analgesics and muscle relaxants, depending on the severity of symptoms, is appropriate. The present authors have had universal success with this treatment. Early in our experience, follow-up cervical spine x-rays were obtained and always showed resolution of the asymmetry. Presently, if symptoms of pain and muscle spasm subside, followup x-rays are not felt to be necessary. The halo brace is not required as these lesions are not unstable. Operative procedures such as open reduction and spinal fusion are completely inappropriate.

How well did you know this?
1
Not at all
2
3
4
5
Perfectly
111
Q

CASE 20: Persistent Neck Pain and Muscle Spasm after Ice Hockey Injury HISTORY: A 19-year-old male was seen 48 hours following injury in an ice hockey game. He was apparently knocked to the ice by an opponent who fell on top of him. The side of his head struck the ice and his neck was forced acutely into right lateral flexion. Ke noted immediate pain in the right shoulder and right side of the neck. He was seen because of persistent pain in the neck, particularly the right side. The shoulder pain had subsided. EXAMINATION: There was mild muscle spasm on the right side of the neck at rest. Neck motion, however, was full with some aggravation of pain on right lateral neck flexion or rotation. The neurological examination was entirely normal. X-RAYS: An anteroposterior (AP) cervical spine x-ray is illustrated in Figure 20.1. A lateral view appeared normal. QUESTIONS

  1. The risk of spinal cord injury in this patient is (Select only one)

A. very high

B. minimal

C. impossible to predict

A

(B) Spinal cord injury has not been reported with so called “rotatory subluxation” of the atlanto-axial region. It is essential to bear in mind that the important aspect of spinal injury relates to the integrity of neural structures, i.e., the spinal cord and nerve roots. Excessive emphasis on the bony aspects of spinal injury are unjustified unless they are related to the associated neural structures. Naturally, the possibility that untreated bony injury may lead to delayed neurological injury must also be kept in mind. In the condition described in this case, there is no evidence of related neurological injury and there is no evidence of longterm instability or delayed neurological deterioration. Thus, treatment, if any, should be conservative and as noninvasive as possible. Unnecessary attempts at reduction and/or immobilization are ill advised.

How well did you know this?
1
Not at all
2
3
4
5
Perfectly
112
Q

CASE 21: Spinal Cord injury in a 4-Year-Old Boy Following Automobile Accident HISTORY: A 4-year-old male was seen in the emergency room thirty minutes following an automobile, accident. The child was riding oh his mother’s lap in the front seat when the slowly moving vehicle struck a stopped truck in front. The child was thrown forward against the dashboard. The exact mechanism of injury was unknown but the forces involved appeared to be minimal. No one else was injured. The child was awake and crying. EXAMINATION: BP 110/70, P 92r R 32. There was tenderness to palpation over the spinous processes of C3 and C4. The child was moving all extremities but not as briskly as expected. Specific motor examination could not be carried out because the child would not cooperate fully. He could stand and walk slowly, he could not run. Tendon reflexes were abnormally weak in both upper and lower extremities. Plantar beware flexor. Sensation to pinprick appeared normal. LABORATORY DATA: Complete blood count and urinalysis is normal. QUESTIONS

  1. The cervical spine x-ray from this patient is illustrated in Figure 21.1. It shows (Select only one) normal findings for age

A. Normal findings for age

B. subluxation C2-3

C. fracture-dislocation

D. facet dislocation

E. odontoid fracture

A

(A) There is apparent subluxation at the C2-3 interspace, the problem of excessive movement at the C2-3 interspace in children and the differentiation between physiological movement of pathological subluxation has been discussed by several authors. In our experience, true pathological subluxation at C2-3 is rare in children and most apparent subluxations are due to normal movement at the joint. A lateral program in this patient (Figure 21.2) showed no evidence of bony injury and confirmed normal alignment in the neutral position. Dunlap was one of the first to emphasize the normal appearance of subluxation at C2-3 in children, particularly reduceable on the lateral x-ray view of the cervical spine in the flexion. It was suggested that persistent subluxation C2-3 in the neutral or hyperexteaded position would difference pathological from physiological movement at C2-3 in children. This physiological subluxation may cause difficulty in diagnosis of the Hangman’s fracture at C2-3 as well, since anterior displacement of the body of C2 on C3 may occur with bilateral fracture of the pedicles of C2. The incidence of such phvsiological subluxation in children ages 1-17 years has been estimated at 24%. Cattell indicated that the high mobility of the cervical spine and elasticity of facet joint ligaments coupled with the relatively horizontal position of the C2-3 articular facts in children accounted for this excessive movement. In addition, the C2-3 region was felt to be an important fulcrum of cervical movement and such physiological subluxation was maximum there. The relatively poor development of paraspinal muscles in infants and young children may also contribute an excessive spinal mobility. Our review of cervical spine x-rays in children indicates that smaller degrees of physiological subluxation are often found at C3-4 and even C4-5. Although one or two millimeters of anterior subluxation of C2 and or C3 or C4 is considered abnormal in adults, up to 4 mm may be considered within the range of normal for children up eight years of age.

How well did you know this?
1
Not at all
2
3
4
5
Perfectly
113
Q

CASE 21: Spinal Cord injury in a 4-Year-Old Boy Following Automobile Accident HISTORY: A 4-year-old male was seen in the emergency room thirty minutes following an automobile, accident. The child was riding oh his mother’s lap in the front seat when the slowly moving vehicle struck a stopped truck in front. The child was thrown forward against the dashboard. The exact mechanism of injury was unknown but the forces involved appeared to be minimal. No one else was injured. The child was awake and crying. EXAMINATION: BP 110/70, P 92r R 32. There was tenderness to palpation over the spinous processes of C3 and C4. The child was moving all extremities but not as briskly as expected. Specific motor examination could not be carried out because the child would not cooperate fully. He could stand and walk slowly, he could not run. Tendon reflexes were abnormally weak in both upper and lower extremities. Plantar beware flexor. Sensation to pinprick appeared normal. LABORATORY DATA: Complete blood count and urinalysis is normal. QUESTIONS

  1. Which of the following is the most likely diagnosis in this patient? (Select only one)

A. Normal child, no significant injury

B. Spinal cord contusion

C. Epidural hematoma

D. Acute herniated intervertebral disc

E. Subdural hematoma

A

(D) The neurological examination is abnormal. There is induced extremity movement and use. In addition, the reflexes are diffusely brisk and muscle tone is increased in the lower extrimities. The child is unabie to run. These findings are strongly suggestive of abnormal motor function. There are no specific sensory abnormalities detectable. The difficulty in carrying test neurological evaluation in young children with spinal injury is discussed by Yashon. Because cooperation for detailed motor and sensory examination is often unobtainable, the examiner must often rely on observation of spontaneous motor activity or reflex responses such as withdrawal or crying in response to noxious stimuli. These signs are easily misinterpreted. For instance, withdrawal of extremities in response to noxious stimuli may represent local spinal reflex activity and may not be perceived by the patient as painful due to spinal cord dysfunction at a higher evel. Even crying in conjunction with motor withdrawal may not reliably indicate stimulus perception, as pain may be initiated above the level of a spinal cord injury by reflex motor withdrawal below the lesion. Nevertheless, in the patient presented, the motor examination supports spinal cord pathology. The absence of extensor plantar responses does not exclude motor system pathology. Epidural or subdural hematoma are unlikely due to the minimal nature of the patient’s symptoms and signs and the apparent lack of progression of neurological deficit often seen with these lesions. An acutely herniated intervertebral disc is an extremely unusual injury in children with spinal trauma and the patient does not manifest the anterior spinal cord injury syndrome which may be seen with acute disc herniation secondary to spinal trauma. Although intraspinal hematoma and intervertebral disc herniation cannot be completely ruled out they are unlikely as the cause of the mild neurological deficit identified in the patient described. Also to be considered n children with spine or spinal cord injuries is child abuse. Most commonly these are seen in children without a history of trauma. Considerable emphasis has been placed on long bone fractures and head injuries in battered children but a significant number also have spinal trauma according to Cullen. Since none of the above pathological conditions seems appropriate a mild spinal cord contusion appears most likely. The neurological deficit related to spinal cord contusion may range from very mild to very severe. A static lesion with neurological deficit as described is most compatible with spinal cord contusion.

How well did you know this?
1
Not at all
2
3
4
5
Perfectly
114
Q

CASE 21: Spinal Cord injury in a 4-Year-Old Boy Following Automobile Accident HISTORY: A 4-year-old male was seen in the emergency room thirty minutes following an automobile, accident. The child was riding oh his mother’s lap in the front seat when the slowly moving vehicle struck a stopped truck in front. The child was thrown forward against the dashboard. The exact mechanism of injury was unknown but the forces involved appeared to be minimal. No one else was injured. The child was awake and crying. EXAMINATION: BP 110/70, P 92r R 32. There was tenderness to palpation over the spinous processes of C3 and C4. The child was moving all extremities but not as briskly as expected. Specific motor examination could not be carried out because the child would not cooperate fully. He could stand and walk slowly, he could not run. Tendon reflexes were abnormally weak in both upper and lower extremities. Plantar beware flexor. Sensation to pinprick appeared normal. LABORATORY DATA: Complete blood count and urinalysis is normal. QUESTIONS

  1. Which of the following are true concerning spinal injuries in children or infants who are passengers in motor vehicles? (Select one or more)

A. Children are safest when held on lap of parent

B. Children arc safer in the rear than in the front seat

C. The incidence of childhood spinal injury could be reduced by a restraining system

D. Infants are safest in a bassinet placed on the seat but otherwise unrestrained

E. Infants are safest in a bassinet placed on the seat and restrained by seat belts

A

(B,C,E) Spinal injuries are frequently sustained by children who are passengers in motor vehicles. Although the child held en its parent’s lap is psychologically the most secure position from the parent’s point of view. it exposes the infant or child to significant risk of injury in case of collision. On impact, the child may be thrown free in the vehicle to strike solid objects such as the dashboard as in the case illustrated or may be crushed between the parent’s body and the automobile. Spinal injuries resulting from such trauma.occur most commonly in the cervical or upper thoracic areas. Many injuries in children who are passengers in motor vehicle accidents apparently could be prevented by proper restraining systems. Appropriate car seats are available for older children and it appears that the rear seat is safer than the front. For infants, the bassinet is safest when placed in the rear seat its long axis parallel to the long axis of the automobile, utilizing the seat belt around the bassinet legs to secure it solidly to the seat. The child should be placed feet forward and a net should be placed over the bassinet to prevent the child from being thrown out and into contact with the inside of the vehicle in case of collision. The idea that passengers in vehicles are safest without restraints is absolutely false. Fears of injured victims being unable to escape from vehicles and sustaining fatal burns in case of explosion or combusion of fuel are, in the main, unfounded. The safest place is within the vehicle, well restrained with lap and shoulder harness for adults and large children. Appropriate car seats or properly restrained bassinet are safest for children.

How well did you know this?
1
Not at all
2
3
4
5
Perfectly
115
Q

CASE 21: Spinal Cord injury in a 4-Year-Old Boy Following Automobile Accident HISTORY: A 4-year-old male was seen in the emergency room thirty minutes following an automobile, accident. The child was riding oh his mother’s lap in the front seat when the slowly moving vehicle struck a stopped truck in front. The child was thrown forward against the dashboard. The exact mechanism of injury was unknown but the forces involved appeared to be minimal. No one else was injured. The child was awake and crying. EXAMINATION: BP 110/70, P 92r R 32. There was tenderness to palpation over the spinous processes of C3 and C4. The child was moving all extremities but not as briskly as expected. Specific motor examination could not be carried out because the child would not cooperate fully. He could stand and walk slowly, he could not run. Tendon reflexes were abnormally weak in both upper and lower extremities. Plantar beware flexor. Sensation to pinprick appeared normal. LABORATORY DATA: Complete blood count and urinalysis is normal. QUESTIONS

  1. Re-examination of the child upon completion of x-ray studies showed marked weakness of both arms and legs. The child could no longer walk or stand alone and no spontaneous arm movement was noted. There was apparent reduced pinprick perception below the C4 dermatomal level. Bilateral extensor plantar responses were noted. Which of the following might explain this neurological deterioration? (Select one or more)

A. Spinal cord infarction

B. Progressive spinal cord necrosis

C. Epidural hematoma

D. Subdural hematoma

E. None of the above

A

(A,B,C,D) The neurological examination is now indicative of progressive deterioration. The child is quadriparetic with marked reduction in upper extremity movement and inability to stand. Bilateral Babinski’s signs are now present. In addition, there is a sensory level at C4. These are all suggestive of severe spinal cord dysfunction at the upper cervical level. Any of the lesions listed could be responsible for progressive rapid neurological deterioration. Spinal cord infarction after spinal trauma has recently received attention as the cause of neurological deterioration in children. Ahmann described two children, ages 22 months and four years, who sustained relatively minor spinal trauma with minimal initial neurological deficit. In both patients, severe neurological deterioration leading to quadriplegia, respiratory insufficiency, and death occurred over a several hour period post injury. Autopsy examination in both patients showed extensive ischemic necrosis extending from C3-T1 in one patient and from the medulla to C5 in the other. Lenn reported a similar patient with progressive spinal cord dysfunction after spinal trauma of a relatively mild degree which involved the thoracic spinal cord in a 10-year-old child. Pathological confirmation was not obtained in Lenn’s patient because she continued to survive at the time of this report. Ahmann reviewed the vascular supply of the spinal cord in search of a potential etiology for delayed spinal cord infarction after spinal trauma. Neither of his patients showed spinal artery occlusion at autopsy. In addition, the pattern of infarction did not suggest venous occlusion since no evidence of congestion or hemorrhage was seen. Ahmann suggested anomalies in spinal cord vascular supply and arterial vasoplasm as other possibilities. Lenn suggested that the two regions of spinal cord involved with ischemic infarction, namely cervical and thoracic, were in watershed areas of vascular supply. Ahmann also considered a combination of reduced vascular supply due to the watershed phenomenon as well as selective vulnerability of central spinal cord gray matter is responsible for spinal cord infarction. Both epidural and subdural spinal hematomas may occur following spinal trauma and although uncommon in children, are not excluded when spinal fracture is not identified (see Cases 27 and 28). Progressive spinal cord necrosis usually occurs within four hours of trauma and is usually associated with more severe initial spinal cord injury. Nevertheless, progressive spiral cord necrosis is a possibility

How well did you know this?
1
Not at all
2
3
4
5
Perfectly
116
Q

CASE 21: Spinal Cord injury in a 4-Year-Old Boy Following Automobile Accident HISTORY: A 4-year-old male was seen in the emergency room thirty minutes following an automobile, accident. The child was riding oh his mother’s lap in the front seat when the slowly moving vehicle struck a stopped truck in front. The child was thrown forward against the dashboard. The exact mechanism of injury was unknown but the forces involved appeared to be minimal. No one else was injured. The child was awake and crying. EXAMINATION: BP 110/70, P 92r R 32. There was tenderness to palpation over the spinous processes of C3 and C4. The child was moving all extremities but not as briskly as expected. Specific motor examination could not be carried out because the child would not cooperate fully. He could stand and walk slowly, he could not run. Tendon reflexes were abnormally weak in both upper and lower extremities. Plantar beware flexor. Sensation to pinprick appeared normal. LABORATORY DATA: Complete blood count and urinalysis is normal. QUESTIONS

  1. The appropriate next step should be (Select only one)

A. lumbar puncture

B. Queckenstedt test

C. myelogram

D. immediate laminectomy

E. none of the above

A

(C) There is general agreement that progressive spinal cord deficit requires prompt evaluation and consideration for surgical treatment. In order to ascertain if a surgically remediable lesion is present and to establish its localization, myelography is required. In the patient presented, spinal hematoma or intervertebral disc herniation, although unlikely, could account for progressive neurological deterioration and appropriate means to exclude such lesions should be pursued. Lumbar puncture alone, even combined with the Queckenstedt test of spinal dynamics, is unlikely to yield useful information as to the presence of a surgically treatable lesion. The Queckenstedt test may be positive with or without a surgically treatable lesion and it may be negative with or without such a lesion. Myelography in children with spinal cord infarction following minor trauma has been normal. The prognosis in such patients is not well established but recovery of neurological function was not noted in any of the three reported cases and two of three expired within a short time after injury. If a normal myelogram is demonstrated, surgical treatment is not indicated. Supportive care including attention to respiratory and urinary systems and skin in particular is important in such children.

117
Q

CASE 21: Spinal Cord injury in a 4-Year-Old Boy Following Automobile Accident HISTORY: A 4-year-old male was seen in the emergency room thirty minutes following an automobile, accident. The child was riding oh his mother’s lap in the front seat when the slowly moving vehicle struck a stopped truck in front. The child was thrown forward against the dashboard. The exact mechanism of injury was unknown but the forces involved appeared to be minimal. No one else was injured. The child was awake and crying. EXAMINATION: BP 110/70, P 92r R 32. There was tenderness to palpation over the spinous processes of C3 and C4. The child was moving all extremities but not as briskly as expected. Specific motor examination could not be carried out because the child would not cooperate fully. He could stand and walk slowly, he could not run. Tendon reflexes were abnormally weak in both upper and lower extremities. Plantar beware flexor. Sensation to pinprick appeared normal. LABORATORY DATA: Complete blood count and urinalysis is normal. QUESTIONS

  1. Which of the following are appropriate treatments for cervical fracture-dislocation in the pediatric age group? (Select only one)

A. Reduction by skeletal traction

B. Decompressive laminectomy

C. Spinal fusion

D. Open reduction and internal fixation

E. None of the above

A

(A) The indications for surgical treatment are not clearly established for spinal fracture-dislocations occurring in the pediatric age group. Skeletal traction for reduction of fracture-dislocations is agreed to be the desirable first step, although because of the thin nature of the skull in childhood, special means to establish traction may be required. Some authors indicated a high incidence of scoliosis in children following spinal injury and suggested operative stabilization. Scoliosis is also reported as a delayed occurrence six months after injury in girls under twelve years or boys under fourteen years with paraplegia or quadriplegia. Burke indicated that scoliosis occurred with or without laminectomy in children with spinal injury. We have generally pursued a conservative course in children with spinal cord injuries whether or not fracture-dislocation is identified. Prompt reduction utilizing skeletal traction has been employed if fracture-dislocation is present. Unless marked instability due primarily or exclusively to ligamentous injury is identified, a period of 8-12 weeks of immobilization is utilized. If persistent instability is subsequently demonstrated, then a stabilization procedure is recommended. Children have a considerably greater tendency to stable healing of spinal fractures than do adults. This is well illustrated in the case of odontoid fractures which heal well in children with immobilization. Contrary to some authors, we have not utilized laminectomy in the treatment of spinal cord injury in children unless a progressive spinal cord deficit is identified clinically and a surgically treatable lesion such as an extradural or subdural hematoma is identified by myelography.

118
Q

CASE 22: Respiratory Distress in a Newborn infant HISTORY: You are requested to see a newborn infant with respiratory distress. The child’s gestational age is 3 5 weeks and he is the first born of a primiparous mother. The child was delivered utilizing forceps application. The prenatal historywas negative. The mother was healthy and there was no exposure to drugs, toxins, infectious diseases or radiation. EXAMINATION: Respirations are very shallow and intermittent. Intercostal retraction is noted during inspiration. No airwayobstruction is present. The lungs are clear. Color is mild cyanotic. There is minimal, if any, spontaneous movement of the extremities. Muscle tone is diffusely decreased and no tendon or skin reflexes can be obtained. The urinary bladder is distended. The child cries but response to noxious stimuli cannot be established with certainty. There is little, if any, withdrawal to noxious stimuli. X-RAY: Chest and spine x-rays are normal. LABORATORY DATA: Blood count is normal. QUESTIONS

  1. Which of the following might be considered in the differential diagnosis of this patient’s neurological condition ? (Select one or more)

A. Amyotonia congenita

B. Infantile spinal muscular atrophy

C. Respiratory distress syndrome

D. Arnold Chiari malformation

E. Spinal cord injury

A

(A, B, C, D, E) Amyotonia congenita is a term proposed by Oppenheim in 1900 to describe a group of infants with hypotonia and poor motor development. The term is probably obsolete as it does not characterize a true single pathological entity. The term benign congenital hypotonia has also been applied to these children with limp and flabby limbs in infancy. In most infants, Hit: process is benign and some recovery occurs. Adams feels that many of these children actually have congenital myopathies. Allen, has described confusion between “amyotonia congenita” and spinal cord injury and indicated that many patients with the former diagnosis actually sustained spinal cord injuries. Infantile spinal muscular atrophy or so-called Werdnig-Hoff man disease, is a hereditary progressive muscle atrophy of spinal origin. The disease is inherited in an autosomal dominant pattern and is characterized clinically by diffuse hypotonia and areflexia. Respiratory difficulties are also frequent. The severity of disease at birth varies but the disease is progressive. For those severely affected at birth, death usually ensues within a year. For those less severely affected at birth, the decline is less precipitous. Confusion with spinal cord injury is common inpatients thought to have infantile spinal muscular atrophy. Congenital polymyopathies, polyneuropathies and certain forms of muscular dystrophy may also be confused both with spinal cord injury and v/ith spinal muscular atrophy. Respiratory insufficiency due to the infantile respiratory distress syndrome is common after premature deliveries. Associated hypoxia and hypotonia may cause confusion with spinal cord injury. In addition, it has been suggested that neonatal hypoxia might contribute to and enhance the effects of minor trauma to the spinal cord. It was also suggested that in some instances pulmonary edema, atelectasis and hyaline membrane disease may have been secondary to cardiorespiratory distress related to spinal cord injury rather than the primary pathological process. The Arnold Chiari malformation is frequently associated with myelomeningocele but noi always. In the type II Arnold Chiari malformation, the cervicomedullary junction is kinked and distorted by glial scar tissue. This malformation leads to respiratory disorders, hypoxia and hypotonia. The presence of malfunction of the lower cranial nerves with difficulty swallowing, aspiration and vocal cord paralysis may allow separation of patients with the Arnold Chiari malformation from those with spinal cord injury. However, in cases of spinal cord injury at the cervicomeduiiary junction, *he clinical picture may be essentially identical to that seen with the Arnold Chiari malformation. Spinal cord injury in the newborn period is probably much more frequent than appreciated. The disease processes listed above are frequently misdiagnosed in patients suffering from neonatal spinal cord injury. Towbin estimated that 10% of autopsies in the neonatal period showed evidence of spinal cord or brainstem injury. Such lesions included spinal epidural hemorrhage, laceration of meninges, injury to nerve roots, arteries, ligaments and vertebral bodies. The role of unnoticed spinal cord injury in unexpected sudden infant deaths (SIDS) has been controversial. Some authors have felt that evidence of spinal cord damage in such cases is secondary to hypoxia and venous congestion, and that physical trauma and spinal cord injury do not account for unexpected neonatal death. Others, feel that spinal injury may play a primary role in unexplained neonatal deaths. Walter found subarachnoid hemorrhage and concurrent upper spinai cord and brainstem injury in six of eight unexpected and unexplained sudden neonatal deaths.

119
Q

CASE 22: Respiratory Distress in a Newborn infant HISTORY: You are requested to see a newborn infant with respiratory distress. The child’s gestational age is 35 weeks and he is the first born of a primiparous mother. The child was delivered utilizing forceps application. The prenatal historywas negative. The mother was healthy and there was no exposure to drugs, toxins, infectious diseases or radiation. EXAMINATION: Respirations are very shallow and intermittent. Intercostal retraction is noted during inspiration. No airwayobstruction is present. The lungs are clear. Color is mild cyanotic. There is minimal, if any, spontaneous movement of the extremities. Muscle tone is diffusely decreased and no tendon or skin reflexes can be obtained. The urinary bladder is distended. The child cries but response to noxious stimuli cannot be established with certainty. There is little, if any, withdrawal to noxious stimuli. X-RAY: Chest and spine x-rays are normal. LABORATORY DATA: Blood count is normal.

  1. Which of the following is the most common sign of spinal cord injury in the newborn? (Select only one)

A. Respiratory depression

B. Limb paralysis

C. Sensory level

D. Sphincter paralysis

E. Hypotonia

A

(A) Respiratory depression is the most common and significant sign following neonatal spinal cord injury. This is a particular significant deficit since it may lead to hypoxia and secondary brain injury. Quadriplegia, sensory loss and sphincter paralysis may also be seen in neonatal spinal cord injury but are less frequently noted and of less immediate clinical significance than respiratory depression. Hypotonia and absent tendon reflexes are usually seen acutely following neonatal spinal cord injury but are replaced by spasticity and return of r e flexes if the child survives past the resolution of spinal shock. Sphincter paralysis with initial flaccid urinary bladder and overflow incontinence also may occur with neonatal spinal cord injury. Suprapubic pressure may allow bladder decompression without catheterization in such infants. As spinal shock subsides, the flaccid bladder is usually followed by the development of a spastic automatic bladder. Because of the spontaneous nature of voiding in infants, bladder dysfunction often goes unrecognized and overflow incontinence may be thought to reflect normal voiding until A iiirge firm mass suprapubic ally is identified, representing the distended bladder. We have observed a patient with neonatal spinal cord dysfunction m whom unrecognized bladder distention led to vomiting and massive abdominal expansion thought to be due to intra-abdominal pathology. In this patient, catheterization of the urinary bladder led to complete resolution.

120
Q

CASE 22: Respiratory Distress in a Newborn infant HISTORY: You are requested to see a newborn infant with respiratory distress. The child’s gestational age is 35 weeks and he is the first born of a primiparous mother. The child was delivered utilizing forceps application. The prenatal historywas negative. The mother was healthy and there was no exposure to drugs, toxins, infectious diseases or radiation. EXAMINATION: Respirations are very shallow and intermittent. Intercostal retraction is noted during inspiration. No airwayobstruction is present. The lungs are clear. Color is mild cyanotic. There is minimal, if any, spontaneous movement of the extremities. Muscle tone is diffusely decreased and no tendon or skin reflexes can be obtained. The urinary bladder is distended. The child cries but response to noxious stimuli cannot be established with certainty. There is little, if any, withdrawal to noxious stimuli. X-RAY: Chest and spine x-rays are normal. LABORATORY DATA: Blood count is normal.

  1. What factors are identified in the case presented which increase the risk of spinal cord injury at birth? (Select one or more)

A. Prematurity

B. Primiparity

C. Forceps delivery

D. None of the above

A

(A, B, C) Multiple risk factors have been identified in relation to neonatal spinal cord injury. These include prematurity”, primiparity, intrauterine malposition, dystocia, precipitate delivery and forceps delivery. The single most sigmfic ant risk factor for neonatal spinal cord injury is breech presentation. Stern indicated that in a group of 56 patients with spinal cord injury in the newborn period, 75% had been born by breech delivery. In breech presentations, significant hyperextension of the neck may occur during delivery with spinal cord injury. Abrarnsand Bhagwanani both suggested that caesarean section should be carried out in breech presentations to prevent spinal cord injury. Similar problems with spinal cord injury and recommendations for caesarean section have been made for transverse presentations as well. Although the infant in the case illustrated demonstrated a cephalic delivery, spinal cord injury occurred. Spinal cord injury after cephalic delivery is felt to be rare but does occur. The risk is felt to be increased by the use of forceps for delivery. Reduction in fetal muscle tone as a result of intrauterine hypoxia is also felt to increase the risk of spinal cord injury during delivery. Attempts at rapid delivery in such patients to reduce anoxic insult must be balanced against the risk of spinal cord transection.

121
Q

CASE 22: Respiratory Distress in a Newborn infant HISTORY: You are requested to see a newborn infant with respiratory distress. The child’s gestational age is 35 weeks and he is the first born of a primiparous mother. The child was delivered utilizing forceps application. The prenatal historywas negative. The mother was healthy and there was no exposure to drugs, toxins, infectious diseases or radiation. EXAMINATION: Respirations are very shallow and intermittent. Intercostal retraction is noted during inspiration. No airwayobstruction is present. The lungs are clear. Color is mild cyanotic. There is minimal, if any, spontaneous movement of the extremities. Muscle tone is diffusely decreased and no tendon or skin reflexes can be obtained. The urinary bladder is distended. The child cries but response to noxious stimuli cannot be established with certainty. There is little, if any, withdrawal to noxious stimuli. X-RAY: Chest and spine x-rays are normal. LABORATORY DATA: Blood count is normal.

  1. The most common mechanism of spinal cord injury at the time of birth is (Select only one)

A. fracture-dislocation

B. stretch injury

C. epidural hemorrhage

D. acute intervertebral disc herniation

E. spinal cord infarction

A

(B) Stretch injury to the spinal cord is the most common mechanism of spinal cord injury at birth. The vertebral bodies of the infant are a series of elastic cartilages surrounded by inelastic connective tissue. Leventhal, showed that the infant vertebral column could be stretched up to two inches but the ability of the spinal cord to stretch was much less. The disparity between the pliable bony-ligamentous spinal column and the less elastic spinal cord and meninges probably explains the occurrence of severe sp/nal cord injury at birth without evidence of bony injury. Actual fi aofnrp-dis i 0cation seen on x-ray is extremely rare in newborns with spinal cord injury. Possibly acute temporary dislocation with spontaneous reduction occurs. The most frequent setting in which spinal cord injury occurs in the newborn infant is during breech delivery. Such injuries are likely due to excessive longitudinal traction combined with flexion and tension on the cervical spine which occur when traction is applied to the trunk in order to deliver the after-coming head. The significance of hyperextension of the cervical spine during mnnuai breech deliveries and its relationship to spinal cord injury in the newborn was stressed by Abrams,Bresnan. and Helistrom. In cephalic presentations, stretching of the cervical spine is less likely to occur and thus the incidence of spinal cord injury is less with cephalic presentations. If forceps delivery is carried out, the incidence of spinal cord injuries increases because of the traction and rotation which may be placed on the cervical spine in rotating the infant and delivering the aftercoming body. The occurrence of shoulder dystocia may increase this risk. Nevertheless, even in apparently uneventful cephalic delivery, spinal cord injury may occur. Epidural spinal hemorrhage has been identified in autopsy studies of infants with spinal cord injury. These appear, however, to be associated with other evidence of diffuse spinal injury such as spinal cord transection, parenchymal hemorrhage, subarachnoid hemorrhage, avulsed nerve roots and concurrent brainstem hemorrhage. Epidural spinal hemorrhage with secondary compression of a relatively uninjured spinal cord has not been observedby the authors not been reported in the newborn, as far as we can ascertain. In addition, the overall elasticity of the vertebral column and relatively underdeveloped differentiation of the intervertebral discs make intervertebral disc herniations unknown in newborns. In addition, although spinal cord infarction has been reported Ln older children after mild spinal injury, a similar pattern has not been reported Ln newborns. Beyers8 and Yates27 described vertebral artery injury at autopsy in newborns with spinal cord injury and Towbin and Walter suggested that prenatal factors as well as neonatal hypoxia might combine with relatively mild trauma to produce clinically significant and even fatal spinal cord injury. In these cases, however, the picture was one of hemorrhagic necrosis rather than the ischemic infarctions reported in older children.

122
Q

CASE 22: Respiratory Distress in a Newborn infant HISTORY: You are requested to see a newborn infant with respiratory distress. The child’s gestational age is 35 weeks and he is the first born of a primiparous mother. The child was delivered utilizing forceps application. The prenatal historywas negative. The mother was healthy and there was no exposure to drugs, toxins, infectious diseases or radiation. EXAMINATION: Respirations are very shallow and intermittent. Intercostal retraction is noted during inspiration. No airwayobstruction is present. The lungs are clear. Color is mild cyanotic. There is minimal, if any, spontaneous movement of the extremities. Muscle tone is diffusely decreased and no tendon or skin reflexes can be obtained. The urinary bladder is distended. The child cries but response to noxious stimuli cannot be established with certainty. There is little, if any, withdrawal to noxious stimuli. X-RAY: Chest and spine x-rays are normal. LABORATORY DATA: Blood count is normal.

  1. What treatment should be recommended in the patient illustrated in this case? (Select only one)

A. Immediate decompressive laminectomy

B. Skeletal fraction

C. Anterior cervical exploration and fusion

D. The outlook is hopeless, no treatment is indicated

E. Supportive therapy, corticosteroids

A

(E) Therapeutic measures which are beneficial in neonatal spinal cord injury are not clearly established. Most an thors recommend supportive therapy and recently the use of corticosteroids in spinal cord injury has received support, although the clinical effectiveness of this modality is open to discussion. A few authors have utilized decompressive laminectomy in the treatment of spinal cord injury in the newborn, howevers so few cases have been operatively treated no definite conclusion can be reached. Significant therapeutic benefit has not been reported after laminectomy. This is not surprising since, unlike adults, spinal cord compression by persistent spinal dislocation, intervertebral disc herniation, bony spicules or extra-axial hematoma is rare or nonexistent in newborn spinal cord injury. The value of myelotomy, spinal cord cooling or decompression alone to reduce progressive central spinal cord necrosis has not been analyzed in the newborn patient with spinal cord injury nor in animal models of neonatal spinal cord injuries. Skeletal traction is unlikely to benefit infants with neonatal spinal cord injury since persistent spinal dislocation is extremely uncommon. In older children, however, this modality may be useful (see Case 21). The anterior approach to the cervical spine has generally not been utilized in infants. It is unlikely that this approach would be of any more value than laminectomy in neonatal spinal cord injury. Problems of long term stability have generally not been addressed in neonatal spinal cord injury. Although the outlook for survival is poor, a totally hopeless attitude does not seem justified. Some children with more mild forms of spinal cord injury may show significant improvement and mrnifest long term survival and be able to attend school. Ln the patient illustrated in this case, the clinical picture suggests spinal cord transection. In such instances, mortality is very high. Nevertheless, clear indicators for prognosis in neonatal spinal cord injury have not been developed. Additionally, the neonatal nervous system has considerable capacity for r e covery and adaptation and supportive therapy seems justified particularly since death is nearly always due to complications, particularly respiratory, rather than due to spinal cord injury per se.

123
Q

CASE 23: Spinal Cord Injury in a 51-Year-Old Man with a Rigid Neck HISTORY: A 51-year-old man was seen in the emergency room He complained of neck pain following a fall after a misstep at the bottom step of a flight of stairs. EXAMINATION: The neck was rigid and in a kyphotic position, flexion and extension could not be performed. There were no marks of trauma on either the head, neck or shoulders. Neurological examination including strength, sensation, reflexes and sphincter function was normal. QUESTIONS

  1. A lateral cervical spine x-ray is illustrated in Figure 23.1; It shows (Select one or more)

A. normal cervical spine

B. cervical fracture

C. ankylosing spondylitis

D. ankylosing hyperostosis

E. cervical spondylosis

A

(B, C) The basic underlving pathology illustrated in Figure is that of ankylosing spondylitis. This disease involves a gradual progressive ankylosis of spinal joints and ultimate fusion of uncovertebral (facet) joints, disc spaces and posterior elements (laminae and spinous processes). It occurs primarily in males, begins caudally in the lumbar area and proceeds rostrally to the cervical region. It has been estimated to have an incidence of 1-2% in the white population in general and up to 20% in individuals who possess the HLA-B27 antigen, it is so-called “Marie-StrumpelTs disease” and eventually results in “bamboo spine”. The spine becomes a rigid inflexible bone, resembling a long bone, extending from the sacrum to the base of the skull. The radiological appearance seen in the x-rays illustrated are typical. The bones are generally osteoporotic and cervical kyphosis, rather than the usual lordosis, is seen. There is bony bridging across disc spaces, facet joints and posterior elements. In addition, a hairline fracture passing through the body of C6 from anterior to posterior;is present. A coned view of the midcervical region (Figure 23.3) shows improved demonstration of the fracture line (arrows). Fracture of the posterior elements at the same level, although poorly seen on these x-rays, must be assumed to be present. In the normal spine, fracture of anterior or posterior elements may occur independently with ligaments absorbing the injury force in nonfractured areas. In the spine of ankylosing spondylitis, no such ligamentous and soft tissue force-absorbing mechanism is present. In order for fracture to occur anteriorly, it must occur posteriorly as well. The difference between the normal and spondylitic spine might be compared to that of a green stick and dried stick of wood respectively. The former may break partially when bent but the latter snaps in two under bending force. In ankylosing hyperostosis bony fusion of the spine may occur but only anteriorly involving the disc spaces. The facet joints and posterior elements are uninvolved. In cervical spondylosis, degenerative changes occur in both disc space and facet joints but ankylosis generally does not occur and, if it does, it is localized to only a few interspaces rather than being generalized.

124
Q

CASE 23: Spinal Cord Injury in a 51-Year-Old Man with a Rigid Neck HISTORY: A 51-year-old man was seen in the emergency room He complained of neck pain following a fall after a misstep at the bottom step of a flight of stairs. EXAMINATION: The neck was rigid and in a kyphotic position, flexion and extension could not be performed. There were no marks of trauma on either the head, neck or shoulders. Neurological examination including strength, sensation, reflexes and sphincter function was normal.

  1. Which of the following would you recommend for this patient? {Select one or more)

A. Discharge home

B. Cervical spine tomography

C. Hospitalization for observation

D. Immobilization in a halo brace

E. None* of the above

A

(B,C,D) The presence of a transverse spinal fracture in apatient with ankylosing spondylitisposesgreat risk of spinal cord injury and significant probability of death for the individual. Therefore, immediate immobilization utilizing skeletal fixation is required. Skeletal tongs may be used temporarily while studies are being completed but the halo brace is useful for a more prolonged period. Cervical soine tomography is very helpful in establishing the diagnosis. in the case presented, the fracture was not recognized on plain xrays. In patients with ankylosing spondylitis who complain of neck pain after even minor trauma, a cervical fracture should be assumed until careful radiological studies including tomography demonstrate otherwise. At times careful flexion and extension radiographs may be necessary to demonstrateafracture and associated instability. The spine is easily fractured in ankylosing spondylitis and because of osteoporosis of the spine and difficulty with patientpositioning, fractures are easily missec on plain x-rays alone. Tomography is mandatory in such situations. Furthermore, the trauma necessary to produce spinal fracture may be trivial and not recailedby the patient. Nevertheless, when a patient with ankylosing spondylitis complains of spine pain, a prompt, accurate and exhaustive radiological investigation is required. The patient should not be discharged until the question of cervical fracture is answered. A cervicai collar is unsatisfactory for temporary immobilization inpatients with cervical spondylosis since even minor movements maylead to sudden dislocation with spinal cord injury. The lack of normal ligamentous structures and the lack of mobility at other spinal joints means that any movement force will be fully transmitted to the fracture site. Thus, spinal fractures are by their mere presence highly unstable in patients with ankylosing spondylitis.

125
Q

CASE 23: Spinal Cord Injury in a 51-Year-Old Man with a Rigid Neck HISTORY: A 51-year-old man was seen in the emergency room He complained of neck pain following a fall after a misstep at the bottom step of a flight of stairs. EXAMINATION: The neck was rigid and in a kyphotic position, flexion and extension could not be performed. There were no marks of trauma on either the head, neck or shoulders. Neurological examination including strength, sensation, reflexes and sphincter function was normal.

  1. The patient was admitted to the hospital. The spinal fracture was unrecognized. While awaiting examination, he turned his head suddenly to the right. He heard a “popping” sound ana noted sudden weakness of arms and legs. Reexamination revealed quadriplegia and loss of sensation below C6. Which of the following would you recommend? (Select one or more)

A. Repeat cervical spine x-rays

B. Immediate immobilization

C. Neither A nor B

A

(B,C,D) The presence of a transverse spinal fracture in apatient with ankylosing spondylitisposesgreat risk of spina cord injury and significant probability of death for the individual. Therefore, immediate immobilization utilizing skeletal fixation is required. Skeletal tongs may be used temporarily while studies are being completed but the halo brace is useful for a more prolonged period. Cervical soine tomography is very helpful in establishing the diagnosis. in the case presented, the fracture was not recognized on plain xrays. In patients with ankylosing spondylitis who complain of neck pain after even minor trauma, a cervical fracture should be assumed until careful radiological studies including tomography demonstrate otherwise. At times careful flexion and extension radiographs may be necessary to demonstrateafracture and associated instability. The spine is easily fractured in ankylosing spondylitis and because of osteoporosis of the spine and difficulty with patientpositioning, fractures are easily missec on plain x-rays alone. Tomography is mandatory in such situations. Furthermore, the trauma necessary to produce spinal fracture may be trivial and not recailedby the patient Nevertheless, when a patient with ankylosing spondylitis complains of spine pain, a prompt, accurate and exhaustive radiological investigation is required. The patient should not be discharged until the question of cervical fracture is answered. A cervical collar is unsatisfactory for temporary immobilization inpatients with cervical spondylosis since even minor movements maylead to sudden dislocation with spinal cord injury. The lack of normal ligamentous structures and the lack of mobility at other spinal joints means that any movement force will be fully transmitted to the fracture site. Thus, spinal fractures are by their mere presence highly unstable in patients with ankylosing spondylitis. spinal cord injury, immediate immobilization and accurate x-ray evaluation is required. Because of the unstable nature spinal fractures in ankylosing spondylitis, up to 70% or more of such patients sustain associated spinal cord injury.

126
Q

CASE 23: Spinal Cord Injury in a 51-Year-Old Man with a Rigid Neck HISTORY: A 51-year-old man was seen in the emergency room He complained of neck pain following a fall after a misstep at the bottom step of a flight of stairs. EXAMINATION: The neck was rigid and in a kyphotic position, flexion and extension could not be performed. There were no marks of trauma on either the head, neck or shoulders. Neurological examination including strength, sensation, reflexes and sphincter function was normal.

  1. A lateral cervical spine x-ray of this patient was subsequently obtained and is illustrated in Figure 23. 2. It shows (Select one or more)

A. cervical fracture

B. subluxation

C. dislocation

D. locked facets

E. none of the above

A

(A,C) The cervical fracture is now apparent and gross dislocation has occurred (Figure 23.2). The degree of displacement is too large to be called a subluxation. The latter term is usually applied to displacements of only a lew millimeters. dislocation with locked facets rarely occurs in patients with ankylosing spondylitis. This is due to the fact

127
Q

CASE 23: Spinal Cord Injury in a 51-Year-Old Man with a Rigid Neck HISTORY: A 51-year-old man was seen in the emergency room He complained of neck pain following a fall after a misstep at the bottom step of a flight of stairs. EXAMINATION: The neck was rigid and in a kyphotic position, flexion and extension could not be performed. There were no marks of trauma on either the head, neck or shoulders. Neurological examination including strength, sensation, reflexes and sphincter function was normal.

  1. Which of the following treatments would you recommend? (Select one or more)

A. Anterior cervical fusion

B. Cervical laminectomy

C. Posterior fusion

D. Skeletal tong traction

E. Halo brace

A

(D, E) The most urgent requirement in this patient is to reduce tne dislocation. This can be accomplished most quickly and easily utilizing skull tongs and cervical traction. The difficulty in obtaining reduction of gross dislocations in ankylosing spondylitis should not be minimized. In addition, redislocation occurs with ease because the lack of mobility of other portions of the spine concentrates forces at the fracture site. It is important in attempting reduction that the traction pull be aligned with the pre-existing kyphosis. if the spine is immobilized in extension by utilizing traction, further spinal cord injury may occur. Excessive traction should be avoided because distraction of the fractured elements with destructive tension on the spinal cord may occur. Subsequently, the halobrace is effective in maintaining reduction while healing occurs. The incidence of nonunion or persistent instability is very low in ankylosing spondylitis. It has been suggested that operative treatment and attempts at anterior or posterior fusion actually result in a higher incidence of long term instability than treatment by halo immobilization. This has been attributed to osteoporosis as well as the additional instability caused by reflection of supporting muscles and other soft tissues in the surgical exposure. In addition, advanced age and cardiopulmonary problems make these patients poor operative risks. Manipulation of the neck during anesthesia also poses a significant hazard of further neurological injury if operative treatment is utilized.

128
Q

Indicate whether the following questions (6-10) are true or false in relation to spinal fractures in patients with ankylosing spondylitis.

  1. Death following spinal cord injury is more lively than in patients without spondylitis. (True or False)
A

TRUE Young and Woodruff reported a fatality rate in patients with ankylosing spondylitis and spinal cord injury up to four times the overall death rate of spinal cord injured patients. Both authors report a death rate of about 50% in spinal cord injured patients with ankylosing spondylitis. The advanced age of such patients was considered a significant factor. Cardiopulmonary complications were the usual cause of death in both series. Because the thoracic cage is also often ankylosed, restrictive pulmonary disease is common. Aggressive pulmonary therapy and mobilization as soon as possible utilizing the halo brace may reduce the fatality rate.

129
Q

Indicate whether the following questions (6-10) are true or false in relation to spinal fractures in patients with ankylosing spondylitis.

  1. Healing of fractures occurs more rapidly in patients with spondylitis than without. (True or False)
A

TRUE Rapic consolidation and healing of spinal fractures in ankylosing spondylitis is well known. Many authors have confirmed this observation, and it is for this: reason that conservative treatment utilizing skeletal traction for reduction of dislocation and the halo brace for prolonged immobilization is recommended. Most fractures will heal in a stable position with tins treatment in a period of twelve weeks,

130
Q

Indicate whether the following questions (6-10) are true or false in relation to spinal fractures in patients with ankylosing spondylitis.

  1. Multiple fractures are common. (True-or False)
A

TRUE Multiple spinal fractures are common in ankylosing spondylitis. Therefore, if one spinal fracture is identified, complete x-ray study of the spine is probably indicated to rule out other fractures which may complicate management or lead to spinal cord injury if untreated. Hansenll and Yau stressed the occurrence of fractures and dislocations in the thoracic and lumbar spines of patients with ankylosing; spondylitis. Atlanto-axial dislocation has also been reported.

131
Q

Indicate whether the following questions (6-10) are true or false in relation to spinal fractures in patients with ankylosing spondylitis.

  1. The most common mechanism of injury is flexion. (True or False)
A

FALSE

132
Q

Indicate whether the following questions (6-10) are true or false in relation to spinal fractures in patients with ankylosing spondylitis.

  1. The most common neurological findings are those of the central spinal cord syndrome. (True or False)
A

TRUE Extension is the primary mechanism of spinal injury in patients with ankylosing spondylitis. This has been stressed by many authors. As a result, the central spinal cord syndrome. is the most frequent neurological picture seen after spinal cord injury in patients with ankylosing spondylitis.

133
Q

CASE 24: Paralysis and Sensory Loss in a 17-Year-Old Male After Stab Wound to the Neck HISTORY: A 17-year-old male was brought to the emergency room after an altercation in which he was stabbed in the neck with an ice pick. He was awake and complained of numbness in the right arm and leg and inability to move the left arm and leg. EXAMINATION: There was a small puncture wound just posterior to the left sternocleidomastoid muscle about three fingerbreadths above the clavicle. The general physical examination was unremarkable. Vital signs were normal. Neurological examination revealed loss of pain and temperature sensation on the right side of the body beginning with the middle, ring and little fingers and including the ulnar side of the hand, forearm and arm, the entire right side of the trunk, abdomen and right leg. Position and vibratory sensation was absent in the left arm only. Motor examination revealed a dense left hemiplegia. The entire left arm was paralyzed with inability to abduct, flex, extend or adduct the shoulder. A left Babinski’s sign was present and tendon reflexes were increased on the left side. The left superficial abdominal and cremasteric reflexes were absent. X-RAYS: Plain x-rays of the cervical spine showed no ababnormalities. QUESTIONS

  1. The history and examination most closely suggests which one of the following? (Select only one)

A. Hemisection of the spinal cord

B. The anterior spinal cord syndrome

C. The central spinal cord syndrome

D. Complete spinal cord transection

E. Anterior spinal artery injury

A

(A) The signs and symptoms are most characteristic of hemisection of the spinal cord or the so-called “Brown-Sequard syndrome”. The corticospinal motor pathways decussate at the level of the medullary pyramids. Thus, unilateral spinal cord injury to the corticospinal tract results in ipsilateral motor signs and symptoms (Figure 24.1). The increased tendon reflexes, absent abdominal reflexes, and Babinski’s sign on the left side are all evidence of left corticospinal tract injury below the medulla. The spinothalamic tract decussates locally within the spinal cord as second and third order afferent interneurons cross through the ventral white commissure to enter the contralateral spinothalamic tract (Figure 24. 1 and Figure 10.2). Unilateral l e sions of the spinothalamic tract wrthin the spinal cord therefore result in contralateral loss of pain and temperature sensation. Thus, a hemisection of the spinal cord results in ipsilateral paralysis and contralateral loss of pain and temperature sensation. Information concerning position and vibratory sensation is carried in the dorsal columns (Figure 24. 1). These afferent spinal tracts travel ipsiiateral to the side of entry of information via the dorsal spinal nerve roots. Decussation occurs at the medullary level. Thus, spinal cord lesions affecting the dorsal columns result in ipsiiateral alterations in position and vibratory sensation. In addition, the dorsal columns are somatotopically organized. The more laterally located fasiculus cuneatus is concerned with arm sensation while the more medially located fasiculus gracilis is concerned with’sensation from the trunk and Teg (Figure 24. 1). #pict# Figure 24.1. Diagram of cross-section of cervical spinal cord. Ascending fibers in the dorsal columns and spinothalamic tracts as well as descending motor fibers in the corticospinal tract are so mato topic ally organized. C = cervical, Th = thoracic, L = lumbar, S = sacral. Injury to one-half of the spinal cord results in contralateral loss of pain and temperature sensation (spinothalamic tract) and ipsilateral loss of vibration and position sense (dorsal columns) as well as. ipsilateral paralysis (corticospinal tract). see text for full explanation. In the patient described, the left hemiparesis relates to injury to the left corticospinal pathway. The loss of pain and temperature sensation on the right relates to injury to the left spinothalamic tract. The loss of vibratory and position sense in the left arm relates to injury to the left fasiculus cuneatus. Thus, the entire neurological examination can be explained by a lesion on the left side of the cervical spinal cord. The plain x-rays of the cervical spine and history of ice pick assault make it likely that the ice pick entered an intervertebral foramen and lacerated the left half of the spinal cord sparing only the left fasiculus gracilis. Penetrating wounds are a frequent cause of hemisection of the spinal cord. Most commonly this results from assault with a sharp object, but recently iatrogenic spinal cord injury by acupuncture needles has been reported. The clinical syndrome is not that of the anterior spinal cord syndrome which is characterized by total paralysis and loss of all sensory modalities except vibratory and position sense below the lesion (see Case 11) The anterior spinal cord syndrome usually results from hyperflexion injuries or from anterior spinal artery occlusion rather than from penetrating wounds. The central spinal cord syndrome is characterized by motor weakness primarily in the arms and dissociated sensory loss. In addition, the central cord syndrome is either due to indirect mechanical trauma or vascular injury and not due to penetrating wounds (see Case 10).

134
Q

CASE 24: Paralysis and Sensory Loss in a 17-Year-Old Male After Stab Wound to the Neck HISTORY: A 17-year-old male was brought to the emergency room after an altercation in which he was stabbed in the neck with an ice pick. He was awake and complained of numbness in the right arm and leg and inability to move the left arm and leg. EXAMINATION: There was a small puncture wound just posterior to the left sternocleidomastoid muscle about three fingerbreadths above the clavicle. The general physical examination was unremarkable. Vital signs were normal. Neurological examination revealed loss of pain and temperature sensation on the right side of the body beginning with the middle, ring and little fingers and including the ulnar side of the hand, forearm and arm, the entire right side of the trunk, abdomen and right leg. Position and vibratory sensation was absent in the left arm only. Motor examination revealed a dense left hemiplegia. The entire left arm was paralyzed with inability to abduct, flex, extend or adduct the shoulder. A left Babinski’s sign was present and tendon reflexes were increased on the left side. The left superficial abdominal and cremasteric reflexes were absent. X-RAYS: Plain x-rays of the cervical spine showed no ababnormalities.

  1. The motor and sensory findings suggest (Select one or more)

A. injury at more than one spinal level

B. injury at a single spinal level

C. an intramedullary hematoma

D. spinal shock

E. combined corticospinal, dorsal column and spinothalamic tract injury

A

(B,E) The neurological findings are explained by a single lesion at the C4 level on the left side of the spinal cord. The pattern is that of hemisection of the spinal corrt the Brown- Sequard syndrome. The hemiplegia begins at the C5 level on the left (paralysis of shoulder motion). The highest loss of vibratory and position sense in the arm, namely the shoulder, is also innervated by C5. The loss of pain and temperature sensation on the right side begins at C7 (middle finger). This might be thought to imply a second lesion at a lower level. As the neurons of the spinothalamic tract decussate, however, they ascend two to three segments or more within the spinal cord. Thus, a lesion at the C4 level on the left siae of the spinal cord may produce a loss of pain and temperature sensation beginning at about the C7 dermatomal level on the right. A single injury to the spinothalamic tract, corticospinal tract and lateral aspect (fasiculus cuneatus) of the dorsal column at C4-5 accounts for the patient’s neurological findings. An intramedullary hematoma usually produces a picture similar to the central spinal cord syndrome. Spinal shock is usually seen only with total or subtotal interruption of spinal cord function and is manifested by loss of spinal cord function below a particular level. Such was not the case in this patient.

135
Q

CASE 24: Paralysis and Sensory Loss in a 17-Year-Old Male After Stab Wound to the Neck HISTORY: A 17-year-old male was brought to the emergency room after an altercation in which he was stabbed in the neck with an ice pick. He was awake and complained of numbness in the right arm and leg and inability to move the left arm and leg. EXAMINATION: There was a small puncture wound just posterior to the left sternocleidomastoid muscle about three fingerbreadths above the clavicle. The general physical examination was unremarkable. Vital signs were normal. Neurological examination revealed loss of pain and temperature sensation on the right side of the body beginning with the middle, ring and little fingers and including the ulnar side of the hand, forearm and arm, the entire right side of the trunk, abdomen and right leg. Position and vibratory sensation was absent in the left arm only. Motor examination revealed a dense left hemiplegia. The entire left arm was paralyzed with inability to abduct, flex, extend or adduct the shoulder. A left Babinski’s sign was present and tendon reflexes were increased on the left side. The left superficial abdominal and cremasteric reflexes were absent. X-RAYS: Plain x-rays of the cervical spine showed no ababnormalities.

  1. Which of the following are indicated in this patient? (Select one or more)

A. Emergency surgical exploration

B. Myelography

C. Skeletal traction via skull tongs

D. Angiography

E. None of these

A

(E) Indications for surgical treatment of penetrating wounds are controversial. Surgical treatment seems indicated if persistent cerebrospinal fluid leakage or progressive neurological deterioration occurs. Perhaps surgical treatment is indicated if radiological evidence is obtained of retained foreign material or bone fragments. Failure to remove retained foreign material may lead to long term neurological deterioration. None of these indications were present in the case described. Likewise, no indication for myelography is present. The neurological condition is stable without suggestion of expanding hematoma, herniated intervertebral disc or other treatable lesion which might be revealed by myelography. Myelography may be useful in diagnosis~of the sequelae of knife wounds to the spinal cord when retained foreign bodies such as broken knife blades are present. No evidence of vascular injury is present so that angiography is not indicated. Plain x-rays revealed no evidence of fracture or dislocation so that traction is unnecessary as well. The only specific treatment to be considered relates to the use of antibiotics. The use of such agents in penetrating, presumably contaminated spinal wounds, is recommended by most authors, although the value of such a course of action is not substantiated by comparison with untreated patients.

136
Q

CASE 24: Paralysis and Sensory Loss in a 17-Year-Old Male After Stab Wound to the Neck HISTORY: A 17-year-old male was brought to the emergency room after an altercation in which he was stabbed in the neck with an ice pick. He was awake and complained of numbness in the right arm and leg and inability to move the left arm and leg. EXAMINATION: There was a small puncture wound just posterior to the left sternocleidomastoid muscle about three fingerbreadths above the clavicle. The general physical examination was unremarkable. Vital signs were normal. Neurological examination revealed loss of pain and temperature sensation on the right side of the body beginning with the middle, ring and little fingers and including the ulnar side of the hand, forearm and arm, the entire right side of the trunk, abdomen and right leg. Position and vibratory sensation was absent in the left arm only. Motor examination revealed a dense left hemiplegia. The entire left arm was paralyzed with inability to abduct, flex, extend or adduct the shoulder. A left Babinski’s sign was present and tendon reflexes were increased on the left side. The left superficial abdominal and cremasteric reflexes were absent. X-RAYS: Plain x-rays of the cervical spine showed no ababnormalities.

  1. One week post injury, the patient began to complain of an annoying “swisning” sound heard in his left ear and synchronous with his pulse deal. Auscultation.in the region of the healing entry wound revealed a systolic bruit. Otherwise, the neurological examination was unchanged. This turn of events is most likely due to (Select only one)

A. cerebrospinal fluid fistula

B. carotid artery to venous fistula

C. vertebral artery cc venous fistula

D. a pseudoaneurysm

E. cervical dislocation

A

(C) The systolic bruit which is audible to both patient and examiner suggests a vertebral artery to venous fistula. Xray and clinical evidence previously discussed suggested that the ice pick used in the original assault passed into the spinal canal via the left C4-5 intervertebral foramen. The vertebral artery lies immediately anterior to the intervertebral foramen at that level and could easily have been injured as well. The vertebral artery is surrounded by a plexus of veins making vertebral artery to venous fistulas relatively common occurrences after penetrating injury to the vertebral artery. Carotid artery to jugular vein fistula is possible but much less likely. Since the entry wound was posterior to the sternocleidomastoid muscle and passed through the intervertebral foramen, it would be considerably posterior to the carotid artery. In addition, carotid- jugular fistulas are rather uncommon after penetrating neck injuries. A pseudoaneurysm could form on either vertebral or carotid artery after penetrating injury. Although an audible bruit may be detectable wTith stethoscope due to turbulent flow as a result of pseudoaneurysm formation, such bruits are not usually loud enough to be noticed by the patient. Neither cerebrospinal fluid fistula nor cervical dislocation should result in a bruit audible to either patient or examiner.

137
Q

CASE 24: Paralysis and Sensory Loss in a 17-Year-Old Male After Stab Wound to the Neck HISTORY: A 17-year-old male was brought to the emergency room after an altercation in which he was stabbed in the neck with an ice pick. He was awake and complained of numbness in the right arm and leg and inability to move the left arm and leg. EXAMINATION: There was a small puncture wound just posterior to the left sternocleidomastoid muscle about three fingerbreadths above the clavicle. The general physical examination was unremarkable. Vital signs were normal. Neurological examination revealed loss of pain and temperature sensation on the right side of the body beginning with the middle, ring and little fingers and including the ulnar side of the hand, forearm and arm, the entire right side of the trunk, abdomen and right leg. Position and vibratory sensation was absent in the left arm only. Motor examination revealed a dense left hemiplegia. The entire left arm was paralyzed with inability to abduct, flex, extend or adduct the shoulder. A left Babinski’s sign was present and tendon reflexes were increased on the left side. The left superficial abdominal and cremasteric reflexes were absent. X-RAYS: Plain x-rays of the cervical spine showed no ababnormalities.

  1. Appropriate investigation of the circumstances described in question 4 would be accomplished by which of the following studies? (Select only one)

A. Carotid angiography

B. Vertebral angiography

C. Four vessel cerebral angiography

D. Lumbar puncture

E. Cervical spine x-rays

A

(C) Four vessel angiography is required for accurate diagnosis. Bilateral study of carotid and vertebral circulation is required to analyze not only in-flow to the potential arteriovenous fistula but also the pattern of flow to the brain. The sink effect created by an arteriovenous fistula may result in steal of blood from the normal territory of distribution of arteries n involved directly in the fistulous communication. Carotid or vertebral angiography alone may not give a satisfactory overall evaluation such as to allow appropriate treatment to be planned, either lumbal puncture nor cervical spine x-rays are indicated since there is nothing in the clinical situation to suggest meningitis or cer icai dislocation.

138
Q

CASE 25: Thoracic-Spinal Cord Injury. HISTORY: A 48-year-old man was seen two hours following an automobile accident. The patient was thrown out of the vehicle. He complained of inability to move his legs, numbness of his legs and midback pain. EXAMINATION: The patient was awake and alert. BP 110/70, pulse 80, R 20, temperature 98.8°F. There was a flaccid paralysis of the legs except for very slight voluntary dorsiflexion of the left great toe. The umbilicus moved upward as the patient lifted his head off the pillow in the supine position. Reflexes were 2-f and symmetrical in the arms and absent in the legs. There was no response to plantar stimulation. Sensory examination revealed loss of all modalities below the level of the umbilicus. Rectal and bladder sphincters were paralyzed. X-RAYS: X-rays of the thoracic spine showed a fracture of the bodies of T8 and T9 and angulation at T8 (Figures 25.1A and 25.1B). QUESTIONS

  1. The lesion responsible for the patient’s deficit can best be characterized clinically as (Select only one)

A. the anterior spinal cord syndrome

B . incomplete spinal cord injury

C. spinal cord transection

D. Brown-Sequard syndrome (hemisection of the spinal cord)

E. none of the above

A

(B) The preservation of dorsiflexion of the left great toe indicates that the spinal cord lesion in this patient is incomplete Aside from this, the clinical picture is one of total loss of function extending candaliy from the level of injury. The anterior spinal cord syndrome (See Case 11 ) is confined almost exclusively to injuries of the cervical spine (and uncommon cases of anterior spinal artery interruption) and is characterized by loss of all motor function below a specific spinal level, loss of pain and temperature sensation, and preservation of joint position sense, vibratory sensation and touch. The loss of all sensory modalities in the patient presented excludes this syndrome. The Brown-Sequard syndrome!, 6 (hemisection of the cord) is characterized by ipsilateral motor loss and contralateral loss of pain and temperature sensation. Ipsilateral loss of proprioception and vibratory, sensation is also seen. The bilateral global loss of sensory and motor function of a symmetrical type exciudes the Brown-Sequard syndrome in the patient described. Thus, the lesion is best described clinically as an incomplete spinal cord injury.

139
Q

CASE 25: Thoracic-Spinal Cord Injury. HISTORY: A 48-year-oid man was seen two hours following an automobile accident. The patient was thrown out of the vehicle. He complained of inability to move his legs, numbness of his legs and midback pain. EXAMINATION: The patient was awake and alert. BP 110/70, pulse 80, R 20, temperature 98. 8°F. There was a flaccid paralysis of the legs except for very slight voluntary dorsiflexion of the left great toe. The umbilicus moved upward as the patient lifted his head off the pillow in the supine position. Reflexes were 2-f and symmetrical in the arms and absent in the legs. There was no response to plantar stimulation. Sensory examination revealed loss of all modalities below the level of the umbilicus. Rectal and bladder sphincters were paralyzed. X-RAYS: X-rays of the thoracic spine showed a fracture of the bodies of T8 and T9 and angulation at T8 (Figures 25.1A and 25.1B). QUESTIONS

  1. The upward migration of the umbilicus when the patient raises his head is (Select one or more)

A. of no localizing significance

B. consistent with a mid or low thorac ic spinal cord lesion

C. accurately localizing for a spinal cord lesion at Tl0

D. due to paralysis of the lower abdominal muscles

E. none of the above

A

(B) The overall innervation of the rectus abdominus muscle extends from T5-T12. The lateral abdominal muscles are innervated by spinal segments T5-L1. Spinal segments T9-T10 innervate the periumbilical musculature. In lesions of the thoracic spinal cord between T9 and T12, the segments of the rectus abdominus and lateral abdominal muscles above the umbilicus are spared. The segments of these muscles below the level of injury are paralyzed. Thus, when the patient elevates his head, while in the supine position, the upper portion of the abdominal wail musculature contracts while the lower portion doe snot. This results in upward migration of the umbilicus (Beevor’s sign). This sign is thus indicative of a mid or lower thoracic spinal cord lesion but doe snot accurately localize the lesion to T10. The relative amount of upward umbilical movement relates to how close the lesion is to T10 but quantitation of the amount of umbilical motion is nearly impossible from a practical standpoint.

140
Q

CASE 25: Thoracic-Spinal Cord Injury . HISTORY: A 48-year-old man was seen two hours following an automobile accident. The patient was thrown out of the vehicle. He complained of inability to move his legs, numbness of his legs and midback pain. EXAMINATION: The patient was awake and alert. BP 110/70, pulse 80, R 20, temperature 98. 8°F. There was a flaccid paralysis of the legs except for very slight voluntary dorsiflexion of the left great toe. The umbilicus moved upward as the patient lifted his head off the pillow in the supine position. Reflexes were 2-f and symmetrical in the arms and absent in the legs. There was no response to plantar stimulation. Sensory examination revealed loss of all modalities below the level of the umbilicus. Rectal and bladder sphincters were paralyzed. X-RAYS: X-rays of the thoracic spine showed a fracture of the bodies of T8 and T9 and angulation at T8 (Figures 25.1A and 25.1B) QUESTIONS

  1. Which Of the following is true concerning the prognosis “for neurological recovery m this patient? (Select only one)

A. Is likely

B. Is impossible

C. Is highly unlikely if the deficit remains unchanged for 72 hours

D. No reliable prognostic statement can be made

A

(D) An accurate prognosis cannot be made regarding potential recovery of incomplete spinal cord injuries in the early days after injury. If a transverse spinal cord lesion is complete at hours postinjury. the prognosis for significant functional recovery is nil. Regarding incomplete lesions, however, the prognosis cannot accurately be stated. Nevertheless, there is a significant chance of recovery in motor incomplete lesions. The extent and final level of neurological recovery varies considerably.

141
Q

CASE 25: Thoracic-Spinal Cord Injury . HISTORY: A 48-year-old man was seen two hours following an automobile accident. The patient was thrown out of the vehicle. He complained of inability to move his legs, numbness of his legs and midback pain. EXAMINATION: The patient was awake and alert. BP 110/70, pulse 80, R 20, temperature 98. 8°F. There was a flaccid paralysis of the legs except for very slight voluntary dorsiflexion of the left great toe. The umbilicus moved upward as the patient lifted his head off the pillow in the supine position. Reflexes were 2-f and symmetrical in the arms and absent in the legs. There was no response to plantar stimulation. Sensory examination revealed loss of all modalities below the level of the umbilicus. Rectal and bladder sphincters were paralyzed. X-RAYS: X-rays of the thoracic spine showed a fracture of the bodies of T8 and T9 and angulation at T8 (Figures 25.1A and 25.1B) QUESTIONS

  1. Appropriate immediate treatment for this patient should include (Select one or more)

A. systemic corticosteroids

B. decompressive laminectcto my

C. spinal fusion

D. positioning on an appropriate bed or spinal frame

E. frequent position changes

A

(A,D,E) Although the role of systemic corticosteroids in the treatment of spinal cord injury is not definitely established, most authorities recommend their use. The role of surgical treatment in midthoracic injuries is controversial but perhaps not as controversial as with cervical injuries. Most experts recommend nonoperative treatment. Some, however, recommend open operative reduction and fusion of nearly all thoracic spinal cord injuries. Kelly H recommended anterior vertebral body removal and posterior fusion Paul also recommended anterior transthoracic decompression of thoracic fracture-dislocations with spinal cord injury. Kaufer felt that late instability was frequent in thoracic spine fracture-dislocations and was responsible for pain and gibbus formation. Yashon described routine use of laminectomy and lateral transverse process fusion in the treatment of thoracic spine injuries with severe spinal cord injury. He provided no evidence of the efficacy or safety of such a course of action but indicated such a course of action assured the patient that every, thing- that could be done during the acute phase was done. On the other hand, Roberts reviewed the incidence of late instability in patients who had previously undergone thoracic laminectomy in the treatment of thoracic spinal cord injury due to fracture-dislocation. He found a high, incidence of instability particularly in patients with so-called rotational fracture-dislocations. This is in distinct contrast to authors who recommend conservative treatment who have reported a very low incidence of instability in midthoracic fracture-dislocations. A number of authors describe basically a conservative approach to treatment with specific operative indications on an individual case basis.The thoracic spine is inherently more stable than the cervical spine due to the attachments of paraspinal muscles and ribs. Under normal conditions, there is minimal movement in the thoracic spine compared to the cervical and thoracic spines. The attachment of ribs to the transverse processes of the thoracic vertebrae, the presence of the intercostal muscles and anterior attachment of ribs to the sternum restricts movement of the thoracic spine to flexion and extension and even these movements are small. These factors aid considerably in the stabilization of thoracic fractures and fracture-dislocations. It is for this reason that spinal injuries in ttw fhoracic region tend to heal well with proper positioning on a spinal frame or spinal bed. Frequent position changes are necessary Ln such patients to prevent skin ulceration and reduce the possibility of cardiovascular and pulmonary complications. Burke compared neurological recovery in patients with spinal cord injury due to thoracic spine fracture-dislocations treated with and without surgery. He found no difference in the rate of neurological recovery in the two groups. This was significant since the patients treated nonsurgically had mainly complete lesions while those treated surgically had mainly incomplete lesions. Thus, although the surgically treated cases had initially a better prognosis, the rate of neurological recovery was no better than those treated nonsurgically. We generally recommend postural reduction, corticosteroids and nonsurgical management in the acute treatment of thoracic spine injuries with spinal cord injury. If marked angulation suggests disruption of posterior supporting elements and facet joint capsules with avulsion through the intervertebral disc space, then posterior fusion may be considered on a delayed basis. Compression plates attached to the spinous processes may be helpful in assuring stability during healing of bone grafts to provide solid fusion. Harrington rod fixation is also useful in obtaining and maintaining reduction while healing of bone grafts to form a solid bony union takes place. If anteriorly displaced bone fragments related to fractures of vertebral bodies impedes reduction, then anterior reduction by removal of the fractured body may assist greatly in obtaining reduction and an anterior fusion may be accomplished. There is no controversy as to the need for fusion in cases of an unstable spine. The controversy relates to definition of spinal instability, the frequency with which it occurs acutely and the frequency with which it persists after various methods of treatment.

142
Q

CASE 25: Thoracic-Spinal Cord Injury . HISTORY: A 48-year-old man was seen two hours following an automobile accident. The patient was thrown out of the vehicle. He complained of inability to move his legs, numbness of his legs and midback pain. EXAMINATION: The patient was awake and alert. BP 110/70, pulse 80, R 20, temperature 98. 8°F. There was a flaccid paralysis of the legs except for very slight voluntary dorsiflexion of the left great toe. The umbilicus moved upward as the patient lifted his head off the pillow in the supine position. Reflexes were 2-f and symmetrical in the arms and absent in the legs. There was no response to plantar stimulation. Sensory examination revealed loss of all modalities below the level of the umbilicus. Rectal and bladder sphincters were paralyzed. X-RAYS: X-rays of the thoracic spine showed a fracture of the bodies of T8 and T9 and angulation at T8 (Figures 25.1A and 25.1B) QUESTIONS

  1. Regardless of whether or not initial conservative or initial operative treatment of thoracic spinal cord injury is recommended, some period of management in “bed” will be required in such patients. Which of the following are acceptable devices for such periods of bed rest? (Select one or more)

A. Stryker or Foster spinal frame

B. Circle-electric bed

C. Stoke-Mandeville Edgerton turning bed

D. Roto -Rest bed

E. Standard hospital bed

A

(A,C,D) In the United States, most patients with spine or spinal cord injuries are managed on a spinal frame. Such frames provide good stability, ability to elevate the head and stable positioning for turning the patient prone or supine as desired. The frames, however, have many disadvantages. They are narrow and difficult to utilize with obese patients. The thoracic spine can be hyper extended, as is needed for reduction of most thoracic fractures when the patient is supine but only with difficulty when the patient is prone. Further, only two positional options are available, supine or prone. The prone position may be hazardous with long bone, hip or pelvic fractures, hemothorax, pneumothorax: or respiratory insufficiency. Further, most patients dislike the prone position for any extended period. Frequently, due to fear of turning, parents are kept supine for long periods. Yashon, in fact, recommends not turning patients with cervicai fractures for up to 24 hours following injury. Unfortunately, in this period serious pressure ulceration may occur. In addition, the need for at least two trained personnel to turn such frames often leads to patients remaining prone or supine (usually supine) for extended periods allowing for pressure ulceration of skin, and pulmonary, vascular and renal stasis with associated complications. We have used this frame almost exclusively and found it acceptable but not ideal. Guttmann in England and Keane in Ireland developed modifications of the spinal.frame. These are the Stoke-Mandeville. Edgerton turning bed and the Roto-Rest bed respectively, both are much wider than spinal frames, are turned electrically a single individual, easily allow for any spinal position and do not utilize the prone position. Thus, frequent movement of the patient is encouraged. The Roto-Rest actually provides for cotinuous movement. The bed is thus comfortable for long periods avoids the disadvantages of the prone position and the complications of infrequent movement of the patient. The Roto Rest is receiving some acceptance in this country primarily based on a very significant reduction in complications such as pressure ulceration of skin and pulmonary complications such as pneumonia and vascular complications such as thrombophlebitis and pulmonary embolism. The circle-electric bed is unsuitable for most cervical and thoracic spine and spinal cord injuries. 6,18 Although this bed allows for a variety of positions and electrical movement, it has a serious disadvantage. The disadvantage is that the manner in which turning is accomplished, namely by elevating or lowering the head relative to the feet, results in loading and unloading of the fracture-dislocation site. This may lead to dangerous shifts at the site of unstable fractures and dislocations. Most authors, even those who disagree on other aspects of care of spinal injuries, agree that the circle-electric bed should be avoided in the management of spinal injury. Likewise, the standard hospital bed is generally unsuitable. Turning in such beds is difficult and requires “log rolling” of the patient. This necessitates several attendants and with obese patients is very difficult. Although air mattresses and fluidfilled mattresses reduce the likelihood of skin necrosis when used with standard beds, they contribute to pulmonary, renal and vascular stasis and associated complications because they discourage frequent turning of patients with spinal cord injurie

143
Q

CASE 26: Thoracolumbar Fracture HISTORY: The patient was involved in an auto accident on the day of admission. He was a passenger and was not wearing a seat belt. Although he was unable to recall exactly what happened to him at the time of impact, he immediately noted back pain and’‘hot flashes” in his legs. He was noted to be unable to move his legs at the scene of the accident. EXAMINATION: Ln the emergency room his vital signs were stable. He had no visible injury or deformity but did have tenderness over the upper lumbar area. He was paraplegic with a sensory level at T12-L1 and his anal tone was diminished. X-RAY: X-ray views of the spine were normal except the thoracolumbar region. There was a wedge fracture of L1, with lateral and anterior angulation of T12 on L1. There was also a slight (3 mm) anterior subluxation of T12 on L1 (Figure 26.1). QUESTIONS

  1. Alter bony disruption at the T12-L1 level, the next most common site of traumatic thoracic, thoracolumbar, or lumbar fracture is (Select only one)

A T5-6

B. T11-12

C. L1-2

D. L4-5

A

B. Tidak ada pembahasan

144
Q

CASE 26: Thoracolumbar Fracture HISTORY: The patient was involved in an auto accident on the day of admission. He was a passenger and was not wearing a seat belt. Although he was unable to recall exactly what happened to him at the time of impact, he immediately noted back pain and ‘‘hot flashes” in his legs. He was noted to be unable to move his legs at the scene of the accident. EXAMINATION: Ln the emergency room his vital signs were stable. He had no visible injury or deformity but did have tenderness over the upper lumbar area. He was paraplegic with a sensory level at T12-L1 and his anal tone was diminished. X-RAY: X-ray views of the spine were normal except the thoracolumbar region. There was a wedge fracture of L1, with lateral and anterior angulation of T12 on L1. There was also a slight (3 mm) anterior subluxation of T12 on L1 (Figure 26.1). QUESTIONS

  1. The injury described for this patient probably represents a hyperextension injury. (True or False}
A

False. Tidak Ada Pembahasan

145
Q

CASE 26: Thoracolumbar Fracture HISTORY: The patient was involved in an auto accident on the day of admission. He was a passenger and was not wearing a seat belt. Although he was unable to recall exactly what happened to him at the time of impact, he immediately noted back pain and ‘‘hot flashes” in his legs. He was noted to be unable to move his legs at the scene of the accident. EXAMINATION: Ln the emergency room his vital signs were stable. He had no visible injury or deformity but did have tenderness over the upper lumbar area. He was paraplegic with a sensory level at T12-L1 and his anal tone was diminished. X-RAY: X-ray views of the spine were normal except the thoracolumbar region. There was a wedge fracture of L1, with lateral and anterior angulation of T12 on L1. There was also a slight (3 mm) anterior subluxation of T12 on L1 (Figure 26.1). QUESTIONS

  1. This patient’s neural injury most likely involves (Select only one)

A. spinal cord

B. conus medullaris

C. cauda equina

D. a combination of more than one of the above

E. none of the above

A

D. Tidak Ada Pembahasan

146
Q

CASE 27: Interscapular Pain and Paraplegia in a 47-Year-Old Man HISTORY: A 47-year-old man was seen complaining of interscapular pain and weakness of his legs of two days duration. He was a chronic alcoholic. He gave no specific history of trauma. The pain was moderate in severity and radiated along the ulnar aspect of both upper arms and forearms. It was aggravated by straining and relieved by rest. The weakness of his legs had progressed to the point that the patient was unable to stand or walk. He had been unable to urinate for 24 hours. EXAMINATION: He was an obese male who was awake and cooperative but somewhat tremulous. Respirations were diaphragmatic. There was mild tenderness to percussion over the spinous processes of C7, T1 and T2. A firm, slightly tender liver edge was palpable 6 cm below the right costal margin. The urinary bladder was percussed to the level of the umbilicus. The rest of the general physical examination was unremarkable. Neurological examination revealed flaccid paralysis of the leg. The abdominal muscles were also flaccid. Arm strength was normal. Tendon reflexes were absent in the legs and there was no response to plantar stimulation. Abdominal and cremasteric reflexes were also absent. The rectal sphincter was patulous. Sensation was normal to touch, pinprick, position and vibratior throughout. QUESTIONS

  1. The spinal cord level of this patient’s problem is most likely (Select only one)

A. upper cervical

B. lower cervical

C. upper thoracic

D. lower thoracic

A

(C) The history and clinical examination point to an upper thoracic lesion. Motor examination shows paralysis of the legs (supplied by spinal segments L1-S2) and abdominal muscles (supplied by T7-L1). Diaphragmatic respirations, suggested by inspiratory intercostal retraction, suggest paralysis of intercostal muscles (supplied by spinal segments T1-T12). Since the motor function and reflexes are normal in the arms (supplied by spinal segments C5-T1), a high or low cervical lesion is excluded. The presence of pain radiating along the ulnar aspect of the arms implies irritation of the first thoracic nerve root but normal function of intrinsic hand muscles (supplied by the T1 spinal segment) suggests that the first thoracic ventral root is functioning normally. A lower thoracic lesion would not produce paralysis of all intercostal muscles nor arm pain. Lower thoracic lesions frequently produce girdle like pain radiating about the lower chest or upper abdomen due to irritation of thoracic dorsal roots.

147
Q

CASE 27: Interscapular Pain and Paraplegia in a 47-Year-Old Man HISTORY: A 47-year-old man was seen complaining of interscapular pain and weakness of his legs of two days’ duration. He was a chronic alcoholic. He gave no specific history of trauma. The pain was moderate in severity and radiated along the ulnar aspect of both upper arms and forearms. It was aggravated by straining and relieved by rest. The weakness of his legs had progressed to the point that the patient was unable to stand or walk. He had been unable to urinate for 24 hours. EXAMINATION: He was an obese male who was awake and cooperative but somewhat tremulous. Respirations were diaphragmatic. There was mild tenderness to percussion over the spinous processes of C7, T1 and T2. A firm, slightly tender liver edge was palpable 6 cm below the right costal margin. The urinary bladder was percussed to the level of the umbilicus. The rest of the general physical examination was unremarkable. Neurological examination revealed flaccid paralysis of the leg. The abdominal muscles were also flaccid. Arm strength was normal. Tendon reflexes were absent in the legs and there was no response to plantar stimulation. Abdominal and cremasteric reflexes were also absent. The rectal sphincter was patulous. Sensation was normal to touch, pinprick, position and vibratior throughout. QUESTIONS

  1. Plain x-rays and tomograms of the spine were normal. Which of the following are possible etiologies for the patients problem (Select one or more)

A. Metastatic spinal cord tumor

B. Herniated intervertebral disc

C. Spontaneous spinal hematoma

D. Spinal cord contusion

E. None of the above

A

(A,B,C,D) Any of the lesions listed could account for the patient’s problem. Metastatic spinal cord tumors are frequent lesions. Approximately 5% of all patients with malignancies develop spinal metastases at some time during their course. Of these, 97% are epidural in location. Spinal metastases most often occur in the thoracic region and produce rapid spinal core compression. The lack of known or obvious malignancy in this patient does not exclude a spinal metastasis as the latter may be the presenting symptom in a significant number of patients. Plain x-ray changes of an osteolytic or osteoblastic nature are usually present but not universally. Metastatic spinal lymphomas, for instance, often present without x-ray changes. Spinal and radicular pain occur in 90% of patients with spinal metastases and motor weakness is frequent. Up to 50-75% of patients with spinal metastases have weakness severe enough to prevent ambulation when first seen. Herniated intervertebral discs are uncommon in the upper thoracic region. A herniated disc at the C7-T1 interspace, a more frequent lesion, could account for the clinical signs and svmptoms. Spinal and radicular pain with paraplegia are common with herniated thoracic intervertebral discs. Spontaneous spinal hematomas, both epidural and subdural, are uncommon lesions. They frequently present with spinal pain and rapidly progressive spinal cord compression with paraplegia. A history of minor trauma or straining is common but not universal. Chronic alcoholics, such as the patient described here, may suffer repeated unnoticed minor trauma or even more severe trauma. Thus, the history may be unreliable. Further, spontaneous spinal hematomas are often associated with coagulation disorders. Such could easily be the case in a patient with hepatic dysfunction secondary to alcoholism. Spinal cord contusion with intramedullary hematoma or edema could account for the clinical syndrome also. Usually significant trauma is required to cause spinal cord contusion out the unreliable nature of the history In alcoholics might fail to uncover such an event. A variety of spinal lesions other than those listed could be included in the differential diagnosis. These might include primary spinal cord tumor, epidural abscess, intramedullary abscess, nonmalignant epidural tumors, transverse myelitis, and spinal cord infarction, among others.

148
Q

CASE 27: Interscapular Pain and Paraplegia in a 47-Year-Old Man HISTORY: A 47-year-old man was seen complaining of interscapular pain and weakness of his legs of two days’ duration. He was a chronic alcoholic. He gave no specific history of trauma. The pain was moderate in severity and radiated along the ulnar aspect of both upper arms and forearms. It was aggravated by straining and relieved by rest. The weakness of his legs had progressed to the point that the patient was unable to stand or walk. He had been unable to urinate for 24 hours. EXAMINATION: He was an obese male who was awake and cooperative but somewhat tremulous. Respirations were diaphragmatic. There was mild tenderness to percussion over the spinous processes of C7, T1 and T2. A firm, slightly tender liver edge was palpable 6 cm below the right costal margin. The urinary bladder was percussed to the level of the umbilicus. The rest of the general physical examination was unremarkable. Neurological examination revealed flaccid paralysis of the leg. The abdominal muscles were also flaccid. Arm strength was normal. Tendon reflexes were absent in the legs and there was no response to plantar stimulation. Abdominal and cremasteric reflexes were also absent. The rectal sphincter was patulous. Sensation was normal to touch, pinprick, position and vibratior throughout. QUESTIONS

  1. In view of the potential etiology, what further diagnostic studies arc indicated? (Select one or more)

A. Coagulation profile

B. Computerized tomographic brain scan

C. Myelogram

D. Metastatic x-ray survey of skeleton

E. GI series, barium enema and intravenous pyelogram

A

(A,C) In adults, spinal hematomas are frequently associated with coagulation disorders. In the case presented, prothrombin time and partial thromboplastin time were normal. Platelet count, however, was 40.000/mm3. Spinal hematomas have also been associated with the use of anticoagulants. Myelography is essential in the diagnosis of spinal lesions showing rapidly progressive neurological dysfunction. Regardless of the type of lesion suspected, only myelography will provide accurate localization of the spinal level, rostral and caudal extent and relationship to meninges and spinal cord of the causative lesion. Such factors are extremely useful in establishing a likely tissue diagnosis and planning therapy. In addition, cerebrospinal fluid obtained at myelography may be helpful in establishing a diagnosis in the event that a myelographic abnormality is not identified. The neurological picture does not suggest an intracranial lesion and, therefore, a brain scan is not indicated. While spiral metastases from an unsuspected cancer are extremely prominent in the differential diagnosis, attempts to search for a primary malignancy in view of the patient’s neurological deterioration are inappropriate. Thus, metastatic bone survey, GI series, barium enema and intravenous pyelogram would be time consuming and possibly unnecessary even in the long run. If myelography and biopsy confirm a malignancy, appropriate investigation to locate a primary tumor may be carried out at a later date.

149
Q

CASE 27: Interscapular Pain and Paraplegia in a 47-Year-Old Man HISTORY: A 47-year-old man was seen complaining of interscapular pain and weakness of his legs of two days’ duration. He was a chronic alcoholic. He gave no specific history of trauma. The pain was moderate in severity and radiated along the ulnar aspect of both upper arms and forearms. It was aggravated by straining and relieved by rest. The weakness of his legs had progressed to the point that the patient was unable to stand or walk. He had been unable to urinate for 24 hours. EXAMINATION: He was an obese male who was awake and cooperative but somewhat tremulous. Respirations were diaphragmatic. There was mild tenderness to percussion over the spinous processes of C7, T1 and T2. A firm, slightly tender liver edge was palpable 6 cm below the right costal margin. The urinary bladder was percussed to the level of the umbilicus. The rest of the general physical examination was unremarkable. Neurological examination revealed flaccid paralysis of the leg. The abdominal muscles were also flaccid. Arm strength was normal. Tendon reflexes were absent in the legs and there was no response to plantar stimulation. Abdominal and cremasteric reflexes were also absent. The rectal sphincter was patulous. Sensation was normal to touch, pinprick, position and vibratior throughout. QUESTIONS

  1. A myelogram was done (Figures 27. 1 and 27.2). It shows {Select only one)

A. an epidural block

B. a subdural block

C. an intramedullary block

D. none of the above

A

(A) The myelogram shows an epidural block. Contrast material was instilled rostrally at the C1-2 level and caudally via a lumbar puncture. The caudally instilled contrast (Figure 27.2) shows a smooth deviation of the contrast column from left to right and gradual tapering to the point of obstruction. The rostrally instilled contrast material (Figure 27.1) shows a more abrupt block typical of an epidural lesion. The myelogram also showed that the lesion extended from C7 to T6. In reference to question 2 concerning the differential diagnose the myelographic findings are compatible with possibilities A or C. Either an epidural metastatic tumor or spontaneous spinal epidural hematoma could produce an epidural myelography block extending over many spinal segments. A herniated intervertebral disc could also produce an epidural spinal block but the block should be localized opposite the intervertebral disc space involved and be ventral in. location. A spinal cord contusion, since it results in expansion of the spinal cord substance from within, should procuce the myelographic picture an intramedullary lesion.

150
Q

CASE 27: Interscapular Pain and Paraplegia in a 47-Year-Old Man HISTORY: A 47-year-old man was seen complaining of interscapular pain and weakness of his legs of two days’ duration. He was a chronic alcoholic. He gave no specific history of trauma. The pain was moderate in severity and radiated along the ulnar aspect of both upper arms and forearms. It was aggravated by straining and relieved by rest. The weakness of his legs had progressed to the point that the patient was unable to stand or walk. He had been unable to urinate for 24 hours. EXAMINATION: He was an obese male who was awake and cooperative but somewhat tremulous. Respirations were diaphragmatic. There was mild tenderness to percussion over the spinous processes of C7, T1 and T2. A firm, slightly tender liver edge was palpable 6 cm below the right costal margin. The urinary bladder was percussed to the level of the umbilicus. The rest of the general physical examination was unremarkable. Neurological examination revealed flaccid paralysis of the leg. The abdominal muscles were also flaccid. Arm strength was normal. Tendon reflexes were absent in the legs and there was no response to plantar stimulation. Abdominal and cremasteric reflexes were also absent. The rectal sphincter was patulous. Sensation was normal to touch, pinprick, position and vibratior throughout. QUESTIONS

  1. In view of the myelographic findings and clinical picture, you would suggest (Select one cr more)

A. observation and corticosteroids

B. radiotherapy

C. immediate laminectomy

D. observation followed by laminectomy if the patient fails to improve

E. none of the above

A

(C) Immediate laminectomy is indicated. These rapidly progressive clinical picture, negative plain x-rays of the spine, myelogram demonstrating an extensive epidural block and thrombocytopenia strongly suggest an epidural spinal hematoma. Metastatic epidural tumor is, however, not excluded. Either of these lesions are neurological emergencies and require immediate treatment. Exploration in this patient reveale an epidural hematoma of recent age, extending from C7 to T6. Spinal epidural hematomas are potentially benign lesions in that prompt evacuation can lead to total recovery without fear of recurrence. Delay in therapy, however, can lead to permanent severe residual neurological deficit. Because of the rapidity of onset of symptoms, the likelihood of reversal of deficit is much poorer than might be expected from the potentially benign nature of a hematoma. The etiology of spinal epidural hematomas is uncertain. Arteriosclerosis and hypertension have been suggested in entirely spontaneous cases, particularly in adults. In addition, coagulation defects, neoplasms, trauma, surgery, infections and angiomatous malformations have been implicated. In children, coagulation defects, sudden increases in intrathoracic pressure and epidural angiomas have been suggested as etiological agents. Spinal pain is an almost constant symptom and usually the first noted. Progression to spinal cord dysfunction including motor weakness, sensory loss and sphincter paralysis usually occurs within hours to a few days. However, recently, a 6-year-old girl was reported in whom spinal pain w2 s present for three months before neurological symptoms and signs developed. Regardless of the etiology, prompt evaluation and evacuation of the hematoma via laminectomy is essential if residual neurological deficit is to be avoided. In addition, surgical treatment will establish a correct tissue diagnosis and allow appropriate adjunctive treatment in undiagnosed epidural lesions. Particularly important is the identification of metastatic tumor which may benefit from chemotherapy or radiotherapy.

151
Q

CASE 28: Paraplegia Following a Fall in an 11-Month-Old Child HISTORY: An 11-month-old male was seen in the emergency room one hour after having fallen down fifteen stairs while in a baby walker. The child was not unconscious. EXAMINATION: Neurological examination was normal. There were minor contusions of the skin over the posterior surface of the shoulders. HOSPITAL COURSE: The child was discharged home. 4 hours later, he was returned to the emergency room. The patents described irritability and progressive decrease in use of his legs over 30-60 minutes. Repeat neurological examination disclosed an alert but slightly irritable child. Cranial nerve examination was normal. There was analgesia to pinprick to low the T5 level. A flaccid paraplegia was present and tendon reflexes were absent in the lower extremities. Motor and reflexes were normal in the arms. Rectal sphincter tone was diminished. QUESTIONS

  1. The clinical findings are most consistent with (Select only one)

A. an intracranial lesion

B. upper thoracic spinal cord lesion

C. a cauda equina lesion

D. multiple nerve root lesions

A

(B) The presence of a sensory level to pinprick, namely loss of pain sensation below a certain dermatomal level, is an excellent localizing sign of spinal cord pathology. The T5 pinprick level seen in this patient suggests an upper thoracic spinal cord lesion. The spinothalamic tract, which is involved in pinprick perception, angles in a cranial direction as it decussates in me spinal cord (see Figure 24.1, Case 24). This decussation occurs over two to three spinal segments. Thus, a spinal cord lesion at the T2-3 level would cause a loss of pinprick sensation from about T5 caudally. Flaccid paraplegia is also consistent with a spinal cord lesion. Since the musculature involved in hip flexion is supplied by spinal cord levels as high as L1, the lesion must be above this level. Since upper extremity function is normal, the lesion in the spinal cord must be below T1. The lowest spinal cord level which supplies motor innervation to the upper extremity is T1 to the intrinsic hand muscles. Thus, the paraplegia is consistent with a motor level between T1 and T12. Flaccid sphincter paralysis is also consistent with a spinal cord lesion. An intracranial lesion would not usually cause a loss of pinprick sensation bilaterally below a dermatomal level. .More likely it would cause a unilateral loss of sensation. Although a parasaggital lesion compressing the paracentral lobule of the frontal motor cortex (leg area) bilaterally could produce paraplegia, such lesions are rare. The essentially normal level of consciousness is also inconsistent with an intracranial lesion. A cauda equina lesion would not cause sensory loss above the T12-L1 level since those are the highest nerve roots involved in the cauda equina. Multiple nerve root injuries usually occur in relation to the brachial plexus or lumbosacral plexus and are most often unilateral. The picture of motor and sensory loss below a spinal level is most consistent with spinal cord pathology,

152
Q

CASE 28: Paraplegia Following a Fall in an 11-Month-Old Child HISTORY: An 11-month-old male was seen in the emergency room one hour after having fallen down fifteen stairs while in a baby walker. The child was not unconscious. EXAMINATION: Neurological examination was normal. There were minor contusions of the skin over the posterior surface of the shoulders. HOSPITAL COURSE: The child was discharged home. 4 hours later, he was returned to the emergency room. The patents described irritability and progressive decrease in use of his legs over 30-60 minutes. Repeat neurological examination disclosed an alert but slightly irritable child. Cranial nerve examination was normal. There was analgesia to pinprick to low the T5 level. A flaccid paraplegia was present and tendon reflexes were absent in the lower extremities. Motor and reflexes were normal in the arms. Rectal sphincter tone was diminished. QUESTIONS

  1. The pathologic process responsible for the clinical findings is most likely (Select only one)

A. a spinal cord contusion

B. spinal cord transection

C. an intraspinal hematoma

D. none of the above

A

(C) Spinal cord contusions and spinal transection present with symptoms and signs either immediately at the time of trauma or with In a few minutes or hours at most. Both lesions are usually stable shortly after the initial period of injury. Although progressive spinal cord necrosis has been shown to occur experimentally after spinal contusion, some deficit is usually present within a few minutes of trauma, if not immediately. The patient in question was normal neurologic ally for at least three hours after injury and then suffered progressive onset of deficit over about one hour. Tnis is most consistent with an expanding intraspinal mass such as a hematoma or herniated intervertebral disc.

153
Q

CASE 28: Paraplegia Following a Fall in an 11-Month-Old Child HISTORY: An 11-month-old male was seen in the emergency room one hour after having fallen down fifteen stairs while in a baby walker. The child was not unconscious. EXAMINATION: Neurological examination was normal. There were minor contusions of the skin over the posterior surface of the shoulders. HOSPITAL COURSE: The child was discharged home. 4 hours later, he was returned to the emergency room. The patents described irritability and progressive decrease in use of his legs over 30-60 minutes. Repeat neurological examination disclosed an alert but slightly irritable child. Cranial nerve examination was normal. There was analgesia to pinprick to low the T5 level. A flaccid paraplegia was present and tendon reflexes were absent in the lower extremities. Motor and reflexes were normal in the arms. Rectal sphincter tone was diminished. QUESTIONS

  1. Plain x-rays of the spine are normal. What course of action would you recommend in this patient ? (Select only one)

A. Hospital admission for observation

B Hospital admission for corticosteroid treatment

C. Immediate myelography

D. Myelography only if further neurological deterioration occurs

E. immediate laminectomy

A

(C) This child has shown evidence of progressive neurological deterioration which is localized to the spinal cord. Aggressive investigation is required and further procrastination is ill advised. Myelography is the best means of localizing the exact spinal level of the responsible lesion and is best carried out before any surgical procedures. Particularly in the thoracic spine, an anterior lesion such as a herniated intervertebral disc is best approached by a lateral or posterolateral approach rather than by laminectomy. Myelography is therefore required before a surgical procedure is undertaken.

154
Q

CASE 28: Paraplegia Following a Fall in an 11-Month-Old Child HISTORY: An 11-month-old male was seen in the emergency room one hour after having fallen down fifteen stairs while in a baby walker. The child was not unconscious. EXAMINATION: Neurological examination was normal. There were minor contusions of the skin over the posterior surface of the shoulders. HOSPITAL COURSE: The child was discharged home. 4 hours later, he was returned to the emergency room. The patents described irritability and progressive decrease in use of his legs over 30-60 minutes. Repeat neurological examination disclosed an alert but slightly irritable child. Cranial nerve examination was normal. There was analgesia to pinprick to low the T5 level. A flaccid paraplegia was present and tendon reflexes were absent in the lower extremities. Motor and reflexes were normal in the arms. Rectal sphincter tone was diminished. QUESTIONS

  1. The child was admitted to the hospital and a myelogram was performed (Figure 28.1). It shows a block to the flow of contrast material. The block is typical of (Select only one)

A. an intramedullary lesion

B. a subdural lesion

C. an extradural lesion

D. none of the above

A

(C) Extradural lesions produce progressive obliteration of the contrast column as the point of obstruction is approached. In this patient, the contrast column is obstructed at a higher level (T2) on the left and at a lower level (about T3) on the right (Figure 28.1) . Widening of the spinal cord shadow, characteristic of an intramedullary lesion (i.e. within the spinal cord) is not seen. Deviation of the spinal cord to one side characteristic of a subdural (intradural extramedullary) lesion is also not seen. Further, the “capping effect” seen in subdural lesions due to direct contact of the contrast column and lesion is also not seen. A lateral view did not show a lesion opposite the intervertebral disc space. Thus, the myelogram is most compatible with an extradural lesion, most likely a hematoma,

155
Q

CASE 28: Paraplegia Following a Fall in an 11-Month-Old Child HISTORY: An 11-month-old male was seen in the emergency room one hour after having fallen down fifteen stairs while in a baby walker. The child was not unconscious. EXAMINATION: Neurological examination was normal. There were minor contusions of the skin over the posterior surface of the shoulders. HOSPITAL COURSE: The child was discharged home. 4 hours later, he was returned to the emergency room. The patents described irritability and progressive decrease in use of his legs over 30-60 minutes. Repeat neurological examination disclosed an alert but slightly irritable child. Cranial nerve examination was normal. There was analgesia to pinprick to low the T5 level. A flaccid paraplegia was present and tendon reflexes were absent in the lower extremities. Motor and reflexes were normal in the arms. Rectal sphincter tone was diminished. QUESTIONS

  1. Recommended treatment for this child would be (Select only one)

A . further observation

B. decompressive laminectomy

C. corticosteroids

D. spinal fusion

E. none of the above

A

(B) Emergency decompressive laminectomy is indicated. Spinal cord injury due exclusively to intraspinal hematomas, particularly those outside the spinal cord substance itself, is rare. It is, however, one of the few causes of spinal cord injury amenable to surgical decompression. The likelihood of recovery of neurological function in spinal cord compression relates directly to the length of time compression has been present and the severity of the clinical lesion. Thus, urgent decompression is indicated. Further observation is contra indicated and could lead to irreversible neurological deficit. The child in this case underwent laminectomy from T3 to T8. No lesion was identified in the epidural space in spite of the myelographic findings. A bluish discoloration was visible through the dura. On opening the dura, a freshly clotted subdural hematoma was identified and removed. Zilkha commented on the difficulty in distinguishing epidural from subdural hematoma on myelography when complete spinal block is present. When subdural hematomas produce a myelographic defect but not a block, they are more easily distinguished from epidural hematomas. Progressive improvement in neurological function occurred postoperatively. The child became ambulatory and bladder and bowel function returned to normal. The value of corticosteroids in this situation is undefined. Although the child received corticosteroids preoperatively, they produced no lasting improvement in symptoms or signs. Corticosteroids may be of value to combat secondary spinal cord edema but removal of the offending compressive mass is the mainstay of therapy. Spinal fusion is unnecessary. No injury to bony vertebral column nor to supporting soft tissue structures was identified. Further, spinal fusion at tiiis age may interfere with proper vertebral column growth. Although instability may result from extensive laminectomy in the cervical and lumbar regions with delayed kyphosis, scoliosis or so-called “swan neck” deformity, it is unlikely in the thoracic region with the conservative laminectomy which would be required for drainage of an epidural spinal hematoma.

156
Q

CASE 28: Paraplegia Following a Fall in an 11-Month-Old Child HISTORY: An 11-month-old male was seen in the emergency room one hour after having fallen down fifteen stairs while in a baby walker. The child was not unconscious. EXAMINATION: Neurological examination was normal. There were minor contusions of the skin over the posterior surface of the shoulders. HOSPITAL COURSE: The child was discharged home. 4 hours later, he was returned to the emergency room. The patents described irritability and progressive decrease in use of his legs over 30-60 minutes. Repeat neurological examination disclosed an alert but slightly irritable child. Cranial nerve examination was normal. There was analgesia to pinprick to low the T5 level. A flaccid paraplegia was present and tendon reflexes were absent in the lower extremities. Motor and reflexes were normal in the arms. Rectal sphincter tone was diminished. QUESTIONS

  1. Which of the following are related to the occurrence of spinal subdural hematomas? (Select one or more)

A. Trauma

B. Anticoagulant use

C. Lumbar puncture

D. Hemophilia

E. None of the above

A

(A,B,C,D) Spinal subdural hematomas are extremely rare. The most common cause appears to be spinal trauma. Although the trauma may be severe enough to cause a spinal fracture, it may also be trivial and may not be recalled unless the patient is specifically questioned. Chronic spinal subdural hematomas have also been described six months and five years after spinal trauma. Occasionally spinal subdural hematomas occur without definable etiology. Coagulation abnormalities, such as the use of anticoagulant drugs and occurring in patients with leukemia have been reported as causes of spinal subdural hematomas. Likewise, lumbar puncture has been implicated. In some instances, the combination of a coagulation defect and lumbar puncture have been reported in association with spinal subdural hematoma.

157
Q

CASE 28: Paraplegia Following a Fall in an 11-Month-Old Child HISTORY: An 11-month-old male was seen in the emergency room one hour after having fallen down fifteen stairs while in a baby walker. The child was not unconscious. EXAMINATION: Neurological examination was normal. There were minor contusions of the skin over the posterior surface of the shoulders. HOSPITAL COURSE: The child was discharged home. 4 hours later, he was returned to the emergency room. The patents described irritability and progressive decrease in use of his legs over 30-60 minutes. Repeat neurological examination disclosed an alert but slightly irritable child. Cranial nerve examination was normal. There was analgesia to pinprick to low the T5 level. A flaccid paraplegia was present and tendon reflexes were absent in the lower extremities. Motor and reflexes were normal in the arms. Rectal sphincter tone was diminished. QUESTIONS

  1. In comparison to spinal epidural hematomas, spinal subdural hematomas (Select one or more)

A. are less common

B. have a better prognosis

C. have a more rapidly progressive course

D. none of the above

A

(A,B) Epidural hematomas are by far the commonest form of spinal hematoma. Subdural hematomas are less common and spinal subarachnoid hematomas (not subarachnoid hemorrhage) are extremely rare. The course of spinal subdural hematomas is less fulminantly, than that of spinal epidural hematomas and the prognosis of the former is probably slightly better.

158
Q

CASE 29: Post-traumatic Lumbar Herniated Disc with Cauda Equina Compression HISTORY: Twenty-four hours prior to hospital admission, this 42-year-old previously healthy female Was accidentally knocked over by her dog. She landed directly on her lower sacral region and experienced sudden low back and bilateral posterior thigh pain. The pain persisted and, although she noted no leg weakness, she became concerned because of difficulty emptying her bladder. She had a sensation of fullness in her lower abdomen and was able, in the last hours before admission, to feel her distended bladder. EXAMINATION: On admission, the patient was in obvious discomfort and efforts to move were accompanied by further complaints of pain. There was no point tenderness in her lumbar region although there was an ecchymosis over her lower sacrum. Lumbosacralparaspinal muscle spasm was noted. Straight leg raising signs were positive at 55° bilaterally. The motor examination of her lower extremities was normal. Tendon reflexes at the knees were normal and were slightly depressed at the ankles. On sensory examination, there was a slight diminution of pinprick perception over the saddle area bilaterally. A distended urinary bladder was palpable through the anterior abdominal wall and her anal tone was patulous. The urinary bladder was catheterized and 800 cc of urine was obtained. X-RAY: Plain lumbosacral spine x-rays were normal without evidence of fractures or dislocation. A pantopaoue myelogram demonstrated a complete block at L4-5. The block appeared to be extradural in nature (Figures 29.1A and 29.2B) . HOSPITAL COURSE: The patient was taken directly to the operating room where a large herniated L4-5 disc fragment was removed via an L5 laminectomy. Postoperatively her neurological status was unchanged except that the zone of saddle sensory loss was complete. TWELVE MONTH FOLLOWUP: The patient as neurological status was unchanged, In that she still had saddle anesthesia. She was, however, free of pain and had returned to work. She had voluntary control of voiding but had occasional urinary and fecal incontinence with coughing or straining. QUESTIONS

  1. Cauda equina compression without spinal cord compression can occur with lesions as high as T12. (True or False)
A

TRUE The caudal spinal cord usually tapers to an end opposite the L1 vertebral body but the point of termination maybe as high as the middle of the T12 body. Therefore, pure cauda equina compression may occur with lesions as high as T12.

159
Q

CASE 29: Post-traumatic Lumbar Herniated Disc with Cauda Equina Compression HISTORY: Twenty-four hours prior to hospital admission, this 42-year-old previously healthy female Was accidentally knocked over by her dog. She landed directly on her lower sacral region and experienced sudden low back and bilateral posterior thigh pain. The pain persisted and, although she noted no leg weakness, she became concerned because of difficulty emptying her bladder. She had a sensation of fullness in her lower abdomen and was able, in the last hours before admission, to feel her distended bladder. EXAMINATION: On admission, the patient was in obvious discomfort and efforts to move were accompanied by further complaints of pain. There was no point tenderness in her lumbar region although there was an ecchymosis over her lower sacrum. Lumbosacralparaspinal muscle spasm was noted. Straight leg raising signs were positive at 55° bilaterally. The motor examination of her lower extremities was normal. Tendon reflexes at the knees were normal and were slightly depressed at the ankles. On sensory examination, there was a slight diminution of pinprick perception over the saddle area bilaterally. A distended urinary bladder was palpable through the anterior abdominal wall and her anal tone was patulous. The urinary bladder was catheterized and 800 cc of urine was obtained. X-RAY: Plain lumbosacral spine x-rays were normal without evidence of fractures or dislocation. A pantopaoue myelogram demonstrated a complete block at L4-5. The block appeared to be extradural in nature (Figures 29.1A and 29.2B) . HOSPITAL COURSE: The patient was taken directly to the operating room where a large herniated L4-5 disc fragment was removed via an L5 laminectomy. Postoperatively her neurological status was unchanged except that the zone of saddle sensory loss was complete. TWELVE MONTH FOLLOWUP: The patient as neurological status was unchanged, In that she still had saddle anesthesia. She was, however, free of pain and had returned to work. She had voluntary control of voiding but had occasional urinary and fecal incontinence with coughing or straining. QUESTIONS

  1. Signs and symptoms of cauda equina compression may elude (select one or more)

A. anal and bladder sphincter disturbance

B. limb muscle weakness

C. sensory deficit

D. pain

E. none of the above

A

(A,B,C,D) In general, sphincter and bladder dysfunction are considered the common denominator of cauda equina syndromes (CES). Occasionally, patients may have other clear-cut indications of cauda equina compression without evidence of bladder or sphincter disturbance. Limb muscle weakness, pain and sensory deficit may also occur as part of CES. Limb weakness, if present, usually occurs below the knees. Pain may be located in the low back region or may be unilateral or bilateral radiating radicular type pain. Sensory disturbances, if present, usually involve the saddle distribution, but may be unilateral.

160
Q

CASE 29: Post-traumatic Lumbar Herniated Disc with Cauda Equina Compression HISTORY: Twenty-four hours prior to hospital admission, this 42-year-old previously healthy female Was accidentally knocked over by her dog. She landed directly on her lower sacral region and experienced sudden low back and bilateral posterior thigh pain. The pain persisted and, although she noted no leg weakness, she became concerned because of difficulty emptying her bladder. She had a sensation of fullness in her lower abdomen and was able, in the last hours before admission, to feel her distended bladder. EXAMINATION: On admission, the patient was in obvious discomfort and efforts to move were accompanied by further complaints of pain. There was no point tenderness in her lumbar region although there was an ecchymosis over her lower sacrum. Lumbosacralparaspinal muscle spasm was noted. Straight leg raising signs were positive at 55° bilaterally. The motor examination of her lower extremities was normal. Tendon reflexes at the knees were normal and were slightly depressed at the ankles. On sensory examination, there was a slight diminution of pinprick perception over the saddle area bilaterally. A distended urinary bladder was palpable through the anterior abdominal wall and her anal tone was patulous. The urinary bladder was catheterized and 800 cc of urine was obtained. X-RAY: Plain lumbosacral spine x-rays were normal without evidence of fractures or dislocation. A pantopaoue myelogram demonstrated a complete block at L4-5. The block appeared to be extradural in nature (Figures 29.1A and 29.2B) . HOSPITAL COURSE: The patient was taken directly to the operating room where a large herniated L4-5 disc fragment was removed via an L5 laminectomy. Postoperatively her neurological status was unchanged except that the zone of saddle sensory loss was complete. TWELVE MONTH FOLLOWUP: The patient as neurological status was unchanged, In that she still had saddle anesthesia. She was, however, free of pain and had returned to work. She had voluntary control of voiding but had occasional urinary and fecal incontinence with coughing or straining. QUESTIONS

  1. Sphincter disturbances may occur with cauda equina compression in the absence of (Select one or more)

A. limb weakness

B. sensory deficit

C. pain

D. deep tendon reflex changes

A

(A,B,C,D) Limb weakness may be absent with CES associated with bladder and sphincter disturbance. Limb weakness was found in approximately 66% of patients with CES. Sensory deficit is more commonly (80-100%) associated with CES but is not necessarily associated with sphincter or bladder disturbance. Likewise, pain occurs in most cases of CES but may be absent from the onset or disappear during the course of this syndrome. Diminution of deep tendon reflexes in association with CES was unusual at the knee but occurred at the ankle in approximately 60% of patients. Absence of reflex abnormalities in CES associated with sphincter or bladder dysfunction is attributed to midline compression of the more medially placed lower sacral nerve roots, This spares the S1 nerve root which subserves the ankle jerk but compresses the roots from S2 and below which subserve parasympathetic and somatic bladder and sphincter activity.

161
Q

CASE 29: Post-traumatic Lumbar Herniated Disc with Cauda Equina Compression HISTORY: Twenty-four hours prior to hospital admission, this 42-year-old previously healthy female Was accidentally knocked over by her dog. She landed directly on her lower sacral region and experienced sudden low back and bilateral posterior thigh pain. The pain persisted and, although she noted no leg weakness, she became concerned because of difficulty emptying her bladder. She had a sensation of fullness in her lower abdomen and was able, in the last hours before admission, to feel her distended bladder. EXAMINATION: On admission, the patient was in obvious discomfort and efforts to move were accompanied by further complaints of pain. There was no point tenderness in her lumbar region although there was an ecchymosis over her lower sacrum. Lumbosacralparaspinal muscle spasm was noted. Straight leg raising signs were positive at 55° bilaterally. The motor examination of her lower extremities was normal. Tendon reflexes at the knees were normal and were slightly depressed at the ankles. On sensory examination, there was a slight diminution of pinprick perception over the saddle area bilaterally. A distended urinary bladder was palpable through the anterior abdominal wall and her anal tone was patulous. The urinary bladder was catheterized and 800 cc of urine was obtained. X-RAY: Plain lumbosacral spine x-rays were normal without evidence of fractures or dislocation. A pantopaoue myelogram demonstrated a complete block at L4-5. The block appeared to be extradural in nature (Figures 29.1A and 29.2B) . HOSPITAL COURSE: The patient was taken directly to the operating room where a large herniated L4-5 disc fragment was removed via an L5 laminectomy. Postoperatively her neurological status was unchanged except that the zone of saddle sensory loss was complete. TWELVE MONTH FOLLOWUP: The patient as neurological status was unchanged, In that she still had saddle anesthesia. She was, however, free of pain and had returned to work. She had voluntary control of voiding but had occasional urinary and fecal incontinence with coughing or straining. QUESTIONS

  1. Cauda equina compression syndromes usually occur in association with herniated lumbar intervertebral discs. (True or False)
A

FALSE The incidence of CES associated with lumbar disc prolapse varies from 1-15%.

162
Q

CASE 29: Post-traumatic Lumbar Herniated Disc with Cauda Equina Compression HISTORY: Twenty-four hours prior to hospital admission, this 42-year-old previously healthy female Was accidentally knocked over by her dog. She landed directly on her lower sacral region and experienced sudden low back and bilateral posterior thigh pain. The pain persisted and, although she noted no leg weakness, she became concerned because of difficulty emptying her bladder. She had a sensation of fullness in her lower abdomen and was able, in the last hours before admission, to feel her distended bladder. EXAMINATION: On admission, the patient was in obvious discomfort and efforts to move were accompanied by further complaints of pain. There was no point tenderness in her lumbar region although there was an ecchymosis over her lower sacrum. Lumbosacralparaspinal muscle spasm was noted. Straight leg raising signs were positive at 55° bilaterally. The motor examination of her lower extremities was normal. Tendon reflexes at the knees were normal and were slightly depressed at the ankles. On sensory examination, there was a slight diminution of pinprick perception over the saddle area bilaterally. A distended urinary bladder was palpable through the anterior abdominal wall and her anal tone was patulous. The urinary bladder was catheterized and 800 cc of urine was obtained. X-RAY: Plain lumbosacral spine x-rays were normal without evidence of fractures or dislocation. A pantopaoue myelogram demonstrated a complete block at L4-5. The block appeared to be extradural in nature (Figures 29.1A and 29.2B) . HOSPITAL COURSE: The patient was taken directly to the operating room where a large herniated L4-5 disc fragment was removed via an L5 laminectomy. Postoperatively her neurological status was unchanged except that the zone of saddle sensory loss was complete. TWELVE MONTH FOLLOWUP: The patient as neurological status was unchanged, In that she still had saddle anesthesia. She was, however, free of pain and had returned to work. She had voluntary control of voiding but had occasional urinary and fecal incontinence with coughing or straining. QUESTIONS

  1. Most cauda equina compression syndromes due to herniated lumbar discs are associated with a history of trauma. (True or False)
A

FALSE The role of trauma in the onset of CES with lumbar disc herniation varies greatly from report to report but, in general, a history of trauma is not the rule, in studies containing a high proportion of patients with a history of trauma (i.e. 75% of the patients in Shephard’s series), many of the trauma events involve “strain” (e.g. lifting of heavy weights).

163
Q

CASE 29: Post-traumatic Lumbar Herniated Disc with Cauda Equina Compression HISTORY: Twenty-four hours prior to hospital admission, this 42-year-old previously healthy female Was accidentally knocked over by her dog. She landed directly on her lower sacral region and experienced sudden low back and bilateral posterior thigh pain. The pain persisted and, although she noted no leg weakness, she became concerned because of difficulty emptying her bladder. She had a sensation of fullness in her lower abdomen and was able, in the last hours before admission, to feel her distended bladder. EXAMINATION: On admission, the patient was in obvious discomfort and efforts to move were accompanied by further complaints of pain. There was no point tenderness in her lumbar region although there was an ecchymosis over her lower sacrum. Lumbosacralparaspinal muscle spasm was noted. Straight leg raising signs were positive at 55° bilaterally. The motor examination of her lower extremities was normal. Tendon reflexes at the knees were normal and were slightly depressed at the ankles. On sensory examination, there was a slight diminution of pinprick perception over the saddle area bilaterally. A distended urinary bladder was palpable through the anterior abdominal wall and her anal tone was patulous. The urinary bladder was catheterized and 800 cc of urine was obtained. X-RAY: Plain lumbosacral spine x-rays were normal without evidence of fractures or dislocation. A pantopaoue myelogram demonstrated a complete block at L4-5. The block appeared to be extradural in nature (Figures 29.1A and 29.2B) . HOSPITAL COURSE: The patient was taken directly to the operating room where a large herniated L4-5 disc fragment was removed via an L5 laminectomy. Postoperatively her neurological status was unchanged except that the zone of saddle sensory loss was complete. TWELVE MONTH FOLLOWUP: The patient as neurological status was unchanged, In that she still had saddle anesthesia. She was, however, free of pain and had returned to work. She had voluntary control of voiding but had occasional urinary and fecal incontinence with coughing or straining. QUESTIONS

  1. The level of the herniated lumbar disc in cases presenting with cauda equina compression can usually be determined by (Select one or more)

A. sensory level

B. reflex changes

C. muscle weakness

D. plain x-ray findings

E. none of the above

A

(E) CES due to herniated lumbar discs, occur most commonly at the L4-5 and L5-S1 levels followed by the L2-3 and L3-4 levels. It is difficult to make an accurate determination of the level of the compressive lesion based on the sensory, reflex or muscular examination. Plain lumbosacral x-rays in these patients commonly show evidence of nonspecific degenerative changes but rarely have localizing value and may occasionally be normal.

164
Q

CASE 29: Post-traumatic Lumbar Herniated Disc with Cauda Equina Compression HISTORY: Twenty-four hours prior to hospital admission, this 42-year-old previously healthy female Was accidentally knocked over by her dog. She landed directly on her lower sacral region and experienced sudden low back and bilateral posterior thigh pain. The pain persisted and, although she noted no leg weakness, she became concerned because of difficulty emptying her bladder. She had a sensation of fullness in her lower abdomen and was able, in the last hours before admission, to feel her distended bladder. EXAMINATION: On admission, the patient was in obvious discomfort and efforts to move were accompanied by further complaints of pain. There was no point tenderness in her lumbar region although there was an ecchymosis over her lower sacrum. Lumbosacralparaspinal muscle spasm was noted. Straight leg raising signs were positive at 55° bilaterally. The motor examination of her lower extremities was normal. Tendon reflexes at the knees were normal and were slightly depressed at the ankles. On sensory examination, there was a slight diminution of pinprick perception over the saddle area bilaterally. A distended urinary bladder was palpable through the anterior abdominal wall and her anal tone was patulous. The urinary bladder was catheterized and 800 cc of urine was obtained. X-RAY: Plain lumbosacral spine x-rays were normal without evidence of fractures or dislocation. A pantopaoue myelogram demonstrated a complete block at L4-5. The block appeared to be extradural in nature (Figures 29.1A and 29.2B) . HOSPITAL COURSE: The patient was taken directly to the operating room where a large herniated L4-5 disc fragment was removed via an L5 laminectomy. Postoperatively her neurological status was unchanged except that the zone of saddle sensory loss was complete. TWELVE MONTH FOLLOWUP: The patient as neurological status was unchanged, In that she still had saddle anesthesia. She was, however, free of pain and had returned to work. She had voluntary control of voiding but had occasional urinary and fecal incontinence with coughing or straining. QUESTIONS

  1. Myelography always demonstrates an extradural defect in cases of herniated lumbar disc. (True or False)
A

FALSE Complete myelography blocks, which manifest as a sharply cut off or feathered interface at the site of blockage, occur in a high proportion of patients with CES from herniated lumbar disc. Occasionally, myelography in these patients may show extradural defects without a complete block. Even more rarely, myelography may be interpreted as normal. Particularly with pantopaque myelograms, relatively small anteriorly placed lesions may be obscured by the dense contrast material.

165
Q

CASE 29: Post-traumatic Lumbar Herniated Disc with Cauda Equina Compression HISTORY: Twenty-four hours prior to hospital admission, this 42-year-old previously healthy female Was accidentally knocked over by her dog. She landed directly on her lower sacral region and experienced sudden low back and bilateral posterior thigh pain. The pain persisted and, although she noted no leg weakness, she became concerned because of difficulty emptying her bladder. She had a sensation of fullness in her lower abdomen and was able, in the last hours before admission, to feel her distended bladder. EXAMINATION: On admission, the patient was in obvious discomfort and efforts to move were accompanied by further complaints of pain. There was no point tenderness in her lumbar region although there was an ecchymosis over her lower sacrum. Lumbosacralparaspinal muscle spasm was noted. Straight leg raising signs were positive at 55° bilaterally. The motor examination of her lower extremities was normal. Tendon reflexes at the knees were normal and were slightly depressed at the ankles. On sensory examination, there was a slight diminution of pinprick perception over the saddle area bilaterally. A distended urinary bladder was palpable through the anterior abdominal wall and her anal tone was patulous. The urinary bladder was catheterized and 800 cc of urine was obtained. X-RAY: Plain lumbosacral spine x-rays were normal without evidence of fractures or dislocation. A pantopaoue myelogram demonstrated a complete block at L4-5. The block appeared to be extradural in nature (Figures 29.1A and 29.2B) . HOSPITAL COURSE: The patient was taken directly to the operating room where a large herniated L4-5 disc fragment was removed via an L5 laminectomy. Postoperatively her neurological status was unchanged except that the zone of saddle sensory loss was complete. TWELVE MONTH FOLLOWUP: The patient as neurological status was unchanged, In that she still had saddle anesthesia. She was, however, free of pain and had returned to work. She had voluntary control of voiding but had occasional urinary and fecal incontinence with coughing or straining. QUESTIONS

  1. Immediate surgery is indicated in all cases of cauda equina compression due to herniated lumbar discs. (True or False)
A

TRUE Surgical removal of the compressive lesion commonly relieves pain and motor weakness, but is less effective in relieving sensory and sphincter disturbances. Most authors agree that the earlier surgery is performed after the onset of symptoms of CES the better the outlook for preservation of all neuronal functions.

166
Q

CASE 29: Post-traumatic Lumbar Herniated Disc with Cauda Equina Compression HISTORY: Twenty-four hours prior to hospital admission, this 42-year-old previously healthy female Was accidentally knocked over by her dog. She landed directly on her lower sacral region and experienced sudden low back and bilateral posterior thigh pain. The pain persisted and, although she noted no leg weakness, she became concerned because of difficulty emptying her bladder. She had a sensation of fullness in her lower abdomen and was able, in the last hours before admission, to feel her distended bladder. EXAMINATION: On admission, the patient was in obvious discomfort and efforts to move were accompanied by further complaints of pain. There was no point tenderness in her lumbar region although there was an ecchymosis over her lower sacrum. Lumbosacralparaspinal muscle spasm was noted. Straight leg raising signs were positive at 55° bilaterally. The motor examination of her lower extremities was normal. Tendon reflexes at the knees were normal and were slightly depressed at the ankles. On sensory examination, there was a slight diminution of pinprick perception over the saddle area bilaterally. A distended urinary bladder was palpable through the anterior abdominal wall and her anal tone was patulous. The urinary bladder was catheterized and 800 cc of urine was obtained. X-RAY: Plain lumbosacral spine x-rays were normal without evidence of fractures or dislocation. A pantopaoue myelogram demonstrated a complete block at L4-5. The block appeared to be extradural in nature (Figures 29.1A and 29.2B) . HOSPITAL COURSE: The patient was taken directly to the operating room where a large herniated L4-5 disc fragment was removed via an L5 laminectomy. Postoperatively her neurological status was unchanged except that the zone of saddle sensory loss was complete. TWELVE MONTH FOLLOWUP: The patient as neurological status was unchanged, In that she still had saddle anesthesia. She was, however, free of pain and had returned to work. She had voluntary control of voiding but had occasional urinary and fecal incontinence with coughing or straining. QUESTIONS

  1. At surgery, lumbar disc herniations causing cauda equinas compression originate centrally as opposed to laterally. (True or False)
A

FALSE The point of lumbar disc herniation in patients with CES is usually lateral. In ten patients surgically explored only one had herniation originating from a defect in the midline annulus fibrosis.

167
Q

CASE 29: Post-traumatic Lumbar Herniated Disc with Cauda Equina Compression HISTORY: Twenty-four hours prior to hospital admission, this 42-year-old previously healthy female Was accidentally knocked over by her dog. She landed directly on her lower sacral region and experienced sudden low back and bilateral posterior thigh pain. The pain persisted and, although she noted no leg weakness, she became concerned because of difficulty emptying her bladder. She had a sensation of fullness in her lower abdomen and was able, in the last hours before admission, to feel her distended bladder. EXAMINATION: On admission, the patient was in obvious discomfort and efforts to move were accompanied by further complaints of pain. There was no point tenderness in her lumbar region although there was an ecchymosis over her lower sacrum. Lumbosacralparaspinal muscle spasm was noted. Straight leg raising signs were positive at 55° bilaterally. The motor examination of her lower extremities was normal. Tendon reflexes at the knees were normal and were slightly depressed at the ankles. On sensory examination, there was a slight diminution of pinprick perception over the saddle area bilaterally. A distended urinary bladder was palpable through the anterior abdominal wall and her anal tone was patulous. The urinary bladder was catheterized and 800 cc of urine was obtained. X-RAY: Plain lumbosacral spine x-rays were normal without evidence of fractures or dislocation. A pantopaoue myelogram demonstrated a complete block at L4-5. The block appeared to be extradural in nature (Figures 29.1A and 29.2B) . HOSPITAL COURSE: The patient was taken directly to the operating room where a large herniated L4-5 disc fragment was removed via an L5 laminectomy. Postoperatively her neurological status was unchanged except that the zone of saddle sensory loss was complete. TWELVE MONTH FOLLOWUP: The patient as neurological status was unchanged, In that she still had saddle anesthesia. She was, however, free of pain and had returned to work. She had voluntary control of voiding but had occasional urinary and fecal incontinence with coughing or straining. QUESTIONS

  1. Recovery of voluntary urination in this patient probably reflects full recovery of neurogenic bladder control. (True or False)
A

FALSE Normal bladder function does not occur with the saddle anesthesia of CES. Normal bladder emptying depends on reflex contraction and external compression of the bladder. Saddle anesthesia indicates that the pathway for the afferent limb of reflex bladder contractions is abolished. However, as with this patient, recovery of independent urinary voiding may be achieved by teaching the patient to empty the bladder with methods which increase intraabdominal pressure and compress the bladder. Thus, effective voluntary bladder emptying is achieved in a high proportion of patients as they learn crede and valsalva maneuvers. Assessment of bladder dysfunction may be accomplished by measuring of postvoiding residual, urine volume, cystoscopy and cystometrographic analysis.

168
Q

CASE 29: Post-traumatic Lumbar Herniated Disc with Cauda Equina Compression HISTORY: Twenty-four hours prior to hospital admission, this 42-year-old previously healthy female Was accidentally knocked over by her dog. She landed directly on her lower sacral region and experienced sudden low back and bilateral posterior thigh pain. The pain persisted and, although she noted no leg weakness, she became concerned because of difficulty emptying her bladder. She had a sensation of fullness in her lower abdomen and was able, in the last hours before admission, to feel her distended bladder. EXAMINATION: On admission, the patient was in obvious discomfort and efforts to move were accompanied by further complaints of pain. There was no point tenderness in her lumbar region although there was an ecchymosis over her lower sacrum. Lumbosacralparaspinal muscle spasm was noted. Straight leg raising signs were positive at 55° bilaterally. The motor examination of her lower extremities was normal. Tendon reflexes at the knees were normal and were slightly depressed at the ankles. On sensory examination, there was a slight diminution of pinprick perception over the saddle area bilaterally. A distended urinary bladder was palpable through the anterior abdominal wall and her anal tone was patulous. The urinary bladder was catheterized and 800 cc of urine was obtained. X-RAY: Plain lumbosacral spine x-rays were normal without evidence of fractures or dislocation. A pantopaoue myelogram demonstrated a complete block at L4-5. The block appeared to be extradural in nature (Figures 29.1A and 29.2B) . HOSPITAL COURSE: The patient was taken directly to the operating room where a large herniated L4-5 disc fragment was removed via an L5 laminectomy. Postoperatively her neurological status was unchanged except that the zone of saddle sensory loss was complete. TWELVE MONTH FOLLOWUP: The patient as neurological status was unchanged, In that she still had saddle anesthesia. She was, however, free of pain and had returned to work. She had voluntary control of voiding but had occasional urinary and fecal incontinence with coughing or straining. QUESTIONS

  1. In the cauda equina syndrome, a good correlation exists between deficits of bladder control, anal control and sexual function (True or False)
A

FALSE Erection and ejaculation are possible in the presence of bladder dysfunction and saddle anesthesia associated with CES from a herniated lumbar disc. In fact, sexual function is usually adequate in these patients. Similarly, bowel and anal symptoms are less frequent with CES than bladder distances

169
Q

CASE 30: Lap-type Seat Belt Injury HISTORY: A 17-year-old female was involved in a head-on automobiie collision. She was the front seat passenger and was wearing a snugly applied lap-type seat belt at the time of the accident. Her only complaint following the accident was of severe low back pain. EXAMINATION: on examination in emergency vital signs were stable. She had an ecchymosis extending across the lower abdomen between the superior iliac crests. There was tenderness over the mid lumbar spinal area. Otherwise her general physical and neurological examination was normal X-RAY: Lumbar spine x-rays showed a transverse fracture through the posterior elements of L4 with anterior angulation of L3 on L4 2nd anterior wedging of the body of L4. There was no subluxation (Figures 30.1A and 30.1B). QUESTIONS

  1. Lumbar fracture with lap-type seat belt injury is usually associated with visceral injury. (True or False)
A

FALSE Only approximately 15% of patients with lumbar fractures from lap-type seat belt injuries have associated visceral injuries. Visceral injuries are more commonly associated with improper seat belt placement (too high, too loose).

170
Q

CASE 30: Lap-type Seat Belt Injury HISTORY: A 17-year-old female was involved in a head-on automobiie collision. She was the front seat passenger and was wearing a snugly applied lap-type seat belt at the time of the accident. Her only complaint following the accident was of severe low back pain. EXAMINATION: on examination in emergency vital signs were stable. She had an ecchymosis extending across the lower abdomen between the superior iliac crests. There was tenderness over the mid lumbar spinal area. Otherwise her general physical and neurological examination was normal X-RAY: Lumbar spine x-rays showed a transverse fracture through the posterior elements of L4 with anterior angulation of L3 on L4 2nd anterior wedging of the body of L4. There was no subluxation (Figures 30.1A and 30.1B). QUESTIONS

  1. As with this patient, the vertebral level usually involved with lap-type seat belt injuries in L4. (True or False)
A

FALSE Although vertebral fractures associated with lap-type scat be it injuries are seen from T12 to L5 they are most commonly at L1, L2 and L3.

171
Q

CASE 30: Lap-type Seat Belt Injury HISTORY: A 17-year-old female was involved in a head-on automobiie collision. She was the front seat passenger and was wearing a snugly applied lap-type seat belt at the time of the accident. Her only complaint following the accident was of severe low back pain. EXAMINATION: on examination in emergency vital signs were stable. She had an ecchymosis extending across the lower abdomen between the superior iliac crests. There was tenderness over the mid lumbar spinal area. Otherwise her general physical and neurological examination was normal X-RAY: Lumbar spine x-rays showed a transverse fracture through the posterior elements of L4 with anterior angulation of L3 on L4 2nd anterior wedging of the body of L4. There was no subluxation (Figures 30.1A and 30.1B). QUESTIONS

  1. The absence of neurological injury seen with this patient is the rule for vertebral fractures associated with the use of lap-type seat belts. (True or False)
A

TRUE Only 15-20% of spinal fractures associated with lap-type seat belt injuries had associated weakness or sensory impairment below the level of the injury.

172
Q

CASE 30: Lap-type Seat Belt Injury HISTORY: A 17-year-old female was involved in a head-on automobiie collision. She was the front seat passenger and was wearing a snugly applied lap-type seat belt at the time of the accident. Her only complaint following the accident was of severe low back pain. EXAMINATION: on examination in emergency vital signs were stable. She had an ecchymosis extending across the lower abdomen between the superior iliac crests. There was tenderness over the mid lumbar spinal area. Otherwise her general physical and neurological examination was normal X-RAY: Lumbar spine x-rays showed a transverse fracture through the posterior elements of L4 with anterior angulation of L3 on L4 2nd anterior wedging of the body of L4. There was no subluxation (Figures 30.1A and 30.1B). QUESTIONS

  1. The lumbar vertebral injury related to the use of lap-type seat belts is characterized by (Select one or more)

A. ruptures of posterior ligaments and facet joints

B. fractures of posterior elements and facet joints

C. compression of the vertebral body

D. subluxation of vertebral elements

E. none of the above

A

(A, B) In the usual hyperflexion injury of the lumbar spine (i.e. fracture associated with a fall), the stress forces rotate around a fulcrum within the vertebral elements themselves causing compression fracture. In lap-type seat belt injuries, the fulcrum of forces is displaced anteriorly to the point of contact between the seat belt and the anterior abdominal wall. Because the spine is completely behind this fulcrum, all of its elements are subjected to tension (e.g. fission, pulling apart) stress as the upper and lower parts of the body are thrown forward at the time of deacceleration of the vehicle. The tension stress causes distraction of vertebral elements with rupture of nonbony elements (i.e. posterior ligaments, facet ligaments). If the ligaments remain intact, fission-type fractures of the posterior bony elements and, at times, the vertebral body may ensue. Therefore, the lap-tvpe seat belt vertebral injury is usually a distraction of ligamentous, facet or bony elements without compression or subluxation.

173
Q

CASE 30: Lap-type Seat Belt Injury HISTORY: A 17-year-old female was involved in a head-on automobiie collision. She was the front seat passenger and was wearing a snugly applied lap-type seat belt at the time of the accident. Her only complaint following the accident was of severe low back pain. EXAMINATION: on examination in emergency vital signs were stable. She had an ecchymosis extending across the lower abdomen between the superior iliac crests. There was tenderness over the mid lumbar spinal area. Otherwise her general physical and neurological examination was normal X-RAY: Lumbar spine x-rays showed a transverse fracture through the posterior elements of L4 with anterior angulation of L3 on L4 2nd anterior wedging of the body of L4. There was no subluxation (Figures 30.1A and 30.1B). QUESTIONS

  1. Therapeutic management of this patient might reasonably include (Select one or more)

A. early mobilization and discharge

B. prolonged bed rest in the hyperextended position and or bracing or casting in the hyperextended position

C . operative fusion

D. laminectomy combined with fusion

E. none of the above

A

(B, C) There are no reports of long term follow up for these patients. Presumably, however, such fractures are initially unstable particularly if there is ligamentous or facet disruption and they require some form of stabilization. Management has, therefore, consisted of bed rest or hyperextension casting or operative fusion. Laminectomy might be indicated if there is deterioration of neurological function below the level of the injury.

174
Q

CASE 31: Spinal Injury Associated with Gunshot Wound HISTORY: A 22-year-old male was struck from the rear, by a bullet, in the upper lumbar region. He was knocked over by the impact and noted immediate tingling in both lower extremities and had difficulty moving his right leg. By the time he arrived in the emergency room, the tingling was predominantly in the right leg. EXAMINATION In the emergency room, his vital signs were normal ana stable. The bullet entry wound was noted at the right dorsal paramidline upper lumbar region. Rectal tone and voluntary sphincter contractions were normal and the patient was able to urinate upon request. His neurological examination was abnormal in the lower extremities. There was 4+/5 weakness in his anterior tibial, extensor hallucis longus, peroneus longus, posterior tibial and gastrocnemius muscles in the right kg. Knee reflexes were normal; the right ankle reflex was slightly diminished Hyperpathia was noted over the whole of the right leg from the L2 region down to and including the sacral, perianal region. Hypalgesia was noted over the same disiribution on the right and over the L2-S1 dermatomes on the left. joint position sense was normal. X-RAY: AP and lateral lumbosacral x-rays showed a fragmented bullet in the spinal canal at the L2 level. There was.no evidence of bony disruption (Figures 31.1A and 31.1B). The patient was observed for several hours and showed no signs i neurological deterioration. The entry wound was superficially debrided and further surgery was not recommended. QUESTIONS

  1. The lack of a fracture identified on plain x-rays may be misleading in this patient. (True or False)
A

TRUE It is not unusual that fractures are detected at surgery which were not seen on plain x-rays.

175
Q

CASE 31: Spinal Injury Associated with Gunshot Wound HISTORY: A 22-year-old male was struck from the rear, by a bullet, in the upper lumbar region. He was knocked over by the impact and noted immediate tingling in both lower extremities and had difficulty moving his right leg. By the time he arrived in the emergency room, the tingling was predominantly in the right leg. EXAMINATION In the emergency room, his vital signs were normal ana stable. The bullet entry wound was noted at the right dorsal paramidline upper lumbar region. Rectal tone and voluntary sphincter contractions were normal and the patient was able to urinate upon request. His neurological examination was abnormal in the lower extremities. There was 4+/5 weakness in his anterior tibial, extensor hallucis longus, peroneus longus, posterior tibial and gastrocnemius muscles in the right kg. Knee reflexes were normal; the right ankle reflex was slightly diminished Hyperpathia was noted over the whole of the right leg from the L2 region down to and including the sacral, perianal region. Hypalgesia was noted over the same disiribution on the right and over the L2-S1 dermatomes on the left. joint position sense was normal. X-RAY: AP and lateral lumbosacral x-rays showed a fragmented bullet in the spinal canal at the L2 level. There was.no evidence of bony disruption (Figures 31.1A and 31.1B). The patient was observed for several hours and showed no signs i neurological deterioration. The entry wound was superficially debrided and further surgery was not recommended QUESTIONS

  1. A lumbar puncture to evaluate the presence of blood and subbarachnoid spinal block (Queckenstedt test) is indicated in this patient. (True or False)
A

FALSE Neither the presence nor the absence of blood nor subarachnoid spinal block (as determined by the Queckenstedt test) has been of value in determining prognosis. Therefore, lumbar puncture is of little use in further management decisions and is not indicated.

176
Q

CASE 31: Spinal Injury Associated with Gunshot Wound HISTORY: A 22-year-old male was struck from the rear, by a bullet, in the upper lumbar region. He was knocked over by the impact and noted immediate tingling in both lower extremities and had difficulty moving his right leg. By the time he arrived in the emergency room, the tingling was predominantly in the right leg. EXAMINATION In the emergency room, his vital signs were normal ana stable. The bullet entry wound was noted at the right dorsal paramidline upper lumbar region. Rectal tone and voluntary sphincter contractions were normal and the patient was able to urinate upon request. His neurological examination was abnormal in the lower extremities. There was 4+/5 weakness in his anterior tibial, extensor hallucis longus, peroneus longus, posterior tibial and gastrocnemius muscles in the right kg. Knee reflexes were normal; the right ankle reflex was slightly diminished Hyperpathia was noted over the whole of the right leg from the L2 region down to and including the sacral, perianal region. Hypalgesia was noted over the same disiribution on the right and over the L2-S1 dermatomes on the left. joint position sense was normal. X-RAY: AP and lateral lumbosacral x-rays showed a fragmented bullet in the spinal canal at the L2 level. There was.no evidence of bony disruption (Figures 31.1A and 31.1B). The patient was observed for several hours and showed no signs i neurological deterioration. The entry wound was superficially debrided and further surgery was not recommended QUESTIONS

  1. Myelography is indicated in this patient. (True or False)
A

FALSE Myelography may be used to localize the spinal cord level of a lesion however, plain x-ray findings, including bullet fragments and fractures if seen, as well as clinical findings usuallv adequately determine the level.

177
Q

CASE 31: Spinal Injury Associated with Gunshot Wound HISTORY: A 22-year-old male was struck from the rear, by a bullet, in the upper lumbar region. He was knocked over by the impact and noted immediate tingling in both lower extremities and had difficulty moving his right leg. By the time he arrived in the emergency room, the tingling was predominantly in the right leg. EXAMINATION In the emergency room, his vital signs were normal ana stable. The bullet entry wound was noted at the right dorsal paramidline upper lumbar region. Rectal tone and voluntary sphincter contractions were normal and the patient was able to urinate upon request. His neurological examination was abnormal in the lower extremities. There was 4+/5 weakness in his anterior tibial, extensor hallucis longus, peroneus longus, posterior tibial and gastrocnemius muscles in the right kg. Knee reflexes were normal; the right ankle reflex was slightly diminished Hyperpathia was noted over the whole of the right leg from the L2 region down to and including the sacral, perianal region. Hypalgesia was noted over the same disiribution on the right and over the L2-S1 dermatomes on the left. joint position sense was normal. X-RAY: AP and lateral lumbosacral x-rays showed a fragmented bullet in the spinal canal at the L2 level. There was.no evidence of bony disruption (Figures 31.1A and 31.1B). The patient was observed for several hours and showed no signs i neurological deterioration. The entry wound was superficially debrided and further surgery was not recommended QUESTIONS

  1. Absolute indications for surgery in patients with gunshot wounds to the spine include (Select one or more)

A. laminectomy and exploration to assess the degree of neural injury in order to determine the patient’s prognosis

B. laminectomy and exploration to repair possible disruption of dura to prevent infection

C. exploration to determine the extent of bone injury with the intent to fuse if instability is demonstrated

D. decompressive laminectomy and exploration in a patient demonstrating progressive neurological deficit

E. debridement and exploration in cases of massive compound injuries

F. exploration to remove bullet and/or bone fragments

G. decompressive laminectomy to improve prognosis with regard to neurological function H. none of the above

A

(D,E) Evaluation of gross spinal cord pathology at surgery does not provide an index of prognosis. Persistent deficit may occur in patients found to have normal appearing spinal elements at surgery. Dural closure has not been demonstrated to decrease the incidence of meningitis in these patients. Heiden, et al. suggest that appropriate antibiotics, thorough debridement and meticulous wound closure are the most important determinants for the prevention of infection. Spinal instability and the need for fusion from gunshot wounds are extremely rare. Moreover, bony injury might exist, particularly from a ventral entry wound, which would not be visible at surgery. Tomography may be helpful. Progressive neurological deficit is an indication for surgery; however, progressive neurological deficit occurs rarely with gunshot wounds. Thorough debridement and optimal wound closure is indicated in massive open gunshot wounds to prevent infection. Although bullet or bone fragments theoretically compress neural elements, removal of the offending objects has not been shown to significantly alter return of neurological function.

178
Q

CASE 31: Spinal Injury Associated with Gunshot Wound HISTORY: A 22-year-old male was struck from the rear, by a bullet, in the upper lumbar region. He was knocked over by the impact and noted immediate tingling in both lower extremities and had difficulty moving his right leg. By the time he arrived in the emergency room, the tingling was predominantly in the right leg. EXAMINATION In the emergency room, his vital signs were normal ana stable. The bullet entry wound was noted at the right dorsal paramidline upper lumbar region. Rectal tone and voluntary sphincter contractions were normal and the patient was able to urinate upon request. His neurological examination was abnormal in the lower extremities. There was 4+/5 weakness in his anterior tibial, extensor hallucis longus, peroneus longus, posterior tibial and gastrocnemius muscles in the right kg. Knee reflexes were normal; the right ankle reflex was slightly diminished Hyperpathia was noted over the whole of the right leg from the L2 region down to and including the sacral, perianal region. Hypalgesia was noted over the same disiribution on the right and over the L2-S1 dermatomes on the left. joint position sense was normal. X-RAY: AP and lateral lumbosacral x-rays showed a fragmented bullet in the spinal canal at the L2 level. There was.no evidence of bony disruption (Figures 31.1A and 31.1B). The patient was observed for several hours and showed no signs i neurological deterioration. The entry wound was superficially debrided and further surgery was not recommended QUESTIONS

  1. As a general rule, initial neurological deficits caused byhigh velocity missiles, such as those seen in wartime, are more likely to improve than comparable deficits caused by low velocity missiles, such as those caused by handheld guns in the civilian population. (True or False)
A

TRUE The low velocity of civilian firearm bullets necessitates direct hits to cause neural damage, whereas high velocity military missiles can cause temporary injury by the concussive effects of a bullet trajectory which passes near but does not directly strike neural elements. The latter type of injury is, therefore, occasionally rapidly and spontaneously reversible.

179
Q

CASE 32: Hypothermia, Steroids, Mannitol for Spinal Cord Injury HISTORY: A 17-year-old female was admitted to the emergency room following a gymnastic accident. She complaint of neck pain. EXAMINATION: Revealed a loss of sensation to pinprick, temperature, touch, vibration and proprioception to the C6 level. There was flaccid quadriparesis below C6 except for slight flexion and extension of the left ankle. Sphincters were paralyzed. Thorough x-ray examination including tomography disclosed no fracture or dislocation. An emergency myelogram revealed widening of the spinal cord shadow from C4 to C6, but a myelography block to the flow of contrast material was not identified. A consultant suggested the use of corticosteroids, mannitol and immediate decompressive laminectomy and intraoperative spinal cord hypothermia. QUESTIONS

  1. Which of the following are true concerning spinal cord hypothermia in the treatment of spinal cord injury? (Select only one)

A. Is a clearly proven therapeutic measure for clinical use

B. Must be utilized within four hours of injury to be effective

C. Is clearly proven to be of value in experimental spinal cord injury in animals

D. All of the above

A

(B) Albin introduced the idea that spinal cord hypothermia might reduce or reverse the effects of spinal cord injury. He felt that hypothermia would reduce tissue metabolic demand for oxygen. Kelly observed that tissue oxygen levels were reduced by local spinal cord injury in dogs and supported Albin’s hypothesis. Others suggested that membrane stabilization, reduction in edema, and reduction in free radicals which could damage membranes also might be beneficial effects of spinal cord hypothermia. In a series of reports, Aibin and White showed that progressive central spinal cord necrosis after spinal cord injury could be reduced by spinal cord cooling in the experimental situation. Later, animals receiving controlled spinal cord injury wore observed clinically with and without subsequent spinal cord hypothermia. Some investigators reported a reduction in neurological deficits in such animals. Likewise, a few trials of hypothermic therapy in human spinal cord injury have been inconclusive. Some beneficial results have been reported but others have been unable to demonstrate benefit. Tator also suggested that normothermic perfusion of injured spinal cord in experimental animals was as effective as hypothermic perfusion in reversing neurological deficit due to spinal cord injury. He postulated that perfusion might remove certain toxic substances released after spinal cord trauma which contributed to progressive spinal cord necrosis. No careful prospective randomized study of this mode of therapy has been carried out. In addition, Stein recently indicated experimentally that toxic substances do not appear to be involved in the spinal cord dysfunction that follows injury. Bucy has suggested recently that localized spinal hypothermia is probably beneficial in reducing neurological deficit in humans after spinal cord injury and recommended further testing of this modality. In those studies demonstrating benefit from spinal hypothermia, the best results are obtained when the hypothermia is instituted within four hours of injury.

180
Q

CASE 32: Hypothermia, Steroids, Mannitol for Spinal Cord Injury HISTORY: A 17-year-old female was admitted to the emergency room following a gymnastic accident. She complaint of neck pain. EXAMINATION: Revealed a loss of sensation to pinprick, temperature, touch, vibration and proprioception to the C6 level. There was flaccid quadriparesis below C6 except for slight flexion and extension of the left ankle. Sphincters were paralyzed. Thorough x-ray examination including tomography disclosed no fracture or dislocation. An emergency myelogram revealed widening of the spinal cord shadow from C4 to C6, but a myelography block to the flow of contrast material was not identified. A consultant suggested the use of corticosteroids, mannitol and immediate decompressive laminectomy and intraoperative spinal cord hypothermia. QUESTIONS

  1. Which of the following are true concerning the treatment of spinal cord trauma with corticosteroids? (Select one or more)

A. Corticosteroids are superior to hypothermia in experimental spinal cord injury

B. Corticosteroids are clearly effective clinically

C. Corticosteroids are effective in animal models of spinal cord injury

D. The combination of hypothermia and corticosteroids is probably superior to either agent alone

E. None of the above

A

(C,D) Corticosteroids have multiple pharmacological actions. Their ability to produce stabilization of membranes and their ability to reduce edema in brain tissue surrounding tumors led to their use in spinal cord injury. Ducker initially recommended use of corticosteroids in the treatment of spinal cord injury based on laboratory studies. He observed improved neurological recovery in dogs subjected to a standard spinal cord injury, and treated subsequently with corticosteroids as compared to controls. Richardson reported, in electron microscopic studies of injured spinal cords, that methylprednisolone prevented changes in capillary walls, especially the basement membranes which normally occurred. The combination of hypothermia and steroids resulted in a nearly normal appearing spinal cord after trauma in Richardson’s studies. Hansebout, Kuchner, and Campbell also showed benefits from steroid administration in experimental animals after spinal cord injury. De la Torre suggested that steroids would be of value in spinal cord injury. In contrast, Ortiz-Ga!van applied hydrocortisone directly to injured spinal cord tissue and observed a decrease in inflammation but no significant effect on edema or necrosis. He concluded that steroids were unlikely to be effective in the treatment of spinal cord injury. Lewin showed that dexamethasone had minimal effect on spinal cord edema following experimental spinal cord injury. Overall, the majority of studies have demonstrated a beneficial effect of corticosteroids in reducing neurological deficit following experimental spinal cord injury. The combination of hypothermia and corticosteroids appears to be more effective than corticosteroids alone. Although steroids are currently being used clinically by many individuals in the treatment of spinal cord injury, their value is unproven. Ducker one of the original proponents of steroid treatment, recently reported no benefit from corticosteroids in the treatment of clinically complete spinal cord injuries. In cases of incomplete lesions, there was a suggested but not statistically significant effect in reducing neurological deficit. Bucy feels that steroids are probably beneficial and should be used clinically in the treatment of spinal cord injury,

181
Q

CASE 32: Hypothermia, Steroids, Mannitol for Spinal Cord Injury HISTORY: A 17-year-old female was admitted to the emergency room following a gymnastic accident. She complaint of neck pain. EXAMINATION: Revealed a loss of sensation to pinprick, temperature, touch, vibration and proprioception to the C6 level. There was flaccid quadriparesis below C6 except for slight flexion and extension of the left ankle. Sphincters were paralyzed. Thorough x-ray examination including tomography disclosed no fracture or dislocation. An emergency myelogram revealed widening of the spinal cord shadow from C4 to C6, but a myelography block to the flow of contrast material was not identified. A consultant suggested the use of corticosteroids, mannitol and immediate decompressive laminectomy and intraoperative spinal cord hypothermia. QUESTIONS

  1. Which of the following are true concerning the treatment of spinal cord injury with mannitol? (Select one or more)

A. Mannitol is effective clinically

B. Mannitol is effective in animal models of spinal cord injury

C. Mannitoi is more effective ihan hypothermia or steroids

D. None of the above

A

(B) Mannitol and urea reduce edema of central nervous system tissue by creating an osmotic gradient from the tissue to the bloodstream. These agents are rapidly effective in reducing cerebral edema. Their effectiveness in spinal cord injury, however, is unproven. The role of edema as a causative factor in the production of neurological deficit due to spinal cord injury is open to question. It was suggested that edema led to reduced local circulation with resultant ischemia and irreversible tissue damage. On this basis, the use of corticosteroids in spinal cord injury was studied. Mannitol and urea act much more rapidly than steroids in reducing edema, and thus an investigation of their usefulness appeared logical. Blaisdell demonstrated that urea protected the spinal cord in experimental spinal cord injury due to thoracic aortic occlusion. Jayner also showed a beneficial effect of urea in reducing neurological deficit in experimental spinal cord injury. Richardson demonstrated that while mannitol reduced the ultrastrucrural changes seen after experimental spinal cord injury, it was not as effective as hypothermia and methylprednisolone. Parker demonstrated a reduction in trauma induced with the use of mannitol after experimental spinal cord injury in dogs. Thus experimental studies, although few, have consistently demonstrated the value of mannitol and urea in the treatment of experimental spinal cord injury. No significant clinical studies of the use of mannitol or urea in the treatment of spinal cord injury are available. Experienced authors suggest the use of mannitol in intermittent dosages of 100 gm intravenously over one hour. This may be repeated for 7-10 days after spinal cord injury. The value of this form of treatment is unproven at present.

182
Q

CASE 33: Hypotension in a 34-Year-Old Man Two Weeks after Spinal Cord Injury HISTORY: A 34-year-old male was rendered paraplegic as a result of spinal cord transection the C7-T1 level. The patient’s condition had been stable ir. the hospital for two weeks. Subsequently, the patient’s blood pressure dropped from 110/60 to 60/0, and his pulse rate increased from 72 to 120. Respiratory rate increased slightly from 15 to 20. The patient was in no distress and had no specific complaints aside from mild lightheadedness. EXAMINATION: Temperature 100°F (rectal). The patient was comfortable in the supine position. His color was pale. The neurological examination was unchanged, with complete paraplegia and sensory loss below T1. The chest was clear to auscultation and cardiac sounds were normal. Tachycardia to 120 was persistent. The abdomen was soft, and no enlarged organs were noted. Bowel sounds were absent. Rectal examination was negative. QUESTIONS

  1. Clinical differential diagnosis would include which of the following? (Select one or more)

A. Recurrent spinal shock

B. Myocardial infarction

C. Perforated abdominal viscus

D. Acute gastrointestinal hemorrhage

E. Septicemia

A

(B,C,D,E) The clinical picture is that of cardiovascular shock with marked hypotension and tachycardia. Spinal shock is not usually a recurrent phenomenon. It apoears coincidental with acute spinal cord injury and subsides over several weeks (see Case 5). In spinal shock hypotension is usually mild and bradycardia, rather than tachycardia, are seen secondary to the effect of sympathectomy. The presence of sensory loss below the T1 level removes the warning signal of pain due to acute pathology in thoracic or abdominal viscera. Patients may thus remain suprisingly comfortable in the face of life-threatening lesions wiihin the chest or abdomen. As a result, careful attention must be paid to chaneres in vital signs as they may be the only warning of impending catastrophe in spinal cord injured patients.

183
Q

CASE 33: Hypotension in a 34-Year-Old Man Two Weeks after Spinal Cord Injury HISTORY: A 34-year-old male was rendered paraplegic as a result of spinal cord transection the C7-T1 level. The patient’s condition had been stable ir. the hospital for two weeks. Subsequently, the patient’s blood pressure dropped from 110/60 to 60/0, and his pulse rate increased from 72 to 120. Respiratory rate increased slightly from 15 to 20. The patient was in no distress and had no specific complaints aside from mild lightheadedness. EXAMINATION: Temperature 100°F (rectal). The patient was comfortable in the supine position. His color was pale. The neurological examination was unchanged, with complete paraplegia and sensory loss below T1. The chest was clear to auscultation and cardiac sounds were normal. Tachycardia to 120 was persistent. The abdomen was soft, and no enlarged organs were noted. Bowel sounds were absent. Rectal examination was negative. QUESTIONS

  1. Appropriate laboratory studies might include (Select only one)

A. complete blood count

B. abdominal x-ray series

C. electrocardiogram

D. blood cultures

E. all of the above

A

(E) The electrocardiogram may give immediate information concerning myocardial infarction. Abdominal x-rays may reveal free air if perforation of a viscus has occurred. An acutely reduced hematocrit suggests blood loss possibly secondary to a bleeding stress ulcer. Blood cultures, although not providing any information of immediate use, may be of value in the long run. If no other etiology for shock is revealed, empirical antibiotic therapy may be required while awaiting culture results. Appropriate cultures of sputum, urine, spinal fluid, wound drainage, etc., may also be indicated in order to investigate a potential source of infection.

184
Q

CASE 33: Hypotension in a 34-Year-Old Man Two Weeks after Spinal Cord Injury HISTORY: A 34-year-old male was rendered paraplegic as a result of spinal cord transection the C7-T1 level. The patient’s condition had been stable ir. the hospital for two weeks. Subsequently, the patient’s blood pressure dropped from 110/60 to 60/0, and his pulse rate increased from 72 to 120. Respiratory rate increased slightly from 15 to 20. The patient was in no distress and had no specific complaints aside from mild lightheadedness. EXAMINATION: Temperature 100°F (rectal). The patient was comfortable in the supine position. His color was pale. The neurological examination was unchanged, with complete paraplegia and sensory loss below T1. The chest was clear to auscultation and cardiac sounds were normal. Tachycardia to 120 was persistent. The abdomen was soft, and no enlarged organs were noted. Bowel sounds were absent. Rectal examination was negative. QUESTIONS

  1. While awaiting laboratory studies, the patient’s blood pressure became unobtainable, althcugn a carotid pulse could be palpated at a rate of 160. Which of the following are indicated? (Select one or more)

A. Blood volume replacement

B. Corticosteroids

C. Repeat cervical spine x-rays

D. Hypothermia blanket

E. Insertion of nasogastric tube

A

(A,B,E) Volume replacement in an attempt to correct hypotension is indicated. The rapid pulse suggests volume depletion. Because of the possibility of septicemia due to gram negative bacteria, the addition of corticosteroids is judicious. If another etiology is identified, steroids may be discontinued later. The placement of a nasogastric tube will allow rapid evaluation of gastric contents and if fresh blood is identified, may establish the diagnosis. In the event of viscus perforation the tube will allow evacuation of gastric contents. In the patient described, fresh blood was evacuated via the nasogastric tube. Hematocrit had decreased from 41%, 48 hours prior to the event described, to 24%. Free air was not identified on x-ray examination. Repeat x-rays of the cervical spine are not indicated. Even if movement had occurred in a fracture-dislocation with further spinal cord damage, it would not account for cardiovascular shock of the magnitude described. Hypothermia blanket may be dangerous,in patients with cervical spinal cord transections as the body temperature may rapidly fail to dangerous levels in the presence of sympathectomy and sensory loss. Careful measurements of body temperature is required if cooling or heating blankets are utilized in-such patients.

185
Q

CASE 33: Hypotension in a 34-Year-Old Man Two Weeks after Spinal Cord Injury HISTORY: A 34-year-old male was rendered paraplegic as a result of spinal cord transection the C7-T1 level. The patient’s condition had been stable ir. the hospital for two weeks. Subsequently, the patient’s blood pressure dropped from 110/60 to 60/0, and his pulse rate increased from 72 to 120. Respiratory rate increased slightly from 15 to 20. The patient was in no distress and had no specific complaints aside from mild lightheadedness. EXAMINATION: Temperature 100°F (rectal). The patient was comfortable in the supine position. His color was pale. The neurological examination was unchanged, with complete paraplegia and sensory loss below T1. The chest was clear to auscultation and cardiac sounds were normal. Tachycardia to 120 was persistent. The abdomen was soft, and no enlarged organs were noted. Bowel sounds were absent. Rectal examination was negative. QUESTIONS

  1. Gastrointestinal crisis in spinal cord injured patients is most commonly due to (Select only one)

A. bowel obstruction

B. stress ulceration

C. perforated viscus

D. gastrointestinal hemorrhage

E. none of the above

A

(C) A perforated viscus with peritonitis is the most common type of pathology responsible for abdominal crisis in spinal cord injured patients. Undiagnosed intraabdominal catastrophes account for up to 10% of fatalities in spinal cord injured patients. The incidence of gastrointestinal hemorrhage may be increasing in spinal cord injured patients due to the frequent use of corticosteroids to treat spinal cord injury per se. The incidence of acute intestinal ulceration and gastrointestinal hemorrhage is estimated at 5-22%. A variety of other conditions may also be responsible for intraabdominal pathology in such patients. Bowel obstruction in spinal injured patients is most often due to fecal impaction. Ileus is very common acutely following spinal cord injury and may lead to respiratory distress if abdominal distention or acute gastric dilation occurs. It may also obscure bowel obstruction from other causes.

186
Q

CASE 33: Hypotension in a 34-Year-Old Man Two Weeks after Spinal Cord Injury HISTORY: A 34-year-old male was rendered paraplegic as a result of spinal cord transection the C7-T1 level. The patient’s condition had been stable ir. the hospital for two weeks. Subsequently, the patient’s blood pressure dropped from 110/60 to 60/0, and his pulse rate increased from 72 to 120. Respiratory rate increased slightly from 15 to 20. The patient was in no distress and had no specific complaints aside from mild lightheadedness. EXAMINATION: Temperature 100°F (rectal). The patient was comfortable in the supine position. His color was pale. The neurological examination was unchanged, with complete paraplegia and sensory loss below T1. The chest was clear to auscultation and cardiac sounds were normal. Tachycardia to 120 was persistent. The abdomen was soft, and no enlarged organs were noted. Bowel sounds were absent. Rectal examination was negative. QUESTIONS

  1. Which of the following signs of intraabdominal disease might be expected in a patient such as the one illustrated with a C7-T1 spinal cord transection? (Select one or more)

A. Abdominal pain

B. Rebound tenderness

C. Muscular rigidity

D. Referred shoulder pain

E. None of the above

A

(D) If spinal cord transection occurs in the cervical region, abdominal crises are painless. Regardless oi the severity or location of intraabdominal pathology, abdominal pain is generally not a symptom of such entities. If upper abdominal lesions such as peritonitis result in diaphragmatic irritation, referred pain may be felt in the shoulder. Such refer red pain originates due to the common dermatomal innervation of the diaphragm and the skin of the shoulder, i.e. C3-4. Localized abdominal tenderness, abdominal muscular rigidity and guarding are uncommon as signs of intraabdominal pathology in spinal cord injured patients, in fact, if muscular spasticity has been present, an acute illness of any type may result in a reversion to flaccidity as is seen during the period of spinal shock. Other signs of intraabdominal disease are less specific but nevertheless raise the general question of acute disease in spinal cord injured patients. Such signs include persistent nausea or vomiting, alteration in pulse rate or change in degree or type of spasticity.

187
Q

CASE 33: Hypotension in a 34-Year-Old Man Two Weeks after Spinal Cord Injury HISTORY: A 34-year-old male was rendered paraplegic as a result of spinal cord transection the C7-T1 level. The patient’s condition had been stable ir. the hospital for two weeks. Subsequently, the patient’s blood pressure dropped from 110/60 to 60/0, and his pulse rate increased from 72 to 120. Respiratory rate increased slightly from 15 to 20. The patient was in no distress and had no specific complaints aside from mild lightheadedness. EXAMINATION: Temperature 100°F (rectal). The patient was comfortable in the supine position. His color was pale. The neurological examination was unchanged, with complete paraplegia and sensory loss below T1. The chest was clear to auscultation and cardiac sounds were normal. Tachycardia to 120 was persistent. The abdomen was soft, and no enlarged organs were noted. Bowel sounds were absent. Rectal examination was negative. QUESTIONS

  1. Abdominal operation should be avoided in patients with spinal cord injury since wound healing and operative risk are increased in such patients. (True or False)
A

FALSE Operative risk is generally felt to be increased in spinal cord injured patients in the acute phase. Most abdominal complications occur somewhat later and Greenfield indicated that patients with chronic paraplegia withstand abdominal operations well and heal as well as normal patients. Yashon agrees with this approach and feels that abdominal exploration should not be delayed in spinal cord injured patients.

188
Q

CASE 34: Apnea Following Spinal Cord Injury in a 13-Year-Old Boy HISTORY: A 13-year-old boy was brought to the emergency room twenty minutes after receiving an accidental gunshot wound to the neck. Immediate resuscitative measures were carried out at the scene of the accident. EXAMINATION: B/P 70/40, P 56, T 37°C. There was an entry wound about 0.5 cm in diameter just caudal to the right mastoid process but no exit wound could be found. The remainder of the general physical examination was unremarkable. The child appeared to be awake but could not speak and was apneic. There was flaccid quadriplegia including nuchal muscles. All reflexes, both superficial and deep, were absent. There was anesthesia over the entire body to the level of the mandible bilaterally. Sphincters were paralyzed. Sweating was absent. X-RAYS: Cervical spine x-rays revealed a metallic fragment about 0.5 x 0.75 cm within the spinal canal at the C1-2 level. QUESTIONS:

  1. Which of the following are suggested by the clinical picture ? (Select one or more)

A. Complete spinal cord transection

B. Subtotal spinal cord injury

C. Spinal shock

D. Sympathectomy

E. None of the above

A

(A,C,D) The clinical picture indicates total loss of spinal cord function from the upper cervical region down. Paralysis of the nuchal muscles, supplied via the cervical plexus (2-4) or via the spinal portion (C3,4) of the spinal accessory nerve, confirms that muscle paralysis extends to the high cervical level . Loss of sensation to the C2 level is consistent with this. The distal paralysis, loss of reflexes and sphincter paralysis are all evidence of total loss of spinal cord function and are consistent with spinal shock. Hypotension, bradycardia and anhydrosis are evidence of sympathectomy.

189
Q

CASE 34: Apnea Following Spinal Cord Injury in a 13-Year-Old Boy HISTORY: A 13-year-old boy was brought to the emergency room twenty minutes after receiving an accidental gunshot wound to the neck. Immediate resuscitative measures were carried out at the scene of the accident. EXAMINATION: B/P 70/40, P 56, T 37°C. There was an entry wound about 0.5 cm in diameter just caudal to the right mastoid process but no exit wound could be found. The remainder of the general physical examination was unremarkable. The child appeared to be awake but could not speak and was apneic. There was flaccid quadriplegia including nuchal muscles. All reflexes, both superficial and deep, were absent. There was anesthesia over the entire body to the level of the mandible bilaterally. Sphincters were paralyzed. Sweating was absent. X-RAYS: Cervical spine x-rays revealed a metallic fragment about 0.5 x 0.75 cm within the spinal canal at the C1-2 level. QUESTIONS:

  1. Apnea and inability to speak are due to (Select one or more)

A. sleep-induced apnea

B. diaphragm paralysis

C. injury to respiratory centers

D. intercostal paralysis

E. temporary spinal cord concussion

A

(B,D) Spinal cord transection at the C1-2 level results in interruption of supra segmental control of all respiratory musculature. The diaphragm, innervated by the phrenic nerve (segmental levels C3-4) and the intercostal muscles innervated by the intercostal nerves (segmental levels T1-12) are both paralyzed as a result. Apnea results and only prompt resuscitation at the accident scene saved the patient’s life. Aphonia is secondary to apnea. Since the patient cannot inspire, no air is available to cause vibration of the vocal cords in order to generate sound. Sleep-induced apnea is not present since the patient is awake. Sleep-induced apnea, the so-called “Ondine’s Curse” results from partial neural injury in the medulla or high cervical region. Such patients have a reduced responsiveness of the respiratory centers to hypercarbia. In the awake state, compensation occurs however with sleep, apnea ensues due to loss of the automatic phase of respiration. Brainstem tumors and the effects of bilateral high cervical cordotomy are common causes of sleep-induced apnea. Chronic pulmonary insufficiency with chronic hypercarbia also may render the respiratory centers relatively insensitive to elevated pCO2 and result in sieep-induced apnea. The centers involved in the automatic aspects of respiration are located primarily in the medulla and pons. Spinal cord injury at the C1-2 level would not directly injure these centers. Such a lesion, however, interrupts connections between the respiratory centers and motor neurons of the phrenic and intercostal nerves. The complete nature of the spinal cord lesion and the presence of a foreign body (likely the bullet fragment) in the spinal canal suggest direct spinal cord injury. Spinal cord concussion, a rare condition, is characterized by a brief period of loss of spinal cord function from an indirect blow. Recovery usually begins within minutes of the injury and complete recovery is usually seen within minutes to hours of the injury. Respiratory paralysis in a surviving patient with spinal cord concussion is extremely rare.

190
Q

CASE 34: Apnea Following Spinal Cord Injury in a 13-Year-Old Boy HISTORY: A 13-year-old boy was brought to the emergency room twenty minutes after receiving an accidental gunshot wound to the neck. Immediate resuscitative measures were carried out at the scene of the accident. EXAMINATION: B/P 70/40, P 56, T 37°C. There was an entry wound about 0.5 cm in diameter just caudal to the right mastoid process but no exit wound could be found. The remainder of the general physical examination was unremarkable. The child appeared to be awake but could not speak and was apneic. There was flaccid quadriplegia including nuchal muscles. All reflexes, both superficial and deep, were absent. There was anesthesia over the entire body to the level of the mandible bilaterally. Sphincters were paralyzed. Sweating was absent. X-RAYS: Cervical spine x-rays revealed a metallic fragment about 0.5 x 0.75 cm within the spinal canal at the C1-2 level. QUESTIONS:

  1. Appropriate treatment for this patient would include (Select one or more)

A. placement of an endotracheal tube

B. controlled respiration

C. assisted respiration

D. immediate surgical exploration of the spinal cord injury

E.placement of a urinary catheter

A

(A,B,E) Placement of an endotracheal tube or tracheostomy and controlled respiration is essential for immediate care of this patient with total respiratory failure. Assisted respiration is impossible since the patient cannot generate respiratory effort to activate a mechanical ventilator. Urinary catheterization is also essential to prevent bladder distension with possible rupture or infection. Immediate surgical exploration is unnecessary following spinal gunshot wounds (see Case 31). The spinal cord trauma caused by passage of the missile cannot be reversed by surgical treatment. Unless persistent cerebrospinal fluid leakage or neurological deterioration ensue, surgical treatment of this patient is not indicated.

191
Q

CASE 34: Apnea Following Spinal Cord Injury in a 13-Year-Old Boy HISTORY: A 13-year-old boy was brought to the emergency room twenty minutes after receiving an accidental gunshot wound to the neck. Immediate resuscitative measures were carried out at the scene of the accident. EXAMINATION: B/P 70/40, P 56, T 37°C. There was an entry wound about 0.5 cm in diameter just caudal to the right mastoid process but no exit wound could be found. The remainder of the general physical examination was unremarkable. The child appeared to be awake but could not speak and was apneic. There was flaccid quadriplegia including nuchal muscles. All reflexes, both superficial and deep, were absent. There was anesthesia over the entire body to the level of the mandible bilaterally. Sphincters were paralyzed. Sweating was absent. X-RAYS: Cervical spine x-rays revealed a metallic fragment about 0.5 x 0.75 cm within the spinal canal at the C1-2 level. QUESTIONS:

  1. Which of the following methods might be effective in providing adequate respiratory exchange in the patient described? (Select one or more)

A. Active exercises of accessory muscles of respiration

B. Tracheostomy with mechanical ventilation

C. Electrical stimulation of the phrenic nerves

D. Respiratory stimulants

E. None of the above

A

(B,C) In the past, ail patients with respiratory paralysis due to spinal cord injury at the C1-2 level who survived were doomed to aphonia, a permanent trachesotomy and the need for constant controlled ventilation via a mechanical respirator. This method provides adequate respiratory exchange but is undesirable because of the inability to speak, the risks of permanent tracheostomy and the need for a constant mechanical ventilator. Respiration using accessory respiratory muscles such as the cervical strap muscles, trapezius and sternocleidomastoid is impossible in such patients because these muscles, supplied by the upper cervical segments, are also paralyzed. Respiratory stimulants are ineffective in such patients because they are centrally acting agents. Since the deficiency in high cervical spinal cord injury is due to interruption of connections between central respiratory centers and phrenic and intercostal motor neurons, respiratory stimulants are useless. Recently the use of electrical stimulation of the phrenic nerve, so-called “diaphragm pacing’’ has given new hope to patients with respiratory paralysis due to C1-2 spinal cord injury. The phrenic motor neuron is are located at C3-4, and thus the peripheral portion of the phrenic nerve remains viable following high cervical spinal cord Injury above C3. Direct electrical stimulation of the phrenic nerve was first observed to elicit diaphragm contraction by Waud in 1937. Sarnoff demonstrated functional diaphragmatic contraction in response to phrenic nerve stimulation in animals. Later, Glenn pioneered electrical stimulation of the phrenic nerve in humans with neurologically related respiratory paralysis. This method utilizes a fully implanted passive radiofrequency receiver and attached stimulating electrode. The latter is placed about the phrenic nerve after surgical exposure in the supraclavicular region. The receiver is implanted in a subcutaneous pocket in the subclavicular region. A battery powered external transmitter allows control of respiratory rate, inspiratory and expiratory liming, tidal volume and inspiratory rate. Power from the transmitter is passed to the receiver transcutaneously via an antenna placed on the skin overlying the implanted receiver. No wires or other connections pass through the skin. Since a single phrenic nerve can be electrical stimulated for a maximum of about twelve hours, bilateral implants are necessary in patients who require total respiratory support. For patients with sleep-induced apnea, a unilateral implant is sufficient since phrenic nerve stimulation is only required during sleep. Glenn has had extensive experience with phrenic nerve stimulation in patients with respiratory paralysis due to high cervical spinal cord injury. Many years of respirator-free existence are possible for such patients. It is important to note that phrenic nerve stimulation is unlikely to be effective if spinal cord injury has occurred at the C3-4 level. In such a circumstance, phrenic motor neurons are destroyed, resulting in degeneration of the phrenic nerve which thus becomes insensitive to electrical stimulation. In spinal cord lesions below C4, phrenic nerve and diaphragm function are usually normal and no benefit would be derived from electrical stimulation. Although electrical stimulation of the phrenic nerve is oniy appropriate for a small proportion of patients with spinal cord injury, it can offer some hope to such patients.

192
Q

CASE 35: Respiratory Complications in a 47-Year-Old Quadriplegic HISTORY: A 47-year-old man was admitted to the hospital following an automobile accident. A fracture-dislocation of the cervical spine at C5-6 was reduced with traction and the patient was placed on a spinal frame. The patient complained of a feeling of breathlessness within an hour of admission. EXAMINATION: Examination showed a complete motor paralysis and sensory loss below the C5 level. Sphincters were paralyzed. Auscultation of the chest was normal. Bowel sounds were absent. B/P 78/40, P 56, T 97°F, R 30. X-RAYS: Chest x-ray was normal. LABORATORY DATA: Tidal volume average - 0. 2 l i t e r s (normal 0.3-0.5); vital capacity - 0.8 liters (normal 3.26-4. 8); maximum breathing capacity - 30 liters/min (normal 80-.170); arterial pH - 7. 31 (normal 7. 35-7.45); arterial pC02- 55mmHg (normal 35-45); arterial p02 - 70 mmHg (normal > 9 5 ) . QUESTIONS:

  1. Which of the following is most likely responsible for this patient’s respiratory status? (Select only one)

A. Pulmonary embolus

B. Neurogenic pulmonary edema

C. Arteriovenous pulmonary shunting

D. Neurogenic hypoventilation

E. Neurogenic hyperventilation

A

(D) As a result of spinal cord injury this patient manifests hypoventilation. There is reduced vital capacity and maximum breathing capacity, hypoxia, hypercarbia and mild respiratory acidosis. This is a common pattern acutely following cervical spinal cord injury. Early clinical studies of respiratory function focused on vital capacity as a standard test of pulmonary function. Later studies suggested that maximum ventilation was achieved by maximum voluntary effort and exceeded that achieved during exercise or with inhalation of carbon dioxide. Gilliatt published early data on the value of vital capacity measurements in evaluation of respiratory function in quadriplegics. He and his co-workers established the presence of a low vital capacity in midcervical quadriplegics. Later studies revealed vital capacities as low as 300 ml. Wingo, Hemingway and Haas described vital capacity reductions averaging about 60% of normal in quadriplegics and maximum breathing capacity reductions to 50% of normal are also seen. Silver found reductions in vital capacity in tetraplegics ranging from 25-70%. He also found reductions in maximum voluntary ventilation ranging from 20-75% and related this loss directly to the magnitude of loss of expiratory muscle function. Hypoxia is also a common problem. This may relate to respiratory insufficiency per se but often is due to complications. These include pulmonary embolism, atelectasis, obstruction of bronchi by secretions, intercurrent infection, pneumothorax and subcutaneous emphysema. No signs of these problems are present in this patient. With pulmonary edema diffuse rales, bronchospasm and abnormal chest x-ray are seen. Moderate hypoxia in this patient relates to alveolar hypoventilation due to reduced ventilating capacity. Up to 50% of patients with high cervical cord injuries manifest hypoxia due to this neuromuscular ventilatory deficit. The patient presented cannot hyperventilate as indicated by a reduced maximum breathing capacity. Arteriovenous pulmonary shunting most often relates to disturbances in the ventilation- perfusion ratio. In such patients, ventilatory function, as judged by pulmonary function tests, is normal but hypoxia is noted even with significant elevations in inspired oxygen concentration. The reduced vital capacity, maximum breathing capacity, and hypercarbia would not be expected if hypoxia were due to pulmonary arteriovenous shunting. Such shunting has been described acutely after head injury as a poor prognostic sign but is net described with spinal cord injury, possibly being masked by hypoventilation.

193
Q

CASE 35: Respiratory Complications in a 47-Year-Old Quadriplegic HISTORY: A 47-year-old man was admitted to the hospital following an automobile accident. A fracture-dislocation of the cervical spine at C5-6 was reduced with traction and the patient was placed on a spinal frame. The patient complained of a feeling of breathlessness within an hour of admission. EXAMINATION: Examination showed a complete motor paralysis and sensory loss below the C5 level. Sphincters were paralyzed. Auscultation of the chest was normal. Bowel sounds were absent. B/P 78/40, P 56, T 97°F, R 30. X-RAYS: Chest x-ray was normal. LABORATORY DATA: Tidal volume average - 0. 2 l i t e r s (normal 0.3-0.5); vital capacity - 0.8 liters (normal 3.26-4. 8); maximum breathing capacity - 30 liters/min (normal 80-.170); arterial pH - 7. 31 (normal 7. 35-7.45); arterial pC02- 55mmHg (normal 35-45); arterial p02 - 70 mmHg (normal > 9 5 ) . QUESTIONS:

  1. Which of the following are true concerning respiratory function in this patient? (Select one or more)

A. Likely to be associated with sleep apnea

B. Likely to improve in time

C. Could be improved by the head down position

D. Dependent acutely on diaphragmatic function only

E. Dependent acutely on function of accessory respiratory muscles

A

(B,C,D) In spinal cord injury at the C5 level, respiratory function is dependent acutely on diaphragmatic function alone. There is little or no contribution to respiratory function in the acute stage by accessory respiratory muscles. The phrenic motor neurons, located at C3-4. remain basically uninjured and functionally connected both to central respiratory control centers and to the diaphragm via axons in the phrenic nerve. Phrenic motor neurons may extend to C5 in some patients and may be affected in C4-5 or even C5-6 lesions especially if upward migration of the injured zone, of even a small degree, occurs. This usually results in a quantitative partial reduction rather than total loss of diaphragm function which would result in apnea. The intercostal and abdominal muscles, the other two main respiratory muscles, under normal conditions are totally paralyzed and flaccid immediately after complete lesions of the spinal cord in the cervical region. Sleep-induced apnea occurs in partial high spinal cord lesions at the cervicomedullary junction, lower brainstem, or C1-2 region which result in partial disconnection of the central respiratory centers from the motor neurons which innervate the respiratory muscles, i.e., diaphragm and intercostals. The automatic phase of respiration during sleep is impaired, although voluntary respiration is unaffected. Sleep-induced apnea is therefore not expected in a spinal cord injury at the C5-6 level. Guttmann and Guttmann and Silver have demonstrated improved respiratory function in quadriplegics with the passage of time. This has also been our observation. Guttmann also demonstrated that the 15° head down position produced a significant increase in vital capacity in quadriplegics and that the 15° head up position significantly reduced the vital capacity. These postural effects were attributed to paralysis of the abdominal musculature which allowed bulging of the abdominal contents through the anterior abdominal wall. The diaphragm thus assumed a lower position in the chest at the end of expiration, reducing-the available excursion during inspiration.

194
Q

CASE 35: Respiratory Complications in a 47-Year-Old Quadriplegic HISTORY: A 47-year-old man was admitted to the hospital following an automobile accident. A fracture-dislocation of the cervical spine at C5-6 was reduced with traction and the patient was placed on a spinal frame. The patient complained of a feeling of breathlessness within an hour of admission. EXAMINATION: Examination showed a complete motor paralysis and sensory loss below the C5 level. Sphincters were paralyzed. Auscultation of the chest was normal. Bowel sounds were absent. B/P 78/40, P 56, T 97°F, R 30. X-RAYS: Chest x-ray was normal. LABORATORY DATA: Tidal volume average - 0. 2 l i t e r s (normal 0.3-0.5); vital capacity - 0.8 liters (normal 3.26-4. 8); maximum breathing capacity - 30 liters/min (normal 80-.170); arterial pH - 7. 31 (normal 7. 35-7.45); arterial pC02- 55mmHg (normal 35-45); arterial p02 - 70 mmHg (normal > 9 5 ) . QUESTIONS:

  1. Which of the following may be responsible for improved respiratory function in the subacute and chronic stages of complete quadriplegia? (Select one or more)

A. Utilization of accessory respiratory muscles

B.Recovery of reflex contraction of intercostal muscles

C.Recovery of reflex contraction of abdominal muscles

D. Spinal cord regeneration

E. Recovery of respiratory function does not occur after complete quadriplegia

A

(A,B,C) The sternocleidomastoid, trapezius and scalene muscles, as well as the cervical strap muscles, all may act as accessory respiratory muscles. They do so by attachment to the clavicle, sternum and scapula. Upward pull on these structures by the accessory respiratory muscles during inspiration increases the anteroposterior diameter of the chest and increases respiratory negative pressure. The intensity of the compensatory respiratory function of these muscles is variable. The development of these muscles takes time and is augmented considerably by training and exercise. Guttmann and Silver demonstrated significant improvements in respiratory function from the acute to the more chronic stages of cervical spinal cord injury. They found initial vital capacities as low as 0.1-0.3 liters and found increases later to 1.2-3 liters. Guttmann postulated that improved respiratory function in the later stages of quadriplegia was due to recovery of reflex tone in the intercostal and abdominal muscles. Guttmann and Silver examined recovery of reflex function of intercostal muscles electromyographically in nineteen patients with complete spinal cord injuries between C4 and C8. They established a marked recovery of reflex contraction in both intercostal and abdominal muscles after recovery from spinal shock. There was significant muscular contraction in response to the stimulus of breathing in these muscles during inspiration. Due to the heightened activity of all reflexes distal to cord injury, remote stimuli such as stroking the foot may elicit contraction of intercostal and abdominal muscles. In the main, however, the reflex activity of these muscles was noted to show a rhythmic pattern closely related to the respiratory cycle. The contractions began in mid or late inspiration and lasted until early expiration. Thus, as recovery from spinal shock occurs, return of reflex contractions of intercostal and abdominal muscles results in significant augmentation of respiratory function. There is no evidence that functional spinal cord regeneration, occurs in complete spinal cord transection. With incomplete lesions, recovery of respiratory function may occur if partially injured descending respiratory spinal cord pathways return to functional capacity. Such return, however, is most likely not due to regeneration.

195
Q

CASE 35: Respiratory Complications in a 47-Year-Old Quadriplegic HISTORY: A 47-year-old man was admitted to the hospital following an automobile accident. A fracture-dislocation of the cervical spine at C5-6 was reduced with traction and the patient was placed on a spinal frame. The patient complained of a feeling of breathlessness within an hour of admission. EXAMINATION: Examination showed a complete motor paralysis and sensory loss below the C5 level. Sphincters were paralyzed. Auscultation of the chest was normal. Bowel sounds were absent. B/P 78/40, P 56, T 97°F, R 30. X-RAYS: Chest x-ray was normal. LABORATORY DATA: Tidal volume average - 0. 2 l i t e r s (normal 0.3-0.5); vital capacity - 0.8 liters (normal 3.26-4. 8); maximum breathing capacity - 30 liters/min (normal 80-.170); arterial pH - 7. 31 (normal 7. 35-7.45); arterial pC02- 55mmHg (normal 35-45); arterial p02 - 70 mmHg (normal > 9 5 ) . QUESTIONS: 4. Which of the following are true concerning tracheostomy in quadriplegics? (Select one or more)

A. Increased likelihood of mucosal erosion

B. Suctioning may lead to cardiac arrhythmia

C. Required in most C5 quadriplegics

D. Required in all C 1 - 2 quadriplegics

E. None of the above

A

(A,B,D) Tracheostomy may be a life-saving procedure in respiratory insufficiency due to cervical spinal cord injury. Tracheostomy is required in all quadriplegic patients due to spinal cord injury at the C1-2 level. Such patients will require prolonged mechanical respiratory assistance which cannot be given via endotracheal tube for more than 7-10 days at maximum. Electrical stimulation of the phrenic nerves may allow eventual freedom from mechanical respiratory support for such patients but this modality is usually not considered for at least three months from the time of injury (see Case 34). The waiting period allows reasonable time for recovery to occur and for overall stabilization of the patient. Tracheostomy is not without significant complications under usual circumstances but the risk of complications is significantly increased in quadriplegics. In the acute stage of cervical spinal cord injury, there is reduced tone of the tracheal tissue as a result of sympathectomy and vasoparalysis. Thus, the mucosa is vulnerable to ulceration from compression by tracheostomy tubes. Infection of such ulcerations leads to chondromalacia and secondary tracheal stricture. Overinflation of tracheostomy tube cuffs, movement of the tracheostomy tube, and secondary infection appear to be the main cause of tracheal stenosis. Cardiac arrhythmias, with potential for sudden death, may result from tracheal suctioning or even from inflation of endotracheal or tracheostomy tube cuffs. The predominant arrhythmia seen is profound bradycardia. This presumably results from parasympathetic stimulation via the vagus nerve which is unopposed by the sympathetic nervous system due to the sympathectomy created by cervical spinal cord injury. Such reflex arrhythmias have also been reported in tracheostomized patients without spinal cord injury but the likelihood of occurrence appears greater in patients with sympathectomy due to cervical spinal cord injury. Such reflexes may be blocked by parasympatholytic agents such as atropine. Fortunately, high spinal cord injuries which require prolonged use of mechanical ventilatory assistance are relatively uncommon. Patients with spinal cord injury at C5 or below usually do not require prolonged mechanical ventilatory assistance and thus do net usually require tracheostomy. Such patients may require temporary ventilatory assistance and this may be effectively given via naso or oral endotracheal intubation. As soon as respiratory effectiveness has increased to vital capacity of 500-800 ml, the patient should be weaned off the respirator. The use of IMV (intermittent mandatory ventilation) may be very helpful in attempting to wean a patient off mechanical ventilatory assistance. In some instances, recurrent respiratory complications such as atelectasis, recurrent infection or inability to clear secretions may accessitate tracheostomy even in patients with mid and lower cervical spinal cord injuries. The use of tracheostomy solely to improve tracheobronchial toilet is probably not justified. Effective tracheal toilet can be performed without tracheostomy, although this approach is time consuming and physically demanding.

196
Q

CASE 35: Respiratory Complications in a 47-Year-Old Quadriplegic HISTORY: A 47-year-old man was admitted to the hospital following an automobile accident. A fracture-dislocation of the cervical spine at C5-6 was reduced with traction and the patient was placed on a spinal frame. The patient complained of a feeling of breathlessness within an hour of admission. EXAMINATION: Examination showed a complete motor paralysis and sensory loss below the C5 level. Sphincters were paralyzed. Auscultation of the chest was normal. Bowel sounds were absent. B/P 78/40, P 56, T 97°F, R 30. X-RAYS: Chest x-ray was normal. LABORATORY DATA: Tidal volume average - 0. 2 l i t e r s (normal 0.3-0.5); vital capacity - 0.8 liters (normal 3.26-4. 8); maximum breathing capacity - 30 liters/min (normal 80-.170); arterial pH - 7. 31 (normal 7. 35-7.45); arterial pC02- 55mmHg (normal 35-45); arterial p02 - 70 mmHg (normal > 9 5 ) . QUESTIONS: 5. Three days after admission, the patient exhibited respiratory distress and became restless and confused. Fever to 100. 6°F was noted. Examination disclosed decreased breath sounds over the right lower lung field. Chest xray showed a right lower lobe infiltration. Arterial p02 was 50 mmHg. Which of the following are possible explanations for this situation? (Select one or more)

A. Atelectasis

B. Pneumonitis

C. Bronchial obstruction

D. Pulmonary edema

E. None of these

A

(A,B,C) In addition to a reduced tidal volume and maximum breathing capacity, there is marked reduction in force of the cough reflex in cervical level quadriplegics. As a result, expectoration of bronchial secretions may be ineffective. Thus, bronchial obstruction, distal atelectasis and pneumonitis are common. Any of these could account for the fever, hypoxia, and pulmonary infiltrate seen in this patient. The problem of respiratory secretions is aggravated by the sympathectomy effect. Renal function is also slowed acutely following cervical spinal cord injury. These factors lead to increased transudation of fluid from the intravascular into the interstitial paces with the formation of edema. If severe, clinical pulmonary edema may develop but the physical examination of the patient described does not suggest pulmonary edema. In addition, the chest x-ray shows a localized infiltrate rather than the generalized increased density seen with pulmonary edema.

197
Q

CASE 35: Respiratory Complications in a 47-Year-Old Quadriplegic HISTORY: A 47-year-old man was admitted to the hospital following an automobile accident. A fracture-dislocation of the cervical spine at C5-6 was reduced with traction and the patient was placed on a spinal frame. The patient complained of a feeling of breathlessness within an hour of admission. EXAMINATION: Examination showed a complete motor paralysis and sensory loss below the C5 level. Sphincters were paralyzed. Auscultation of the chest was normal. Bowel sounds were absent. B/P 78/40, P 56, T 97°F, R 30. X-RAYS: Chest x-ray was normal. LABORATORY DATA: Tidal volume average - 0. 2 l i t e r s (normal 0.3-0.5); vital capacity - 0.8 liters (normal 3.26-4. 8); maximum breathing capacity - 30 liters/min (normal 80-.170); arterial pH - 7. 31 (normal 7. 35-7.45); arterial pC02- 55mmHg (normal 35-45); arterial p02 - 70 mmHg (normal > 9 5 ) . QUESTIONS:

  1. Which of the following are effective in preventing respiratory complications in quadriplegic patients? (Select one or more)

A. Steam inhalation

B. restriction of fluid intake

C. intermittent positive pressure breathing (IPPB)

D. Mucolytic agents E. None of the above

A

(A,B,C,D) Steam inhalation is effective in reducing inspissation of respiratory secretions in quadriplegics. This is particularly necessary in patients with tracheostomy or endotracheal tube where the warming and humidifying effects of the upper airway are bypassed. A dust filter also may be effective in providing clean air to the patient. The effect of sympathectomy and reduced renal function enlarge intravascular volume and lead to an increased tendency for accumulation of respiratory secretions. Thus, there is need for judicious replacement of fluids particularly in the acute phase of quadriplegia due to cervical spinal cord injury. In particular, the tendency to attempt rapid correction of hypotension due to the vasodilatory effect of sympathectomy should be resisted. Large volumes of intravenous fluids given rapidly may lead to massive pulmonary edema. Such edema if mild may lead to increased hypoventilation and increased pulmonary complications and if severe may lead to death. Intravenous diuretics, particularly mannitol, may be an effective means of treating such over-hydration. Intermittent positive pressure breathing (IPPB) is helpful in preventing atelectasis and inspissation of respiratory sensation. It is also an effective means of administering mucolytic agents which reduce sputum viscosity. These may be particularly helpful if signs of bronchitis are present. In addition to these measures, the head down position which increases ventilation and encourages drainage of respiratory secretions may help in preventing respiratory complications in quadriplegics. Treatment of gastric distension may further improve respiratory exchange. Such distension may occur acutely after spinal cord injury due to sudden paralysis of intestinal motility and lead to impaired diaphragmatic excursion and further hypoventilation. Vomiting with aspiration and acute respiratory obstruction or aspiration pneumonitis may also result. Nasogastric decompression and cholinergic drugs such as prostigmine may be helpful in treating this condition. Oral intake should be avoided until motility of the gastrointestinal trace returns. In addition to fluid requirements, electrolyte, caloric and specific nutritional needs must be met. Usually gastrointestinal function returns within 7-10 days or sooner and oral intake may begin.

198
Q

CASE 36: Acute Respiratory Distress in a 47-Year-Old Quadriplegic HISTORY: A 47-year-old man was transferred from another hospital two weeks after a diving accident in which he sustained a complete transverse spinal cord injury at the C5-6 level. A fracture-dislocation of the cervical spine was reduced with skeletal traction and the patient was. Maintained on a spinal frame. On the afternoon of admission, he complained of acute dyspnea and a feeling of impending disaster. EXAMINATION: BP 80/40, P 96, T 100°F (rectal), R 32. Neurological examination revealed a flaccid quadriplegia and , other findings consistent with a recent complete lesion of the spinal cord at C5-6. Respiration was exclusively diaphragmatic. His color appeared ashen. There were expiratory wheeze at the base of the left lung. The left calf was 3 era larger in diameter than the right. X-RAYS: A chest x-ray was normal. LABORATORY DATA: HCT 39%, WBC 11, 500. Urine-300 WBC/mm3. moderate gram negative bacteria. Arterial Pk 7.11 (normal 7.35-7.45). Arterial pCO2 41 mmHg (normal 35-45). Arterial pCO2 >46 mmHg (normal >95) . Vital capacity 0.9 liters (normal 3.26-4.8). Maximum breathing capacity 50 l/min (normal 80-170). QUESTIONS

  1. Which of the following is the most likely pathological process responsible for the respiratory problem in this patient? (Select one or more)

A. Rostral progression of spinal cord lesion

B. Pneumonitis

C. Atelectasis

D. Neurogenic hypoventilation

E. Pulmonary embolus

A

(E) Acute onset of hypoxia and respiratory distress with low grade fever in a patient with spinal cord injury and evidence of calf enlargement is strongly suggestive of pulmonary embolism. Deep venous thrombosis and pulmonary embolism are common complications in spinal cord injured patients. The incidence of deep venous thrombosis, mostly in the legs but also in pelvic veins, is estimated at from 10-61%. The lower figures of 10-20% are from series utilizing only clinical and radiological methods of diagnosis. Utilizing impedance plethysmography and contrast venography, Todd estimated an incidence of 61%. Utilizing I125 fibrinogen uptake study methods, Todd indicated that 10% of patients with significant spinal cord injury had evidence of deep venous thrombosis. Pulmonary embolus, related to deep venous thrombosis in nearly all cases, has been estimated to occur in from 2-2.5% of patients with spinal cord injury. Up to 30-40% of deaths in the first months after spinal cord injury are due to pulmonary emboli. There was no evidence in this patient of rostral progression of the neurological deficit to account for pulmonary deterioration. Examination continued to support a lesion at the C5-6 level, Although the vital capacity and maximum breathing capacity were reduced and hypoxia was profound, a normal pC02 suggests that significant hypoventilation was not present in this patient. Pneumonitis and atelectasis are common causes of hypoxia and sudden pulmonary deterioration in spinal cord injured patients. These are nearly always accompanied by rales, reduced breath sounds and an abnormal chest x-ray. These findings were not present in the patient presented. The localized expiratory wheezes at the left lung base likely relate to local bronchospasrn secondary to pulmonary embolization. The vaiue of frequent calf measurements cannot be overemphasized as a guide to the occurrence of deep vein thrombosis in the legs, nearly always the source of pulmonary emboli. The appearance of increased calf circumference should lead promptly to definitive studies to establish conclusively the presence of deep venous thrombosis so that appropriate treatment may be carried out in an attempt to forestall pulmonary embolization.

199
Q

CASE 36: Acute Respiratory Distress in a 47-Year-Old Quadriplegic HISTORY: A 47-year-old man was transferred from another hospital two weeks after a diving accident in which he sustained a complete transverse spinal cord injury at the C5-6 level. A fracture-dislocation of the cervical spine was reduced with skeletal traction and the patient was. Maintained on a spinal frame. On the afternoon of admission, he complained of acute dyspnea and a feeling of impending disaster. EXAMINATION: BP 80/40, P 96, T 100°F (rectal), R 32. Neurological examination revealed a flaccid quadriplegia and , other findings consistent with a recent complete lesion of the spinal cord at C5-6. Respiration was exclusively diaphragmatic. His color appeared ashen. There were expiratory wheeze at the base of the left lung. The left calf was 3 era larger in diameter than the right. X-RAYS: A chest x-ray was normal. LABORATORY DATA: HCT 39%, WBC 11, 500. Urine-300 WBC/mm3. moderate gram negative bacteria. Arterial Pk 7.11 (normal 7.35-7.45). Arterial pCO2 41 mmHg (normal 35-45). Arterial pCO2 >46 mmHg (normal >95) . Vital capacity 0.9 liters (normal 3.26-4.8). Maximum breathing capacity 50 l/min (normal 80-170). QUESTIONS

  1. Which of the following laboratory studies would be of value in establishing the correct diagnosis? (Select one or more)

A. Radionuclide lung scan

B. Plethysmography

C. Contrast venography

D. I125 fibrinogen uptake

E. None of the above

A

(A,B,C,D) The pulmonary radioisotope scan, combined with clinical symptoms, x-ray findings if positive and enzyme studies are the best measures available for the diagnosis of pulmonary embolism per se. The diagnosis of pulmonary embolization is difficult at times in otherwise normal individuals but is even more difficult in patients with spinal cord injury. For instance, in normals the presence of marked hypoxia out of proportion to clinical or x-ray findings of lung consolidation is strong evidence of pulmonary embolism. Hypoxia of significant degree, however, is often present in patients with cervical spinal cord injury, thus reducing the value of this diagnostic test. Regular evaluation of arterial pO2 is of value in patients with spinal cord injury to establish baseline levels against which to judge changes. While the pulmonary isotope scan is highly diagnostic of pulmonary emboli, it may be impossible for technical reasons to carry out scanning in patients with spinal cord injury, in skeletal traction and on spinal frames. Electrical impedance plethysmography is noninvasive and readily available. It is very efficacious in the diagnosis of deep venous thrombosis of the legs. False positive findings occur with some regularity related mainly to particular positioning of the electrodes and the extremity being studied. Repeated study after a positive finding with repositioning of electrodes is desirable to confirm a positive result. Confirmation with contrast venography is desirable if positive findings are obtained with plethysmography. The I125 fibrinogen test is excellent for early detection of venous obstruction in the calf. In addition, because of the half life of the isotope, repealed scan can be carried out daily to assess progression of thrombus size. Plethysmography, contrast venography and the fibrinogen uptake test are valuable in the diagnosis of deep venous thrombosis in the legs. The presence of deep venous thrombosis alone is due to pulmonary embolism. On the other hand, pulmonary embolism is more unlikely in the absence of deep venous thrombosis.

200
Q

CASE 36: Acute Respiratory Distress in a 47-Year-Old Quadriplegic HISTORY: A 47-year-old man was transferred from another hospital two weeks after a diving accident in which he sustained a complete transverse spinal cord injury at the C5-6 level. A fracture-dislocation of the cervical spine was reduced with skeletal traction and the patient was. Maintained on a spinal frame. On the afternoon of admission, he complained of acute dyspnea and a feeling of impending disaster. EXAMINATION: BP 80/40, P 96, T 100°F (rectal), R 32. Neurological examination revealed a flaccid quadriplegia and , other findings consistent with a recent complete lesion of the spinal cord at C5-6. Respiration was exclusively diaphragmatic. His color appeared ashen. There were expiratory wheeze at the base of the left lung. The left calf was 3 era larger in diameter than the right. X-RAYS: A chest x-ray was normal. LABORATORY DATA: HCT 39%, WBC 11, 500. Urine-300 WBC/mm3. moderate gram negative bacteria. Arterial Pk 7.11 (normal 7.35-7.45). Arterial pCO2 41 mmHg (normal 35-45). Arterial pCO2 >46 mmHg (normal >95) . Vital capacity 0.9 liters (normal 3.26-4.8). Maximum breathing capacity 50 l/min (normal 80-170). QUESTIONS

  1. What factors are identifiable in this patient which increase the risk of pulmonary embolization? (Select one or more)

A. Sluggish venous return

B. Hypercoagulation

C. Urinary infection

D . Less than 30 days from time of injury

E. Complete cervical spinal cord lesion

A

(A,B,C,D,E) The most important factor in the occurrence of deep venous thrombosis and subsequent pulmonary embolism is sluggish venous return. The patient with spinal cord injury is immobile, especially acutely. There is absence of lower extremity movement and absence of the compressive action of muscular contraction to augment venous return from the lower extremities. In addition, there is loss of vasomotor tone due to the effect of sympathectomy. Flaccid paralysis of intercostal and abdominal muscles in the acute stages following spinal cord injury results in a decreased negative intrathoracic pressure during inspiration. This negative pressure normally augments venous return to the heart from the lower extremities and its reduction further contributes to venous stasis and clotting in leg veins. Although two hourly turning is the routine for patients on spinal frames, the time between position changes is significant and allows gravitational sludging of blood and aggravation of the tendency for intravascular coagulation. This is further aggravated by the tendency to nurse patients with spinal cord injury in the head up position particularly in cervical spine fracture-dislocations where skeletal traction is utilized for reduction and stabilization. The head up position contributes to gravitational pooling of blood in the legs and intravascular coagulation. It is well established that deep venous thrombosis and resultant pulmonary embolism are most frequent in the first month after injury and are more frequent with complete, as compared to incomplete, spinal cord injuries. There is no specific relationship between the spinal level of injury and the occurrence of pulmonary emboli, although most emboli occur with cervical and thoracic lesions. There is evidence that a hypercoagulable state occurs in spinal cord injured patients. Some have suggested that this is due to release of thrombokinase particularly if associated injuries are present. Thrombokinase facilitates local clotting. Cook and Lyons suggested that additional factors such as age, obesity, varicose veins, anemia, infection and hypoproteinemia may also account for hypercoagulability after spinal cord injury. The period during which the hypercoagulable state remains present is unknown but it appears most severe the first few weeks after spinal cord injury. Urinary tract infection has also been identified as a significant factor in the occurrence of deep venous thrombosis and pulmonary emboli in spinal cord injured patients. The rate of urinary tract infection is high in spinal cord injured patients. The frequent catheterizations in these patients lead to recurrent bacteremia even in those patients with sterile urine. In the patient presented, pyuria and bacteriuria are both present. It is postulated that endotoxins released in the blood stream by gram negative bacteria lead to intravascular coagulation. In addition, patients with urinary infections may be particularly susceptible to deep pelvic vein thrombosis from the thrombogenic effect of gram negative bacteria entering the venous circulation In patients with clinical evidence of infection, e.g. fever, tachycardia and leukocytosis, “L” forms of bacteria have been implicated in the occurrence of thromboembolism.

201
Q

CASE 36: Acute Respiratory Distress in a 47-Year-Old Quadriplegic HISTORY: A 47-year-old man was transferred from another hospital two weeks after a diving accident in which he sustained a complete transverse spinal cord injury at the C5-6 level. A fracture-dislocation of the cervical spine was reduced with skeletal traction and the patient was. Maintained on a spinal frame. On the afternoon of admission, he complained of acute dyspnea and a feeling of impending disaster. EXAMINATION: BP 80/40, P 96, T 100°F (rectal), R 32. Neurological examination revealed a flaccid quadriplegia and , other findings consistent with a recent complete lesion of the spinal cord at C5-6. Respiration was exclusively diaphragmatic. His color appeared ashen. There were expiratory wheeze at the base of the left lung. The left calf was 3 era larger in diameter than the right. X-RAYS: A chest x-ray was normal. LABORATORY DATA: HCT 39%, WBC 11, 500. Urine-300 WBC/mm3. moderate gram negative bacteria. Arterial Pk 7.11 (normal 7.35-7.45). Arterial pCO2 41 mmHg (normal 35-45). Arterial pCO2 >46 mmHg (normal >95) . Vital capacity 0.9 liters (normal 3.26-4.8). Maximum breathing capacity 50 l/min (normal 80-170). QUESTIONS

  1. Which of the following are true concerning pulmonary embolization in spinal cord injured patients? (Select one or more)

A. A rare occurrence

B. Often fatal

C. Related to deep vein thrombosis in lower extremities

D. Increased frequency in obese patients E Increased frequency with advanced age

A

(B,C) Depending on methods used in diagnosis, the incidence of pulmonary emboli in spinal cord injured patients is as high as 25%. It is generally accepted that pulmonary emboli are a frequent and serious problem in spinal cord injured patients. The primary reason for concern with pulmonary emboli is the high incidence of fatality with this lesion. Estimates of the contribution of pulmonary emboli to the total mortality following spinal cord injury range from about 20-30%. Specific mortality rates from pulmonary emboli range from about 2-25%. Nearly all pulmonary emboli can be related to deep venous thrombosis in either the lower extremities or pelvic veins. Interestingly, recent studies indicate that factors such as advanced age and obesity are not related to an increased incidence of deep venous thrombosis and pulmonary emboli. These were previously thought to be significant risk factors for occurrence of thromboembolism,

202
Q

CASE 36: Acute Respiratory Distress in a 47-Year-Old Quadriplegic HISTORY: A 47-year-old man was transferred from another hospital two weeks after a diving accident in which he sustained a complete transverse spinal cord injury at the C5-6 level. A fracture-dislocation of the cervical spine was reduced with skeletal traction and the patient was. Maintained on a spinal frame. On the afternoon of admission, he complained of acute dyspnea and a feeling of impending disaster. EXAMINATION: BP 80/40, P 96, T 100°F (rectal), R 32. Neurological examination revealed a flaccid quadriplegia and , other findings consistent with a recent complete lesion of the spinal cord at C5-6. Respiration was exclusively diaphragmatic. His color appeared ashen. There were expiratory wheeze at the base of the left lung. The left calf was 3 era larger in diameter than the right. X-RAYS: A chest x-ray was normal. LABORATORY DATA: HCT 39%, WBC 11, 500. Urine-300 WBC/mm3. moderate gram negative bacteria. Arterial Pk 7.11 (normal 7.35-7.45). Arterial pCO2 41 mmHg (normal 35-45). Arterial pCO2 >46 mmHg (normal >95) . Vital capacity 0.9 liters (normal 3.26-4.8). Maximum breathing capacity 50 l/min (normal 80-170). QUESTIONS

  1. Which of the following are effective in the prevention of pulmonary emboli in spinal cord injured patients? (Select one or more)

A. Head down position

B. Passive exercises

C Low dose heparin therapy

D. Resistance breathing exercises

E. Static compression stockings for lower extremities

A

(A,B,C,D) The head down position, passive exercises and resistance breathing exercises are all designed and effective in reversing the effects of circulatory stasis in the lower extremities. Frequent position changes and elevation of the feet are probably the single most important factors in facilitating venous return from the lower extremities. Electrically controlled turning and tilting beds such as the Stoke-Mandeville Edgerton and Roto-Rest are very helpful in this regard and reduce stress on the nursing staff compared to the use of spinal frames. Although static calf and thigh compression utilizing elastic stockings is commonly employed in immobile patients with spinal cord injury, there is little proof of its effectiveness. Passive exercises of the legs carried out at frequent intervals also promote venous return from the legs. Motorized methods have been employed to allow more frequent, nearly continuous passive leg exercises but the value of this system is unproven. Electrical stimulation of leg muscles nas also been utilized in an attempt to augment venous return. The most effective means of preventing deep venous thrombosis and subsequent pulmonary emboli is the use of anticoagulants. Oral anticoagulants have not been utilized extensively due to relatively slow onset, difficulty in reversing overdose and relatively narrow range between effective and safe dosage. Oral anticoagulants, however, may be useful for long term use. Primary interest in prophylaxis and treatment of deep vein thrombosis has centered on the use of heparin following the demonstration that low doses (5,000-10,000) of heparin every twelve hours could prevent thromboembolism after surgical procedures. This method has been employed prophylactically in spinai cord injured patients as well. Significant reductions in the occurrence of deep venous thrombosis, pulmonary embolism and secondary fatality have been demonstrated with these methods. Although these recommendations are well founded, significant concerns about potential complications and contraindications to routine anticoagulant prophylaxis of thromboembolism in the acute stages following spinal cord injury have been raised. Coincidental injuries with hemorrhage, head injuries, bloody spinal fluid on lumbar puncture, acute gastrointestinal hemorrhage or ulceration are all felt to be contraindications to the use of anticoagulant therapy for the prophylaxis of thromboembolism. In the presence of deep venous thrombosis, the risk of fatal pulmonary embolus may necessitate vena cavaplication or ligation if contraindications to the use of anticoagulants are present.

203
Q

CASE 37: Hypotension and Unconsciousness in a 27-Year-Old Quadripleeic HISTORY: A 27-year-old woman was admitted to the hospital following a gunshot wound to the base of the neck. EXAMINATION: The patient was awake and alert at the time of admission. Examination disclosed a complete flaccid paralysis below C7 with total loss of sensation bel°w C7. Tendon reflexes were absent. Sphincters were paralyzed. There was no cerebrospinal fluid leakages LABORATORY DATA: BP 82/46 P 60, R 20, T 97° F. Central venous pressure-0. HCT 40%, HGB 13. 9 gm, WBC 6,750 QUESTIONS

  1. Which of the following are true concerning the central venous pressure recording in this patient? (Select one or more)

A. Reliably indicates need for fluid replacement

B. Unreliable as an index of need for fluid replacement

C. Low due to volume depletion

D. Low due to increased venous capacitance

E. Low due to sympathectomy

A

(B,D,E) Central venous pressure (CVP) measurements are unreliable in cervical spinal cord injury as a measure of circulatory status. A large increase in venous capacitance occurs as a result of vasomotor paralysis secondary to the sympathectomy effect of complete transverse cervical spinal cord injury. Preganglionic ympathetic motor neurons are located in the intermediolateral cel l column of the spinal cord from T1 to L1. The suprasegmental input to these sympathetic neurons is completely interrupted by transverse spinal cord lesions above C8. Lesions between T1 and L1 spinal cord levels produce incomplete sympathectomy. For instance, in lesions be­low T2, sympathetic innervation to the upper extremities, face and head is retained. Thus, as spinal cord injury occurs pro­gressively more caudally, the sympathectomy effect is reduced. Fluid replacement based on CVP’s may lead to gross overhy­dration since the CVP may not rise until venous capacitance is nearly exhausted. At this point, symptoms of massive fluid overload including pulmonary edema, which is on occasion fatal, may occur. Even if clinical pulmonary edema does not occur, the increased fluid load leads to decreased pulmo­nary compliance and aggravation of hypoventilation and hypoxia. The measurement of pulmonary artery pressure utilizing the Swan-Ganz catheter may detect early increases in pulmonary artery pressure before elevations in central venous pressure or clinical signs of decreased pulmonary compliance or pulmonary edema occur. Brisman demonstrated that dogs with transected spinal cords demonstrated a greater ratio of increase in pulmonary wedge pressure to increase in central venous pres­sure when loaded with dextran arid levarteranol than did dogs with spinal cord injury. This underscores the fact that central ve­nous pressure measurement are even more unreliablein patients with spinal cord injuries as a measure of fluid require­ments than in patients without spinal cord injury. In general fluid replacement is not required fortreatment of hypotension re­lated exclusively to spinal cord injury and sympathectomy. On the other hand, injuries to other systems such as major fractures of long bones, intraabdominal injury and thoracic injury may require rapid and substantial fluid replacement. In such instances, placement of a Swan-Ganz catheter and observation of pulmo­nary artery pressure is the most reliable index to adequate but not excessive fluid replacement. Pulmonary edema after spinal cord injury may not relate ex­clusively to fluid overload. Poe reported pulmonary edema in a 15-year-old male eleven hours after traumatic quadriple­gia at the C5-6 level . Although pulmonary wedge pressure was not measured, fluid replacement did not appear excessive. The authors postulated a mechanism similar to that which has been used to explain so-called “acute neurogenic pulmonary edema” seen after craniocerebral injury. This theory involves sudden sympathetic discharge with increased total peripheral resistance, decrease in cardiac output, increase in left atrial pressure with acute pulmonary congestion. Massive im­mediate outpouring of sympathomimetic amines with brief hypertension and tachycardia is known to occur with cervical spinal cord transection but is usually unnoticed clinically since subsequent hypotension bradycardia and other effects of sym­pathectomy, dominate the clinical picture. Perhaps this is the reason “neurogenic” pulmonary edema is rare with spinal cord injury and more common with craniocerebral insult and an in­tact spinal cord. Luisadall also postulated that histamine and other vasoactive substances could be released secondary to central nervous system insult and lead to increased pulmonary capillary permeability and edema. Such a mechanism could ac­count for pulmonary edema in either cerebral or spinal cord injuries. Concomittant chest wounds also may relate to the occurrence of pulmonary edema after spinal cord injury.

204
Q

CASE 37: Hypotension and Unconsciousness in a 27-Year-Old Quadripleeic HISTORY: A 27-year-old woman was admitted to the hospital following a gunshot wound to the base of the neck. EXAMINATION: The patient was awake and alert at the time of admission. Examination disclosed a complete flaccid paralysis below C7 with total loss of sensation bel°w C7. Tendon reflexes were absent. Sphincters were paralyzed. There was no cerebrospinal fluid leakages LABORATORY DATA: BP 82/46 P 60, R 20, T 97° F. Central venous pressure-0. HCT 40%, HGB 13. 9 gm, WBC 6,750 QUESTIONS

  1. Two weeks after injury there was no change in neurological function. The patient was placed in a wheelchair with the back elevated about 50°. She complained of blurred vision, then loss of vision accompanied by giddiness and became unresponsive. Vital signs at that time showed blood pressure unobtainable and carotid pulse barely palpable at a rate of 180. The most likely cause for this patient’s problem is (Select only one)

A. pulmonary embolus

B.postural hypotension due to vasomotor instability

C. epileptic seizure

D. myocardial infarction

E. irritable carotid sinus

A

(B) Vasomotor control is lost following cervical spinal cord transection. Thus, when a patient’s head is elevated, pooling of blood occurs in the lower extremities and viscera with resultant decreased cardiac output, hypotension, reduced cerebral perfusion and altered or lost consciousness. Systematic study of this problem has been carried out by several authors. Blurring and subsequent loss of vision were invariable premonitory symptoms. Giddiness, tinnitus, paraesthesias of the hands, headaches and nausea were also noted. Response of blood pressure and pulse depended on whether or not conscious­ness was retained. If consciousness was lost, severe systolic and diastolic hypotension occurred and the blood pressure be ­came unobtainable. Immediate tachycardia occurred while the pa­tient remained vertical but was supplanted by sustained bradycardia if the patient was returned to the horizontal position. If consciousness was maintained, there was a transient slight in­crease in heart rate and systolic pressure was elevated slightly while diastolic pressure showed a maintained rise. Changes in rhythm were rare in Guttmann’s studies.

205
Q

CASE 37: Hypotension and Unconsciousness in a 27-Year-Old Quadripleeic HISTORY: A 27-year-old woman was admitted to the hospital following a gunshot wound to the base of the neck. EXAMINATION: The patient was awake and alert at the time of admission. Examination disclosed a complete flaccid paralysis below C7 with total loss of sensation bel°w C7. Tendon reflexes were absent. Sphincters were paralyzed. There was no cerebrospinal fluid leakages LABORATORY DATA: BP 82/46 P 60, R 20, T 97° F. Central venous pressure-0. HCT 40%, HGB 13. 9 gm, WBC 6,750 QUESTIONS

  1. Which of the following are true concerning the cardiovascular complications which occurred in this patient? (Select one or more)

A. Occur more frequently in incomplete than complete spinal cord lesions

B. Occur most often with low thoracic and lumbar lesions

C. Occur more frequently in older compared to recent spinal cord injury

D. None of the above

A

(D) Vasomotor instability with severe, clinically significant postural hypotension occurs primarily with complete transection of the spinal cord above the T5 level with com­plete lesions above C8, sympathectomy is complete and vaso­motor instability occurs in essentially all such patients. With complete injuries between C8 and T5 the severity varies . Below T5, although partial sympathectomy occurs, sufficient vasomotor tone is retained, particularly in the viscera via the splanchnic nerves, that clinically significant postural hypoten­sion is uncommon. Circulating levels of the sympathetic amines, epinephrine and nor epinephrine were reduced in subjects with high spinal cord lesions. Little or no elevation in these catecholamines occurred in patients who lest consciousness with vertical position­ing whereas in those who maintained consciousness significant elevations were observed. The former group had mainly com­plete cervical lesions while the latter had incomplete or thoracic or lumbar cord lesions. In addition, vasomotor instability is much more severe in the immediate post injury period, Frequent positional changes apparently help overcome vasomotor instability by stimulating reflex vasoconstrictive responses in the isolated spinal cord below the level of injury. Gilliat, et al . studied reflex activities of the isolated spinal cord which may relate to vasomotor readjustment mechanisms. They demonstrated that respiratory inspiration results in reflex vasoconstriction even with com­plete transection of the spinal cord in the cervical region. Thus,they assumed that inspiratory vasoconstriction was a local spinal reflex phenomenon occurring in the thoracic spinal cord Interestingly, Silver demonstrated that inspiratory reflex vasoconstriction can be elicited in complete cervical spinal cord lesions even during the period of spinal shock. Perhaps this implies that the reflex is not spinally mediated but is controller by more centrally located connections, perhaps in the brainstem

206
Q

CASE 37: Hypotension and Unconsciousness in a 27-Year-Old Quadripleeic HISTORY: A 27-year-old woman was admitted to the hospital following a gunshot wound to the base of the neck. EXAMINATION: The patient was awake and alert at the time of admission. Examination disclosed a complete flaccid paralysis below C7 with total loss of sensation bel°w C7. Tendon reflexes were absent. Sphincters were paralyzed. There was no cerebrospinal fluid leakages LABORATORY DATA: BP 82/46 P 60, R 20, T 97° F. Central venous pressure-0. HCT 40%, HGB 13. 9 gm, WBC 6,750 QUESTIONS

  1. Which of the following are effective in preventing cardiovascular complications such as experienced by this patient? (Select one or more)

A.Gradual elevation from the supine position

B. Abdominal binder

C. Ephedrine prior to position changes

D. None of the above

A

(A,B,C) Gradual, repeated elevation of the patient from supine to upright is the keystone in prevention of postural hypotension and loss of consciousness. The tilt-table or continu­ously moving beds such as the Roto-Rest are very useful in providing repetitive stimulation of vasomotor reflexes leading to recovery of vasomotor tone and control. Guttmann also sug­gests that application of an abdominal binder or compression stockings and the use of 20-25 mg of ephedrine before raisins a patient from supine to vertical may minimize,postural hypotension. Resistance exercises of unparalyzed limbs, trunk or head are also useful in stimulating heart rate, circulatory reflexes and decreasing pooling of blood in viscera and dependent limbs. Overcoming the effects of vasomotor paralysis is es­sential if spinal cord injured patients are to be successfully re­habilitated by use of braces, crutches, wheelchairs, etc.

207
Q

CASE 38: Decubitus Ulcer Associated with Spinal Cord Injury HISTORY: A 27-year-old woman was seen for evaluation of a decubitus ulcer overlying the sacrum. She had been paraplegic since a motor vehicle accident six months previously. She was able to ambulate by wheelchair. EXAMINATION There was a 6 cm decubitus ulcer overlying the sacrum. It had extended down to the fascial level. There was an area of erythema about 2-3 cm wide surrounding the lesion. The lesion itself was composed mainly of necrotic tissue which was gray and pale. Neurological examination revealed complete anesthesia below T4. There was a spastic paraplegia and spastic neurogenic bladder. QUESTIONS

  1. The sacral location of this lesion, is an uncommon one for decubitus ulcers seen in paraplegics. (True or False).
A

FALSE The sacral region and skin over the ischial tuberosity each represent the location of about 25% of all decubiti. The skin over the greater femoral trochanter is the location of another 15% of decubiti. Other less common locations are the heels (8%) , malleoli (7%), knees (6%) and elbows (3%). Rare sites are the pretibial area, vertebral spinous processes, chin, occiput and scapula.

208
Q

CASE 38: Decubitus Ulcer Associated with Spinal Cord Injury HISTORY: A 27-year-old woman was seen for evaluation of a decubitus ulcer overlying the sacrum. She had been paraplegic since a motor vehicle accident six months previously. She was able to ambulate by wheelchair. EXAMINATION There was a 6 cm decubitus ulcer overlying the sacrum. It had extended down to the fascial level. There was an area of erythema about 2-3 cm wide surrounding the lesion. The lesion itself was composed mainly of necrotic tissue which was gray and pale. Neurological examination revealed complete anesthesia below T4. There was a spastic paraplegia and spastic neurogenic bladder. QUESTIONS

  1. The surrounding erythema most likely indicates infection.(True or False)
A

TRUE Shortly after necrosis and sloughing of the superficial skin layers, a surrounding area of erythema develops which is indicative of cellulitis. As the ulceration deepens, the erythematous halo becomes brighter in color and enlarges. Subsequently, gross pus may be evident in the depths of the ulceration. Essentially all decubitus ulcers become infected, usually with a combination of skin (staphylococcal) and gram negative organisms (proteus, E.coli, pseudomonas).

209
Q

CASE 38: Decubitus Ulcer Associated with Spinal Cord Injury HISTORY: A 27-year-old woman was seen for evaluation of a decubitus ulcer overlying the sacrum. She had been paraplegic since a motor vehicle accident six months previously. She was able to ambulate by wheelchair. EXAMINATION There was a 6 cm decubitus ulcer overlying the sacrum. It had extended down to the fascial level. There was an area of erythema about 2-3 cm wide surrounding the lesion. The lesion itself was composed mainly of necrotic tissue which was gray and pale. Neurological examination revealed complete anesthesia below T4. There was a spastic paraplegia and spastic neurogenic bladder. QUESTIONS

  1. A decubitus ulcer is shaped like the letter “V” becoming narrower from the skin level to its deepest point. (True or False)
A

FALSE Decubitus ulcers are shaped like the letter “V” but the apex of the “V “ or cone-shaped end of the lesion is at the skin level . Thus, according to Agris, “there is no such thing as a small pressure ulcer”. Pressure is transmitted from the surface to deep tissues in such a way that a cone-shaped area of tissue destruction is created with the apex of the cone at the skin level and the base near the underlying bony prominence. About 70% of the ulcer may be below skin leveL

210
Q

CASE 38: Decubitus Ulcer Associated with Spinal Cord Injury HISTORY: A 27-year-old woman was seen for evaluation of a decubitus ulcer overlying the sacrum. She had been paraplegic since a motor vehicle accident six months previously. She was able to ambulate by wheelchair. EXAMINATION There was a 6 cm decubitus ulcer overlying the sacrum. It had extended down to the fascial level. There was an area of erythema about 2-3 cm wide surrounding the lesion. The lesion itself was composed mainly of necrotic tissue which was gray and pale. Neurological examination revealed complete anesthesia below T4. There was a spastic paraplegia and spastic neurogenic bladder. QUESTIONS

  1. Which of the following are factors in the occurrence of decubitus ulceration in paraplegics? (Select one or more)

A. Loss of sensation

B. Sympathectomy

C. Prolonged pressure

D. Loss of fat and muscle padding between skin and bone

E. None of the above

A

(A,B,C,D) The basic process by which decubitus ulcers form is by ischemic tissue necrosis. Tissue pressures of greater than 40-80 mmHg cause tissue ischemia and if prolonged, result in necrosis of skin and deeper layers. In spinal cord injury, the effect of sympathectomy is to reduce perfusion pressure, thus allowing ischemia to occur more easily at lower tissue compression pressures and with shorter ischemic periods. Loss of sensation results to abolition of the normal warning signs of tissue ischemia such as par­esthesias or pain. In nonanesthetic individuals, these sensa­tions cause constant slight shifts in position of the body which prevent prolonged tissue ischemia and necrosis. Naturally, paralysis accompanying anesthesia further aggravates this prob­lem because it prevents the frequent changes in body position which occur normally, even during sleep. With prolonged reduced mobility and intercurrent problems such as infection, the spinal cord injured patient may lose body fat. Muscle atrophy may occur secondary to disuse or to lower mo­tor neuron paralysis. Thus, the padding effect of these tissues between skin and bone is lost. The pressure of underlying bony prominence is, therefore, transmitted more directly and force­-fully to the skin with increased likelihood of tissue ischemia and necrosis.

211
Q

CASE 38: Decubitus Ulcer Associated with Spinal Cord Injury HISTORY: A 27-year-old woman was seen for evaluation of a decubitus ulcer overlying the sacrum. She had been paraplegic since a motor vehicle accident six months previously. She was able to ambulate by wheelchair. EXAMINATION There was a 6 cm decubitus ulcer overlying the sacrum. It had extended down to the fascial level. There was an area of erythema about 2-3 cm wide surrounding the lesion. The lesion itself was composed mainly of necrotic tissue which was gray and pale. Neurological examination revealed complete anesthesia below T4. There was a spastic paraplegia and spastic neurogenic bladder. QUESTIONS

  1. Which of the following would be the best immediate treatment for this lesion? (Select only one}

A. Excision of entire ulcer and flap rotation to close defect

B. Local excision of devitalized tissue

C. Systemic antibiotics

D. Sterile dressings, topical antibiotics and enzymatic agents

E. None of the above

A

(B) The most appropriate treatment of an ulcer of the type and location described is local debridement of the devita­lized tissue and eschar. This should be followed by sterile wet to dry dressings which aid in further tissue debride­ment. The use of substances other than saline on either the debrided ulcer or dressings is controversial . Local antibiotics, enzymes, antiseptics, vegetable poultices, tincture of benzoin, gold leaf, animal collagen and negative ions among others have been suggested as beneficial in the healing of pressure ulcers but the merits of such methods are unproven scientifically (see reviews by Sheal and Guttmann). It is absolutely essential to note that in addition to the above treatment, all local pressure must be avoided by appropriate positioning and movement. The patient cannot be placed in one position only to avoid pressure on a decubitus ulcer, as pressure necrosis may develop in another area. Thus, in the patient described with a sacral ul­cer, prolonged positioning in the prone position may lead to ul­cerations of skin overlying the chin, sternum, iliac crests, knees, thighs or tibia. The Keane Roto-Rest and other similar beds are particularly useful in such patients because they allow continuous patient movement. Systemic antibiotics alone are of little or no value in such local lesions. It is also doubtful if they are of any value when added to local surgical debridement. Most authors recommend their use pre-, intra- and postoperatively for major surgical pro­cedures on decubitus ulcers , but again their value in such situ­ations is unproven. Sterile dressing changes, whether or not combined with local antiseptics, antibiotics, enzymes, etc, are generally ineffec­tive alone. Debridement of dead tissue is necessary to reduce local contamination and allow secondary healing from healthy, well vascularized tissue. Excision of the entire ulcer with flap rotation is an effective means of treatment of decubitus ulcers but is a major surgical undertaking particularly in patients who may be generally de­bilitated. Further, healing of many decubitus ulcers may be accomplished by local debridement and dressing changes, making more radical surgical treatment unnecessary.

212
Q

CASE 38: Decubitus Ulcer Associated with Spinal Cord Injury HISTORY: A 27-year-old woman was seen for evaluation of a decubitus ulcer overlying the sacrum. She had been paraplegic since a motor vehicle accident six months previously. She was able to ambulate by wheelchair. EXAMINATION There was a 6 cm decubitus ulcer overlying the sacrum. It had extended down to the fascial level. There was an area of erythema about 2-3 cm wide surrounding the lesion. The lesion itself was composed mainly of necrotic tissue which was gray and pale. Neurological examination revealed complete anesthesia below T4. There was a spastic paraplegia and spastic neurogenic bladder. QUESTIONS

  1. Which of the following might be appropriate if the above method fails? (Select one or more)

A. Excision and flap rotation

B. Systemic antibiotics

C Resection of underlying bone

D. None of the above

A

(A,B,D) If local methods fail, complete excision of the devitalized area with surgical closure primarily by skin graft or by flap rotation may be effective. Many authors also recommend the use of systemic antibiotics combined with sur­gical procedures. If necrosis has proceeded through the deep fascia to include underlying bone, excision of such infected bone is also required for healing to occur . More radical bone resections (particularly amputation) may rarely also be required but should be avoided if at all possible.

213
Q

CASE 38: Decubitus Ulcer Associated with Spinal Cord Injury HISTORY: A 27-year-old woman was seen for evaluation of a decubitus ulcer overlying the sacrum. She had been paraplegic since a motor vehicle accident six months previously. She was able to ambulate by wheelchair. EXAMINATION There was a 6 cm decubitus ulcer overlying the sacrum. It had extended down to the fascial level. There was an area of erythema about 2-3 cm wide surrounding the lesion. The lesion itself was composed mainly of necrotic tissue which was gray and pale. Neurological examination revealed complete anesthesia below T4. There was a spastic paraplegia and spastic neurogenic bladder. QUESTIONS

  1. Important adjunctive measures to promote healing of decubitus ulcers include (Select one or more)

A. correction of anemia

B. correction of nutritional deficiencies

C. treatment of spasticity

D. correction of contractures

E. none of the above

A

(A,B,C,D) Systemic factors are important in the preven­tion and treatment of decubitus ulcers. Anemia and hypoproteinemia resulting from poor nutrition and chronic infection aggravate the tendency to ulcer formation. Likewise, unless such deficiencies are corrected by transfusion and proper nu­trition, occasionally with intravenous hyperalimentation, suc­cessful treatment is impossible. Spasticity, particularly in adductor muscles may cause constant pressure on the medial thighs, knees and malleoli and lead to pressure ulceration. Appropriate treatment of spasticity in such cases is required for successful resolution of decubitus ulcers (see Case 41 for discussion of treatment of spasticity). Contractures may result in abnormal tightness of skin overlying bony prominences, par­ticularly ischial tuberosities, femoral trochanters and knees, leading to an increased incidence of decubitus ulceration. Appropriate correction of contractures is required in such cases to obtain resolution of the decubiti.

214
Q

CASE 38: Decubitus Ulcer Associated with Spinal Cord Injury HISTORY: A 27-year-old woman was seen for evaluation of a decubitus ulcer overlying the sacrum. She had been paraplegic since a motor vehicle accident six months previously. She was able to ambulate by wheelchair. EXAMINATION There was a 6 cm decubitus ulcer overlying the sacrum. It had extended down to the fascial level. There was an area of erythema about 2-3 cm wide surrounding the lesion. The lesion itself was composed mainly of necrotic tissue which was gray and pale. Neurological examination revealed complete anesthesia below T4. There was a spastic paraplegia and spastic neurogenic bladder. QUESTIONS

  1. Which of the following are important considerations in the prevention of decubitus ulceration? (Select one or more)

A. Frequent position changes

B. Maintaining skin cleanliness

C. Use of heel rings to prevent heel ulceration

D. Use of air or water mattresses

E. None of the above

A

(A,B) The two keys to prevention of decubitus ulceration in spinal cord injured patients are frequent position changes to redistribute pressure and allow tissue perfusion and the maintenance of skin cleanliness. The importance of frequent position changes, at least every two hours and more often in some cases, cannot be stressed too vigorously to attendants and patients alike. Early in the care of spinal cord injured patients the importance of frequent position changes and skin cleanliness must be pointed out to the patient since it will be an important lifelong consideration. The use of appropriate cushioning properly fitting clothing and footwear, properly positioned uri­nals and caution to avoid unnecessary trauma in transfers must also be taught to patients and attendants. Proper bathing to avoid occlusion of pores leading to skin infections and build-up of foreign material in intertriginous areas is also of value in preventing ulceration. Care with temperature of bath water is also essential to avoid burns of anesthetic skin. The use of massage and lanolin-containing lotions to keep skin soft and supple also reduces the likelihood of ischemic necrosis. Heel rings should be avoided scrupulously. They may actually increase rather than decrease the likelihood of heel ulceration. Heel rings actually interfere with blood supply to skin by com­pressing arterial inflow. Although air and water mattresses and cheepskins were felt to be useful in preventing pressure ulcerations, they are probably of little value. In addition, these static measures may give a false sense of security to attendants who may neglect patient movement, feeling that such devices alone will be success­ful. Frequent position changes, if possible utilizing an electri­cally controlled bed, are the best measures available to prevent ischemic pressure necrosis and ulceration of skin.

215
Q

CASE 39: Pain Following Spinal Injury HISTORY: At 21 years of age, this patient, a Vietnam veteran, sustained a shell fragment injury over the first lumbar vertebrae with ensuing complete flaccid paraplegia complicated by severe and persistent burning pain in both legs. He had an early laminectomy for dural closure, and subsequent re-exploration of the wound one year later in an unsuccessful effort to relieve pain by lysing adhesions and sectioning dorsal roots. Two years after the second operation, the pain was unimproved and the patient complained that he was unable to participate in vocational rehabilitation because of pain. EXAMINATION: The patient had a flaccid paraplegia with complete sensory without sacral sparing. There was wasting of all muscle groups in the lower limbs and he had flaccid anal tone. QUESTIONS

  1. Pain following spinal injury (Select one or more)

A. is uncommon

B. usually develops several weeks after the injury

C. usually persists indefinitely unless treated

D. is more common in open than closed injuries

E. none of the above

A

(E) The overall incidence of pain with spinal injury is high (94%, Botterell, et al.) However, pain considered to be of clinical significance, that is, pain which requires regular analgesic therapy or causes increasing disability, is less common, ranging from 5 to 35%. The onset of pain after injury varies from patient to patient, but patients may suffer severe pain from the moment of injury. On the other hand, pair onset may be delayed for several years. The pain associated with spinai injury may slowly subside, but it may also persist indefinitely in some patients. A comparison of the incidence of pain following spinal injury did not support a conclusion that pain occurred more commonly with open versus closed wounds. or vice versa.

216
Q

CASE 39: Pain Following Spinal Injury HISTORY: At 21 years of age, this patient, a Vietnam veteran, sustained a shell fragment injury over the first lumbar vertebrae with ensuing complete flaccid paraplegia complicated by severe and persistent burning pain in both legs. He had an early laminectomy for dural closure, and subsequent re-exploration of the wound one year later in an unsuccessful effort to relieve pain by lysing adhesions and sectioning dorsal roots. Two years after the second operation, the pain was unimproved and the patient complained that he was unable to participate in vocational rehabilitation because of pain. EXAMINATION: The patient had a flaccid paraplegia with complete sensory without sacral sparing. There was wasting of all muscle groups in the lower limbs and he had flaccid anal tone. QUESTIONS

  1. Pain following spinal injury occurs in a (Select one or more)

A. higher percentage of upper versus lower spmal lesions

B. higher percentage of lower versus upper spinal lesions

C. higher percentage of complete versus incomplete spinal lesions

D. higher percentage of incomplete versus complete spinal lesions

E. none of the above

A

(B,D) Pain following spinal injury is least common following cervical injury and is more common with complete thoracolumbar or cauda equina lesions, but is most common with incomplete lumbar or cauda equina lesions.

217
Q

CASE 39: Pain Following Spinal Injury HISTORY: At 21 years of age, this patient, a Vietnam veteran, sustained a shell fragment injury over the first lumbar vertebrae with ensuing complete flaccid paraplegia complicated by severe and persistent burning pain in both legs. He had an early laminectomy for dural closure, and subsequent re-exploration of the wound one year later in an unsuccessful effort to relieve pain by lysing adhesions and sectioning dorsal roots. Two years after the second operation, the pain was unimproved and the patient complained that he was unable to participate in vocational rehabilitation because of pain. EXAMINATION: The patient had a flaccid paraplegia with complete sensory without sacral sparing. There was wasting of all muscle groups in the lower limbs and he had flaccid anal tone. QUESTIONS

  1. Severe pain following a spinal injury (Select one or more)

A. frequently improves with mild analgesics

B. may improve with vocational rehabilitation

C. may be exacerbated by conditions which undermine general health

D. may be exacerbated by cold and damp weather

E. none of the above

A

(A,B,C,D) Mild analgesics, such as aspirin, phenacetin or occasionally codeine, may relieve pain of spinal injury enough to render residual pain tolerable. Likewise, patients occasionally report reduction of pain as an apparent function of progress toward an independent life outside the hospital. Conversely, pain is frequently aggravated by deterioration of general physical or mental health from diverse causes (e. g. sepsis, infected pressure sores, marital discord, etc. ). Fourteen percent of patients complain that pain is exacerbated by cold, damp weather and is relieved by a change to warmer, drier climate.

218
Q

CASE 39: Pain Following Spinal Injury HISTORY: At 21 years of age, this patient, a Vietnam veteran, sustained a shell fragment injury over the first lumbar vertebrae with ensuing complete flaccid paraplegia complicated by severe and persistent burning pain in both legs. He had an early laminectomy for dural closure, and subsequent re-exploration of the wound one year later in an unsuccessful effort to relieve pain by lysing adhesions and sectioning dorsal roots. Two years after the second operation, the pain was unimproved and the patient complained that he was unable to participate in vocational rehabilitation because of pain. EXAMINATION: The patient had a flaccid paraplegia with complete sensory without sacral sparing. There was wasting of all muscle groups in the lower limbs and he had flaccid anal tone. QUESTIONS

  1. Most authors agree that pain following spinal injury is subserved by the sympathetic nervous system. (True or False)
A

FALSE Davis and Mariin postulated that afferent fibers may travel in the autonomic nervous system and peripheral impulses may bypass the spinal injury via the sympathetic pathway to be recognized as painful sensations. Most authors, however, disagree. Pain of spinal injury has been attributed to “thalamic escape”. Thalamic activity is enhanced with spinal injury by loss of the inhibitory effect of ascending spinal pathways. Alternatively, pain following spinal trauma has been thought to arise from neuroma formation in the proximal spinal cord or nerve root stumps. Finally, pain has been postulated to originate from irritation of neural elements by scarring of pia-arachnoid around the injured spinal cord or nerve roots.

219
Q

CASE 39: Pain Following Spinal Injury HISTORY: At 21 years of age, this patient, a Vietnam veteran, sustained a shell fragment injury over the first lumbar vertebrae with ensuing complete flaccid paraplegia complicated by severe and persistent burning pain in both legs. He had an early laminectomy for dural closure, and subsequent re-exploration of the wound one year later in an unsuccessful effort to relieve pain by lysing adhesions and sectioning dorsal roots. Two years after the second operation, the pain was unimproved and the patient complained that he was unable to participate in vocational rehabilitation because of pain. EXAMINATION: The patient had a flaccid paraplegia with complete sensory without sacral sparing. There was wasting of all muscle groups in the lower limbs and he had flaccid anal tone. QUESTIONS

  1. Early surgical intervention to decompress neural elements or remove foreign objects has been shown to reduce the incidence of pain following spinal injury. (True or False)
A

FALSE Early surgery is thought to increase the incidence of pain associated with spinal injury. Possible scarring secondary to surgery serves to exacerbate the pain state.

220
Q

CASE 39: Pain Following Spinal Injury HISTORY: At 21 years of age, this patient, a Vietnam veteran, sustained a shell fragment injury over the first lumbar vertebrae with ensuing complete flaccid paraplegia complicated by severe and persistent burning pain in both legs. He had an early laminectomy for dural closure, and subsequent re-exploration of the wound one year later in an unsuccessful effort to relieve pain by lysing adhesions and sectioning dorsal roots. Two years after the second operation, the pain was unimproved and the patient complained that he was unable to participate in vocational rehabilitation because of pain. EXAMINATION: The patient had a flaccid paraplegia with complete sensory without sacral sparing. There was wasting of all muscle groups in the lower limbs and he had flaccid anal tone. QUESTIONS

  1. Surgical intervention for treatment of established severe incapacitating pain following spinal injury has been uniformly unsuccessful.. (True or False)
A

FALSE Laminectomies with removal of compressive tissues or foreign objects, posterior rhizotomies and anterolateral cordotomy, are thought by some authors to be of value in treating selected patients for pain following spinal injury. Porter, et al. reported 61. 7% long term relief of pain with cordotomies. White and Sweet^ reported failure of pain relief with rhizotomy, but had a 65% success rate with cordotomics, Davis and Martin, on the other hand, reported consistent failure of pain relief in these patients with cordotomy. White and Sweet discount the value of sympathectomy for pain relief for spin ally injured patients.

221
Q

CASE 40: “Phantom” Sensation Following Spinal Injury HISTORY: A 35-year-old male was thrown from a motorcycle and incurred a spinal injury at the T7 level. EXAMINATION: Neurological examination three months after his injury showed a spastic paraplegia with complete motor impairment below to T7 with minimal sacral sparing to noxious cutaneous stimulation. In spite of the fact that the patient’s legs assumed an extended position, he felt as if his legs were flexed at the hips and knees, as “in a sitting position”. This sensation was intermittent and disappeared if the patient looked at his legs, QUESTIONS

  1. Disorders of body image following spinal injury, as with this patient, are called ‘‘phantom’ sensations by some authors. This sequelae rarely occurs in spinally injured patients. (True or False)
A

FALSE A debate concerning definition of terms complicates the problem of disorders of body images following spinal cord injury. Pollock, et al. would limit the term “phantom” to instances in which amputation of an extremity has been performed. On the other hand, other authors have extended the definition of “phantom” to include awareness of limbs or torso in an area below the level of a complete transverse spinal lesion. Ninety to 100% of patients with spinal injury have “phantom” sensations if the latter definition is accepted.

222
Q

CASE 40: “Phantom” Sensation Following Spinal Injury HISTORY: A 35-year-old male was thrown from a motorcycle and incurred a spinal injury at the T7 level. EXAMINATION: Neurological examination three months after his injury showed a spastic paraplegia with complete motor impairment below to T7 with minimal sacral sparing to noxious cutaneous stimulation. In spite of the fact that the patient’s legs assumed an extended position, he felt as if his legs were flexed at the hips and knees, as “in a sitting position”. This sensation was intermittent and disappeared if the patient looked at his legs, QUESTIONS

  1. “Phantom” sensations following spinal injury occur, as with this patient, only if there is some sensory sparing. (True or False)
A

FALSE “Phantom” sensations have been demonstrated to occur in a high proportion of patients with both complete or incomplete spinal lesions. Pollock, et al, doubt the potential for awareness of the lower extremities with complete lesions, yet they report an awareness of posture and movement of the legs in 12 of 117 patients with “anatomically and physiologically” complete spinal lesions.

223
Q

CASE 40: “Phantom” Sensation Following Spinal Injury HISTORY: A 35-year-old male was thrown from a motorcycle and incurred a spinal injury at the T7 level. EXAMINATION: Neurological examination three months after his injury showed a spastic paraplegia with complete motor impairment below to T7 with minimal sacral sparing to noxious cutaneous stimulation. In spite of the fact that the patient’s legs assumed an extended position, he felt as if his legs were flexed at the hips and knees, as “in a sitting position”. This sensation was intermittent and disappeared if the patient looked at his legs, QUESTIONS

  1. “Phantom” sensations associated with spinal injury commonly include a (Select one or more)

A. sense of limb placement that docs not correlate with actual limb position

B. sense of limb placement that does correlate with actual limb position

C. disordered perception of movement of limbs

D. disordered perception of the size or continuity of limbs

E. none of the above

A

(A,B,C) A disordered perception of the limbs in space connotes an awareness of limb and/or trunk placement in the presence of sensory impairment. The sensation of limb placement may or may not correlate with the actual position of the limb. A dissociation of the position of the “phantom” and the actual position of the paralyzed limb is more likely to occur early after injury. Within a few days to a few months after the injury the position of the “phantom” and the actual limb is more likely to correlate. Altered perception of active motor and dynamic postural phenomena also commonly occur following spinal injury. These sensations may be perceived as a voluntary capability to move a paralyzed limb or part (i.e., the sense of being able to wiggle paralyzed toes) or as involuntary movements (i.e., the sense of the toes being forced down toward the sole of the foot). Disordered perceptions of paralyzed limb bulk, size or continuity rarely occur with spinal injury.

224
Q

CASE 40: “Phantom” Sensation Following Spinal Injury HISTORY: A 35-year-old male was thrown from a motorcycle and incurred a spinal injury at the T7 level. EXAMINATION: Neurological examination three months after his injury showed a spastic paraplegia with complete motor impairment below to T7 with minimal sacral sparing to noxious cutaneous stimulation. In spite of the fact that the patient’s legs assumed an extended position, he felt as if his legs were flexed at the hips and knees, as “in a sitting position”. This sensation was intermittent and disappeared if the patient looked at his legs, QUESTIONS

  1. “Phantom” sensations occur, as with this patient, in injuries to the cauda equina. (True or False)
A

FALSE “Phantom” sensations were described in five patients who had spinal injuries below the L1 vertebral level and presumably, therefore, involved cauda equina.

225
Q

CASE 40: “Phantom” Sensation Following Spinal Injury HISTORY: A 35-year-old male was thrown from a motorcycle and incurred a spinal injury at the T7 level. EXAMINATION: Neurological examination three months after his injury showed a spastic paraplegia with complete motor impairment below to T7 with minimal sacral sparing to noxious cutaneous stimulation. In spite of the fact that the patient’s legs assumed an extended position, he felt as if his legs were flexed at the hips and knees, as “in a sitting position”. This sensation was intermittent and disappeared if the patient looked at his legs, QUESTIONS

  1. “Phantom” sensations can also occur in nonpaired organs below the level of the spinal injury. (True or False)
A

TRUE “Phantom” sensations have been described for the rectum, bladder, anal sphincter, penis, vagina. Phantom sensations of erection, urination and defecation have also been described.

226
Q

CASE 40: “Phantom” Sensation Following Spinal Injury HISTORY: A 35-year-old male was thrown from a motorcycle and incurred a spinal injury at the T7 level. EXAMINATION: Neurological examination three months after his injury showed a spastic paraplegia with complete motor impairment below to T7 with minimal sacral sparing to noxious cutaneous stimulation. In spite of the fact that the patient’s legs assumed an extended position, he felt as if his legs were flexed at the hips and knees, as “in a sitting position”. This sensation was intermittent and disappeared if the patient looked at his legs, QUESTIONS

  1. “Phantom” sensations never last beyond a few months after the spinal injury. (True or False)
A

FALSE “Phantom” sensations with dissociation of position and disordered perceptions of movement tend to disappear over the early days or months following spinal injury. Occasionally, however, these “phantom” sensations persist indefinitely. On the other hand, “phantom” sensations which correlate with actual limb position appear to persist for many years.

227
Q

CASE 40: “Phantom” Sensation Following Spinal Injury HISTORY: A 35-year-old male was thrown from a motorcycle and incurred a spinal injury at the T7 level. EXAMINATION: Neurological examination three months after his injury showed a spastic paraplegia with complete motor impairment below to T7 with minimal sacral sparing to noxious cutaneous stimulation. In spite of the fact that the patient’s legs assumed an extended position, he felt as if his legs were flexed at the hips and knees, as “in a sitting position”. This sensation was intermittent and disappeared if the patient looked at his legs, QUESTIONS

  1. “Phantom” sensations have been ascribed to the formation of “artificial synapses” from damaged neural elements at the level of the spinal injury. (True or False)
A

TRUE Bors has attributed “phantom” sensations to cross stimulation, the “artificial synapse”, of damaged neural elements at the point of spinal injury. There is, however, little agreement amongst authors as to the etiology of “phantom” sensations. For instance, Sweet suggests that “phantom” sensation is dependent on the loss of posterior column function. Cook and Druckemiller, on the other hand, speculate that the “…phantom represents a memory established after prolonged sensory experience of a part and that its appearance is dependent on a functioning cerebral cortex, primarily post-Rolandic in position, and encompassing the temporal area…” Pollock, et al. postulated that “awareness” of the paralyzed extremities requires soaring of some sensation below the level of the lesion. Conomy has reviewed these and the other notions regarding the etiology of disorders of body image with spinal injury.

228
Q

CASE 40: “Phantom” Sensation Following Spinal Injury HISTORY: A 35-year-old male was thrown from a motorcycle and incurred a spinal injury at the T7 level. EXAMINATION: Neurological examination three months after his injury showed a spastic paraplegia with complete motor impairment below to T7 with minimal sacral sparing to noxious cutaneous stimulation. In spite of the fact that the patient’s legs assumed an extended position, he felt as if his legs were flexed at the hips and knees, as “in a sitting position”. This sensation was intermittent and disappeared if the patient looked at his legs, QUESTIONS

  1. Another disorder of body image that may occur after spinal injury is a sense of amputation of the paralyzed parts. (True or False)
A

TRUE A sensation of amputation (“…a sensation of not only not being felt but not being present…”) tends to occur early in the post-traumatic period and disappears usually as reflexes reappear. Occasionally, the paralyzed extremities are not even recognized as belonging to the patient (anosognosia).

229
Q

CASE 41: Spasticity in Spinal Cord Injury HISTORY: A 37-year-old female was seen four months after a gunshot wound at the T3 level. After an initial period of spinal shock, she complained of marked spasticity in the lower extremities with ankles. In addition, she was bothered by intermittent flexor spasms in which the lower extremities would briskly and forcefully on a spinal frame but for the past two months has been maintained in supine position. In addition to the spasticity, she complained of difficulty with constipation. EXAMINATION: Temperature was 100°F and other vital signs were normal. General examination revealed a presacral decubitus ulcer about lumbosacral fascia. There was total loss of sensation below the T3 dermatomal level. She had no voluntary motor control below T3. The lower extremities were held in flexion at the hips, knees and ankles and, while these joints could be extended somewhat, there appeared to be some contractures, particularly at the knees and ankles. The legs tended to be held in adduction. While the legs would remain extended when they were manually brought into that position, minimal stimuli produced marked flexor spasms. The tendon reflexes were hyperactive in the lower extremities with bilateral sustained ankle clonus and bilateral Babinski’s signs. X-RAYS: Chest x-ray was normal. LABORATORY DATA: CBC: Hct. 30%; hemoglobin 10.1 gm, WBC 13,500. Urinalysis: 80-100 WBC per cubic millimeter: urine culture - Pseudomonas Aeruginosa. QUESTIONS

  1. Which of the following are true concerning motor recovery from spinal shock? (Select one or more)

A. Paraplegia in flexion indicates a complete spinal cord transection

B. Paraplegia in extension indicates an incomplete spinal cord transection

C. Reflex recovery generally proceeds from caudal to rostral

D. The receptive field for eliciting reflex withdrawal gradually spreads

E. None of the above

A

(C,D) The sequence of events in the motor recovery from spinal shock has been thoroughly reviewed by Guttmann based on over thirty years of experience with many spinal cord injured patients. The patient illustrated exhibits the usual motor pattern following recovery from spinal shock including hyperreflexia, spasticity and the presence of pathological reflexes. Originally Riddock, studying the recovery of reflexes in patients with spinal shock, proposed that one could determine whether the spinal cord had been completely destroyed at any level or not based on the recovery of reflexes. The idea was proposed that paraplegia in flexion was associated with complete transection of the spinal cord while paraplegia in extension was characteristic of incomplete lesions. This was based on animal experiments which indicated that extensor hypertonias depended on descending suprasegmental influences and would not occur in the case of complete spinal cord transection. Guttmann and Michaelis both indicate that extensive clinical experience did not support this idea. The recovery oi reflexes and the presence of primarily flexor or primarily extensor spasticity is apparently dependent to a great degree on post injury factors rather than on the completeness or incompleteness of the spinal cord lesion per se. Sherrington studied the recovery of spinal reflexes following spinal shock in the dog and Denny-Brown and LeDell similarly studied reflex activity in the cat and dog following spinal cord injury. Their studies indicated that paraplegia in flexion was the common early response of reflexes following the spinal cord injury: however, as time passed, strong extensor spasticity and rigidity appeared to an extent that the animals were actually able to stana; although the limbs were paralyzed as far as voluntary motor activity was concerned. In general, the recovery of reflex function following spinal cord injury proceeds from caudal to rostral. The first reflexes which appear after spinal shock decreases are the anal and bulbocavernosus reflexes followed by dorsiflexion of the toes or feet in response to plantar stimulation. In contrast to what has been stated by some, the return of the bulbocavernosus reflex after spinal cord injury does not indicate that the period of spinal shock is over. The bulbocavernosus reflex is one of the indications that spinal shock is decreasing: however, the bulbocavernosus reflex is frequently seen within hours of spinai cord injury while return of motor tone and reflexes is delayed for many weeks. Initially, strong stimuli or summation of several stimuli are necessary, particularly on the sole of the foot, in order to elicit any reflex response of muscles in the legs and the reflexes fatigue easily. As time progresses, however, the strength of stimulus necessary to elicit reflex withdrawal decreases and the area of skin from which the reflex may be elicited spreads markedly. In the later stages of refiex recovery, the receptive field increases greatly and a variety of reflexes may be elicited by light stimulation of the skin in any area below the spinal cord injury. The recovery of tone in muscles below the level of spinal cord injury is a normal process. It is important to distinguish between the normal return of tone to paralyzed muscles below a spinal cord injury and excessive spasticity which is pathological and a serious impediment to proper rehabilitation of spinal cord injured patients. The presence of flexor spasms and particularly persistent flexion positioning of the lower extremities is extremely distressing to the patient with spinal cord injury. It prevents proper positioning in bed and makes the sitting position impossible.

230
Q

CASE 41: Spasticity in Spinal Cord Injury HISTORY: A 37-year-old female was seen four months after a gunshot wound at the T3 level. After an initial period of spinal shock, she complained of marked spasticity in the lower extremities with ankles. In addition, she was bothered by intermittent flexor spasms in which the lower extremities would briskly and forcefully on a spinal frame but for the past two months has been maintained in supine position. In addition to the spasticity, she complained of difficulty with constipation. EXAMINATION: Temperature was 100°F and other vital signs were normal. General examination revealed a presacral decubitus ulcer about lumbosacral fascia. There was total loss of sensation below the T3 dermatomal level. She had no voluntary motor control below T3. The lower extremities were held in flexion at the hips, knees and ankles and, while these joints could be extended somewhat, there appeared to be some contractures, particularly at the knees and ankles. The legs tended to be held in adduction. While the legs would remain extended when they were manually brought into that position, minimal stimuli produced marked flexor spasms. The tendon reflexes were hyperactive in the lower extremities with bilateral sustained ankle clonus and bilateral Babinski’s signs. X-RAYS: Chest x-ray was normal. LABORATORY DATA: CBC: Hct. 30%; hemoglobin 10.1 gm, WBC 13,500. Urinalysis: 80-100 WBC per cubic millimeter: urine culture - Pseudomonas Aeruginosa. QUESTIONS

  1. Which of the following physiological mechanisms are responsible for one or more)

A. Reduced cortical inhibition of spinal reflexes

B. Reduced vestibulospinal input to spinal reflexes

C. Increased gamma motor neuron activity

D. Facilitation of flexion reflexes by positioning

E. None-of the above

A

(A,B,C,D) The net influence of descending cortical input upon spinal reflexes is one of inhibition. The reflex arc for spinal reflexes is made up of a minimum of two neurons. These constitute the afferent or sensory side of the reflex arc and the efferent or motor side of the reflex arc. Input from the afferent side may come by way of the so-called flexor reflex afferent fibers from skin and other structures, from the muscle spindles or occasionally from the GoIgi tendon organs. The efferent side of the reflex arc is composed of the alpha motor neuron located in the ventral horn of the spinal cord and its distal axon which connects with the muscle fiber by way of the myoneural junction. This so-called monosynaptic reflex is the minimal neurophysiological unit responsible for reflex activity. One or more interneurons may be interposed between the afferent and efferent neurons. The excitability or tendency of the motor neurons to fire in response to an afferent input depends on the amount of afferent activity, the activity in interneurons and descending supraspinal inputs. The net influence of the supraspinal inputs under normal circumstances is one of inhibition. As a result of spinal cord injury, this inhibitory input is interrupted. Thus, there is increased central excitability in the isolated spinal cord below the level of injury. This increased excitability appears to be due primarily to increased activity in the gamma motor neurons resulting in an increased sensitivity of the muscle spindle to stretch. This increased responsiveness to stretch leads to the phenomenon of reflex spread or irradiation, whereby a minor stimulus will produce reflex contraction in muscles a great distance from the point of application of the stimulus. It has been considered that the chemical explanation for this hyperactivity is the phenomenon of denervation supersensitivity. The post synaptic membranes which do not receive regular stimulation because of the degeneration of distal axons resulting from spinal cord injury become supersensitive to neural transmitters. Therefore, a small afferent stimulus with the release of some transmitter results in mass reflex contraction because of the excessive sensitivity of post synaptic membranes. The vestibulospinal tract is responsible for facilitatery input to extensor motor neurons under normal circumstances. A reduction in vestibulospinal input which might occur as a result of spinal cord injury would, therefore, be expected to produce a reduction in extensor tone and might contribute to the occurrence of spasticity in flexion. The facilitation of flexion reflexes by positioning has not received a great deal of attention. Guttmann and Michaelis, however, stressed the importance of limb positioning in the occurrence of flexor spasticity. Positioning the legs in flexion promotes predominant synergism in flexor muscles and paraplegia in flexion. Guttmann indicates that the constant approximation of the insertion points of the flexors of hip and knees when the limbs are placed in flexion passively causes facilitation of their stretch reflexes and at the same time the constant overstretching of their antagonists results in weakening of the stretch reflex of these latter muscles. As a consequence, positioning of the lower extremities in flexion tends to facilitate the occurrence of spasticity in flexion of the lower extremities following spinal cord injury,

231
Q

CASE 41: Spasticity in Spinal Cord Injury HISTORY: A 37-year-old female was seen four months after a gunshot wound at the T3 level. After an initial period of spinal shock, she complained of marked spasticity in the lower extremities with ankles. In addition, she was bothered by intermittent flexor spasms in which the lower extremities would briskly and forcefully on a spinal frame but for the past two months has been maintained in supine position. In addition to the spasticity, she complained of difficulty with constipation. EXAMINATION: Temperature was 100°F and other vital signs were normal. General examination revealed a presacral decubitus ulcer about lumbosacral fascia. There was total loss of sensation below the T3 dermatomal level. She had no voluntary motor control below T3. The lower extremities were held in flexion at the hips, knees and ankles and, while these joints could be extended somewhat, there appeared to be some contractures, particularly at the knees and ankles. The legs tended to be held in adduction. While the legs would remain extended when they were manually brought into that position, minimal stimuli produced marked flexor spasms. The tendon reflexes were hyperactive in the lower extremities with bilateral sustained ankle clonus and bilateral Babinski’s signs. X-RAYS: Chest x-ray was normal. LABORATORY DATA: CBC: Hct. 30%; hemoglobin 10.1 gm, WBC 13,500. Urinalysis: 80-100 WBC per cubic millimeter: urine culture - Pseudomonas Aeruginosa. QUESTIONS

  1. What factors are identified in the case illustrated which may contribute to flexor spasticity? (Select one or more )

A. Urinary tract infection

B. Decubitus ulceration

C. Recumbent position

D. Presence of contractures

E. None of the above

A

(A,B,C,D) Any activity or pathologic process which results in afferent spinal cord input augments the occurrence of pathological spasticity in spinal cord injured patients. In addition, any process which would normally produce pain in the individual with an intact spinal cord results in afferent input to the spinal cord and aggravates spasticity in the spinal cord injured patient. Difficulties with the urinary bladder are prominent causes of increased spasticity in spinal cord injury. Urinary tract infections and constipation are common sources of such aggravation. In the patient illustrated, the presence of pyuria and a positive urine culture are evidence of urinary tract infection. Distention of the urinary bladder is also an important initiator of reflex muscle spasms. In this context, the smaller the capacity of the bladder, the smaller amount.of urine is necessary to elicit reflex response of the skeletal muscles and the more frequent the spasms. Decubitus ulceration leads to afferent input over the flexor reflex afferent fibers since, under normal circumstances, one would expect pain from a skin ulceration. In addition, the chronic infection that often occurs in decubitus ulceration also accounts for excessive spasticity. In the supine position, the flexor response to hips and knees is predominant, while in the prone and upright positions, the extensor reflexes tend to be facilitated. The patient described has been maintained in the recumbent position for the three to four months since injury and this positioning has probably contributed further to flexor spasticity. Guttmann recommends the earliest possible use of the standing position in paraplegics utilizing parallel bars as a means of overcoming the exaggerated action of the flexors of hips and knees. It may be possible to promote the change from flexor to extensor synergy to such an extent that standing and walking in parallel bars without the aid of artificial support to the legs may be possible in paraplegic patients. In high thoracic and cervical lesions where utilization of the parallel bars is difficult or impossible, the extensor synergy of the trunk and lower extremity muscles could be enhanced by placing the patient for several hours into the prone position. The presence of contractures contributes markedly to the occurrence of flexor spasticity. The contractures almost always occur in flexion except for the ankle which tends to contract in extension. The shortening of flexor muscles which occurs in flexion contractures leads to constant afferent stimulation and facilitation of the flexor reflexes and greatly aggravates the tendency to flexor spasticity,

232
Q

CASE 41: Spasticity in Spinal Cord Injury HISTORY: A 37-year-old female was seen four months after a gunshot wound at the T3 level. After an initial period of spinal shock, she complained of marked spasticity in the lower extremities with ankles. In addition, she was bothered by intermittent flexor spasms in which the lower extremities would briskly and forcefully on a spinal frame but for the past two months has been maintained in supine position. In addition to the spasticity, she complained of difficulty with constipation. EXAMINATION: Temperature was 100°F and other vital signs were normal. General examination revealed a presacral decubitus ulcer about lumbosacral fascia. There was total loss of sensation below the T3 dermatomal level. She had no voluntary motor control below T3. The lower extremities were held in flexion at the hips, knees and ankles and, while these joints could be extended somewhat, there appeared to be some contractures, particularly at the knees and ankles. The legs tended to be held in adduction. While the legs would remain extended when they were manually brought into that position, minimal stimuli produced marked flexor spasms. The tendon reflexes were hyperactive in the lower extremities with bilateral sustained ankle clonus and bilateral Babinski’s signs. X-RAYS: Chest x-ray was normal. LABORATORY DATA: CBC: Hct. 30%; hemoglobin 10.1 gm, WBC 13,500. Urinalysis: 80-100 WBC per cubic millimeter: urine culture - Pseudomonas Aeruginosa. QUESTIONS

  1. Which of the following might hp considered in the initial treatment of spasticity in the patient illustrated? (Select one or more)

A. Appropriate positioning

B. Bowel training

C. Valium

D. Baclofen

E. None of the above

A

(A,B,C,D) Appropriate positioning contributes greatly to a reduction in flexor spasticity. The paralyzed limbs should be placed in abduction and extension at hips and knees during the period of spinal shock and during the period of recovery of spinal reflexes. This positioning is appropriate while the patient lies in the supine position. The feet and toes in this situation should be dorsiflexed. When the patient is in the prone position, the extension reflexes of the trunk and lower extremities are facilitated without specific positioning. Early passive movement of the paralyzed limbs prevents flexion contractures which may aggravate the tendency to flexor reflex spasticity. It should be mentioned in this context that the usual nursing maneuver of placing a pillow beneath the knees of patients with spinal cord injury when they are in the supine position should be discouraged. This positioning facilitates the occurrence of flexor spasticity. In the case illustrated, the patient complained of constipation. Distention of the colon aggravates the tendency for flexor spasticity to occur. This is a result of the increased afferent input which occurs to the spinal cord as a result of constant distention of the colon. The utilization of appropriate bowel training with the daily use of suppositories is essential in the management of spinal cord injured patients and will contribute specifically to a reduction in pathological spasticity. A variety of pharmacological agents have been utilized in the treatment of spasticity. Valium produces considerable relaxation of spastic muscles in relatively small doses. It may, however, lead to drug addiction and, if larger doses are necessitated in order to control spasticity, clouding of consciousness may result. Dantrolene sodium has also been utilized in the control of spasticity. In general, Dantrolene is more effective when spasticity is relatively mild and less effective in the more severe cases of mass generalized spasm. In addition, there is a tendency for weakness to occur on a generalized basis with the use of Dantrolene producing reduction in strength in normal musculature. This can be a severe handicap to its use in paraplegic patients who depend on good strength in the upper extremities to carry out many of their activities. Baclofen, a recently introduced agent for the control of spasticity, may also be useful.

233
Q

CASE 41: Spasticity in Spinal Cord Injury HISTORY: A 37-year-old female was seen four months after a gunshot wound at the T3 level. After an initial period of spinal shock, she complained of marked spasticity in the lower extremities with ankles. In addition, she was bothered by intermittent flexor spasms in which the lower extremities would briskly and forcefully on a spinal frame but for the past two months has been maintained in supine position. In addition to the spasticity, she complained of difficulty with constipation. EXAMINATION: Temperature was 100°F and other vital signs were normal. General examination revealed a presacral decubitus ulcer about lumbosacral fascia. There was total loss of sensation below the T3 dermatomal level. She had no voluntary motor control below T3. The lower extremities were held in flexion at the hips, knees and ankles and, while these joints could be extended somewhat, there appeared to be some contractures, particularly at the knees and ankles. The legs tended to be held in adduction. While the legs would remain extended when they were manually brought into that position, minimal stimuli produced marked flexor spasms. The tendon reflexes were hyperactive in the lower extremities with bilateral sustained ankle clonus and bilateral Babinski’s signs. X-RAYS: Chest x-ray was normal. LABORATORY DATA: CBC: Hct. 30%; hemoglobin 10.1 gm, WBC 13,500. Urinalysis: 80-100 WBC per cubic millimeter: urine culture - Pseudomonas Aeruginosa. QUESTIONS

  1. Which of the following might be considered in the treatment of spasticity in the patient illustrated if the above measures fail to be effective? (Select one or more)

A. Anterior rhizotomy

B.Myelotomy

C. Subarachnoid alcohol or phenol

D. Tenotomy or myotomy

E. None of the above

A

(A,B,C,D) Appropriate positioning, the control of intercurrent problems such as urinary tract infection and decubitus ulceration prevention of contractures, as well as the utilization of pharmacological agents, has improved the control of spasticity in spinal cord injured patients. A small number of patients fail to respond to these methods and more radical means of treatment are required. Anterior rhizotomy was found to be effective in converting spastic paraplegia to flaccid paralysis. In this procedure, the anterior spinal roots conveying motor impulses to the musculature are interrupted from approximately T10 to S1. Most surgeons attempt to spare the S2 spinal root in this situation, feeling that the remaining motor input to the urinary bladder is probably beneficial in allowing reflex micturition. Not all authors agree with this, however, and some attempt to produce rhizotomy from T10 to S5. Anterior rhizotomy has several disadvantages, including the need for a relatively extensive laminectomy and difficulty with identification of the proper spinal roots to be sectioned prior to rhizotomy. The dentate ligaments and the size of individual anterior roots, once felt to be relatively constant indications for the identification of specific anterior roots, have been found to be less reliable than once thought. Anterior nerve root stimulation during surgical procedures aids the identification of nerve roots but adds considerably to the magnitude of the operative procedure. In order to preserve motor activity particularly in patients with incomplete lesions, Bischof introduced longitudinal myelotomy. In this procedure, a transverse or “T”-shaped incision is made in the spinal cord from T12-S1. The procedure interrupts the monosynaptic reflex arc by a transverse incision through the gray matter of the spinal cord midway between the dorsal and ventral horns. As a result, reflex augmentation of muscle tone is abolished, as are all tendon and withdrawal reflexes. The procedure avoids descending motor pathways so that if voluntary motor activity is present in patients with incomplete spinal cord injury, it maybe preserved. Since the procedure does not go caudal to S1, bladder reflex activity is preserved. Although some authors advocate tms procedure, the results in relief of spasticity and preservation of voluntary motor activity are unpredictable. Rhizotomy may also be accomplished by chemical means by the subarachnoid instillation of phenol or alcohol via spinal puncture. These agents produce chemical destruction of nerve libers. They are useful because no surgical procedure is required. Localization of the agent to produce rhizotomy of the desired roots may be a problem. Guttmann and others utilize this method frequently, although the overall effectiveness of the method appears open to question, undesirable effects include reduction of bladder sensation, loss of sexual reflexes including erection, destruction, of remaining sensation and the production of inflammatory changes such as arachnoiditis, meningitis, vascular thrombosis or demyelination. Such complications may aggravate pain or spasticity and make later surgical procedures, if required, more difficult. Tenotomy or myotomy may be helpful in overcoming the effects of flexion contractures and aiding in limb positioning. While tenotomy or myotomy may be helpful in reference to a particular joint, the value of these procedures is limited in generalized reflex hyperactivity and spasticity of the lower limbs due to spinal cord injury. A variety of other procedures including peripheral neurectomy or nerve block, cordotomy, cordectomy, posterior rhizotomy and electrocoagulation of the spinal cord have been utilized. These procedures are considerably less effective and little utilized today.

234
Q

CASE 42: Autonomic Dysreflexia (Hyperreflexia) Following Spinal Injury HISTORY: A 24-year-old male was seen complaining of episodic pounding wound to the chest four years previously. The present problem started three months ago. On closer questioning, the patient had noted over the same three month period of time increasing frequency of spontaneous of urinary catheter drainage for three years EXAMINATION: His vital signs and general physical examination were normal level bilaterally. HOSPITAL COURSE: Soon after admission, the patient was observed to have a paroxysmal episode lasting five minutes with symptoms as described above. In addition, tachycardia, hypertension, The patient was catheterized after a spontaneous voiding and had a urine residual of 300cc. QUESTIONS

  1. The episodes of which the patient complains are characteristic of “autonomic dusreflexia” (True or False)
A

TRUE The features of “autonomic dysreflexia” are fully typified by this patient.

235
Q

CASE 42: Autonomic Dysreflexia (Hyperreflexia) Following Spinal Injury HISTORY: A 24-year-old male was seen complaining of episodic pounding wound to the chest four years previously. The present problem started three months ago. On closer questioning, the patient had noted over the same three month period of time increasing frequency of spontaneous of urinary catheter drainage for three years EXAMINATION: His vital signs and general physical examination were normal level bilaterally. HOSPITAL COURSE: Soon after admission, the patient was observed to have a paroxysmal episode lasting five minutes with symptoms as described above. In addition, tachycardia, hypertension, The patient was catheterized after a spontaneous voiding and had a urine residual of 300cc. QUESTIONS

  1. “Autonomic dysreflexia” can be seen with lesions at any level of the spinal cord. (True or False)
A

FALSE “Autonomic dysreflexia” is seen with spinal lesions above T7.

236
Q

CASE 42: Autonomic Dysreflexia (Hyperreflexia) Following Spinal Injury HISTORY: A 24-year-old male was seen complaining of episodic pounding wound to the chest four years previously. The present problem started three months ago. On closer questioning, the patient had noted over the same three month period of time increasing frequency of spontaneous of urinary catheter drainage for three years EXAMINATION: His vital signs and general physical examination were normal level bilaterally. HOSPITAL COURSE: Soon after admission, the patient was observed to have a paroxysmal episode lasting five minutes with symptoms as described above. In addition, tachycardia, hypertension, The patient was catheterized after a spontaneous voiding and had a urine residual of 300cc. QUESTIONS

  1. “Autonomic dysreflexia” in this patient is probably due to (Select one or more)

A. sympathetic over-response to bladder filling

B. parasympathetic over-response to bladder filling

C. paroxysmal autonomic over activity unrelated to bladder filling

D. psychiatric disease E. none of the above

A

(A) The sympathetic outflow to the urinary bladder originates in the intermediolateral cell column at the L1-L3 levels forming the hypogastric plexus and acts, in part, to increase internal sphincter tone. The parasympathetic outflow is via S2-5 forming the pelvic plexus and acts, in part, to relax the internal sphincter and contract the bladder. Volitional control of bladder sphincter function is via the pudendal nerve (S1-4) and innervates, in part, the external sphincter. The sympathetic outflow below the level of the midthoracic lesion is unregulated by higher neural centers. Over-response of sympathetic outflow to bladder filling is manifested in the urogenital system by abnormally high proximal urethral pressure. Systemically, those alterations known as ‘‘autonomic dysreflexia” suggest an inappropriate sympathetic response to bladder distension.

237
Q

CASE 42: Autonomic Dysreflexia (Hyperreflexia) Following Spinal Injury HISTORY: A 24-year-old male was seen complaining of episodic pounding wound to the chest four years previously. The present problem started three months ago. On closer questioning, the patient had noted over the same three month period of time increasing frequency of spontaneous of urinary catheter drainage for three years EXAMINATION: His vital signs and general physical examination were normal level bilaterally. HOSPITAL COURSE: Soon after admission, the patient was observed to have a paroxysmal episode lasting five minutes with symptoms as described above. In addition, tachycardia, hypertension, The patient was catheterized after a spontaneous voiding and had a urine residual of 300cc. QUESTIONS

  1. Autonomic dysreflexia” is usually associated with over-distension of the bladder. (True or False)
A

TRUE Bladder neck constriction, blockage of an indwelling catheter, urinary calculi and other causes of urinary retention are the most common causes of “autonomic dysreflexia”. However, the syndrome complex is also seen in spinally injured patients with excessive activity of other abdominal viscera in the anesthetic area of the body. Therefore, ‘‘autonomic dysreflexia” may be seen with constipation, uterine contractions with labor, cholelithiasis, etc.

238
Q

CASE 42: Autonomic Dysreflexia (Hyperreflexia) Following Spinal Injury HISTORY: A 24-year-old male was seen complaining of episodic pounding wound to the chest four years previously. The present problem started three months ago. On closer questioning, the patient had noted over the same three month period of time increasing frequency of spontaneous of urinary catheter drainage for three years EXAMINATION: His vital signs and general physical examination were normal level bilaterally. HOSPITAL COURSE: Soon after admission, the patient was observed to have a paroxysmal episode lasting five minutes with symptoms as described above. In addition, tachycardia, hypertension, The patient was catheterized after a spontaneous voiding and had a urine residual of 300cc. QUESTIONS

  1. Treatment of “autonomic dysreflexia” includes (Select one or more)

A. no treatment is necessary

B. relief of bladder distension

C. alpha-sympathetic blocking drugs (i.e., phenoxybenzamine, phentolamine)

D. cholinergic stimulating drugs (i.e., urecholine)

E. none of the above.

A

(B,C) Fatalities from cerebral hemorrhage associated with “autonomic dysreflexia” and hypertension have been reported. Appropriate management of the syndrome is, therefore, indicated. The underlying cause (i.e., bladder distention, constipation, etc.) should be corrected. If treatment of the inciting cause is not possible, therapy should be directed at the manifestations of “autonomic dysreflexia” and alpha-sympathetic blocking drugs have been recommended. Additionally, these drugs may act to relax the internal sphincter and alleviate a potential cause of bladder outlet obstruction. Cholinergic drugs have not been used for the treatment of “autonomic dysreflexia”.

239
Q

CASE 43: A quadriplegic with Urinary Dysfunction HISTORY: A 16-year-old male dove off a diving board into an empty swimming pool. He struck the back of his head but did not lose consciousness. He felt a snapping in his neck and was unable to feel or move his arms. He was carefully moved to the hospital for evaluation. EXAMINATION: On examination, his vital signs and the general physical neurological examination included a complete flaccid quadriplegia with sensory sphincter paralysis was also present. X-RAY: Cervical spine x-rays revealed a fracture-dislocation of C6-7. HOSPITAL COURSE: Reduction of the fracture-dislocation was accomplished within hours using skull-tong traction. He was placed on a Stryker frame bed and a urinary catheter was inserted.His neurological examination was unchanged 48 hours after admission. QUESTIONS

  1. Forty-eight hours after the accident, the patient was found to have an absent bulbocavernosus reflex. At the same time, a cystometrogram demonstrated a hypotonic, high capacity bladder. The patient had no sensation of bladder distension even with 500cc, of bladder filling. These findings are consistent with

A. a reflex neurogenic bladder

B. spinal shock

C. a flaccid neurogenic bladder

D. normal bladder activity

E. none of the above

A

(B) The atonic, large capacity bladder is the hallmark of low sacral spinal or cauda equina injury. It represents a denervated bladder and is called a “flaccid” bladder. However, similar bladder findings are seen, as with this patient, during the period of spinal shock with spinal cord lesions above the conus. Parkash points out that the bladder, during the period of spinal shock, is areflexic rather than atonic. Like the limbs, bladder reflexes return and eventually become hyperactive as spinal shock subsides. Bladder reflexes can be detected as early as 8-14 days but usually do not return for 4-8 weeks. As the bladder becomes hyperactive, its activity is characterized by spasticity, involuntary contractions and small volume capacity. This hyperactive bladder is known as a “reflex” bladder.

240
Q

CASE 43: A quadriplegic with Urinary Dysfunction HISTORY: A 16-year-old male dove off a diving board into an empty swimming pool. He struck the back of his head but did not lose consciousness. He felt a snapping in his neck and was unable to feel or move his arms. He was carefully moved to the hospital for evaluation. EXAMINATION: On examination, his vital signs and the general physical neurological examination included a complete flaccid quadriplegia with sensory sphincter paralysis was also present. X-RAY: Cervical spine x-rays revealed a fracture-dislocation of C6-7. HOSPITAL COURSE: Reduction of the fracture-dislocation was accomplished within hours using skull-tong traction. He was placed on a Stryker frame bed and a urinary catheter was inserted.His neurological examination was unchanged 48 hours after admission. QUESTIONS

  1. Prior to the return of reflex activity, bladder drainage is best maintained with continuous catheter drainage. (True or False)
A

FALSE Indwelling catheters are a major source of infection. With a catheter, infected urine is usually cultured by forty-eight hours and a bacteremia found in an overwhelming proportion of patients (98%) by ninety-six hours. Other genitourinary complications (primary bladder stones, hemorrhagic cystitis and urethral strictures, diverticula and fistuli) occur at significantly higher rates in patients with chronic indwelling catheters compared to patients with short term catheter placement. Guttmann introduced intermittent catheterization in 4949, using an aspetic, no-touch technique and demonstrated a reduction of infectious complications. In addition, intermittent catheterization is said to facilitate the early return of reflex bladder activity.

241
Q

CASE 43: A quadriplegic with Urinary Dysfunction HISTORY: A 16-year-old male dove off a diving board into an empty swimming pool. He struck the back of his head but did not lose consciousness. He felt a snapping in his neck and was unable to feel or move his arms. He was carefully moved to the hospital for evaluation. EXAMINATION: On examination, his vital signs and the general physical neurological examination included a complete flaccid quadriplegia with sensory sphincter paralysis was also present. X-RAY: Cervical spine x-rays revealed a fracture-dislocation of C6-7. HOSPITAL COURSE: Reduction of the fracture-dislocation was accomplished within hours using skull-tong traction. He was placed on a Stryker frame bed and a urinary catheter was inserted.His neurological examination was unchanged 48 hours after admission. QUESTIONS

  1. Return of reflex bladder activity maybe assessed by means of (Select one or more)

A. cremasteric reflex

B. bulbocavernosus reflex

C. anal “wink”

D. cystometrographic analysis

E. none of the above

A

(B.C,D) The cremasteric reflex is a cutaneous reflex that is diminished in any upper motor neuron lesion. This reflex, therefore, remains diminished even after spinal shock has resolved. The bulbocavernosus reflex (anal contraction with clitoral or glans penis pressure) and the anal ‘“wink” (anal constriction with pinprick stimulation of the anal mucocutaneous junction) are subserved by the same sacral dermatomes as the bladder and reappear as spinal shock subsides. Cystometry provides a graphic record of bladder activity during bladder filling. After the period of spinal shock, reflex contractile waves can be recorded as the bladder fills.

242
Q

CASE 43: A quadriplegic with Urinary Dysfunction HISTORY: A 16-year-old male dove off a diving board into an empty swimming pool. He struck the back of his head but did not lose consciousness. He felt a snapping in his neck and was unable to feel or move his arms. He was carefully moved to the hospital for evaluation. EXAMINATION: On examination, his vital signs and the general physical neurological examination included a complete flaccid quadriplegia with sensory sphincter paralysis was also present. X-RAY: Cervical spine x-rays revealed a fracture-dislocation of C6-7. HOSPITAL COURSE: Reduction of the fracture-dislocation was accomplished within hours using skull-tong traction. He was placed on a Stryker frame bed and a urinary catheter was inserted.His neurological examination was unchanged 48 hours after admission. QUESTIONS

  1. Catheter-free bladder drainage can be anticipated for this patient. (True or False)
A

TRUE As reflex activity of the bladder returns, filling of the bladder with urine stimulates reflex contractions for bladder emptying. A program of intermittent catheterization can achieve a balance of the volume of the bladder capacity, the strength of the reflex bladder contractions and the volume of urine residuals aimed at catheter-free status. Up to 90% of spinally injured patients with reflex bladders may become catheter-free.

243
Q

CASE 43: A quadriplegic with Urinary Dysfunction HISTORY: A 16-year-old male dove off a diving board into an empty swimming pool. He struck the back of his head but did not lose consciousness. He felt a snapping in his neck and was unable to feel or move his arms. He was carefully moved to the hospital for evaluation. EXAMINATION: On examination, his vital signs and the general physical neurological examination included a complete flaccid quadriplegia with sensory sphincter paralysis was also present. X-RAY: Cervical spine x-rays revealed a fracture-dislocation of C6-7. HOSPITAL COURSE: Reduction of the fracture-dislocation was accomplished within hours using skull-tong traction. He was placed on a Stryker frame bed and a urinary catheter was inserted.His neurological examination was unchanged 48 hours after admission. QUESTIONS

  1. Failure to achieve catheter-free status within three months after spinal shock subsides may indicate (Select one or more)

A. pelvic floor dyssynergia

B. spastic pelvic floor

C. enlarged prostate gland

D.. bladder neck fibrosis

E. none of the above

A

(A,B,C,D) Automatic voiding depends not only on reflex bladder contraction but on simultaneous reduction of urethral pressure, due to pelvic floor muscle relaxation. Incoordination of these pelvic floor muscular activities (dyssynergia) or relative hyperactivity of perineal muscles (spastic pelvic floor) may prevent bladder emptying even with adequate bladder contractions. Likewise, scarring (fibrosis) of the bladder neck (as may occur with recurrent infections) and an enlarged prostate gland may not allow enough bladder outlet opening to permit bladder emptying with reflex contraction.

244
Q

CASE 43: A quadriplegic with Urinary Dysfunction HISTORY: A 16-year-old male dove off a diving board into an empty swimming pool. He struck the back of his head but did not lose consciousness. He felt a snapping in his neck and was unable to feel or move his arms. He was carefully moved to the hospital for evaluation. EXAMINATION: On examination, his vital signs and the general physical neurological examination included a complete flaccid quadriplegia with sensory sphincter paralysis was also present. X-RAY: Cervical spine x-rays revealed a fracture-dislocation of C6-7. HOSPITAL COURSE: Reduction of the fracture-dislocation was accomplished within hours using skull-tong traction. He was placed on a Stryker frame bed and a urinary catheter was inserted.His neurological examination was unchanged 48 hours after admission. QUESTIONS

  1. Inadequate bladder emptying may lead to ureteral reflux and hydronephrosis or upper urinary tract infection with renal failure. Measurement of blood urea nitrogen and creatinine (IS, IS NOT) a satisfactory way to assess renal function in the spinally injured patient.
A

IS NOT Blood urea nitrogen and creatinine levels maybe normal in paralyzed patients in spite of inadequate renal function. Poor nutritional status and diminished muscle mass may account for this discrepancy. Creatinine clearance more accurately reflects renal function in paralyzed patients.

245
Q

CASE 43: A quadriplegic with Urinary Dysfunction HISTORY: A 16-year-old male dove off a diving board into an empty swimming pool. He struck the back of his head but did not lose consciousness. He felt a snapping in his neck and was unable to feel or move his arms. He was carefully moved to the hospital for evaluation. EXAMINATION: On examination, his vital signs and the general physical neurological examination included a complete flaccid quadriplegia with sensory sphincter paralysis was also present. X-RAY: Cervical spine x-rays revealed a fracture-dislocation of C6-7. HOSPITAL COURSE: Reduction of the fracture-dislocation was accomplished within hours using skull-tong traction. He was placed on a Stryker frame bed and a urinary catheter was inserted.His neurological examination was unchanged 48 hours after admission. QUESTIONS

  1. Complications of urological management are more common in patients with complete spinal cord lesions than in patients with incomplete spinal cord lesions. (True or False)
A

TRUE Infections and the eventual need for bladder outlet operations are more common in patients with complete versus incomplete spinal cord injuries. Similarly, the length of time requireu to wean patients from intermittent catheterization is greater for patients with complete than patients with incomplete spinal cord lesions.

246
Q

CASE 44 : Sexual Activity in Spinally injured Male HISTORY: A 27-year-old male was transferred to a rehabilitation facility from an acute care hospital. Two weeks previously he had suffered a C7-T1 fracture-dislocation with immediate complete paraplegia as a result of striking his head on the shallow sandy bottom of the ocean while surfing near the beach. At the acute care hospital his fracture was managed with skeletal tong traction. There was no improvement in his neurological status. EXAMINATION: On examination following transfer, the patient was still in skull tong traction on a Stryker-frame bed. His general physical examination was normal. The neurological examination showed complete loss of motor and sensory function below the C8 level. The deep tendon reflexes in the lower extremities, the bulbocavernosus reflex, and the anal reflexes were absent. X-RAY: Cervical spine x-rays show good alignment at C7-T1. HOSPITAL COURSE: Over the first several weeks the patient began to establish friendships with other patients and staff members. regarding his future sexual capabilities QUESTIONS

  1. Because the prospects of gratifying sexual activities are so dismal for this patient, it is best to delay discussions until the patient insists on specific information. (True or False)
A

FALSE Sexual capabilities are often an early concern in spinally injured patients. Whereas the prognosis for many aspects of sexual activities are difficult to predict, satisfying sexual relationships are established in approximately 75% of spinally injured patients.

247
Q

CASE 44 : Sexual Activity in Spinally injured Male HISTORY: A 27-year-old male was transferred to a rehabilitation facility from an acute care hospital. Two weeks previously he had suffered a C7-T1 fracture-dislocation with immediate complete paraplegia as a result of striking his head on the shallow sandy bottom of the ocean while surfing near the beach. At the acute care hospital his fracture was managed with skeletal tong traction. There was no improvement in his neurological status. EXAMINATION: On examination following transfer, the patient was still in skull tong traction on a Stryker-frame bed. His general physical examination was normal. The neurological examination showed complete loss of motor and sensory function below the C8 level. The deep tendon reflexes in the lower extremities, the bulbocavernosus reflex, and the anal reflexes were absent. X-RAY: Cervical spine x-rays show good alignment at C7-T1. HOSPITAL COURSE: Over the first several weeks the patient began to establish friendships with other patients and staff members. regarding his future sexual capabilities QUESTIONS

  1. Penile erection in the normal male occurs (Select one or more)

A. as a result of parasympathetic activity

B. as a result of sympathetic activity

C. with cutaneous (reflexogenic) stimuiation

D. with psychogenic stimulation

E spontaneously

A

(A,C,D,E) Penile erection occurs with arterial vasoadilatation and engorgement of the corpora cavernosa and corpora spongiosa as a result, of parasympathetic activity via the S2-4 roots. This final efferent component of penile erection reflexly occurs with penile cutaneous stimulation reflexogenic erection) via the pudendal and dorsal nerve of the penis. Spontaneous penile erection probably depends on the same reflex neural pathway. Psychogenic erections (erections in response to fantasies, visual stimuli, etc.) are probably dependent on supraspinal connections to the sacral parasympathetic center. Sympathetic neural activity may play a role in the detumescence of the engorged erect penis, but probably has no role in the development of erection.

248
Q

CASE 44 : Sexual Activity in Spinally injured Male HISTORY: A 27-year-old male was transferred to a rehabilitation facility from an acute care hospital. Two weeks previously he had suffered a C7-T1 fracture-dislocation with immediate complete paraplegia as a result of striking his head on the shallow sandy bottom of the ocean while surfing near the beach. At the acute care hospital his fracture was managed with skeletal tong traction. There was no improvement in his neurological status. EXAMINATION: On examination following transfer, the patient was still in skull tong traction on a Stryker-frame bed. His general physical examination was normal. The neurological examination showed complete loss of motor and sensory function below the C8 level. The deep tendon reflexes in the lower extremities, the bulbocavernosus reflex, and the anal reflexes were absent. X-RAY: Cervical spine x-rays show good alignment at C7-T1. HOSPITAL COURSE: Over the first several weeks the patient began to establish friendships with other patients and staff members. regarding his future sexual capabilities QUESTIONS 3. The prospects for this patient to have erections (Select one or more)

A. are good

B. are related to return of bladder activity

C. will be affected by the need for bladder neck operations aimed at making the patient catheter-free

D. that are “reflexogenic” are better than the prospects for 1 “psychogenic” erections

E. none of the above

A

(A,D) Forty-one of 45 patients (91%) with complete cervical spinal cord lesions developed erections; however, none of the patients had psychogenic erections. Although the same nerves are involved, the presence of functioning bladder reflexes does not imply that sexual function will be present, or vice versa. Whereas bladder neck operations in spinal injured patients may diminish the patient’s ability to have erections, a large portion of patients (75%) were unaltered or improved with regard to erection following these operations.

249
Q

CASE 44 : Sexual Activity in Spinally injured Male HISTORY: A 27-year-old male was transferred to a rehabilitation facility from an acute care hospital. Two weeks previously he had suffered a C7-T1 fracture-dislocation with immediate complete paraplegia as a result of striking his head on the shallow sandy bottom of the ocean while surfing near the beach. At the acute care hospital his fracture was managed with skeletal tong traction. There was no improvement in his neurological status. EXAMINATION: On examination following transfer, the patient was still in skull tong traction on a Stryker-frame bed. His general physical examination was normal. The neurological examination showed complete loss of motor and sensory function below the C8 level. The deep tendon reflexes in the lower extremities, the bulbocavernosus reflex, and the anal reflexes were absent. X-RAY: Cervical spine x-rays show good alignment at C7-T1. HOSPITAL COURSE: Over the first several weeks the patient began to establish friendships with other patients and staff members. regarding his future sexual capabilities QUESTIONS

  1. Ejaculation in the normal male depends only on parasympathetic activation (True or False).
A

FALSE Secretions of fluids and emission of sperm are regulated by the sympathetic pathways originating in the upper lumbar spinal segment. Parasympathetic activity initiates peristalsis of smooth muscle and somatic nerves activate contractions of striated pelvic floor muscle to complete the ejaculation of semen.

250
Q

CASE 44 : Sexual Activity in Spinally injured Male HISTORY: A 27-year-old male was transferred to a rehabilitation facility from an acute care hospital. Two weeks previously he had suffered a C7-T1 fracture-dislocation with immediate complete paraplegia as a result of striking his head on the shallow sandy bottom of the ocean while surfing near the beach. At the acute care hospital his fracture was managed with skeletal tong traction. There was no improvement in his neurological status. EXAMINATION: On examination following transfer, the patient was still in skull tong traction on a Stryker-frame bed. His general physical examination was normal. The neurological examination showed complete loss of motor and sensory function below the C8 level. The deep tendon reflexes in the lower extremities, the bulbocavernosus reflex, and the anal reflexes were absent. X-RAY: Cervical spine x-rays show good alignment at C7-T1. HOSPITAL COURSE: Over the first several weeks the patient began to establish friendships with other patients and staff members. regarding his future sexual capabilities QUESTIONS

  1. The prospect for this patient to be able to ejaculate are poor. (True-or False)
A

TRUE Ejaculation is a highly complex function and, hence more vulnerable to spinal trauma than erection. No patient with a complete cervical lesion has been able to ejaculate.

251
Q

CASE 44 : Sexual Activity in Spinally injured Male HISTORY: A 27-year-old male was transferred to a rehabilitation facility from an acute care hospital. Two weeks previously he had suffered a C7-T1 fracture-dislocation with immediate complete paraplegia as a result of striking his head on the shallow sandy bottom of the ocean while surfing near the beach. At the acute care hospital his fracture was managed with skeletal tong traction. There was no improvement in his neurological status. EXAMINATION: On examination following transfer, the patient was still in skull tong traction on a Stryker-frame bed. His general physical examination was normal. The neurological examination showed complete loss of motor and sensory function below the C8 level. The deep tendon reflexes in the lower extremities, the bulbocavernosus reflex, and the anal reflexes were absent. X-RAY: Cervical spine x-rays show good alignment at C7-T1. HOSPITAL COURSE: Over the first several weeks the patient began to establish friendships with other patients and staff members. regarding his future sexual capabilities QUESTIONS

  1. This patient is likely, to have inadequate spermatogenesis.(True or False)
A

TRUE Testicular atrophy with inadequate spermatogenesis commonly occurs after spinal cord injury and is thought to be due to alterations in temperature regulating mechanisms.

252
Q

CASE 44 : Sexual Activity in Spinally injured Male HISTORY: A 27-year-old male was transferred to a rehabilitation facility from an acute care hospital. Two weeks previously he had suffered a C7-T1 fracture-dislocation with immediate complete paraplegia as a result of striking his head on the shallow sandy bottom of the ocean while surfing near the beach. At the acute care hospital his fracture was managed with skeletal tong traction. There was no improvement in his neurological status. EXAMINATION: On examination following transfer, the patient was still in skull tong traction on a Stryker-frame bed. His general physical examination was normal. The neurological examination showed complete loss of motor and sensory function below the C8 level. The deep tendon reflexes in the lower extremities, the bulbocavernosus reflex, and the anal reflexes were absent. X-RAY: Cervical spine x-rays show good alignment at C7-T1. HOSPITAL COURSE: Over the first several weeks the patient began to establish friendships with other patients and staff members. regarding his future sexual capabilities QUESTIONS

  1. In spinal cord injury, factors which significantly affect sexual activities include (Select one or more)

A. complete versus incomplete neural lesions

B. level of the neural lesion

C. age of the patient at injury

D. marital status

E. none of the above

A

(A,B,C) Preservation of some spinal function, particularly pinprick sensation in the sacral segments, offers prognostic evidence in favor of “psychogenic erections”. The prospects for ejaculation and fertility are also improved with incomplete lesions versus complete lesions. Complete upper motor neuron-type (UMN) spinal lesions at any level offer much the same poor prognosis with regard to psychogenic erections and ejaculation. However, occasionally with complete UMN spinal lesions below the T10 level, these functions have been preserved. Patients with complete lower motor neuron-type (LMN) lesions (conus medullaris and cauda equina) are less likely to have “reflexogenic” erections than patients with UMN lesions. However, approximately 25% of this complete LMN group will have “psychogenic” erections and a smaller percentage will be able to ejaculate and sire children. Determinations of overall sexual adjustment following spinal injury were made using factors such as frequency of sexual contact, self-rating of sexual satisfaction, comparison with preinjury sexual satisfaction, perception of partner sexual satisfaction and others. Whereas the age of the patient at injury inversely correlated with sexual satisfaction, no correlation was demonstrated between marital status and the above determinants. Interestingly, these same investigators also failed to find a significant correlation between overall sexual satisfaction and the level of the spinal lesion. The discrepancy between sexual function and sexual satisfaction as related to the level of the spinal lesion points out the many determinates of sexual adjustment to spinal injury. Perhaps as important as the physiological alterations engendered by the injury are the emotional needs of the patient and his ability to establish and maintain satisfying interpersonal relationships. For this reason, sexual rehabilitation of the spinally injured patient can produce a high proportion of satisfied patients even in the face of profound physiological alterations.

253
Q

CASE 45: Sexual Activity in a Spinally Injured Female HISTORY: A 22-year-old married female had sustained a T7-8 fracture from a skiing accident with complete paraplegia below the T8 She had not required surgery and had had no improvement in her neurological status following the accident. She was ready for discharge and she and her husband had several questions related to sexual activity. EXAMINATION: On examination, her general physical and gynecological examination were normal. The neurological examination was abnormal for a complete T8 sensory-motor spastic paraplegia. Her bulbocavernosus and anal wink reflexes were both brisk. QUESTIONS

  1. The patient was still using an indwelling catheter. Must she remove the catheter prior to intercourse? (Yes or No)
A

NO Coitus can be performed with a urinary catheter left in place. Female patients can be instructed on proper catheter position prior to intercourse.

254
Q

CASE 45: Sexual Activity in a Spinally Injured Female HISTORY: A 22-year-old married female had sustained a T7-8 fracture from a skiing accident with complete paraplegia below the T8 She had not required surgery and had had no improvement in her neurological status following the accident. She was ready for discharge and she and her husband had several questions related to sexual activity. EXAMINATION: On examination, her general physical and gynecological examination were normal. The neurological examination was abnormal for a complete T8 sensory-motor spastic paraplegia. Her bulbocavernosus and anal wink reflexes were both brisk. QUESTIONS

  1. The patient’s menses had been irregular since the time of the accident. Will she be able to become pregnant? (Yes-or No)
A

YES Menstrual irregularities often occur after spinal injury but irregularities usually last only a few months. Fifty percent of spinally injured females never miss a menstrual period. Whether or not fertility is reduced in spinally injured females compared to spinally intact females is uncertain, but normally menstruating spinally injured females may become pregnant.

255
Q

CASE 45: Sexual Activity in a Spinally Injured Female HISTORY: A 22-year-old married female had sustained a T7-8 fracture from a skiing accident with complete paraplegia below the T8 She had not required surgery and had had no improvement in her neurological status following the accident. She was ready for discharge and she and her husband had several questions related to sexual activity. EXAMINATION: On examination, her general physical and gynecological examination were normal. The neurological examination was abnormal for a complete T8 sensory-motor spastic paraplegia. Her bulbocavernosus and anal wink reflexes were both brisk. QUESTIONS

  1. If the patient becomes pregnant, will she need to have a cesarean section delivery? (Yes or No)
A

NO Most pregnancies in spinally injured females are uncomplicated and delivery is transvaginal. Obstetricians are cautioned to guard against autonomic dysreflexia during labor and must differentiate this syndrome from eclampsia. Pregnant, spinally injured females have a higher incidence of renal complications than spinally intact females.

256
Q

CASE 45: Sexual Activity in a Spinally Injured Female HISTORY: A 22-year-old married female had sustained a T7-8 fracture from a skiing accident with complete paraplegia below the T8 She had not required surgery and had had no improvement in her neurological status following the accident. She was ready for discharge and she and her husband had several questions related to sexual activity. EXAMINATION: On examination, her general physical and gynecological examination were normal. The neurological examination was abnormal for a complete T8 sensory-motor spastic paraplegia. Her bulbocavernosus and anal wink reflexes were both brisk. QUESTIONS

  1. The patient and her husband were concerned that she will no longer enjoy sexual activities creating frustration anxiety and, possibly, hostility for both of them. Is the patient unlikely to enjoy sexual activities? ( Yes or No)
A

NO Female orgasm is thought by some to be analogous physiologically to male ejaculation, requiring autonomic and somatic neural input. Physiological orgasm in females with a complete spinal injury is unlikely to occur, however, heightened arousal and climax-like sensations have been reported in spinally injured females during coitus or other forms of tactile stimulation. Proper counseling is an important component of sexual rehabilitation for females with spinal injuries. Such patients should be informed about those aspects of sexual activity which are preserved as well as these aspects which are altered. Alternatives to coitus for sexual gratification for herself and her sexual partner should be discussed.

257
Q

CASE 46: Swelling of the Hip Three Months after Spinal Cord Injury HISTORY: A seventeen-year-old male was hospitalized three months after a spinal cord injury at T3. He complained of swelling during passive exercises. He also had developed a decubitus ulcer over the right ischial tuberosity within the two weeks before admission. The patient utilized a wheelchair for ambulation. EXAMINATION: The patient was afebrile. There was an incomplete sensory level at T3 with some scattered preservation of pinprick sensation caudally. Sweating was absent below T3. There was no voluntary motor activity below T3. The legs were spastic with sustained ankle clonus, 4+ reflexes and bilateral extensor plantar signs. There was a diffuse swelling, without erythema or tenderness, about the region of the right hip with a marked decrease in range of motion compared to the left side. The patient indicated mobility of the hips had been equal a month previously. There was a 2cm decubitus ulceration overlying the right ischial tuberosity. QUESTIONS

  1. Which of the following is the most likely diagnosis to explain the hip sweling?

A. Thrombophlebitis

B. Ectopic para-articular calcification

C. Cellulitis

D. Septic arthritis

E. None of the above

A

(B) The symptoms and signs are characteristic of heterotopic paraosteoarthropathic calcification (PAO). This complication occurs in 16-49% of paraplegic patients. The other diagnoses mentioned are often confused with PAO. The absence of fever makes a septic joint and cellulitis unlikely. Thrombophlebitis is usually characterized by diffuse swelling of the lower extremity, particularly the calf, rather than localized swelling about the hip. Hussard suggested that, in the presence of a unilateral ischial tuberosity decubitus ulcer, PAO should be strongly suspected. With reduced hip movement, the ischial tuberosity sustains increased pressure during sitting, resulting in pressure necrosis of overlying skin.

258
Q

CASE 46: Swelling of the Hip Three Months after Spinal Cord Injury HISTORY: A seventeen-year-old male was hospitalized three months after a spinal cord injury at T3. He complained of swelling during passive exercises. He also had developed a decubitus ulcer over the right ischial tuberosity within the two weeks before admission. The patient utilized a wheelchair for ambulation. EXAMINATION: The patient was afebrile. There was an incomplete sensory level at T3 with some scattered preservation of pinprick sensation caudally. Sweating was absent below T3. There was no voluntary motor activity below T3. The legs were spastic with sustained ankle clonus, 4+ reflexes and bilateral extensor plantar signs. There was a diffuse swelling, without erythema or tenderness, about the region of the right hip with a marked decrease in range of motion compared to the left side. The patient indicated mobility of the hips had been equal a month previously. There was a 2cm decubitus ulceration overlying the right ischial tuberosity. QUESTIONS

  1. Which of the following laboratory studies would probably be abnormal in this patient? (Select one or more)

A. Serum calcium

B. Serum phosphorus

C. Urinary calcium

D. Serum alkaline phosphatase

E. None of the above

A

(B,C,D) Recent studies indicate that serum calcium levels are generally normal in patients with spinal cord injury. The scatter of values in a population of such patients, however, tends to be greater than in the normal population. Serum phosphorus values are usually considerably elevated during the first three months following spinal cord injury (mean 4.5 mg%). After five months, the elevation in phosphorus is less marked (mean 4.1 mg%). Urinary calcium levels are elevated to peak values three months after injury. In addition, urinary phosphorus and hydroxyproline values are also elevated. While there is some variability in the frequency and degree of elevation of serum phosphorus, and serum calcium is usually normal, there is a consistent elevation in serum alkaline phosphatase after spinal cord injury and the elevation persists for many months.

259
Q

CASE 46: Swelling of the Hip Three Months after Spinal Cord Injury HISTORY: A seventeen-year-old male was hospitalized three months after a spinal cord injury at T3. He complained of swelling during passive exercises. He also had developed a decubitus ulcer over the right ischial tuberosity within the two weeks before admission. The patient utilized a wheelchair for ambulation. EXAMINATION: The patient was afebrile. There was an incomplete sensory level at T3 with some scattered preservation of pinprick sensation caudally. Sweating was absent below T3. There was no voluntary motor activity below T3. The legs were spastic with sustained ankle clonus, 4+ reflexes and bilateral extensor plantar signs. There was a diffuse swelling, without erythema or tenderness, about the region of the right hip with a marked decrease in range of motion compared to the left side. The patient indicated mobility of the hips had been equal a month previously. There was a 2cm decubitus ulceration overlying the right ischial tuberosity. QUESTIONS

  1. Serum alkaline phosphatase in the patient described was 350 IU. (normal
A

TRUE The radioisotope bone scan may be abnormal early in the course of PAO when plain x-rays are normal. The course of PAO has been divided into four stages. In the first stage, soft tissue swelling and elevated alkaline phosphatase levels are noted but x-rays are negative. This characterizes the patient illustrated. Hsu indicated that bone scanning may be positive daring this period. In the second stage, diffuse floccular calcifications become evident on x-ray. By the third stage, swelling subsides. In the fourth stage, swelling is absent and the serum alkaline phosphatase drops to normal but dense para-articular calcification on x-ray with restricted range of motion is present.

260
Q

CASE 46: Swelling of the Hip Three Months after Spinal Cord Injury HISTORY: A seventeen-year-old male was hospitalized three months after a spinal cord injury at T3. He complained of swelling during passive exercises. He also had developed a decubitus ulcer over the right ischial tuberosity within the two weeks before admission. The patient utilized a wheelchair for ambulation. EXAMINATION: The patient was afebrile. There was an incomplete sensory level at T3 with some scattered preservation of pinprick sensation caudally. Sweating was absent below T3. There was no voluntary motor activity below T3. The legs were spastic with sustained ankle clonus, 4+ reflexes and bilateral extensor plantar signs. There was a diffuse swelling, without erythema or tenderness, about the region of the right hip with a marked decrease in range of motion compared to the left side. The patient indicated mobility of the hips had been equal a month previously. There was a 2cm decubitus ulceration overlying the right ischial tuberosity. QUESTIONS

  1. Parathyroid hormone levels are elevated in patients with spinal cord injury. (True or False)
A

FALSE tidak ada pembahasan

261
Q

CASE 46: Swelling of the Hip Three Months after Spinal Cord Injury HISTORY: A seventeen-year-old male was hospitalized three months after a spinal cord injury at T3. He complained of swelling during passive exercises. He also had developed a decubitus ulcer over the right ischial tuberosity within the two weeks before admission. The patient utilized a wheelchair for ambulation. EXAMINATION: The patient was afebrile. There was an incomplete sensory level at T3 with some scattered preservation of pinprick sensation caudally. Sweating was absent below T3. There was no voluntary motor activity below T3. The legs were spastic with sustained ankle clonus, 4+ reflexes and bilateral extensor plantar signs. There was a diffuse swelling, without erythema or tenderness, about the region of the right hip with a marked decrease in range of motion compared to the left side. The patient indicated mobility of the hips had been equal a month previously. There was a 2cm decubitus ulceration overlying the right ischial tuberosity. QUESTIONS

  1. Thyroid hormone levels are usually normal in patients with spinal cord injury. (True or False)
A

TRUE tidak ada pembahasan

262
Q

CASE 46: Swelling of the Hip Three Months after Spinal Cord Injury HISTORY: A seventeen-year-old male was hospitalized three months after a spinal cord injury at T3. He complained of swelling during passive exercises. He also had developed a decubitus ulcer over the right ischial tuberosity within the two weeks before admission. The patient utilized a wheelchair for ambulation. EXAMINATION: The patient was afebrile. There was an incomplete sensory level at T3 with some scattered preservation of pinprick sensation caudally. Sweating was absent below T3. There was no voluntary motor activity below T3. The legs were spastic with sustained ankle clonus, 4+ reflexes and bilateral extensor plantar signs. There was a diffuse swelling, without erythema or tenderness, about the region of the right hip with a marked decrease in range of motion compared to the left side. The patient indicated mobility of the hips had been equal a month previously. There was a 2cm decubitus ulceration overlying the right ischial tuberosity. QUESTIONS

  1. Corticosteroid metabolism is usually normal in patients with spinal cord injury. (True or False)
A

FALSE Bergman investigated endocrine function in patients with spinal cord injury. He found both parathyroid hormone and thyroid hormone levels to be normal in almost all patients studied. Growth hormone was also normal. Burnhart indicated that normal parathyroid and thyroid function were both necessary for the occurrence of diffuse osteoporosis. In the past, elevated, normal, and reduced levels of parathyroid hormone had been reported. Corticosteroid metabolism appeared to be abnormal in nearly all patients with spinal cord injury studied by Bergman, although the abnormalities were not consistent. Most patients manifested decreased urinary corticosteroid excretion with elevated levels of ACTH.

263
Q

CASE 46: Swelling of the Hip Three Months after Spinal Cord Injury HISTORY: A seventeen-year-old male was hospitalized three months after a spinal cord injury at T3. He complained of swelling during passive exercises. He also had developed a decubitus ulcer over the right ischial tuberosity within the two weeks before admission. The patient utilized a wheelchair for ambulation. EXAMINATION: The patient was afebrile. There was an incomplete sensory level at T3 with some scattered preservation of pinprick sensation caudally. Sweating was absent below T3. There was no voluntary motor activity below T3. The legs were spastic with sustained ankle clonus, 4+ reflexes and bilateral extensor plantar signs. There was a diffuse swelling, without erythema or tenderness, about the region of the right hip with a marked decrease in range of motion compared to the left side. The patient indicated mobility of the hips had been equal a month previously. There was a 2cm decubitus ulceration overlying the right ischial tuberosity. QUESTIONS

  1. Which of the following are true concerning bone turnover in patients with spinal cord injury? (Select one or more)

A. Increased primarily in bone of nonparalyzed areas in first two months after injury

B. Increased primarily in bone of paralyzed areas after two months post injury

C. Never increased significantly in nonparalyzed areas

D. Increased primarily in paralyzed areas at all times following injury

A

(A,B) Turnover of bone or bone accretion is the amount of calcium trapped on a daily basis by the skeleton by bone apposition, secondary mineralization or slow exchange. Bone accretion is a measure of bone remodelling. Ca is utilized to determine bone accretion. Utilizing this method, Bergman confirmed earlier reports that total bone accretion is elevated in the spinal cord injured patient. During the first two months after injury, the turnover is greatest in bone in the nonparalyzed portion of the body. At about three months, a reversal occurs and the remodelling is more active in bone in tile paralyzed portion of the body. This is not meant to indicate that bone remodelling in paralyzed areas does not occur in the first few months after injury; it does. In fact, enlargement of long bone shafts such as the femur have been noted in the first two months after injury. The bone tends to be poorly calcified, which is generally taken as evidence that it is new bone.

264
Q

CASE 46: Swelling of the Hip Three Months after Spinal Cord Injury HISTORY: A seventeen-year-old male was hospitalized three months after a spinal cord injury at T3. He complained of swelling during passive exercises. He also had developed a decubitus ulcer over the right ischial tuberosity within the two weeks before admission. The patient utilized a wheelchair for ambulation. EXAMINATION: The patient was afebrile. There was an incomplete sensory level at T3 with some scattered preservation of pinprick sensation caudally. Sweating was absent below T3. There was no voluntary motor activity below T3. The legs were spastic with sustained ankle clonus, 4+ reflexes and bilateral extensor plantar signs. There was a diffuse swelling, without erythema or tenderness, about the region of the right hip with a marked decrease in range of motion compared to the left side. The patient indicated mobility of the hips had been equal a month previously. There was a 2cm decubitus ulceration overlying the right ischial tuberosity. QUESTIONS

  1. Which of the following result in an increased risk of ectopic paraosteoarthropathic calcification (POA) ? (Select one or more)

A. Immobilization

B. Trauma

C. Circulatory changes

D. Elevated phosphorus level

E. None of the above

A

(A,B,C,D) Multiple factors are related to the occurrence of bone resorption, remodelling and the occurrence of PAO. Immobilization in paralyzed limbs, although a significant factor, does not appear to be exclusively responsible for the abnormalities of bone metabolism seen after spinal cord injury. Immobilization from any cause results in diffuse osteoporosis due to loss of bone mineral. Comparing changes in bone remodelling in spinal cord injury and other immobile states due to neurological problems (e.g. poliomyelitis and stroke) leads to the conclusion that the physiological changes in bone remodelling may be different in the two processes. In poliomyelitis and immobilized normal subjects, Bergman and others indicate that hydroxyprolinuria and calciuria are maximal at two and four weeks respectively, while in spinal cord injury these parameter speak at two to three months. In hemiplegia, bone demineralization is less obvious, although initially immobilization occurs in this disability. In polio, it has been observed that initial calciuria is independent of extent and degree of paralysis as well as the time of walking and appearance of voluntary muscle function. It has been suggested that circulatory changes secondary to spinal cord injury, particularly the effect of sympathectomy, may contribute to hone remodelling in paralyzed limbs as well as to PAO. Bergman demonstrated significant arteriovenous shunting in the legs of spinal cord injured patients as well as an abnormal distribution of the functional capillary bed. Changes in oxygen and carbon dioxide in the proximal circulation of paralyzed limbs has also been suggested as an important factor in the pathogenesis of PAO. The role of trauma in PAO has also been vigorously debated. Some physiotherapists feel that excessively brisk passive exercises may lead to muscle injury, local hemorrhage and increase the incidence of PAO, but others do not agree. It is well known that chronic trauma to muscles in paraplegics may lead to local ossification. Guttmann indicated that with careful avoidance of trauma, prevention of infection and utilization of passive exercises, PAO may be reduced to an incidence of 5% rather than the 15-50% usually reported. The relationship in pathology between PAO and myositis ossificans and between PAO and the calcification which may occur in normal muscles due to repeated trauma is unknown. It seems reasonable that trauma plays a role in PAO but the significance of that role is unknown. Elevated, phosphorus levels, as seen in many spinal cord injured patients, increases the risk of PAO. As discussed in question, serum phosphorus is commonly, but not always, elevated after spinal cord injury and has its peak at about two to three months after injury which is also the most common time for onset of symptoms of PAO although the full development of the latter may take several weeks or more.

265
Q

CASE 46: Swelling of the Hip Three Months after Spinal Cord Injury HISTORY: A seventeen-year-old male was hospitalized three months after a spinal cord injury at T3. He complained of swelling during passive exercises. He also had developed a decubitus ulcer over the right ischial tuberosity within the two weeks before admission. The patient utilized a wheelchair for ambulation. EXAMINATION: The patient was afebrile. There was an incomplete sensory level at T3 with some scattered preservation of pinprick sensation caudally. Sweating was absent below T3. There was no voluntary motor activity below T3. The legs were spastic with sustained ankle clonus, 4+ reflexes and bilateral extensor plantar signs. There was a diffuse swelling, without erythema or tenderness, about the region of the right hip with a marked decrease in range of motion compared to the left side. The patient indicated mobility of the hips had been equal a month previously. There was a 2cm decubitus ulceration overlying the right ischial tuberosity. QUESTIONS

  1. Which of the following treatments would you recommend for the patient illustrated? (Select one or more)

A. Passive exercises

B. Immobilization

C. Wedge resection of the hip

D. Dietary alteration

E. None of the above

A

(E) Most authors feel that there is little in the way of effective treatment to reverse or stabilize PAO once it has begun. The value of passive exercises is debated. Following surgical procedures, passive exercises may help maintain mobility gained as a result of osteotomy or bone resection. Immobilization is of no value since it aggravates the demineralization of the skeleton locally and generally. Surgical procedures such as wedge resection or osteotomy and femoral neck resection are of value in patients with ankylosis of the hip or other joints due to PAO. It is generally agreed, however, that such procedures should not be carried out when active bone formation is occurring. The elevated alkaline phosphatase seen in this patient is accepted as evidence of active bone formation and is considered a contraindication to surgery. The incidence of complications such as hemorrhage, infection and reankylosis are increased if surgery is performed during the active phase of the disease. In addition, in the patient described, joint ankylosis has not occurred. The presence of the early stages of PAO does not uniformly indicate that ankylosis will occur, although the factors responsible for arrest or progression of PAO to the point of ankylosis are poorly understood. Dietary changes have not been recommended as effective in prevention of PAO.

266
Q

CASE 47: Cortical Evoked Potentials (CEP) in Spinal Cord Injury HISTORY: A 17-year-old female sustained a C7 compression fracture. She was a motorcycle passenger wearing a helmet and struck her head at the time of the collision. She was seen within an hour of injury at an emergency room complaining of tingling in her legs and neck pain. EXAMINATION: Vital signs were stable. She was fully awake and her cranial nerve examination was normal. The motor and sensory examination of her upper extremities was normal. The strength and tone in her lower extremities was normal but deep tendon reflexes were slightly diminished in the lower extremities. She noted some pinprick examination of the lower extremities. Joint position sensation was intact. Anal tone was normal. X-RAY: Cervical spine x-rays revealed a compression fracture of C7 without angulation or subluxation, as well as soft tissue swelling anterior to the body of C7. HOSPITAL COURSE: Within two hours, the neurological status in her lower extremities began to deteriorate. Strength diminished and she was was noted at T4 bilaterally and joint position sense was poor. A myelogram showed a complete extradural block at C7. Plans were made for immediate operative decompression of the injured cervical level QUESTIONS

  1. Cortical evoked potentials (CEP) would be a useful adjunct to surgery in this circumstance. (True or False)
A

TRUE Cortical evoked potentials (CEP) have been recommended for use in monitoring spinal cord function in the anesthetized patient. Mechanical deformation of the spinal cord is thought to inhibit CEP and hazardous operative manipulations may, therefore, be avoided by monitoring CEP.

267
Q

CASE 47: Cortical Evoked Potentials (CEP) in Spinal Cord Injury HISTORY: A 17-year-old female sustained a C7 compression fracture. She was a motorcycle passenger wearing a helmet and struck her head at the time of the collision. She was seen within an hour of injury at an emergency room complaining of tingling in her legs and neck pain. EXAMINATION: Vital signs were stable. She was fully awake and her cranial nerve examination was normal. The motor and sensory examination of her upper extremities was normal. The strength and tone in her lower extremities was normal but deep tendon reflexes were slightly diminished in the lower extremities. She noted some pinprick examination of the lower extremities. Joint position sensation was intact. Anal tone was normal. X-RAY: Cervical spine x-rays revealed a compression fracture of C7 without angulation or subluxation, as well as soft tissue swelling anterior to the body of C7. HOSPITAL COURSE: Within two hours, the neurological status in her lower extremities began to deteriorate. Strength diminished and she was was noted at T4 bilaterally and joint position sense was poor. A myelogram showed a complete extradural block at C7. Plans were made for immediate operative decompression of the injured cervical level QUESTIONS

  1. Monitoring of CEP in patients Involves (Select one or more)

A. percutaneous stimulation of peripheral nerves

B. scalp recording and amplification of cortical activity

C. assessment of alterations of normal EEC activity in response to peripheral nerve stimulation

D. measurement of latency and analysis of waveforms of individual cortical responses to peripheral nerve stimulation

E. none of the above

A

(A,B) CEP monitoring technique is noninvasive. Peripheral nerves are repetitively stimulated pereutaneousiy. Cortical responses to stimuli are recorded with scalp electrodes and amplified. Because normal EEC activity is higher voltage than CEP, the random EEG activity is separated out by the method of signal averaging. CEP latency and waveform analysis is, therefore, performed on a summated average of multiple individual evoked potentials.

268
Q

CASE 47: Cortical Evoked Potentials (CEP) in Spinal Cord Injury HISTORY: A 17-year-old female sustained a C7 compression fracture. She was a motorcycle passenger wearing a helmet and struck her head at the time of the collision. She was seen within an hour of injury at an emergency room complaining of tingling in her legs and neck pain. EXAMINATION: Vital signs were stable. She was fully awake and her cranial nerve examination was normal. The motor and sensory examination of her upper extremities was normal. The strength and tone in her lower extremities was normal but deep tendon reflexes were slightly diminished in the lower extremities. She noted some pinprick examination of the lower extremities. Joint position sensation was intact. Anal tone was normal. X-RAY: Cervical spine x-rays revealed a compression fracture of C7 without angulation or subluxation, as well as soft tissue swelling anterior to the body of C7. HOSPITAL COURSE: Within two hours, the neurological status in her lower extremities began to deteriorate. Strength diminished and she was was noted at T4 bilaterally and joint position sense was poor. A myelogram showed a complete extradural block at C7. Plans were made for immediate operative decompression of the injured cervical level QUESTIONS

  1. In an incomplete cervical spinal lesion such as this patient exhibits, tibial or peroneal nerve stimulation during surgery would be CEP with (Select one or more)

A. unilateral nerve stimulation

B. bilateral nerve stimulation

C. increasing the voltage of stimulation

D. momentarily stopping anesthetic agents

E. none of the above

A

(E) In an incomplete cervical spinal cord lesion, CEP is recorded in 55 % to 85% of patients. In Perot’s group of patients the presence of CEP correlated inversely with the severity of neurological deficit. In normal individuals, CEP in response to peroneal nerve stimulation is enhanced by bilateral stimulation. However, improvement in reliability with bilateral stimulation was not demonstrated in patients with incomplete injuries. There is no relationship between stimulus intensity and the amplitude or presence of CEP. Likewise, anesthesia seems to have little effect on recording of CEP.

269
Q

CASE 47: Cortical Evoked Potentials (CEP) in Spinal Cord Injury HISTORY: A 17-year-old female sustained a C7 compression fracture. She was a motorcycle passenger wearing a helmet and struck her head at the time of the collision. She was seen within an hour of injury at an emergency room complaining of tingling in her legs and neck pain. EXAMINATION: Vital signs were stable. She was fully awake and her cranial nerve examination was normal. The motor and sensory examination of her upper extremities was normal. The strength and tone in her lower extremities was normal but deep tendon reflexes were slightly diminished in the lower extremities. She noted some pinprick examination of the lower extremities. Joint position sensation was intact. Anal tone was normal. X-RAY: Cervical spine x-rays revealed a compression fracture of C7 without angulation or subluxation, as well as soft tissue swelling anterior to the body of C7. HOSPITAL COURSE: Within two hours, the neurological status in her lower extremities began to deteriorate. Strength diminished and she was was noted at T4 bilaterally and joint position sense was poor. A myelogram showed a complete extradural block at C7. Plans were made for immediate operative decompression of the injured cervical level QUESTIONS

  1. CEP is dependent on intact dorsal columns. (True or false)
A

TRUE CEP is generally thought to he transmitted almost exclusively by the posterior columns. This finding has been demonstrated in animals and confirmed in humans.

270
Q

CASE 47: Cortical Evoked Potentials (CEP) in Spinal Cord Injury HISTORY: A 17-year-old female sustained a C7 compression fracture. She was a motorcycle passenger wearing a helmet and struck her head at the time of the collision. She was seen within an hour of injury at an emergency room complaining of tingling in her legs and neck pain. EXAMINATION: Vital signs were stable. She was fully awake and her cranial nerve examination was normal. The motor and sensory examination of her upper extremities was normal. The strength and tone in her lower extremities was normal but deep tendon reflexes were slightly diminished in the lower extremities. She noted some pinprick examination of the lower extremities. Joint position sensation was intact. Anal tone was normal. X-RAY: Cervical spine x-rays revealed a compression fracture of C7 without angulation or subluxation, as well as soft tissue swelling anterior to the body of C7. HOSPITAL COURSE: Within two hours, the neurological status in her lower extremities began to deteriorate. Strength diminished and she was was noted at T4 bilaterally and joint position sense was poor. A myelogram showed a complete extradural block at C7. Plans were made for immediate operative decompression of the injured cervical level QUESTIONS

  1. Waveform, but not latency, of CEP is altered with spinal cord lesions. (True or False)
A

FALSE Lesions of the spinal cord may cause changes in the amplitude, latency and waveform of CEP.

271
Q

CASE 47: Cortical Evoked Potentials (CEP) in Spinal Cord Injury HISTORY: A 17-year-old female sustained a C7 compression fracture. She was a motorcycle passenger wearing a helmet and struck her head at the time of the collision. She was seen within an hour of injury at an emergency room complaining of tingling in her legs and neck pain. EXAMINATION: Vital signs were stable. She was fully awake and her cranial nerve examination was normal. The motor and sensory examination of her upper extremities was normal. The strength and tone in her lower extremities was normal but deep tendon reflexes were slightly diminished in the lower extremities. She noted some pinprick examination of the lower extremities. Joint position sensation was intact. Anal tone was normal. X-RAY: Cervical spine x-rays revealed a compression fracture of C7 without angulation or subluxation, as well as soft tissue swelling anterior to the body of C7. HOSPITAL COURSE: Within two hours, the neurological status in her lower extremities began to deteriorate. Strength diminished and she was was noted at T4 bilaterally and joint position sense was poor. A myelogram showed a complete extradural block at C7. Plans were made for immediate operative decompression of the injured cervical level QUESTIONS

  1. CEP is not generated from peripheral nerve stimulation below the level of a clinically complete spinal lesion. (True or False)
A

TRUE No patient with a clinically complete cord lesion at any level has CEP to stimulation of peripheral nerves below the level of the lesion.

272
Q

CASE 47: Cortical Evoked Potentials (CEP) in Spinal Cord Injury HISTORY: A 17-year-old female sustained a C7 compression fracture. She was a motorcycle passenger wearing a helmet and struck her head at the time of the collision. She was seen within an hour of injury at an emergency room complaining of tingling in her legs and neck pain. EXAMINATION: Vital signs were stable. She was fully awake and her cranial nerve examination was normal. The motor and sensory examination of her upper extremities was normal. The strength and tone in her lower extremities was normal but deep tendon reflexes were slightly diminished in the lower extremities. She noted some pinprick examination of the lower extremities. Joint position sensation was intact. Anal tone was normal. X-RAY: Cervical spine x-rays revealed a compression fracture of C7 without angulation or subluxation, as well as soft tissue swelling anterior to the body of C7. HOSPITAL COURSE: Within two hours, the neurological status in her lower extremities began to deteriorate. Strength diminished and she was was noted at T4 bilaterally and joint position sense was poor. A myelogram showed a complete extradural block at C7. Plans were made for immediate operative decompression of the injured cervical level QUESTIONS

  1. Normalization of CEP may precede clinical recovery and provide prognostic information useful in the case of some spinally injured patients. (True or False)
A

TRUE Patients in whom CEP persists immediately following spinal injury or in whom return or normalization of CEP occurs within the first week after injury generally experience substantial clinical recovery. A similar finding was demonstrated in laboratory animals.

273
Q

CASE 48: Hostility and Aggression Following a Spinal Cord Injury HISTORY: A 27-year-old male was transferred to a spinal cord injury rehabilitation center three months after injury. He suffered not undertaken and three months after the injury the fracture appeared to be in a stable position and the patient remained paraplegic with a T12 sensory level bilaterally. At the time of transfer for rehabilitation the patient, a high school graduate who had been fully employed as a skilled laborer was beginning to learn to use a wheelchair but was unable to self-transfer from bed. He had been on a program of intermittent urinary were no pressure ulcerations or other evidence of infection, HOSPITAL COURSE. Over the first several weeks after transfer, patient was abusive and demanding. He frequently and inappropriately accused the staff attention. On several occasions, he angrily threatened to institute legal proceeding QUESTIONS

  1. Psychiatric assistance for this patient at this time would be

A. indicated

B. best delayed, as this hostility will probably pass spontaneously

C. contraindicated, because it will increase the patient’s distrust of the staff

D. contraindicated, because it will delay other more pressing

E. none of the above

A

(A) Patients who exhibit aggressive behavior during rehabilitation from injury form a large portion of those patients who do poorly with overall rehabilitative efforts. Management of this problem is difficult, as aggressive behavior probably represents the effect; rather than the cause, of a number of factors which coalesce to reduce the patientfs self-esteem. Because aggressive behavior can be self-regenerating, and is detrimental to the rehabilitative progress, psychiatric assistance is indicated for this patient. Further hostility generated by psychiatric intervention may be a necessary temporary setback, balanced by the anticipated long term benefits.

274
Q

CASE 48: Hostility and Aggression Following a Spinal Cord Injury HISTORY: A 27-year-old male was transferred to a spinal cord injury rehabilitation center three months after injury. He suffered not undertaken and three months after the injury the fracture appeared to be in a stable position and the patient remained paraplegic with a T12 sensory level bilaterally. At the time of transfer for rehabilitation the patient, a high school graduate who had been fully employed as a skilled laborer was beginning to learn to use a wheelchair but was unable to self-transfer from bed. He had been on a program of intermittent urinary were no pressure ulcerations or other evidence of infection, HOSPITAL COURSE. Over the first several weeks after transfer, patient was abusive and demanding. He frequently and inappropriately accused the staff attention. On several occasions, he angrily threatened to institute legal proceeding QUESTIONS

  1. Most patients with spinal cord injury experience a prolonged phase of hostile and aggressive behaviour (True or False)
A

FALSE Anger is a common manifestation of injury, generated from the accompanying sense of forced increased dependency. However, anger has a variety of ways of manifesting itself, including anxiety and depression in addition to hostility and aggression. Prolonged periods of hostility and aggression are not common to all patients with spinal injury.

275
Q

CASE 48: Hostility and Aggression Following a Spinal Cord Injury HISTORY: A 27-year-old male was transferred to a spinal cord injury rehabilitation center three months after injury. He suffered not undertaken and three months after the injury the fracture appeared to be in a stable position and the patient remained paraplegic with a T12 sensory level bilaterally. At the time of transfer for rehabilitation the patient, a high school graduate who had been fully employed as a skilled laborer was beginning to learn to use a wheelchair but was unable to self-transfer from bed. He had been on a program of intermittent urinary were no pressure ulcerations or other evidence of infection, HOSPITAL COURSE. Over the first several weeks after transfer, patient was abusive and demanding. He frequently and inappropriately accused the staff attention. On several occasions, he angrily threatened to institute legal proceeding QUESTIONS

  1. After spinal injury, most patients experience an initial period of denial. (True or False)
A

TRUE As a protective and adaptive measure, an initial period of denial (a process of blocking awareness of unpleasant external realities) is a vital component in the psychological adjustment to spinal cord injury. Prolonged denial may, however, be detrimental to the rehabilitative process, but fortunately, most patients eventually accept the nature of their injury.

276
Q

CASE 48: Hostility and Aggression Following a Spinal Cord Injury HISTORY: A 27-year-old male was transferred to a spinal cord injury rehabilitation center three months after injury. He suffered not undertaken and three months after the injury the fracture appeared to be in a stable position and the patient remained paraplegic with a T12 sensory level bilaterally. At the time of transfer for rehabilitation the patient, a high school graduate who had been fully employed as a skilled laborer was beginning to learn to use a wheelchair but was unable to self-transfer from bed. He had been on a program of intermittent urinary were no pressure ulcerations or other evidence of infection, HOSPITAL COURSE. Over the first several weeks after transfer, patient was abusive and demanding. He frequently and inappropriately accused the staff attention. On several occasions, he angrily threatened to institute legal proceeding QUESTIONS

  1. At some time after spinal injury, most patients experience a period of depression. (True or False)
A

TRUE Self-esteem is based on a sense of strength, loving and lovability. Spinal cord injury disrupts each of these facets of the foundation for self-esteem. With realization of the nature and impact of spinal deficit, the patient becomes depressed as he sees his self-esteem undermined. Occasionally, patients become suicidal and require psychiatric care, but usually the stage of depression, like denial, passes with time.

277
Q

CASE 48: Hostility and Aggression Following a Spinal Cord Injury HISTORY: A 27-year-old male was transferred to a spinal cord injury rehabilitation center three months after injury. He suffered not undertaken and three months after the injury the fracture appeared to be in a stable position and the patient remained paraplegic with a T12 sensory level bilaterally. At the time of transfer for rehabilitation the patient, a high school graduate who had been fully employed as a skilled laborer was beginning to learn to use a wheelchair but was unable to self-transfer from bed. He had been on a program of intermittent urinary were no pressure ulcerations or other evidence of infection, HOSPITAL COURSE. Over the first several weeks after transfer, patient was abusive and demanding. He frequently and inappropriately accused the staff attention. On several occasions, he angrily threatened to institute legal proceeding QUESTIONS

  1. With active occupational and physical therapy, most patients with spinal cord injury are psychologically prepared to return to fall occupational activities in a matter of months. (True or False)
A

FALSE With spinal cord injury, the damage to self-esteem must be repaired by building a modified identity. As the stage of depression lifts, the patient enters a phase of restitution of self-esteem by learning, within the context of his disability, to cope with increasingly more difficult and complex day-to-day problems. This process may take many months and usually years. It appears that the return to full employment is, at least in part linked to this period of restitution of self-esteem.

278
Q

CASE 49: Regenerative Capacity of a Transected Spinal Cord HISTORY: A 26-year-old male suffered a C5-6 fracture-dislocation as a result of an automobile immediately quadriplegic. At the time of operative spinal fusion, and the surgeon reported that the cord was anatomically and completely transected at the level of the injury. The surgeon was able to reapproximate the proximal and distal spinal stump with a few sutures. Postoperatively, the fracture healed in a stable position and there were no complications. Over the next twelve months, the patient remained quadriplegic; however, he regained some voluntary control of the biceps and triceps muscles. QUESTIONS

  1. In this patient, recovery of the triceps and biceps muscles probably represents evidence of spinal cord regeneration. (True or False)
A

FALSE With a C5-6 fracture-dislocation, the spinal cord injury may be as low as the C6-7 neural level with some sparing of C6 neural elements. The biceps muscle is innervated primarily by the C6 level and the triceps may also have C6 innervation, with individual patient variation. Recovery of volitional use of these muscles does not, therefore, represent evidence of spinal regeneration but, instead, probably represents reduction of edema of the spinal cord which may occur acutely a few segments above the level of the injury.

279
Q

CASE 49: Regenerative Capacity of a Transected Spinal Cord HISTORY: A 26-year-old male suffered a C5-6 fracture-dislocation as a result of an automobile immediately quadriplegic. At the time of operative spinal fusion, and the surgeon reported that the cord was anatomically and completely transected at the level of the injury. The surgeon was able to reapproximate the proximal and distal spinal stump with a few sutures. Postoperatively, the fracture healed in a stable position and there were no complications. Over the next twelve months, the patient remained quadriplegic; however, he regained some voluntary control of the biceps and triceps muscles. QUESTIONS

  1. Suture repair of the transected spinal elements has been shown, albeit rarely, to induce functional spinal regeneration. (True or False)
A

FALSE Although there have been some isolated and skeptically received reports of functional return of activity with spinal cord suture, it is generally considered at the present time that functional restitution of the spinal cord cannot be achieved with suture repair.

280
Q

CASE 49: Regenerative Capacity of a Transected Spinal Cord HISTORY: A 26-year-old male suffered a C5-6 fracture-dislocation as a result of an automobile immediately quadriplegic. At the time of operative spinal fusion, and the surgeon reported that the cord was anatomically and completely transected at the level of the injury. The surgeon was able to reapproximate the proximal and distal spinal stump with a few sutures. Postoperatively, the fracture healed in a stable position and there were no complications. Over the next twelve months, the patient remained quadriplegic; however, he regained some voluntary control of the biceps and triceps muscles. QUESTIONS

  1. Operative exploration of most persons with complete functional spinal cord injury would as with this patient reveal a completely severed cord (True or False)
A

FALSE It is estimated that only 15% of paraplegics have anatomically severed spinal cords.

281
Q

CASE 49: Regenerative Capacity of a Transected Spinal Cord HISTORY: A 26-year-old male suffered a C5-6 fracture-dislocation as a result of an automobile immediately quadriplegic. At the time of operative spinal fusion, and the surgeon reported that the cord was anatomically and completely transected at the level of the injury. The surgeon was able to reapproximate the proximal and distal spinal stump with a few sutures. Postoperatively, the fracture healed in a stable position and there were no complications. Over the next twelve months, the patient remained quadriplegic; however, he regained some voluntary control of the biceps and triceps muscles. QUESTIONS

  1. The adult mammalian central nervous system is capable of regeneration of neural elements. (True or False)
A

TRUE The factors that determine whether neuronal regeneration will occur include preservation of a viable neuron, axonal capacity for sprouting, transmission of a signal to the cell body for increased protein synthesis, and delivery of this protein to the growing axon tip. Although there is disagreement, neurons of the adult mammalian central nervous system are thought to be capable of regenerating their severed processes. This regenerative capacity is, in part, age dependent with greater plasticity in the immature nervous system. However, there is also evidence of plasticity in the adult nervous system. Indeed, histological evidence of abortive spinal neuronal regeneration has been demonstrated in man. Several factors are thought to be responsible for abortive regenerative efforts in the mammalian central nervous system and they include: 1) scar formation at the site of injury; 2) inhibition of neuronal growth by the random establishment of synaptic connections at the site of injury; and 3) the formation of microcysts within the cord adjacent to the injured segment.

282
Q

CASE 49: Regenerative Capacity of a Transected Spinal Cord HISTORY: A 26-year-old male suffered a C5-6 fracture-dislocation as a result of an automobile immediately quadriplegic. At the time of operative spinal fusion, and the surgeon reported that the cord was anatomically and completely transected at the level of the injury. The surgeon was able to reapproximate the proximal and distal spinal stump with a few sutures. Postoperatively, the fracture healed in a stable position and there were no complications. Over the next twelve months, the patient remained quadriplegic; however, he regained some voluntary control of the biceps and triceps muscles. QUESTIONS 5. Spinal regeneration following trauma, in the future, may be potentiated by the use of (Select one or more)

A. nerve growth factor

B. surgical reconstruction

C. pyrogenic agents

D. proteolytic enzymes

E. none of the .above

A

(A,B,C,D) In general, efforts to enhance the regenerative process have been directed at regulating and stimulating protein synthesis in damaged nerve cells and toward aiding axon sprouts into and through the region of the spinal cord injury where scar tissue forms. Nerve growth factor (NGF) stimulates embryonic neuronal growth in spinal and sympathetic ganglia. NGF stimulates the synthesis of protein in these ganglia and also has possible beneficial effects on regeneration of intraspinal neurons. Although simple resuturing of severed ends of spinal cord has been notably unsuccessful, other surgical reconstructive techniques using peripheral nerve and/or neuronal “grafts” have been productive of some limited, laboratory success. Pyrogenic agents (primarily piromen, a bacterial extract) have been used to facilitate spinal regeneration by retarding the development of scar tissue in the area of spinal cord transection. Likewise, the supposition that proteolytic enzymes, known to alter connective tissue metabolism, might deter scar formation has prompted the use of hyaluronidase, trypsin and elastase in the treatment of spinal injuries in the laboratory and in patients. The results have been mixed.

283
Q

CASE 50: 42-Year-Old Woman with C7 Spinal Cord Injury HISTORY: A 42-year-oid woman was seen one month after a spinal cord injury. The patient s family had asked for consultation concerning possible rehabilitation. EXAMINATION: The patient was in cervical traction with Gardner-Wells tongs revealed quadriplegia below C7. The C7 segment and above were preserved. There was total loss of sensation below C7. Rectal and bladder sphincters were paralyzed. X-RAYS: X-rays of the cervical spine revealed a fracture-dislocation of C7 which was in good alignment as a result of traction. QUESTIONS. 1. Which of the following rehabilitative measures would you recommend for this patient? (Select one or more)

A. Physical therapy at least three times a day

B. Vigorous passive hip flexion exercises

C. Placement of pillows beneath knees when supine

D. Turning the spinal frame every two hours

E. Active exercises for nonparalyzed muscles

A

(A,D,E) Physical therapy is an essential part of both early and late rehabilitative care of spinal cord injured patients. In order for physical therapy to be effective, it should be carried out at least three times per day and preferably more often if personnel are available. Passive exercises are an essential part of physical therapy of the spinal cord injured patient; however, these should be carried out in a gentle and slow manner to produce gradual stretching of the muscle and to avoid muscle trauma. Vigorous passive hip flexion and extension exercises should be avoided as they may increase the frequency of paraarticular ectopic ossification. The frequency of such ossification is increased by muscle trauma and resulting hemorrhage which may result from excessively vigorous hip flexion and extension exercises. Pillows beneath the knees in the supine position should be avoided since they result in chronic shortening of the hamstring and hip flexor muscles and may increase the frequency of joint contractures. In addition, they promote the occurrence of spasticity in flexion at the knees and hips. While the spinal frame does not offer as many advantages as the Stoke-Mandeville Edgerton turning bed or the Keane Roto-Rest system, it is a commonly used method in the United States. The spinal frame should be turned every two hours at a minimum for patients with spinal cord injury, particularly those with complete paralysis and sensory loss. Active exercise for nonparalyzed muscles should be encouraged. Of particular importance are the accessory muscles of respiration including the strap muscles of the neck and, the sternocleidomastoid and trapezius muscles which are innervated by spinal cord segments above the level of the spinal cord injury. Since the spinal cord injured patient will depend in large part upon strength in nonparalyzed muscles for his activities of daily living, these muscles must be strengthened through a vigorous program of active exercise. Because of the paralysis of other muscles, the stress on unparalyzed muscles will be greater than in a normal individual and a strengthening program is mandatory so that they may be maximally effective.

284
Q

CASE 50: 42-Year-Old Woman with C7 Spinal Cord Injury HISTORY: A 42-year-oid woman was seen one month after a spinal cord injury. The patient s family had asked for consultation concerning possible rehabilitation. EXAMINATION: The patient was in cervical traction with Gardner-Wells tongs revealed quadriplegia below C7. The C7 segment and above were preserved. There was total loss of sensation below C7. Rectal and bladder sphincters were paralyzed. X-RAYS: X-rays of the cervical spine revealed a fracture-dislocation of C7 which was in good alignment as a result of traction. QUESTIONS. 2. Which of the following are reasonable goals of physical therapy? (Select one or more)

A.. Improve respiratory function

B. Restore vasomotor control

C. Strengthen normal muscles

D. Avoid contractures

A

(A,B,C,D). Improvements in respiratory function and vasomotor control are important goals of physical therapy. These are realized through strengthening of the,accessory muscles of respiration and by placing the patient gradually into the upright position. The avoidance of contractures through passive exercise and the strengthening of normal muscles through active exercises are also essential goals of physical therapy.

285
Q

CASE 50: 42-Year-Old Woman with C7 Spinal Cord Injury HISTORY: A 42-year-oid woman was seen one month after a spinal cord injury. The patient s family had asked for consultation concerning possible rehabilitation. EXAMINATION: The patient was in cervical traction with Gardner-Wells tongs revealed quadriplegia below C7. The C7 segment and above were preserved. There was total loss of sensation below C7. Rectal and bladder sphincters were paralyzed. X-RAYS: X-rays of the cervical spine revealed a fracture-dislocation of C7 which was in good alignment as a result of traction. QUESTIONS. 3. Which of the following would you recommend to improve hand function in this patient? (Select one or more)

A. Fusion of thumb in position of opposition

B. Extensor carpi radialis longus to flexor digitorum profundus tendon transfer

C. Brachioradialis to flexor pollicus longus tendon transfer

D. None of the above

A

(B,C) In patients with spinal cord injury below C7, the flexor carpi radialis muscle, innervated by C7, remains functional. Lamb suggested potentiating the natural tenodesing effect of the long flexors when the wrist is dorsiflexed by transferring the extensor carpi radialis longus tendon through the interosseous membrane to the flexor digitorum profundus. The brachioradialis is then transferred to the flexor pollicis longus. This leaves the extensor carpi radialis brevis to dorsiflex the wrist. Lamb reported significant improvement in hand function with this procedure in eleven of thirteen patients. Fusion of the thumb is not beneficial in this situation since with continued function of the flexor carpi radialis, this muscle is used as the motor force for the abduction-opposition movement of the thumb by utilizing a free tendon graft from the palmaris longus muscle. In lesions in which the C7 segment is also involved, permanent stabilization of the thumb in opposition by way of surgical bone graft and fusion may be helpful.

286
Q

CASE 50: 42-Year-Old Woman with C7 Spinal Cord Injury HISTORY: A 42-year-oid woman was seen one month after a spinal cord injury. The patient s family had asked for consultation concerning possible rehabilitation. EXAMINATION: The patient was in cervical traction with Gardner-Wells tongs revealed quadriplegia below C7. The C7 segment and above were preserved. There was total loss of sensation below C7. Rectal and bladder sphincters were paralyzed. X-RAYS: X-rays of the cervical spine revealed a fracture-dislocation of C7 which was in good alignment as a result of traction. QUESTIONS. 4. Which of the following are true concerning surgical reconstruction for improved hand function in spinal cord injured patients? (Select one or more)

A. Applicable in at least 75% of patients

B Should be performed as soon after injury as possible

C. The wrist joint is most important in regard to handfunction

D. All the above

E. None of the above

A

(C) The wrist joint is undoubtedly the most important joint in the upper extremity in regard to restoration of hand function. Thus, in lesions below C6, if the wrist can be brought into dorsiflexion by use of tendon transfers, passive flexion of the fingers on both the metacarpal pharyngeal and interpharyngeal joints results and enables the patient to acquire some degree of grasping power. In lesions below C7, wrist dorsiflexion is preserved through,use of the extensor carpi radialis and extensor digitorium muscles. Wrist flexion and finger flexion can be improved through tendon translers to the extensor digitorium profundus and flexor poilicus longus muscles. Unfortunately, tendon transfers are not applicable in the large majority of spinal cord injured patients. Estimates of the usefulness of this procedure range from 5% by McSweeney and Bodrook to as high as approximately 25% by Lamb. Guttmann suggests that attempts at improving hand function by surgical reconstruction should be delayed for at least one and half to two years after injury. This allows sufficient time for the paralyzed patient to assess his own capabilities in compensating for neurological deficit in the upper extremities and allows observation of the potential risk and benefit for tendon transplants. Therefore, the patient with spinal cord injury and difficulty in utilization of hand function should not be rushed.into an operative procedure for hand reconstruction.

287
Q

CASE 50: 42-Year-Old Woman with C7 Spinal Cord Injury HISTORY: A 42-year-oid woman was seen one month after a spinal cord injury. The patient s family had asked for consultation concerning possible rehabilitation. EXAMINATION: The patient was in cervical traction with Gardner-Wells tongs revealed quadriplegia below C7. The C7 segment and above were preserved. There was total loss of sensation below C7. Rectal and bladder sphincters were paralyzed. X-RAYS: X-rays of the cervical spine revealed a fracture-dislocation of C7 which was in good alignment as a result of traction. QUESTIONS. 5. What would you tell the patient and family concerning her chances of returning to employment? (Select only one)

A. Impossible

B. Very poor

C. At least 80% chance

D. Depend on the degree of neurological recovery

A

(C) Estimates concerning the potential for return to useful function are extremely important to patients and the families of patients with spinal cord injury. Guttmann estimates that 80-90% of patients with cervical spinal cord lesions, whether complete or incomplete, may return to employment. Of all spinal cord injuries, approximately 50% are able to return to full-time employment, 10% are able to return to part-time employment and 20% are able to return to home occupations. Of approximately 3.000 patients cared for by Guttmann over a long period of time, only 20% were not able to be employed.

288
Q

CASE 50: 42-Year-Old Woman with C7 Spinal Cord Injury HISTORY: A 42-year-oid woman was seen one month after a spinal cord injury. The patient s family had asked for consultation concerning possible rehabilitation. EXAMINATION: The patient was in cervical traction with Gardner-Wells tongs revealed quadriplegia below C7. The C7 segment and above were preserved. There was total loss of sensation below C7. Rectal and bladder sphincters were paralyzed. X-RAYS: X-rays of the cervical spine revealed a fracture-dislocation of C7 which was in good alignment as a result of traction. QUESTIONS. 6. The family asks your advice concerning transfer to a specialized spinal cord injury center. Which of the following is true concerning patients treated at such centers in comparison to spinal cord injuref patients treated at acute general hospitals in the United States ? (Select one o r more)

A. Greater degree of functional recovery

B. incidence of urinary tract infection reduced

C. Length of hospitalization reduced

D. Incidence of decubitus ulcers reduced

E. None of the above

A

(C,D) The desirability of utilizing comprehensive spinal cord injury centers for the care of spinal cord injured patients has been stressed by Guttmann since the 1950’s. Recently, eleven such centers have been funded by the United States Department of Health, Education and Welfare. Recent evaluation of the effectiveness of this system has been carried out by Cole and others. He estimated that the average hospitalization for quadriplegic patients was reduced from 94 days in the usual acute care setting to 55 days in the spinal cord injury center. In the case of paraplegic patients, the average hospitalization was reduced from 84 to 52 days. A significant reduction in the incidence of decubitus ulceration was noted from approximately 56% for a similar group of spinal cord injured patients cared for in nonspinal cord injury centers to 29% in the United States Public Health Service spinal cord injury centers. There were no significant differences in the incidence of urinary tract infections in the two settings, however. The incidence of such infections remains at 80-90% in both groups of patients. Although there was a slight tendency for patients cared for at the spinal cord injury centers to show a higher level of functional recovery, both at the time of discharge and at one year post injury follow-up. this slight difference did not appear to be statistically significant. Cole found that the percent of patients who are.returned to gainful employment increased from 21% in the first year after injury to 46% in the third year after injury, and 67% by the fourth year after injury.